You are on page 1of 258

Portál:Otázky

z biofyziky (1. LF UK, VL)


Stáhněte si vypracované zkouškové otázky z biofyziky!

Stáhnout PDF
Nápověda:Knihy • Poslední změny
Některé názvy otázek mohou být z technických důvodů pozměněny.

Vypracované otázky z biofyziky


1. lékařská fakulta UK, obor Všeobecné lékařství

Obsah

Stavba hmoty

Typy silových interakcí

Elementární částice

Kvantové jevy

Kvantová čísla

Orbitální moment hybnosti

Orbitální magnetický moment elektronu

Spektrum atomu vodíku

Elektronový obal těžkých atomů

Vazebná energie elektronu, ionizace, excitace

Atomové jádro

Magnetické vlastnosti jader, jaderný magneton

Typy vazeb mezi atomy

Princip magnetické rezonanční tomografie

Princip hmotové spektroskopie

Molekulární biofyzika

Síly působící mezi molekulami

Stavová rovnice plynů

Maxwellovo-Boltzmannovo rozdělení rychlostí

Ekvipartiční teorém

Fázové změny
Latentní teplo

Gibbsův zákon fází

Analytické disperze

Henryho zákon

Koloidální disperze

Pohyb koloidních částic v elektrickém poli

Voda jako rozpouštědlo, voda v organizmu

Nefelometrie

Krevní tlak a jeho měření

Bernoulliho rovnice
Krevní řečiště, rovnice kontinuity

Viskozita

Hagen-Poiseuillův zákon

Stokesův zákon

Laminární proudění
Turbulentní proudění

Reynoldsovo číslo

Měření viskozity

Difúze

Fickův zákon

Koligativní vlastnosti roztoků

Raoultovy zákony

Osmotický tlak

Jevy na rozhraní fází

Thermodynamika

Vratné a nevratné děje

Práce a teplo
Měrná tepelná kapacita

Transport tepla

Vnitřní energie, entalpie

Entropie

Volná energie, volná entalpie

Chemický potenciál

Transformace a akumulace energie v buňkách

Měření teploty

Kalorimetrie

Tepelné ztráty organizmu

Termodynamické věty

Aktivní transport
Pasivní transport

Kapalinové teploměry

Termistor, termočlánek

Tepelná zařízení

Biofyzika el. jevů

Coulombův zákon a permitivita prostředí

Potenciál bodového náboje a na rozhraní fází

Elektrochemický potenciál

Klidový membránový potenciál

Akční potenciál a jeho šíření nervovým vláknem

Cyklus excitability nervového vlákna

Vedení elektrického proudu tělem,


Molární vodivost

Elektrická impedance
Účinky různých druhů proudu na organizmus

Měření vodivosti roztoků

Princip funkce osciloskopu

Měření proudu
Měření napětí
Měření odporu

Použití elektřiny v diagnostice

EKG

Použití elektřiny v terapii

Bioakustika

Vlastnosti zvuku, jeho intenzita a šíření

Akustický odpor

Dopplerův jev

Hlasitost

Weber-Fechnerův zákon

Práh sluchu a sluchové pole

Princip slyšení

Audiometrie

Vlastnosti a účinky ultrazvuku

Diagnostické aplikace ultrazvuku

Terapeutické použití ultrazvuku

Optika

UV-
IR-
viditelné světlo, vlastnosti, šíření

Index lomu světla, totální odraz


Endoskop

Disperse světla

Pružný a nepružný rozptyl

Absorpce světla
Extinkce světla

Polarizace světla, optická stáčivost

Interference světla

Energie a intenzita světla

Difrakce světla

Zobrazení lomem, čočková rovnice, optická mohutnost

Zvětšení lupy

Optický mikroskop
Elektronový mikroskop

Lidské oko, citlivost, adaptace

Vady oka, korekce optické mohutnosti oka

Princip vidění

Laser

Fyzika rentgenového záření


Vlastnosti rentgenového záření

Buzení rentgenového záření, rentgenová lampa

Absorpce rentgenového záření

Rentgenový kontrast, kontrastní látky

Comptonův rozptyl rentgenového záření, clony

Použití rentgenového záření v diagnostice

Princip počítačové tomografie

Použití rentgenového záření v terapii, polovrstva, hloubková dávka

Ochrana před rengenovým zářením, dozimetrické jednotky

Radioaktivita a ionizující záření

Radioaktivní rozpad

Efektivní, fyzikální a biologický poločas

Radioaktivní rovnováha

Druhy radioaktivního rozpadu

Rozpad α

Rozpad β

Urychlovače částic

Kosmické záření, měření pozadí detektoru

Interakce záření α

Interakce záření β

Interakce záření γ

Interakce neutronů

Detekce ionizujícího záření

Ionizační komora

Geiger-Mullerova trubice

Scintilační detektor

Integrální a selektivní detekce záření γ

Jednotky radiační dozimetrie

Měření aktivity in vitro a in vivo

Osobní dozimetrie
Typy silových interakcí
Jedna ze dvou forem hmoty je pole. Pro pole je charakteristické vzájemné silové působení zdrojů jednotlivých druhů
těchto polí. Silové působení má výměnný charakter, je zprostředkováno výměnou kvant těchto polí. Hledá se teorie
sjednocující popis všech typů polí, snaha o popsání všech interakcí jedním systémem rovnic (tzv. teorie všeho).

Čtyři typy interakcí


Podrobnější informace naleznete na stránce Standardní model částicové fyziky.

V přírodě se vyskytují jen čtyři typy interakcí:

Tabulka fundamentálních částic


(standardní model)

Gravitační síla

Nejslabší ze všech interakcí, kvantem je hypotetický graviton, zdrojem je hmotnost, působí v makrosvětě a na velké
vzdálenosti je převažujícím typem interakce.

Elektromagnetická síla

Kvantem je foton a zdrojem je elektrický náboj. Působí mezi nabitými částicemi a způsobuje elektromagnetické jevy.
Tato interakce určuje velikost atomů tím, že vytváří vazbu mezi elektrony a jádrem. Tato síla tedy určuje objem
objektů. Většina běžných sil v makrosvětě (třecí síly, odporové síly, …) je způsobena právě elektromagnetickou
interakcí.

Silná jaderná síla

Kvantem je gluon, který k sobě váže kvarky. Zdrojem je barva, nasycené pole s krátkým dosahem. Silná jaderná síla
působí na úrovni jádra.

Slabá jaderná síla


Kvantem je intermediární boson. Zdrojem je vůně. Jedná se o interakci, která způsobuje rozpad beta.

Odkazy
Externí odkazy

Fyzikální síly – překlad ze stránek nadace Alfreda Nobela

Použitá literatura
KUBATOVA, Senta. Biofot [online]. [cit. 2011-01-31]. <https://uloz.to/!CM6zAi6z/biofot-doc>.
Standardní model částicové fyziky

Elementární částice (rozdělení)

Kvarky

Standardní model
Podle kvantové teorie pole vypracované v sedmdesátých letech (Standardní model) se veškerá hmota ve vesmíru
skládá ze šesti druhů kvarků a šesti druhů leptonů, mezi nimiž mohou nastávat různé interakce; tyto interakce jsou
prostředkovány tzv. výměnnými (neboli intermediálními čili zprostředkujícími) částicemi příslušného pole.

Fermiony a bosony
Podle hodnoty spinu dělíme částice na fermiony (poločíselný spin, jejich chování lze popsat Fermiho-Diracovou
statistikou a chovají se podle Pauliho vylučovacího principu, který říká, že nemohou existovat dva fermiony s úplně
totožnou energetickou charakteristikou) a bosony (celočíselný spin, popsány Einstein-Boseovou statistikou). Podle
tohoto dělení kvarky a leptony (tj. základní stavební částice látkové formy hmoty) patří mezi fermiony, zatímco
intermediální částice fyzikálních polí patří mezi bosony; uvedeme několik příkladů:

fermiony (poločíselný spin):


leptony - např. elektrony, pozitrony, neutrina,
kvarky - skládají se z nich těžší částice např. nukleony,
některé složené částice, např. baryony,
bosony (celočíselný spin):
intermediální částice - fotony, gluony atd.,
některé složené částice, např. mezony.

Základní fyzikální interakce


Tradičně fyzika uvažuje o čtyřech základních typech interakcí:

Elektromagnetická síla
Gravitace
Slabá interakce
Silná interakce

Tyto interakce dávají vzniknout následujícím polím:

Druh pole Zdroj pole Dosah Kvantum


Elektromagnetické pole elektrický náboj neomezený foton
Gravitační pole hmotnost neomezený graviton (hypotetická částice)
Silné jaderné pole barva 10-15 m gluony

Slabé jaderné pole vůně 10-18 m


intermediární bosony (W+, W-, Z0)

Standardní model je založen na objevech z 60. let, které ukazují, že je možno zkombinovat elektromagnetickou sílu a
slabou interakci do jedné tzv. elektroslabé interakce.
Gravitace

Z řeckého gravis – těžký, je to obecná vlastnost všech těles.

Gravitační interakce, na rozdíl od ostatních interakcí, působí bez výjimky na všechny částice. Gravitační
interakce je vždy přitažlivá a má nekonečný dosah, tzn. její účinky se nedají vyrušit. Tudíž je rozhodující silou mezi
velmi vzdálenými objekty. Také ale platí, že ze všech interakcí je právě interakce gravitační tou nejslabší. Gravitační
interakce působící mezi dvěma tělesy jsou tedy vždy vzájemné, což vyplývá i z třetího Newtonova zákona. Proto
můžeme říci, že gravitace je obecnou vlastností všech hmotných objektů. Vzájemné silové působení dvou těles ale
nedokážeme vždy pozorovat, projevuje se jen pohybový účinek síly mnohonásobně hmotnějšího tělesa na
druhém, a to i přesto, že vzájemná přitažlivá síla těchto těles je shodná.

Gravitace je spojována s mnoha tématy, od struktury galaxií, vzniku černých děr, velkým třeskem, až po v praxi
naprosto samozřejmé jevy jako je padání předmětů, nemožnost unést některá těžká tělesa apod.

Newtonův gravitační zákon – je nejstarší fyzikální teorie popisující gravitační interakce, je součástí klasické fyziky,
tudíž se využívá především pro popis gravitačních interakcí slabých gravitačních polí a částic o nízkých rychlostech
ve srovnání s rychlostí světla.

Obecná teorie relativity – popisuje gravitaci jako zakřivení časoprostoru způsobené vlastnostmi prostoru a času.
Předpokládá existenci gravitačních vln (šířící se změny gravitačního pole), které se pohybují rychlostí světla. Využívá
se pro silná pole a pro rychlosti blížící se rychlosti světla. Do budoucna se uvažuje o teorii kvantové gravitace, která
by měla být nástupcem obecné teorie relativity a měla by propojit kvantovou fyziku, elektrodynamiku, jadernou a
částicovou fyziku s gravitací (teorií o prostoru a času). Hypotetickou částicí teorie kvantové gravitace je graviton.

Elektromagnetická síla

Jedná se o nejlépe prostudovanou silovou interakci, se kterou se běžně setkáváme například v televizorech,
počítačích, radiích, při přenosu světla nebo dokonce v nás samotných v podobě nervových vzruchů. Je zodpovědná za
tvar a objem těles, neboť se stará o soudržnost atomů, jejich velikost, strukturu látek a tvorbu chemických
vazeb. Její působení můžeme pozorovat jako projev odporových nebo třecích sil.

Elektromagnetická síla je dalekodosahová a se vzdáleností klesá její síla kvadraticky. Působí pouze na nabité
částice. Může být buď odpudivá nebo přitažlivá. Pokud má jedna částice náboj kladný a druhá záporný, vznikne síla
přitažlivá, pokud mají obě částice náboj shodný, bude mezi nimi síla odpudivá. Částice ale mohou mít i neutrální
elektrický náboj, potom mezi nimi elektromagnetická síla působit nebude.

O elektromagnetické síle často uvažujeme, jako kdyby byla složena ze dvou polí- elektrického a magnetického. Mezi
těmito poli však existuje úzký vztah, nelze je zkoumat zcela odděleně a proto např. u elektromagnetického vlnění
hovoříme o elektrické a magnetické složce, které jsou vzájemně kolmé.

Intermediálními částicemi elektromagnetické síly jsou fotony. I když elektromagnetická síla působí mezi nabitými
částicemi, fotony samy o sobě žádný náboj nenesou.

Elektromagnetická interakce má svou teorii v klasické i kvantové fyzice:

V klasické fyzice elektromagnetické interakce popisuje např.:

Coulombův zákon (síly mezi dvěma náboji),


Gaussův zákon elektrostatiky (tok intenzity elektrického pole),
Lorentzova síla (náboj pohybující se v elektrickém a magnetickém poli),
Ampérův zákon (elektrický proud vytváří magnetické pole),
Biotův-Savartův zákon (elektromagnetická indukce),
Maxwellovy rovnice (elektromagnetické vlnění).

Elektromagnetické síly v kvantovém světě popisuje kvantová elektrodynamika. Nejdůležitějším odtud pramenícím
poznatkem (zejména pro biofyziku) je zjištění, že naprostá většina pozorovaných jevů (např. chemické vazby,
interakce záření s hmotou atd.) je důsledkem vzájemných interakcí mezi fotony a elektrony. Tyto foton-
elektronové interakce dobře znázorňují Feynmanovy diagramy.

Slabá interakce
β rozpad neutronu

Slabých interakcí se účastní leptony a hadrony, projevují se v rozpadech neutronu či mionu. Mají velmi malý dosah
10-17 m a jejich intermediálními částicemi jsou bosony W+, W-, Z0.

Při nízkých energiích (do 20 GeV) jsou dosti slabé, proto jsou označovány jako slabé interakce. Při vyšších energiích
jsou přibližně stejně silné jako elektromagnetické interakce.

Podílí se na β rozpadu neutronu, kdy za účasti bosonu W- dojde ke vzniku protonu, elektronu a elektronového
antineutrina.

Slabou interakci společně s elektromagnetickou silou popisuje elektroslabá teorie.

Silná interakce

Působí mezi kvarky, které tvoří hadrony. Její náboj se označuje jako barevný náboj. Má pouze malý dosah, řádově
10-15 m, a je nejsilnější ze základních interakcí. Zprostředkujícími částicemi této interakce jsou gluony.

Poutá k sobě nukleony (zbytková silná interakce) a je zodpovědná za velmi rychlý rozpad hadronů.

Kvantová chromodynamika (QCD) popisuje chování částic při silných interakcích.

Intermediální částice
gluon
foton
bosony W+, W- a Z0
graviton

Základní částice hmoty


Elementární částice

Terminologický problém: Pojem „elementárních (základních) částic“ jako nejjednodušších stavebních prvků hmoty je
závislý na stavu poznání v dané době. Postupně se tento termín používal pro hadrony (protony, neutrony…), leptony
a kvarky (v rámci tzv. Standardního modelu) a s pokračujícím poznáním se tento termín může přesunout k dalším
částicím, tak jak se nám budou zdát „elementárními“. Proto je lépe se pojmu „elementárních částic“ vyhnout.

Leptony a kvarky

1. Leptony (neinteragují se silnou jadernou silou; elektrony a elektronová neutrina, miony a mionová neutrina,
tauony a tauonová neutrina).
2. Kvarky (vůně: up, down, charm, strangeness, bottom, top; barva: červená, zelená, modrá; náboj -1/3 nebo +2/3).

Fundamentální částice mají spin ±1/2 a ke každé existuje antičástice (stejná hmotnost, opačná točivost spinu, opačný
magnetický moment, náboj i barva).

Kvarky se mohou skládat v hadrony (podmínky: celočíselný náboj a bílá barva), mesony (kvark a antikvark;
celočíselný spin) a baryony (3 kvarky, neceločíselný spin; proton a neutron).

Složené částice
Hadrony

mezony
baryony:
nukleony
hyperony

Částice alfa

V částicové fyzice jsou za alfa částice považována jádra 4He. Částice se skládá z 2 neutronů a 2 protonů, celkový náboj
této částice je +2e, značíme ji α nebo He2+. Alfa částice má nenulovou klidovou hmotnost, tudíž se pohybuje rychlostí
vždy menší než rychlost světla. Proud částic α označujeme jako alfa záření, vzniká při alfa rozpadu.

Antičástice
Antičástice je ve své podstatě „opakem” částice. Antičástice a k ní příslušná částice mají stejnou hmotnost, spin a
střední dobu života, liší se od sebe elektrickým nábojem, magnetickým momentem, baryonovým a leptonovým číslem,
izospinem a podivností. Antičástice se řídí stejnými fyzikálními zákony jako částice.

Podrobnější informace naleznete na stránce Antičástice.

Kvazičástice
Kvazičástice jsou koncept především fyziky kondenzovaných látek. Nejedná se o částice v pravém slova smyslu, jde o
vzruch šířící se daným prostředím, který je výhodné pro potřeby další analýzy pokládat za částici. Pomocí kvazičástic
lze analyzovat některé složité fyzikální děje.

Podrobnější informace naleznete na stránce Kvazičástice.

Odkazy
Související články

Atom
Atomové jádro
Comptonův rozptyl
Hadrony
Základní částice hmoty

Externí zdroje

Elementární částice
Záznam přednášky: Prof. RNDr. Petr Kulhánek, CSc., FEL ČVUT, Astrofyzika 03 (Výtečná přednáška! – cca 500
MB)
Systematický přehled elementárních částic na serveru Aldebaran.cz

Použitá literatura
MACHÁČEK, Martin a Zuzana FOGLAROVÁ. Encyklopedie fyziky. 1. vydání. Praha : Mladá fronta, 1999. 
408 s. Kapitola 6: Kvantová fyzika. ISBN 80-204-0237-3.

KATEDRA DIDAKTIKY FYZIKY, MFF UK V PRAZE,, et al. FyzWeb [online]. [cit. 2013-11-29]. 


<http://fyzweb.cz/materialy/sily/obecne/inter.php>.

REICHL, Jaroslav a Martin VŠETIČKA. Encyklopedie fyziky [online]. [cit. 2013-11-29]. 


<http://fyzika.jreichl.com/main.article/view/894-ctyri-silove-interakce-v-prehledu>.

NEZNÁMÝ, Autor. aldebaran [online]. [cit. 2013-12-04]. 


<https://www.aldebaran.cz/astrofyzika/interakce/particles.html>.

WAGNER, Vladimír. Objective Source E-Learning [online]. [cit. 2013-12-04]. <http://www.osel.cz/3457-jak-se-


vyznat-ve-vsemoznych-casticich.html>.

VELTMAN, Martinus. Fakta a záhady ve fyzice elementárních částic. 1. vydání. Praha : Academia, 2007. ISBN


978-80-200-1500-6.
Kvantové jevy
V kvantové mechanice je základní konstantou kvantum účinku, tzv. Diracova konstanta ħ = 1,05 × 10-34 J.s.
Diracova konstanta souvisí s Planckovou konstantou (h = 6,63 × 10-34 J.s) převodním vztahem ħ = h / 2π. Obě
konstanty vystupují v důležitých vztazích, které kvantitativně spojuje dualní charakter hmoty. Plancova konstanta
představuje nejmenší možnou dávku energie vyzařovanou tělesem.

Moment hybnosti
Jedná se o vektrorový součin polohového vektoru r a vektoru hybnosti p. Moment hybnosti kruhového orbitálního
pohybu částice může nabývat pouze hodnot, které jsou násobky Diracovy konstanty. Podobně je to s průmětem
orbitálního momentu hybnosti do souřadnicových os atomů.

Má-li elektricky nabitá částice orbitální moment hybnosti, musí existovat i magnetický moment, jelikož pohyb
elektrického náboje dává vzniknout magnetickému poli. Orbitální magnetický moment daný rotací částice okolo
vlastní osy se nazývá spin (fermiony X bosony).

Dualismus
Elementární částice a z nich utvořené systémy mají zároveň korpuskulární i vlnové vlastnosti. Pohyb každé částice je
tedy spjat s šířením hmotnostních vln (λ = h / m × v = h / √2mE; de Broglieho vlnová délka).

Korpuskulárně-vlnový charakter částic má ten důsledek, že není možné přesně určit současně polohu částice a její
hybnost: Heisenbergova relace neurčitosti Δ r Δp
ħ

Energie fotonu je svázána s vlnovou délkou světelné vlny vztahem E = h × f = hc / λ. Vlnová délka je vzdálenost,
kterou světlo urazí za dobu jedné periody (λ = c × T = c / f)

Také platí, že čím déle existuje příslušný energetický stav, tím přesněji můžeme určit jeho energii: Δ E
Δt ħ

Zákony pohybu v kvantové mechanice jsou popsány Schrödingerovou rovnicí, jejíž druhá mocnina její absolutní
hodnoty = hustota pravděpodobnosti výskytu částice.

Elektron urychlený potenc. rozdílem 1 V má energii 1eV: 1 J = 1 C ×. 1 V => 1 eV = 1,6 × 10-19 J

Článek neobsahuje vše, co by měl.


Můžete se přidat k jeho autorům a jej.
O vhodných změnách se lze poradit v diskusi.

Odkazy
Zdroj

KUBATOVA, Senta. Biofot [online]. [cit. 2011-01-31]. <https://uloz.to/!CM6zAi6z/biofot-doc>.

BENEŠ, Jiří, Daniel JIRÁK a František VÍTEK, et al. Základy lékařské fyziky. 4. vydání. 2015. 17, 19 s. ISBN
9788024626451.
Kvantová čísla
Kvantová čísla popisují chování kvantového systému. Matematicky jde o parametry řešení Schrödingerova rovnice
příslušného systému. Toto řešení se obvykle značí a nazývané vlnová
funkce. Druhá mocnina vlnové funkce[pozn. 1] má význam hustoty pravděpodobnosti výskytu elektronu.[1]

Tabulka kvantových čísel

Atomový orbital
Vlnová funkce popisující konkrétní elektron konkrétního atomu se označuje jako
atomový orbital (AO); stejný výraz je v přeneseném slova smyslu používán i pro prostor, v němž se elektron s
největší pravděpodobností vyskytuje. Elektron se totiž okolo jádra pohybuje po trajektorii, kterou ovšem vzhledem
k Heisenbergovu principu neurčitosti nelze dostatečně zjistit. Zvolením určité hladiny pravděpodobnosti lze pomocí
vlnové funkce vymezit a matematicky popsat část prostoru, v níž se bude elektron s danou pravděpodobností
nacházet. Pro zjednodušení namísto se tento popis neuvádí ve formě konkrétních vlnových funkcí, ale pouze ve formě
parametrů, po jejichž dosazení do obecného řešení bychom získali konkrétní vlnovou funkci. Parametry se nazývají
kvantová čísla.

Jednotlivá kvantová čísla


Řešení Schrödingerovy rovnice (a tím i vlnová funkce a atomový orbital) je
kompletně určeno třemi kvantovými čísly . Elektron v daném orbitalu může
navíc nabývat dvou kvantových stavů, proto se popisuje ještě dalším kvantovým číslem
. Elektronu samotnému je mezi elementárními částicemi přiřazeno kvantové číslo
.

Hlavní kvantové číslo n

Hlavní kvantové číslo ( ) určuje celkovou energii elektronu a tedy i slupku,


ve které se elektron nachází (K pro , L pro , M
pro , N pro ,…). Pro energii elektronu platí
vztah:

Protože hmotnost elektronu , náboj elektronu ,


permitivita vakua i Planckova konstanta ,
lze je shrnout do jediné konstanty a vztah zapsat jako:

Vedlejší kvantové číslo l

Vedlejší kvantové číslo ( souvisí s kvantováním orbitálního momentu


hybnosti . Určuje tvar a symetrii elektronového oblaku, tedy podslupky s, p, d, f, g a h.

Konstanta se nazývá redukovaná Planckova konstanta a platí pro ni:

Magnetické kvantové číslo ml

Magnetické kvantové číslo ( ) určuje orientaci orbitalu v


prostoru; magnetický orbitální moment (pohyb elektrického náboje vytváří magnetické pole) má opačný směr než
orbitální moment hybnosti.

Spinové kvantové číslo s

Spinové kvantové číslo elektronů ale také protonů a neutronů má vždy hodnotu ½, čímž tyto částice řadí mezi
fermiony[pozn. 2]. V případě elektronu je moment hybnosti daný jeho spinovým číslem dán vztahem:
Spin nabitého fermionu se projevuje i jako jeho magnetický moment. Právě představa, že rotující elektron představuje
vlastně pohybující se náboj, kolem kterého vznikne magnetické pole, vedla k tomu, že bylo toto číslo pojmenováno
jako spinové. Jednou ze základních vlastností mikrosvěta, která se vymyká běžným makroskopickým analogiím, je
kvantování. V případě, že se např. elektron vloží do vnějšího magnetického pole, nemůže jeho magnetický moment
zaujímat libovolnou orientaci. Možné jsou pouze dvě orientace, jejich zohledněním jsou tak možné dvě hodnoty
spinového magnetického čísla: +½ a −½., tedy průmět do vnějšího magnetického pole může mít pouze dvě hodnoty:

Pauliho vylučovací princip


Pauliho vylučovací princip (též Pauliho princip výlučnosti) říká, že v jednom kvantovém stavu se nemohou
současně nacházet dva fermiony, tedy fermiony s identickými kvantovými čísly. Princip je pojmenován po švýcarském
fyzikovi Wolfgangu Paulim (1900–1958).

Protože elektrony jsou fermiony, Pauliho vylučovací princip pro ně také platí. Formulace specifická pro elektrony říká,
že v jednom atomu nemohou existovat dva elektrony, které mají všechna kvantová čísla shodná, tedy že se musí
nejméně v jednom kvantovém čísle lišit. Tím je mimo jiné dán maximální možný počet elektronů v každé slupce.

Dovolené přechody
Při přechodu elektronu z jedné energetické hladiny do druhé nejsou možné zcela libovolné přechody. Možné (tzv.
povolené) jsou jedině ty přechody, kdy se hlavní kvantové číslo n mění libovolně, ale vedlejší kvantové číslo l jen
o jedničku.. Ostatní přechody jsou označovány jako zakázané, pravděpodobnost jejich uskutečnění je výrazně nižší než
pravděpodobnost, že se uskuteční povolený přechod[pozn. 3].

Poznámky pod čarou


1. Ve skutečnosti se jedná o druhou mocninu pouze ve speciálních případech, kdy má Schrödingerova rovnice
reálné řešení. V případě, že je řešením komplexní funkce, jde o součin Ψ a funkce k ní komplexně sdružené.
2. Fermiony jsou elementární částice s poločíselným spinem (tj. jejich spin je lichým násobkem jedné poloviny).
Jejich hlavní charakteristikou je to, že pro ně platí Pauliho vylučovací princip. Pojmenovány jsou po italském
fyzikovi Enricovi Fermim (1901–1954). Jejich "opakem" jsou bosony pojmenované po indickém matematikovi a
fyzikovi Satyendrovi Nathovi Bosemu (1894–1974), které se Pauliho vylučovacím principem neřídí. Pro bosony je
obvyklý celočíselný spin. Typickými bosony jsou např. fotony.
3. Důvodem nenulové pravděpodobnosti zakázaného přechodu je existence tunelového jevu. Ten v tomto případě
umožňuje elektronu provést s malou ale nenulovou pravděpodobností "dvojskok" přes povolený mezistav, i když
na něj nemá dostatek energie.

Odkazy
Literatura

HRAZDIRA, Ivo a Vojtěch MORNSTEIN. Lékařská biofyzika a přístrojová technika. 1. vydání. Brno : Neptun, 


2001. 396 s. ISBN 80-902896-1-4.

MUCK, Alexander. Základy strukturní anorganické chemie. 1. vydání. Praha : Academia, 2006. 508 s. s.


36. ISBN 80-200-1326-1.

Související články
Atom
Elektronový obal těžkých atomů

Externí odkazy

KUBATOVA, Senta. Biofot [online]. [cit. 2011-01-31]. <https://uloz.to/!CM6zAi6z/biofot-doc>.

JANEČEK, Ivan, René KALUS a Daniel HRIVŇÁK. Kvantová, atomová a jaderná fyzika [online]. Katedra fyziky
PřF OSU, [cit. 2011-09-21]. <http://artemis.osu.cz/mmfyz/index.htm>.
článek Atomic orbital na anglické Wikipedii
článek Pauli exclusion principle na anglické Wikipedii
článek Quantum number na anglické Wikipedii

Reference
1. MUCK, Alexander. Základy strukturní anorganické chemie. 1. vydání. Praha : Academia, 2006. 508 s. s.
36. ISBN 80-200-1326-1.
Orbitální moment hybnosti

Orbitální moment hybnosti

Přirozenou jednotkou orbitálního momentu


hybnosti je ħ

Odkazy
Související články

Kvantové jevy

Zdroj

KUBATOVA, Senta. Biofot [online]. [cit. 2011-01-31]. <https://uloz.to/!CM6zAi6z/biofot-doc>.

Článek neobsahuje vše, co by měl.


Můžete se přidat k jeho autorům a jej.
O vhodných změnách se lze poradit v diskusi.
Orbitální magnetický moment elektronu

e/2me se nazývá gyromagnetický poměr γ

m určuje směr vektoru orbitálního momentu hybnosti L (jeho složku ve směru vnějšího mag. pole)

jednotkou je Bohrův magneton (e.ħ/2me=0,927.10-23 A.m2)

Odkazy
Související články

Magnetické vlastnosti jader, jaderný magneton


Kvantové jevy

Zdroj

KUBATOVA, Senta. Biofot [online]. [cit. 2011-01-31]. <https://uloz.to/!CM6zAi6z/biofot-doc>.


Spektrum atomu vodíku
Přejde-li elektron ze stavu Ek do En, kdy k>n, je emitováno kvantum záření, o energii rovné rozdílu energie těchto
hladin.

Frekvence záření je dána vzorcem E=h.f. Vzhledem ke kvantování energie je záření nespojité a tvoří tzv. sérii čar.

Lymanova série leží v ultrafialové části spektra a odpovídá přechodu elektronů na základní energetickou
hladinu n=1.
Balmerova série je přechod na n=2 a je ve viditelné části spektra. Další série (n=3 apod.) jsou v infračervené
části spektra.

Odkazy
Zdroj

KUBATOVA, Senta. Biofot [online]. [cit. 2011-01-31]. <https://uloz.to/!CM6zAi6z/biofot-doc>.


Elektronový obal těžkých atomů
Dvě základní pravidla:

1. systém částic je stabilní, jestliže je jeho celková energie minimální (pokud tomu tak není, snaží se systém
dosáhnout stavu s nejnižší možnou energií),
2. v každém kvantovém stavu (určeném 4 parametry) smí v jednom systému (elektronovém obalu) existovat jen 1
elektron (viz Kvantová čísla).

Počet elektronů v podslupce je 2(2l+1). Atomové slupky se zaplňují následovně: 1s, 2s, 2p, 3s, 3p, 4s, 3d, 4p, 5s, 4d,
5p, 6s, … (3d stav má vzhledem k vyšší hodnotě l vyšší energii než 4s, proto se zaplňuje později). Uplatňuje se zde
Pauliho vylučovací princip a Hundovo pravidlo (= dokud je to možné, zůstávají elektrony v atomu nespárované,
tzn. mají rovnoběžné spiny, protože takové elektrony jsou více navzájem odděleny kvůli stejnému magnetickému
kvantovému číslu => tento stav má nižší energii).

Hundovo pravidlo

Odkazy
Zdroj

KUBATOVA, Senta. Biofot [online]. [cit. 2011-01-31]. <https://uloz.to/!CM6zAi6z/biofot-doc>.


Vazebná energie elektronu, ionizace, excitace
Stav atomu o minimální energii se nazývá základní stav. Stavy o vyšších energiích jsou stavy excitované či vybuzené.
Do vyšší energetické hladiny se atom dostane absorpcí energie o rozdílu základní a některé vyšší hladiny (čárová
absorpční spektra).

Při přechodu do nižší energetické hladiny je vyzářen rozdíl energií ve formě fotonu (nebo fotonů, je-li přechod po
etapách, z n=3 na n=2 a pak na n=1). Tento děj je podstatou luminiscence.

Energie vazby
Energie vazby (Ev) je práce, kterou je nutno vynaložit pro vzdálení částice do nekonečna - místo kde na částici
nebude působit žádná jaderná síla.

Ev + E = 0 => Ev = – E

Největší vazebnou energii mají elektrony s nejmenším n (hlavní kvantové číslo), které jsou nejblíže jádru.

Vazebná energie
Vazebná energie nebo-li ionizační potenciál, tzn. energie, kterou je nutno dodat pro výstup elektronu z atomu.
Získá-li elektron takovou energii (např. ozářením), pak část energie je spotřebována pro výstupní práci a zbytek se
přemění na kinetickou energii vyraženého elektronu:

h.f=Ev+1/2.mv2 (Einsteinův vztah pro fotoefekt)

Ionizací se zvýší celková energie atomu, tím ubude záporná energie elektronu. Tento důsledek vede k nestabilitě.
Vyrazí-li se elektron z nižší slupky, atom se energie zbaví zaplněním nižších hladin elektrony z hladin vyšších za
současné emise fotonů -> fluorescenční záření. Vyrazí-li se valenční elektron, energie se sníží připoutáním volného
elektronu z okolí.

Potenciální bariéra je maximální hodnota potenciálu jádra. Vyjadřuje se v elektronvoltech (eV).

Odkazy
Zdroj

KUBATOVA, Senta. Biofot [online]. [cit. 2011-01-31]. <https://uloz.to/!CM6zAi6z/biofot-doc>.


Atom

Model atomu

Leukipos a Demokritos – před 2.5 tis. let na základě pozorování přírodních dějů vyslovili teorii, že všechny látky,
které existují, jsou složené z malých, nezničitelných a nedělitelných částic – atomů.

Dnešní poznatky: Atomy jsou základní stavební částice chemických látek s poloměrem řádově 10-10m.

Jádro atomu
Atomové jádro se skládá z nukleonů (určených hmotnostním číslem A): protonů (atomové číslo Z) a neutronů
(neutronové číslo N). Platí A = Z + N. Číslo Z také udává počet elektronů (atom je elektricky neutrální).

Struktura jádra
Je složené ze dvou mikročástic – protonu a neutronu, které označujeme společným názvem nukleony. Poloměr
jádra je přibližně 5·10−15 m.

Proton

Kvarkový model protonu

Proton je částice, která je nositelem kladného elementárního náboje, má označení p (případně p+, 11p, H+). Počet
protonů v jádře atomu je charakteristický pro každý prvek. Počet protonů odpovídá počtu elektronů –
elektroneutralita atomu.

Protonové číslo Udává počet protonů v jádře atomu a počet elektronů v obalu elektroneutrálního atomu. Shoduje se
s pořadovým číslem prvku v periodické soustavě prvků.

H+ jako proton

Někdy je vodíkový kation, který vznikl disociací kyselin, označovaný též jako proton. Při ztrátě svého jediného
elektronu zůstává jen jádro tvořené jedním protonem (u nejlehčího izotopu 11H).

Neutron
Neutron je elektricky neutrální částice rozměrově přibližně stejná s protonem. Počet neutronů v jádře udává
neutronové číslo – N' a počet nukleonů v jádře (tzn. protonů + neutronů) udává nukleonové číslo – A.

Vlastnosti částic
Relativní
Pokojová
Název Symbol Náboj [C]
hmotnost [g] elementární náboj

proton p 1,6726·10−24 1,602·10−19 +1


neutron n 1,6750·10−24 0 0

elektron e 9,110·10−28 −1,602·10−19 −1

Charakteristika jádra

Doba přeměny poloviny nuklidu


podle proton-neutronového složení.

Atomová hmotnost se vyjadřuje v hmotnostních jednotkách: 1 hj = 1,66·10−27 kg. Každý atom je charakterizovaný
protonovým a nukleonovým číslem, na základě toho rozlišujeme:

Izotopy – Složené z atomů, které mají stejné protonové, ale různé nukleonové číslo. Liší se tedy počtem
neutronů. Mají stejné chemické vlastnosti, ale odlišují se fyzikálními vlastnostmi. V přírodě se většina prvků
vyskytuje v izotopové směsi.
Izobary – Nuklidy rozličných prvků, které mají stejné nukleonové číslo, ale rozdílné protonové číslo.
Izotony – Nuklidy rozličných prvků, které mají rozdílné protonové ale taky nukleonové číslo, ale mají stejný
počet neutronů v jádře.
Izomery – Atomy s dočasně zvýšenou celkovou energií, tzn. nestabilní.

Celkový náboj jádra je Z · 1,6·10−19 C. Poloměr jádra vypočítáme podle vzorce:


.

Síly v jádře jsou projevem silné jádrové interakce. Působí jen v jádře na vzdálenost cca 10−15 m. Jsou to nejsilnější
síly, jaké v přírodě známe.

Radioaktivita
Atomová jádra některých prvků jsou nestabilní a podléhají samovolné přeměně (přírodní radioaktivita), přičemž
dochází k emisi záření – ionizující záření.

U přirozených radionuklidů rozeznáváme tyto složky záření:

záření α- proud atomových jader helia


záření β- tvořené proudem elektronů;
záření γ – nejpronikavější, s krátkou vlnovou délkou;
umělá radioaktivita vzniká například při bombardování jader atomů částicemi α, přičemž vzniká umělý
radionuklid.

Elektronový obal atomu


Může obsahovat jeden nebo více elektronů.
Elektron

Mikročástice a nositel záporného náboje s označením e nebo e-. Elektrony v atomu existují jen ve stavech s určitou
energií.

Elektrony mohou energii vyzařovat nebo přijímat jen po určitých kvantech, a to při přechodu z jedné energetické
hladiny na druhou. Energie kvanta je dána rozdílem energií, které náleží určitým energetickým hladinám v obalu
atomu.

Elektron má duální charakter – vlnový a korpuskulární. Tomu odpovídá také princip neurčitosti (není možné určit,
kde se elektron nachází a jakou rychlostí se pohybuje). Můžeme jen určit pravděpodobnost − místo s největší
pravděpodobností výskytu elektronu se nazývá orbital.

Kvantová čísla

Stav elektronu v obalu atomu popisují 4 kvantová čísla (3 charakterizují orbital a čtvrté projekci spinu konkrétního
elektronu v orbitalu).

Hlavní kvantové číslo (n) – Vyjadřuje energii orbitalu. Nabývá hodnot od 1 do 7 (nebo také označováno jako K,
L, M, N, O, P, Q).
Vedlejší kvantové číslo (l) – Nabývá hodnot od 0 do (n – 1). Udává prostorový tvar a energetickou odlišnost
jednotlivých orbitalů.
Magnetické kvantové číslo (m) – Udává směrovou orientaci orbitalu v magnetickém poli. Nabývá hodnot od -
l…0…+l (vedlejší kvantové číslo).
Spinové kvantové číslo (s) – Nabývá hodnot +1/2 nebo −1/2. V jednom orbitalu můžou být maximálně dva
elektrony, které mají opačný spin a vytvářejí elektronový pár.

Elektr. vrstva n l Typ orbitalu m Počet orbitalů Max. počet elektronů


K 1 0 1s 0 1 2
0 2s 0 1
L 2
1 2p −1, 0, +1 3 8
0 3s 0 1
M 3 1 3p −1, 0, +1 3
2 3d −2, −1, 0, +1, +2 5 18
0 4s 0 1
1 4p −1, 0, +1 3
N 4
2 4d −2, −1, 0, +1, +2 5
3 4f −3, −2, −1, 0, +1, +2, +3 7 32

Pravidla zaplňování orbitalů elektrony

Tvary orbitalů 1s, 2s, 2px, 2py a 2pz

Konfigurace elektronů (iontů)− rozmístění elektronů v jednotlivých orbitalech na jednotlivých vrstvách


elektronového obalu. Možné z něj odvodit schopnost atomu (iontu) tvořit určitý typ chemické vazby. Postupné
obsazování určuje výstavbový a Pauliho princip a Hundovo pravidlo.

Výstavbový princip – princip minimální energie: Atom v základním stavu má snahu nabývat stavu s nejmenší možnou
energií – obsazování orbitalů od těch s nejmenší energií: 1s-2s-2p-3s-3p-4s-3d-4p-5s-4d-5p-6s-4f-5d-6p-7s-5f-6d-7p.

Degenerované orbitaly- mají stejnou energii (mají stejné hlavní a vedlejší kvantové číslo, liší se magnetickým).

Pauliho princip V atomu neexistují dva elektrony, které by měly všechny 4 kvantová čísla stejná (musí se lišit
minimálně spinem – znázorníme opačnými šipkami).
Hundovo pravidlo Degenerované orbitaly se zaplňují tak, aby byl v degenerovaných orbitalech maximální možný
počet nespárovaných elektronů (co nejvíc se stejným spinem) a až potom se začnou vytvářet elektronové páry.

Odkazy
Související články

Atomové jádro
Periodická soustava prvků
Radioaktivita

Použitá literatura

SILNÝ, Peter a Beata BRESTENSKÁ. Prehľad chémie 1. 1. vydání. Bratislava : Slovenské pedagogické


nakladateľstvo, 2000. sv. 1. s. 246. ISBN 80-08-00376-6.
Magnetické vlastnosti jader, jaderný magneton
Nukleony mají vlastní magnetický moment – spin (1/2 ħ).

Dělení dle spinu

1. Jádra s lichým počtem protonů a neutronů (licho-lichá jádra) nebo nukleonů (sudo-lichá) mají výsledný spin
různý od nuly a tedy nenulový magnetický moment, protože jak protony, tak neutrony jsou utvářeny el.
nabitými kvarky.
2. Jádra sudo-sudá mají výsledný spin nulový.

Spin jádra může být vyjádřen jako součin jaderného spinového čísla I a Dir. konstanty a jádra se spin. číslem I ≥ 1/2
mají (2I + 1) stabilních energetických stavů.

Magnetický moment
U jader s I > 0 můžeme pozorovat jevy nukleární magnetické rezonance.

Velikost magnetického momentu jádra vyjadřujeme v jednotkách jaderného magnetonu (e. ħ/2mp= 5,05.10-27 A.m-
2), který je 658x menší než Bohrův magneton.

Protože je proton složen z kvarků, jeho magnetický moment je 2,8 jaderného magnetonu (neutron 1,9).

Pozor!

Feromagnetické vlastnosti látek jsou způsobeny magnetickými vlastnostmi nepárových elektronů v


atomovém obalu.

Odkazy
Související články

Kvantové jevy
Orbitální magnetický moment elektronu

Zdroj

KUBATOVA, Senta. Biofot [online]. [cit. 2011-01-31]. <https://uloz.to/!CM6zAi6z/biofot-doc>.


Typy vazeb mezi atomy

Mezi atomy se vytvoří vazba jen tehdy, když má nově vzniklá molekula nižší energii (vyšší stabilitu) než původní
atom. Vnitřní elektrony jsou zpravidla vazbou nedotčeny. Molekula jako celek podléhá Pauliho principu, pokud by tedy
byly atomy nuceny přemístit své elektrony do vyšších energetických stavů, k jejich vzájemné interakci nedojde. V
praxi to znamená, že reagují jenom elektrony valenčních vrstev atomu, tj. z nejvrchnější elektronové slupky.

Iontová vazba –
NaCl

Iontová vazba
Při vzájemném přiblížení může elektron přejít z jednoho atomu do druhého a vzniklé ionty se přitahují -> vazba
tvořena coulombickými přitažlivými silami a vzdálenost mezi atomy je větší než součet jejich poloměrů. Vazba je to
tzv. nenasycená, tzn. počet iontů, které se navzájem přitahují není omezený => vznik krystalů. V polárním
rozpouštědle se vazba rozruší a látka se rozpustí. Viz Iontová vazba.

Kovalentní vazba

Kovalentní vazba – H2

Jedna nebo více dvojic elektronů v této vazbě patří oběma atomům (sdílení elektronů, vazba na krátkou vzdálenost).
Vazba je velmi silná, pokud jsou ve dvojici elektronů spiny antiparalelní. Vazby se mohou účastnit jen elektrony z
vnější a ne zcela nasycené slupky. V případě této vazby není elektrický náboj v molekule rozložen symetricky a
molekula má tak charakter dipólu. K čistě kovalentní vazbě dochází např. v molekule H2.

Odkazy
Související články

Koordinačně kovalentní vazba

Zdroj
KUBATOVA, Senta. Biofot [online]. [cit. 2011-01-31]. <https://uloz.to/!CM6zAi6z/biofot-doc>.
Nukleární magnetická rezonance
Úvod
Zobrazování magnetickou rezonancí (MRI) využívá fyzikálního fenoménu zvaného nukleární magnetická rezonance
(NMR), který je v literatuře popisován od roku 1940 (Bloch 1940, Purcell 1946, oba 1952 Nobelova cena za fyziku). Z
počátku se objevovaly aplikace zejména v chemii s využitím MR spektroskopie (MRS). Zobrazování pomocí NMR se
objevuje po roce 1970 a z důvodu lepšího přijetí laickou veřejností bylo z názvu vypuštěno slovo nukleární (či jaderná)
a ujal se název MRI.

Obraz vzniká netriviálním zpracováním naměřených odpovědí na radiofrekvenční impulzy, protože je ale výsledek
závislý na neznámých parametrech, nelze uvažovat výsledný stupeň šedi v obraze jako absolutní číslo (podobně jako u
Hounsfieldovy jednotky v CT), ale pouze relativně ve vztahu k stupňům šedi z okolních tkání. Dobře zobrazuje měkké
tkáně. MRI nevyužívá ionizující záření, proto se považuje za bezpečnější a méně zatěžující než CT.

Přístroj magnetické rozonance

Magnetický dipólový moment

Jádro s lichým
počtem částic.
Oranžově naznačen
výsledný moment.

Atomové jádro se skládá z neutronů a protonů, které neustále rotují kolem své vlastní osy v pohybu nazývaném spin.
Protony jsou kladně nabité částice a každá pohybující se nabitá částice vytváří magnetické pole a vykazuje magnetický
moment. Magnetický moment, neboli také magnetický dipólový moment, je vektorová fyzikální veličina
charakterizující magnetický dipól. Magnetický dipólový moment se značí m a jeho jednotkou je ampér metr čtverečný
A.m2. Magnetický dipólový moment je určen vztahem m = IS, kde I je elektrický proud procházející smyčkou dipólu a
S je orientovaná plocha ohraničená smyčkou dipólu. Pro konkrétní základní představu magnetického momentu
uvažujme jen elektron. K jeho magnetickému momentu lze názorně dospět tímto způsobem: Budeme uvažovat
nejjednodušší atom – atom vodíku, který je v klidu.

V elektrostatickém poli kladně nabitého jádra se bude záporně nabitý elektron pohybovat po uzavřených trajektoriích
(orbitech). Elektron je elektricky nabitý, a proto vytvoří při uvažovaném pohybu po uzavřené trajektorii proudovou
smyčku, která je ekvivalentní magnetickému dipólu. Tak vznikne magnetické pole, které je sice velmi slabé, ale přesto
bude na elektron působit. Tímto způsobem lze dojít intuitivně k magnetickému momentu elektronu. U ostatních
objektů (protony, jádra, atomy) je pak základní myšlenka výkladu magnetického momentu podobná. Veličina se
jmenuje magnetický moment. Srovnáme-li tento pojem s mechanikou, nutně musíme dojít k závěru, že tato veličina
bude popisovat rotující objekt. Vzhledem k tomu, že existuje veličina spin, která také v základním přiblížení souvisí s
rotací, můžeme dospět k závěru, že i magnetický moment (elektronu, jádra, atomu) souvisí se spinem (elektronu,
jádra, atomu).

Atomová jádra se sudým nukleonovým číslem se nechovají ke svému okolí magneticky (nemají spin), protože se jejich
magnetické momenty ruší a nelze je používat pro MR zobrazení. Atomová jádra s lichým nukleonovým číslem si svůj
magnetický moment zachovávají. Charakteristickým zástupcem této skupiny je atom vodíku 1H, který má jeden
proton a vykazuje relativně velký magnetický moment. V organismu je více než 60 % vody a 1H je tedy nejvhodnějším
objektem pro MR zobrazování. Další zástupci jsou 13C, 19F, 23Na, 31P.

Atomové jádro v magnetickém poli


Vložíme-li jádro do silného magnetického pole, uspořádají se rotační osy protonů rovnoběžně se siločárami vnějšího
magnetického pole. Větší počet z nich je v poloze, kdy jejich magnetický moment je orientován souhlasně (paralelně) s
vektorem vnějšího magnetického pole a menší počet protonů je orientován opačně (o 180°, antiparalelně).
Antiparalelní uspořádání protonů je energeticky náročnější, a proto je jich méně než polovina.

Princip NMR
Princip NMR spočívá v tom, že je-li rotující jádro umístěno v konstantním magnetickém poli B0, dojde ke srovnání
magnetických momentů (os rotace) s vnějším magnetickým polem a osa jádra bude lehce rotovat kolem směru
působícího pole B0. Tento pohyb vzniká při každé změně působícího magnetického pole, dokud se jádro v dané poloze
neustálí. Pokud vnější pole přestane působit, vrací se jádro do své původní klidové polohy. Pokud se přidá druhé
kolmo působící (transverzální) pole BT, začne jádro opět rotovat. Aby byla jádra udržena ve stálém pohybu, používá se
vysokofrekvenční magnetické pole, které současně rotuje v rovině XY. Volbou velikosti prvního statického
magnetického pole B0 a volby velikosti pro transverzální magnetické pole BT se dá velice přesně určit, která jádra
budou v rezonanci. Rezonancí je magnetický moment m jádra překlopen o 90° do roviny XY a osa pak rotuje podle
transverzálního pole. Pokud je transverzální pole odpojeno, rotuje jádro stále v rovině XY. Přiblížením cívky do
blízkosti rotujícího magnetického momentu se v ní indukuje napětí, které je následně měřeno. Zjednodušeně je
velikost naměřeného napětí závislá na poloze a typu tkáně.

Larmorova frekvence

Precesní pohyb dipólu částice.


Sytě vyjádřen součet
jednotlivých vektorů.

Náhodně natočené dipólové


momenty s náhodnou fází precese.

Jak již bylo uvedeno, protony vykonávají rotační pohyb kolem své osy, neboli spin. Tím vytváří ve svém okolí
magnetické pole a vykazují magnetický moment. Mimo to protony umístěné v magnetickém poli vykazují ještě pohyb
precesní. Ten si lze představit jako pohyb po plášti pomyslného kužele (ještě názornějším příkladem nám může být
pohyb káči). Frekvence tohoto pohybu se nazývá Larmorovou frekvencí. Ta závisí na dvou faktorech:

1. na intenzitě vnějšího magnetického pole,


2. na typu atomového jádra, vyjádřeném gyromagnetickým poměrem (konstanta závislá pouze na vlastnostech
jádra).

Příklad

Pro vodík 1H je gyromatický poměr = 42,58MHz/T (269,2T-1), tzn., že v poli B0 = 1,5T budou mít vodíková jádra
frekvenci precesního pohybu f0 = cca 64MHz.
Rezonance

Magnetické momenty natočené


paralelně a antiparalelně v magnet.
poli (černé rovnoběžky). Fáze
precesních pohybů je
synchronizovaná Lamorovou
frekvencí.

Jednak se směr magnetického momentu každého jednotlivého precedujícího protonu v čase mění a jednak se protony
pohybují v různých fázích, neboli jsou nakloněny v daném čase různým směrem. Tím dochází ke vzájemnému vyrušení
jejich vlivu na úhrnný vektor magnetizace tkáně v rovině kolmé na směr magnetického pole. Vektor výsledné tkáňové
magnetizace má tedy směr totožný se směrem siločar vnějšího magnetu a v tomto stavu ho není možné měřit.
Můžeme říci, že je v zákrytu vnějšího magnetického pole.

Abychom mohli měřit rezonanční frekvenci protonů, tedy jejich spektrum, je nutné vychýlit úhrnný vektor
magnetizace z jeho rovnovážné polohy a tím docílit vzniku vektoru příčné tkáňové magnetizace. Jak bylo zmíněno
výše, velikost příčného vektoru je nulová z důvodu chaotického pohybu částic. Změny lze dosáhnout dodáním energie
v podobě elektromagnetického impulzu. Aby došlo k absorpci elektromagnetického vlnění protony, je potřeba, aby
Larmorova frekvence částic byla shodná s frekvencí vyslaného impulsu.

Pokud tomu tak je, dochází k jevu magnetické rezonance, to má několik důsledků:

1. protony začnou vykonávat svůj precesní pohyb ve fázi,


2. dojde ke zmenšení rozdílu paralelně a antiparalelně (energeticky náročněji) uspořádaných protonů a tím se
zmenší vektor podélné magnetizace.

Výsledkem je pak již zmíněný vektor příčné magnetizace.

„Situace bývá výstižně přirovnávána k výletní lodi s mnoha cestujícími na palubě: dokud jsou cestující rozmístěni a
náhodně a celkem rovnoměrně po palubě, pluje loď rovně. Jakmile se však cestující shluknou dohromady začnou
obcházet podél zábradlí palubu, bude se loď periodicky naklánět postupně na všechny strany.“

Relaxační časy T1 a T2

Postupný návrat do původního stavu,


žlutě T1 (orientace magnet.
momentu), fialově T2 (změna
precesní fáze).

Po skončení elektromagnetického impulsu již není protonům dodávána energie a proto se vrací do původního,
energeticky výhodnějšího, paralelního postavení a mizí jejich synchronní pohyb. Tento děj se nazývá relaxace.

Dochází při něm k postupnému zvětšování podélného vektoru magnetizace a čas, za jaký dojde k obnovení tohoto
vektoru na 63 % se označuje T1 (podélná relaxace, neboli spin-mřížka relaxace). Zároveň však dochází ke ztrátě
vektoru příčné magnetizace vymizením synchronního pohybu protonů, což je důsledek vzájemného působení
magnetických polí jednotlivých částic. Čas, za který klesne vektor příčné magnetizace na 37 % své hodnoty se
označuje T2 (příčná relaxace, neboli spin-spin relaxace).

V absolutním měřítku jsou relaxační časy T1 2−10x delší než relaxační časy T2. V biologické tkáni se hodnoty T1
pohybují v rozmezí 300–2000 ms, T2 30–150 ms. V praxi je pokles příčné složky tkáňové magnetizace ovlivněn ještě
drobnými změnami v nehomogenitě vnějšího magnetického pole. Pokles je tak podstatně strmější.

Typy elekromagnetických impulsů


Typy elektromagnetických pulsů, které se pro MR používají

90° puls – otáčející vektor tkáňové magnetizace o 90° a dochází ke vzniku vektoru příčného. Čas mezi
jednotlivými pulsy se označuje TR (time to repeat). Čas TR mezi jednotlivými pulsy se zkracuje tak, aby tkáně po
jeho skončení nestačily opět získat plnou hodnotu vektoru podélné magnetizace. Přijímaný signál z tkáně se pak
liší tím, jak velký je vektor jejich tkáňové magnetizace v čase vyslání nového impulsu.
kombinace 90°a 180°pulsu – 90°způsobí nárůst vektoru příčné magnetizace a po jeho skončení začne vektor
opět klesat. Ovšem v čase označovaném TE/2 je vyslán 180° impuls, který o 180° změní orientaci precesního
pohybu protonů a původně rychleji precedující protony jsou za protony precedujícími pomaleji a zmenšující se
příčný vektor se začne zvětšovat. Za další dobu TE/2 dojde k synchronizaci pohybu protonů a výsledkem je
opětovná maximalizace signálu. Po sečtení časů TE/2 získáme čas TE – čas ozvěny.

Magnetická pole
Vnější magnetické pole MR magnetu je homogenní. V medicíně používané magnety vytvářejí pole obvykle v rozsahu
0,1–3 T. Pro výzkumné účely se však používají i pole o velikosti až 9,4 T, pro zvířata až 21 T. Jediné magnetické pole
nám však neumožní získat prostorovou představu o jednotlivých protonech a s ní vytvořit scan pacienta. Aby však bylo
možné odlišit signály vedené z různých vrstev těla, je potřeba, aby protony v různých místech reagovaly při zasažení
elektromagnetickým vlnami o vhodné frekvenci.

Prostorové rozlišení a rekonstrukce obrazu MRI


K homogennímu poli hlavního magnetu jsou proto přidána pole další. Pole, jehož intenzita roste s osou těla, vytváří
tzv. magnetický gradient. Magnetický gradient s podélnou osou nám umožňuje zvolit rovinu řezu, a proto je nazýván
„slice selecting gradient“ (rovinu řezu určující gradient). V praxi pak například u nohou působí pole o síle 0,45 T
(odpovídající f= 19,160 MHz), kdežto u hlavy 0,55 T (f= 23,417 MHz). Vysláním vhodné frekvence vybíráme tedy jen
řez, který chceme (Obr. 1).

Pro řez například oblouku aorty bude mít impuls frekvenci 22,566 MHz. Regulovat tloušťku řezu tedy můžeme dvěma
způsoby:

různým rozsahem frekvence impulsu, tedy čím větší rozsah pulsu, tím širší řez a vice versa,
sklonem gradientu, jinak řečeno rozsahem pole, ve kterém se tělo nachází. Zde platí, že čím strmější je
gradient (větší je rozdíl mezi silou pole u nohou a hlavy), tím užší řez získáme (Obr. 1).

Jelikož jedna souřadnice k prostorovému určení nestačí, je přidáno další pole. Tentokrát je ale pole na dlouhou osu
těla kolmé, a síla se tedy mění v pravolevém směru. Díky tomu budou protony umístěné v různých „sloupcích“ těla
emitovat různou frekvenci. Tento gradient je nazýván „frequency encoding gradient“ (frekvenci určující gradient) či
„readout gradient“ (odečítací gradient) (Obr.3). Konečné určení bodu v prostoru poskytne třetí gradient, který však
funguje poněkud odlišně. Nachází se opět v směru sloupců jako readout gradient, je však zapnut pouze na velice
krátký okamžik před aplikací samotného readout gradientu. To ovlivní frekvenci precese jednotlivých protonů ve
sloupci, avšak s ohledem na vzdálenost; tedy ty, které byly ovlivněny polem s vyšší intenzitou, budou mít vyšší
frekvenci než zbylé. Jakmile tento gradient pomine, bude Larmorova frekvence protonů ve sloupci opět stejná, jenže
už nebudou kmitat ve společné fázi, ale v různé podle toho, jak moc byly gradientem ovlivněny. Tento gradient je
proto nazýván „phase encoding gradient“ (fázi určující gradient) (Obr. 2). Postup jednotlivých fází v jedné spin-echo
frekvenci je znázorněn na obrázku (Obr. 4). Poté, co jsou získány echa frekvencí jednotlivých bodů v řezu, může být z
k-prostoru pomocí Fourierovy 2D transformace vytvořen konečný obraz (Obr. 5).

Bohužel ani znalost toho, jak zařídit reakci přesně těch žádaných protonů, ještě neumožní získat scan, který by notně
odpovídal představám vyšetřujícího lékaře. Všechny pulsy, které jsou aplikovány, nejen že musí být aplikovány, ale
musí být aplikovány ve správnou chvíli. Na základě znalosti funkce T1, T2, TR a TE časů je možné získat velké
množství různých obrazů. Vyšetřovacích metod, které se skládají z předem určeného pořadí funkcí, je nespočet, jejich
základem však vždy bývají tři obrazy:

PD obraz nese jméno podle zkratky Proton Density (protonová hustota). Na obrázku (Obr. 6) vidíme řez lebkou s
jeho pomocí,
druhý obraz je nazýván T1-vážený (Obr. 7) a je určen T1 časem jednotlivých tkání,
poslední T2-vážený, jak už název napovídá, je určen T-2 časem tkání (Obr. 8).

Příklady

Pro pole o síle 3T platí například hodnoty T1, T2:

Šedá hmota mozková: 1200 ms, 80 ms.


Bílá hmota mozková: 800 ms, 70ms.
Mozkomíšní mok: 4000 ms, 600 ms.
Arteriální krev: 1700 ms, 120 ms.
Žilní krev: 1500 ms, 40 ms.

Situace, které mohou vzniknout, vypadají například takto:*

Situace 1 odpovídá obrázku (Obr. 9)


Situace 2 odpovídá obrázku (Obr. 10)

Situace 3 odpovídá obrázku (Obr. 11)

Situace 4 odpovídá obrázku (Obr. 12)

Vysvětlivky k obrázkům

WM = bílá hmota mozková GM = šedá hmota mozková CSF= mozkomíšní mok Osa x: čas Osa y: síla signálu (T1
signál horní graf, T2 signál dolní graf)

Situace 1 odpovídá obrázku (Obr. 9)

T1: TR doba je už relativně krátká. Ačkoliv WM už kompletně zrelaxovalo a nevytváří žádný obraz, naproti tomu GM a
CSF stále ještě nejsou zcela relaxované a obraz by vydávaly. Na čistě T1 obrazu by tak bylo možné pozorovat rozdíl
mezi GM, CSF i prázdná místa (WM) T2: TE doba je dostatečně vysoká, aby zrelaxovala většina tkání v mozku na více
než 37%. Díky tomu je rozdíl v síle signálu již dost vysoký, aby na čistě T2 obrazu mohly být pozorovány rozdíly v
tkáních. Především výrazná je CSF. Celkový obraz: Ačkoliv oba obrazy samostatně vydávají solidní rozdíly mezi
tkáněmi, jejich zrcadlovost způsobí, že po jejich složení, je výsledný relativní signál jednotlivých tkání téměř totožný, a
na scanu tedy nejsou žádné struktury znát. Při kombinaci krátké TE a dlouhé TR tedy obraz nevznikne.

Situace 2 odpovídá obrázku (Obr. 10)

T1: TR doba je velmi dlouhá. Prakticky všechny tkáně za tuto dobu dokážou zrelaxovat. Rozdíl mezi T1 signálem WM
a GM je v podstatě neměřitelný, CSF se drobně odlišuje a na samotném T1 obrázku by ho bylo možné slabě rozeznat.
T2: TE čas je velmi krátký. Za tuto chvilku se ještě nestačily moc projevit rozdíly v tkáních. WM a GW by tedy
vydávaly podobně silný signál, CSF téměř maximální. Celkový obraz: Kombinace velmi shodných signálů WM a GM v
podstatě znemožňuje jejich výrazné rozeznání. Vyšší T2 signál je na úroveň WM a GM dorovnán nižším T1 signálem, a
proto i celkový CSF signál je jen velmi nevýrazně odlišný. Takovýto scan s velmi krátkým TE a velmi dlouhým TR dává
za vznik PD scanu.

Situace 3 odpovídá obrázku (Obr. 11)

T1: Relativně krátká TR doba neumožnila celkovou relaxaci tkání, a tak stále můžeme pozorovat odlišnosti v T1
signálu WM, GM i CSF. T2: Minimální TE téměř neumožňuje pozorovat odlišnosti tkání na základě T2. Celkový
obraz: Výsledný scan v podstatě odpovídá čistě výsledkům T1 signálu, jelikož vliv T2 je minimální. Takovýto scan se
proto nazývá T1- vážený obraz.

Situace 4 odpovídá obrázku (Obr. 12)

T1: Čas TR je velmi dlouhý. Za tuto dobu už téměř všechny tkáně zcela zrelaxovaly a jejich T1 signál je téměř
neodlišitelný. T2: Dlouhá TE doba nám už dává možnost pozorovat odlišnosti v postupně relaxujících tkáních na
základě T2 signálu. Obzvláště výrazně se projevuje u pomalu relaxujících tkání jako CSF. Celkový obraz: Protože zde
naopak téměř nezasahuje T1 signál je celý obraz pod vlivem T2, tedy nazývá se T2-vážený.

Zobrazování MRI
Jevu MRI lze dosáhnout dvojím způsobem:

Můžeme pracovat s vnějším magnetickým polem o konstantní magnetické indukci B a hledat energii
(|frekvenci) elektromagnetického vlnění schopnou vyvolat jadernou rezonanci
Můžeme pracovat s konstantní energií (frekvencí) elektromagnetického vlnění a hledáme takovou hodnotu
magnetické indukce B, při které dosáhneme rezonance – v tomto případě měníme hodnotu Larmonovy
precese

Pokud se vyšetřovaná část těla nachází v homogenním magnetickém poli, pak radiofrekvenční impuls o patřičné
energii vyvolá vznik NMR-signálu v celém objemu vyšetřované části těla a informace o lokálních hodnotách rezonance
je ztracena. V nehomogenním magnetickém poli však můžeme vytvořit určitý gradient magnetické indukce B v
určitém směru (například ve směru osy těla vyšetřovaného), čímž získáme stav, ve kterém je rezonanční podmínka
splněna jen v úzkém řezu tělem vyšetřovaného kolmém na směr gradientu.

Nastavování těchto gradientů se děje v různých impulsových režimech a je rozdílné u různých variant MR i u různých
typů přístrojů a speciálních aplikací. Tímto způsobem můžeme získat prostorově specifické informace o velikosti
rezonančního signálu, který je úměrný počtu rezonančních jader v daném místě prostoru, tudíž jsme při použití
Fouriérovy transformace schopni získat i celkovou prostorovou informaci o rozložení rezonujících jader. Rozlišovací
schopnosti MRI jsou závislé na jejich konstrukci, při použití supravodivých magnetů v prostředí kapalného helia jsme
na základě chemických posunů schopni identifikovat i struktury látek přítomných v dané oblasti, provoz těchto
zařízení je však velice nákladný.

Velkou výhodou MRI například oproti CT je, že jsme schopni získat řezy v různých rovinách – nejsme omezeni jen
na řez příčný, ale můžeme získat i řez sagitální nebo frontální díky nastavení příslušného gradientu magnetické
indukce B. Další výhodou je i fakt, že MRI nepoužívá ionizující záření, čímž je garantována i jistá neinvazivita pro
buněčnou tkáň. Pro MRI rovněž mluví i fakt, že dosud nebyly pozorovány žádné účinky vystavení silným
magnetům či radiofrekvečním impulsům na biologickou tkáň, proto je tato metoda vhodná i pro použití u dětí a
v případě nutnosti i u těhotných po prvním trimestru.

Výhody
Přesnost zobrazení − na základě rozlišných intenzit signálu měkkých tkání (mozek, srdce, chrupavčitá tkáň...)

Umožňuje rozlišit patologické stavy, které jsou jinak nezobrazitelné

Neinvazivní metoda − nevyužíva škodlivého ionizujícího záření(vhodné i pro těhotné a novorozence)

Informace o krevním oběhu, krevních cevách

Některé angiografické snímky mohou být zobrazeny bez potřeby dodání kontrastní látky

Funkční MRI umožňuje zobrazení obou aktivních části mozku při jednotlivých aktivitách

Kontrastní látky
Používají se ke zlepšení obrazu – zviditelňují struktury, které nejsou v nativním obraze rozlišitelné. Principem jejich
funkce je usnadnění relaxace protonů, čímž je zkrácena relaxační doba T1 a T2. Zkrácení relaxační doby T1 vede k
zesílení T1 váženého obrazu, naopak u T2 vede k jeho zeslabení. Kontrastní látky lze rozdělit na látky paramagnetické
a superparamagnetické.

Paramagnetické látky zesilují magnetické pole, a to způsobuje zkrácení relaxačního času okolí. Mnohdy obsahují
gadolinium, které je pro člověka vysoce toxické, proto je vázáno ve formě chelátových komplexů. Mají široké využití,
často se používají při vyšetření CNS, protože mohou pronikat poškozenou hematoencefalickou bariérou. Patří sem
např. Magnevist (kyselina gadopentetová), Omniscan (gadodiamid), Dotarem (Gd-DOTA, kyselina gadoterová) a Mn-
DPDP (mangafodipir, používá se k vyšetření jater). Superparamagnetické látky jsou pevné látky, které se do těla
zavádějí formou suspenzí, mají mimořádnou účinnost. Příkladem jsou SPIO (superparamagnetic iron oxide) a USPIO
(ultrasmall superparamagnetic iron oxide).

Zeemanův jev

Zeemanův jev

Jedná se o rozštěpení spektrálních čar atomu, na který působí vnější magnetické pole. Své pojmenování nese po
holandském fyzikovi Pieteru Zeemanovi. Ten v roce 1897 jako první pozoroval štěpení spektrální čáry na triplet v
slabém magnetickém poli, což je označováno jako normální Zeemanův jev. Většinou však dochází ke štěpení na více
čar, tzv. anomální Zeemanův jev.

Energetické hladiny atomů ve vnějším magnetickém poli jsou kromě závislosti na hlavním kvantovém čísle závislé i
na orbitálním a magnetickém kvantovém čísle, a také na magnetické indukci pole B.

Uvažujme atom vodíku bez hyperjemných struktur. Kvantové číslo mj nabývá 2j+1 možných hodnot. Ve vnějším
magnetickém poli budou těmto stavům v důsledku různé orientace magnetických dipólů v magnetickém poli náležet
různé energie. Z toho vyplývá, že každá energetická hladina se rozpadá na 2j+1 stejně od sebe vzdálených hladin,
které jsou symetricky rozloženy okolo původní hladiny. Rozštěpení čar je větší pro silnější pole.

Speciální aplikace MRI


Difuzní MRI

Difuzní MRI zobrazuje změny signálu způsobené difuzí molekul vody ve tkáních. Takové zobrazení je relativně
nezávislé na relaxačních časech T1, T2 i na hustotě protonových jader (PD). Difuzní zobrazení se uplatňuje především
při hodnocení patologických stavů mozku (stáří ischemického postižení mozku, traumatické změny mozku, posuzování
buněčného složení mozkových nádorů nebo změny v důsledku Alzheimerovy choroby, autismu, schizofrenie apod.).
Směr difuze může být zcela náhodný všemi směry (např. v mozkomíšním moku nebo šedé hmotě mozku) nebo
omezený pouze na některé směry (např. v bílé hmotě mozku převládá difuze ve směru dlouhých vláken axonů), kdy je
všesměrové difuzi bráněno bariérou např. buněčných stěn. Směr difuze lze zjistit mnohonásobným skenováním
zvolené vrstvy tkáně v několika směrech. Jednotlivé směry zobrazení se získávají změnou orientace magnetických
gradientů. Každý směr gradientu potom zobrazuje jiný směr difuze. V praxi se směr difuze vypočítává nejméně ze 6
směrů, obvykle však z 12 až 256 směrů. Výsledkem difuzního zobrazení může být šedoškálová mapa velikosti difuze
(metoda DWI – Diffusion Weighted Imaging) nebo barevná mapa mozku (metoda DTI – Diffusion Tensor
Imaging), kdy jednotlivé barvy určují jednotlivé směry difuze v dané oblasti.

DWI metoda

Metoda DWI – Diffusion Weighted Imaging

Při DWI intensita každého elementu obrazu (voxelu) odráží rychlosti difuze vody v dané oblasti. jelikož pohyby vody
jsou vysoce dependentní na tepelných podmínkách a vlastním buněčném prostředí, měli bychom být díky DWI schopni
pozorovat změny v dané tkáni a tudíž i pozorovat rané změny indikující patologické stavy. Pro ilustraci, DWI je více
senzitivní k raným patologickým změnám provázejícím mrtvici než tradiční MRI. DWI se nejvíce hodí pro popis tkání,
v nichž dominuje isotropický pohyb vody, tedy šedé hmoty mozkové v kůře mozkové, velkých mozkových jader anebo v
těle, kde se zdá rychlost difuze vody přibližně stejná ve všech směrech.

Metoda DTI – Diffusion Tensor Imaging

DTI metoda

Difuzní tenzorové zobrazování (DTI) je technika magnetické rezonance, která umožňuje měřit omezenou difuzi vody
ve tkáních. Rovněž poskytuje užitečné informace o struktuře různých tkání, například tkáně svalové (i srdeční) nebo
prostaty.

Při DTI, každému voxelu přísluší jeden nebo více parametrů: rychlost difuze a preferovaný směr difuze. Vlastnosti
každého voxelu jsou obvykle vypočítány díky příslušným vektorům, respektive tenzorům, z minimálně 6 směrů. Při
některých metodách jsou stovky snímků složeny dohromady a je vygenerován výsledný obraz. Díky většímu objemu
informací získaných při DTI se jedná o velice citlivou metodu umožňující nám pozorovat i velice jemné patologie.
Navíc můžeme využít i získané směrové informace a sledovat neurální vlákna bílé hmoty mozkové a látky jimi
transportované mozkem, tento proces se nazývá traktografie.

MR angiografie (MRA)

Zobrazovaným parametrem v MRI může být mimo hustoty jader a relaxačních časů T1 a T2 také průtok excitovaných
jader. Pohybu jader pak využívá MR angiografie (MRA), metoda zobrazující průtok krve cévami nebo tok
mozkomíšního moku. K zobrazení průtoku jader MR angiografií lze

MR angiografie karotid

využít několika metod:

MR angiografie karotid MRA s využitím kontrastních látek


CE MRA (Contrast Enhanced)

MRA bez kontrastních látek


TOF MRA (Time of Flight)
PC MRA (Phase Contrast)
MRA s využitím kontrastních látek

Technika CE MRA využívá k zobrazení průtoku jader kontrastní látky. Metoda dokáže odlišit arteriální fázi (plnění
tepen krví) od fáze venózní (plnění žil krví). Nejprve je pořízen srovnávací obraz bez kontrastu, poté snímek v
okamžiku, kdy je kontrastní látka přítomna v tepnách a nakonec je

pořízen obraz po přestupu kontrastní látky do žilního systému. MRA obraz je vypočten jako rozdíl mezi snímkem bez
kontrastu a snímky s kontrastní látkou v tepnách/žilách. Používají se kontrastní látky na bázi sloučenin kovů.

MRA bez kontrastních látek

Metody MRA bez použití kontrastní látky jsou založeny na změnách fáze precesního pohybu částic a velikosti
vektoru magnetizace v přítomnosti gradientního magnetického pole. Změna fáze ΔΦ precese je úměrná rychlosti
pohybu částic v, druhé mocnině doby trvání tG gradientního pole a velikosti gradientního pole G:

ΔΦ=vt2G

Fázové změny se pak v MRA obraze projevují buď ztrátou signálu (pokles jasu) nebo ziskem signálu (zvýšení
jasu). Projevují se tedy podobně jako kontrastní látky. Ztráty signálu využívá metoda PC MRA (Phase Contrast), zisku
signálu využívá metoda TOF MRA (Time Of Flight).

Phase Contrast MRA

Pokud zobrazovanou oblast vybudíme RF pulzem, potom excitované částice stacionárních tkání poskytují při detekci
silný signál. Vybuzené částice krevního toku ovšem opouštějí snímanou scénu dříve, než je signál zaznamenán, a na
snímku se nezobrazí. Ztráta signálu je typická pouze pro vysoké rychlosti toku, při nízkých rychlostech ke ztrátě
signálu nedochází. K vymizení signálu ovšem může docházet také při rozfázování precesního pohybu částic, např. při
turbulentním proudění (opačné fáze se navzájem vyruší), které se objevuje při vysokých rychlostech toku. Výsledný
MRA obraz je vypočten z rozdílů mezi obrazy nasnímanými před a po aplikaci magnetického gradientu.

Time Of Flight MRA

Pokud zobrazovanou oblast vybudíme několika rychle po sobě jdoucími RF pulzy, potom se částice stacionární tkáně
excitují prvním RF pulzem, do příchodu následujícího RF pulzu nestačí plně relaxovat, aby mohla být novým pulzem
vybuzena, a poskytují proto pouze slabý signál. Částice krevního toku, které vtékají nově do snímané scény naopak
nebyly prvním RF pulzem excitovány, ale následujícím RF pulzem jsou vybuzeny a poskytují silný signál. Zisk signálu,
tzv. efektem vtoku, se obvykle projevuje pouze v první vrstvě, do které vstupuje krevní tok. Pomalé toky zpravidla
nelze zobrazit. Nevýhodou je také pokles signálu v následujících tomovrstvách, do kterých vtéká krev (např. při
snímání 3D obrazu), protože částice krevního toku nestačí relaxovat.

Funkční MR
Funkční magnetická rezonance (fMR) se spolu s progresivním vývojem statistických metod a výpočetní techniky
rozvíjí jako prostředek pro vizualizaci anatomických struktur mozku zapojených do mechanismů vnímání, řízení
motoriky a myšlení. Slouží tedy k funkčnímu zobrazení mozkové aktivity. Podstatou metody je změna prokrvení a
objemu krve v aktivní oblasti mozkové kůry (perfuzní fMRI) a na rozdíl od standardní magnetické rezonance má
schopnost detekovat dynamické změny signálu způsobené lokálním kolísáním poměru oxyhemoglobinu a
deoxyhemoglobinu v závislosti

fMRI snímek během zátěže


paměti

na neuronální aktivitě (BOLD – Blood Oxygenation Level Dependent). Neokysličená forma hemoglobinu má
paramagnetické vlastnosti a chová se jako přirozená MR kontrastní látka a tedy v místech s jeho vyšší koncentrací
dochází ke vzniku magnetických nehomogenit, kvůli kterým je rychlejší ztráta energie excitovaných protonů a tím i
větší lokální úbytek radiofrekvenčního signálu. Aktivní oblast mozku, která spotřebovává více kyslíku, potom
poskytuje silnější signál než okolí.

Při fMRI vyšetření se opakovaným skenováním získávají obrazy celého objemu mozku v klidu i při aktivním řešení
úkolů (reakce na podnět, pohyb končetin, tvorba slov, atd.). Změny mozkové aktivity jsou měřeny z rozdílu dvojic
obrazů pořízených v klidu a při mozkové činnosti.

K detekci se používá metoda Echo Planar Imagining (EPI). Ta umožňuje pomocí rychlých změn magnetického pole
zaznamenávat signál z celého řezu (objemu) po jednom nebo několika radiofrekvenčních pulzech. Získaný signál se
poté rozdělí na konečný počet vzorků, které v sobě mají informaci o signálu z každého místa řezu.
Funkční magnetická rezonance umožňuje specifikovat diagnostiky některých neurologických i psychiatrických
onemocnění a poskytuje možnosti plánování chirurgických výkonů.

Přístrojové vybavení

Pacient se položí do gantry (tunelu) MR přístroje a tam je vystaven silnému homogennímu magnetickému poli MR
magnetu (v rozmezí 0,5–3T). Aby bylo možné odlišit signály z různých vrstev tkání, je přidáno další magnetické pole
tvořené tzv. gradientovými cívkami. Pro vyšetření jednotlivých částí těla se používají povrchové cívky („orgánové
cívky“). Tyto cívky slouží pro přijímání signálu v bezprostřední blízkosti vyšetřované oblasti, výsledný signál je tedy
silnější (hlavová, krční páteřní, ramenní cívky atd.) a kvalita zobrazení je vyšší.

fMRI je vyšetření, při kterém je nutná spolupráce pacientů. Správné provedení zadaného úkolu je totiž zásadním
bodem v celém vyšetřovacím řetězci. U některých kognitivních úloh ( např. test slovní plynulosti) je nemožné při
snímání fMRI dat výkon pacienta přímo objektivizovat a ověřit tak, zda vykonával úlohu podle zadání.

Vyšetření je ovládáno počítačem a opatřená obrazová data jsou následně zpracována programy pro analýzu obrazu.
Předzpracování, statistická analýza a vyhodnocení výsledků již většinou probíhá mimo vlastní MR pracoviště. Je nutné,
aby bylo pracoviště stíněno a nedocházelo tak k rušení vysílacími frekvencemi radiových vln. Stěny vyšetřovací
místnosti jsou izolovány Faradayovou klecí.

MR spektroskopie

Nukleární magnetická rezonanční spektroskopie, častěji známá jako MR spektroskopie, je výzkumná technika, která
využívá magnetických vlastností některých atomových jader k určení fyzikálních a chemických vlastností atomů nebo
molekul, v nichž jsou tato jádra obsažena. MR spektroskopie je založena na principu nukleární magnetické rezonance
a poskytuje podrobné informace o struktuře, dynamice, reakčním stavu a chemickém prostředí molekul.
Intramolekulární magnetické pole kolem atomu v molekule mění rezonanční frekvenci a tím zajišťuje přístup k
podrobným informacím o elektronové struktuře molekuly.

MR spektroskopie umožňuje neinvazivní sledování biochemických pochodů v různých orgánech jak za normálních, tak
patologických podmínek. MR spektroskopie je jednou z mála technik, které umožňují stanovení zastoupení metabolitů
ve tkáních in vivo, a proto bývá někdy označována jako in vivo MR biopsie.

Typy MR spektroskopie

Protonová 1H MR

U protonové 1H MR spektroskopie je možné získat informace o zastoupení a koncentraci některých sloučenin


obsahující vodík. Cívka je zaostřena na frekvenční pásmo, ve kterém rezonuje vodík v různých sloučeninách (63MHz)
− N-acetyl aspartát NAA, kreatin, cholin, myoinositol, laktát, glutamát, aspartát, GABA. Tato metoda se uplatňuje
především ve sledování metabolismu mozkové tkáně.

Fosforová 31P spektroskopie

U fosforové 31P spektroskopie získáváme informace o relativním zastoupení metabolitů obsahující fosfor, který je
přítomen především v makroergních fosfátech (fosfomonoestery PME, fosfodiestery PDE, ATP, ADP, fosfokreatin,
anorganický fosfát (Pi)). Fosforová spektroskopie (31P MRS) je vhodná pro popis energetického metabolismu tkáně či
orgánu. Nejčastěji je využívána k vyšetřování pacientů s metabolickým onemocněnm svalů. V posledních letech byly
metodou fosforové spektroskopie sledovány změny energetického metabolismu jater.

Využití

Nejvíce je MR spektroskopie využívána v chemii a biochemii ke zkoumání vlastností organických molekul. Metoda
umožňuje neinvazivní, in vivo měření koncentrace řady chemických sloučenin v CNS. Slouží také k hodnocení různých
biologických procesů − membránový a energetický metabolismus, koncentrace excitačních a inhibičních
neurotransmiterů. Údaje získané pomocí MR spektroskopie mohou poskytnout nový pohled na příčiny, diagnostiku i
terapii řady neuropsychiatrických chorob.

Odkazy
Související články

Principy diagnostických zobrazovacích metod


Zobrazovací metody v neurochirurgii

Použitá literatura
NAVRÁTIL, Leoš a Jozef ROSINA, et al. Medicínská biofyzika. 1. vydání. Praha : Grada, 2005. 524 s. ISBN 80-
247-1152-4.

ŽIŽKA, Jan a Vlastimil VÁLEK, et al. Moderní diagnostické metody. III. díl, Magnetická rezonance. 1. vydání. 
Brno : Institut pro další vzdělávání pracovníků ve zdravotnictví, 1996. ISBN 80-7013-225-6.
CHLEBUS, P, M MIKL a M BRÁZDIL. Funkční magnetická rezonance – úvod do problematiky. Neurologie pro
praxi [online]. 2005, roč. 6, vol. 3, s. 133-138, dostupné také z
<http://fmri.mchmi.com/articles/chlebus_prehled.pdf>. ISSN 1213-1814. 

SEDLÁŘ, Martin, Erik STAFFA a Vojtěch MORNSTEIN, et al. Zobrazovací metody využívající neionizující záření
[online]. Brno : Biofyzikální ústav Lékařské fakulty Masarykovy univerzity v Brně, 2013, dostupné také z
<http://www.med.muni.cz/biofyz/zobrazovacimetody/files/zobrazovaci_metody.pdf>. 

HRAZDIRA, Ivo a Vojtěch MORNSTEIN. Lékařská biofyzika a přístrojová technika. 1. vydání. Brno : Neptun, 


2001. 396 s. ISBN 80-902896-1-4.

ROSINA, Jozef, Jana VRÁNOVÁ a Hana KOLÁŘOVÁ, et al. Biofyzika : Pro zdravotnické a biomedicínské obory. 
1. vydání. Praha : Grada, 2013. 0 s. ISBN 978-80-247-4237-3.

Internetové zdroje
Autor neznámý, Princip MRI, http://fmri.mchmi.com/main_index.php?strana=13
Autor neznámý, NMR, https://physics.mff.cuni.cz/vyuka/zfp/txt_410.pdf
Autor neznámý, MRS, [1]
Evžen Amler, Elektřina a magnetismus, https://moodle.mefanet.cz/login/index.php
Jakub Nezval, NMR, http://artemis.osu.cz/biofyzika/Studium/Studmat/material/NMR.pptx
Jitka Mazánková, Kontrastní látky a jejich nežádoucí účinky,
https://is.muni.cz/th/326352/lf_b/Kontrastni_latky_a_jejich_nezadouci_ucinky.pdf
Karla Miller, Manus Donahue, Rob Tijssen, Dan Bulte, Jamie Near, MRI Physics Course, FMRIB Centre –
Department of Clinical Neurology, University of Oxford, https://www.win.ox.ac.uk//education/graduate-training-
course/program/lectures/mri-physics/mri-physics-course
Miloš Vojtíšek, RF cívka pro MR systém, https://www.vutbr.cz/www_base/zav_prace_soubor_verejne.php?
file_id=40375
Viktor Babjak, Zeemanov jav,
https://www.viktorbabjak.cz/domains/viktorbabjak.cz/public/documents/practical/prak4-17-zeemanov-jav.pdf

Obrázky

Obrázky 1–5, zdroj: VÁLEK, Vlastimil a Jan ŽIŽKA. Moderní diagnostické medoty. III. díl, Magnetická
rezonance. 1. vydání. Brno : Institut pro další vzdělávání pracovníků ve zdravotnictví, 1996. ISBN 8070132256.

Obrázky 6–12, zdroj: Manus Donahue, Contrast Manipulation, FMRIB Centre – Department of Clinical Neurology,
University of Oxford
Princip hmotové spektroskopie
Hmotová spektroskopie je metoda určujeme hmotnost atomů, iontů či izotopové složení určitého prvku.

Přístroj, který u této metody použijeme je hmotovým spektrograf, který funguje na základě faktu, že trajektorie
nabité částice je závislá na její hmotnosti.

Nejdříve musíme sledované izotopy přeměnit na ionty s nábojem q.

(q= náboj iontu, U=napětí… součin je celková energie urychlených iontů)

Po tomto kroku urychlené ionty vstupují do mag. pole a působí na ně mag. síla o velikosti

Dráha iontů se poté zakřivuje a v mag. poli opisuje kružnici o poloměru r, který můžeme určit z rovnosti mag. a
odstředivé síly:

Za rychlost dosadíme z prvního vzorce:

kde A nezávisí na hmotnosti, ale jen na náboji.Z tohoto důvodu se ionty o stejném náboji a různé hmotnosti budou
pohybovat po různých kružnicích. Z toho vyplývá, že později pomocí detektorů od sebe rozlišíme jednotlivé izotopy.

Postup hmotové spektroskopie


1. Vzorek je umístěn v přístroji a podstoupí odpařování.
2. Vzorek ionizujeme a dochází k vytvoření iontů.
3. Ionty jsou odděleny podle m/Q poměru v analyzátoru elmag. pole.
4. Dochází ke kvantitativní detekci iontů přístrojem.
5. Po detekci přístroj ionty zpracuje.

Odkazy
Zdroj
KUBATOVA, Senta. Biofot [online]. [cit. 2011-01-31]. <https://uloz.to/!CM6zAi6z/biofot-doc>.
Síly působící mezi molekulami

Všechny síly působící mezi atomy, molekulami a ionty jsou coulombické povahy (+ a − se přitahují).

Vodíkové můstky

Van der Waalsovy síly


Jsou přitažlivé síly mezi neutrálními molekulami. Jejich podstatou je vzájemné působení molekulových dipólů (které
existují kvůli okamžitému nerovnoměrnému rozložení elektronů v molekule).

Orientační síly působí mezi trvalými dipóly (dipól-dipól).


Indukční síly – polární molekula zpolarizuje nepolární (dipól-indukovaný dip.).
Disperzní (Londonovy) síly – molekuly kmitají a občas se náhodně vytvoří dipól, který interaguje s ostatními
molekulami.

Vodíkové můstky
Jsou speciálním typem dipól-dipól interakce mezi polárními vazbami obsahujícími vodík kovalentně vázaný na
elektronegativní prvek (např: kyslík, dusík, fluor). Vodík poté interaguje s volným elektronovým párem v blízkosti
elektronegativního prvku v jiné molekule (intermolekulární interakce) nebo v původní molekule (intramolekulární
interakce). Typickou sloučeninou tvořící vodíkové můstky je voda. Na základě vodíkových můstků se vysvětlují některé
vlastnosti vody, (popř. i jiných sloučenin) jako je její relativně vysoký bod varu oproti ostatním hydridům 6. hl. skupiny
periodické tabulky.

Odkazy
Související články

Kovalentní vazba
Koordinačně kovalentní vazba

Použitá literatura

KUBATOVA, Senta. Biofot [online]. [cit. 2011-01-31]. <https://uloz.to/1162346/biofot.doc>.

Zdroj

KUBATOVA, Senta. Biofot [online]. [cit. 2011-01-31]. <https://uloz.to/!CM6zAi6z/biofot-doc>.


Stavová rovnice plynů
Stavová rovnice plynů spojuje základní termodynamické veličiny popisující chování plynu. Stavová rovnice, nebo
lépe stavové rovnice, spojují základní stavové veličiny termodynamického systému. V případě plynů jsou takovými
stavovými veličinami obvykle tlak, objem, teplota a látkové množství, případně veličiny z nich odvozené.

Stavová rovnice ideálního plynu


Ideální plyn je nejjednodušší model plynu. Model předpokládá, že molekuly plynu jsou v zásadě bodové částice bez
vlastního objemu a jejich jediné silové působení nastává při pružných srážkách.U takového plynu se např. neprojevuje
vnitřní tření, takže ideální plyn má nulovou viskozitu.

Pro ideální plyn platí jednoduchá stavová rovnice svazující tlak (p), objem (V), látkové množství (n) a teplotu (T). V
rovnici se dále objevuje plynová konstanta R [J.mol-1.K-1].

Reálné plyny se od ideálního plynu liší tím, že mezi molekulami plynu dochází i k jiným silovým interakcím než jen k
pružným srážkám. Tyto interakce jsou krátkého dosahu, protože jejich podstatou jsou elektrické síly mezi mezi
molekulami (viz nevazebné interakce). Z pochopitelných důvodů budou vzácné plyny dostatečně přesně popsatelné
stavovou rovnicí ideálního plynu při vyšších tlacích než molekulární plyny.

Van der Waalsova stavová rovnice


Pro popis chování reálného plynu se stavová rovnice obvykle příliš nehodí, protože pro většinu zajímavých úloh je
chování reálného plynu významným způsobem odlišné od chování ideálního plynu. Fyzikální podstatou takového
chování je zejména to, že nelze zanedbat vzájemné interakce molekul plynu ani jejich konečný objem, přesněji
odpuzování při přílišném přiblížení (viz chemická vazba). První úspěšnou stavovou rovnici reálného plynu sestrojil na
konci 19. století van der Waals, který do stavové rovnice ideálního plynu zavedl dva korekční faktory.

Stavovou rovnici ideálního plynu si vyjádřil pomocí molárního objemu (objem vztažený na látkové množství):

V prvním kroku vyšel z představy, že jeden mol molekul plynu zaujímá konečný objem, který označila jako vyloučený
objem (b). Pro stlačování jednoho molu plynu je pak k dispozici pouze objem zmenšený o vyloučený objem, tj.:

Vzájemné přitažlivé síly mezi molekulami způsobují, že molekuly jsou více přitahovány k sobě navzájem. To snižuje
rychlost, s jakou molekuly narážejí na stěnu nádoby, makroskopickým projevem je to, že na stěnu nádoby působí
menší tlak. Analýzou sil působících mezi molekulami van der Waals určil, že pokles tlaku je nepřímo úměrný druhé
mocnině molárního objemu a závisí na konstantě a.

Stavová rovnice po zahrnutí těchto dvou korekcí má tvar.

Van der Waalsova rovnice je na rozdíl od rovnice ideálního plynu schopna i popsat kondenzaci plynu. Ani van der
Waalsova rovnice není příliš přesný model, pro řadu úloh má význam spíše jako nástroj kvalitativního popisu chování
plynu. V praxi se však stále používá několik stavových rovnic reálného plynu, které z van der Waalsovy rovnice
vycházejí.

Odkazy
Zdroj

KUBATOVA, Senta. Biofot [online]. [cit. 2011-01-31]. <https://uloz.to/!CM6zAi6z/biofot-doc>.

NOVÁK, J., et al. Fyzikální chemie : bakalářský a magisterský kurz. 1. vydání. VŠCHT Praha, 2008. ISBN 978-
80-7080-675-3.
Maxwellovo-Boltzmannovo rozdělení rychlostí
Molekuly plynu se neustále pohybují a srážejí, každá z nich má při stejné hmotnosti a jiné rychlosti rozdílnou
kinetickou energii. Statistické rozdělení rychlostí náhodného pohybu částice plynu je velmi dobře popsáno
Maxwellovým-Boltzmannovým rozdělením. Hustota pravděpodobnosti[pozn 1] rozdělení velikosti rychlosti molekuly
ideálního plynu má tvar:

kde m je hmotnost molekuly, k je Boltzmannova konstanta (1,38.10-23


J.K-1) a T absolutní teplota.

Nejdůležitější charakteristikou je střední kvadratická hodnota (protože se používá k vyjádření střední kinetické
energie molekul):

Důležitým parametrem je také maximum, tedy nejpravděpodobnější rychlost (v řeči statistiky jde vlastně o modus):

Najde se snadno derivováním hustoty.

Maxwellovo-Boltzmannovo rozdělení není symetrické, ale kladně sešikmené, tzn. nejedná se zároveň o jeho střední
hodnotu. Fyzikální interpretace je taková, že jde o rychlost, s jakou se pohybuje nejvíce částic v systému, ne však o
průměrnou rychlost jednotlivých částic. Ta se dá spočítat integrací: (řeší se substitucí u=v2 a dále metodou per
partes)

Chování plynů, které lze popsat pomocí Maxwellova-Boltzmannova rozdělení, závisí na teplotě. Čím je vyšší teplota,
tím je maximum rychlostí posunuto více směrem k vyšším hodnotám a sama křivka je plošší.

Příklady Maxwellova-Boltzmannova rozdělení pro


několik hodnot parametru

(nejpravděpodobnější rychlost vztažená k libovolné


referenční hodnotě rychlosti, např. pro vzduch o
teplotě 300 K). Na vodorovné ose je
, na svislé
hustota pravděpodobnosti
podle značení výše uvedeného.

Odkazy
Poznámky pod čarou
1. Pro připomenutí nebo vysvětlení, hustota pravděpodobnosti je funkce, která popisuje pravděpodobnostní chování.
Například pokud nás zajímá pravděpodobnost, že rychlost náhodně vybrané částice leží v intervalu v1 až v2, pak
je řešením integrál z hustoty pravděpodobnosti s integračními mezemi od v1 do v2.

Zdroj
KUBATOVA, Senta. Biofot [online]. [cit. 2011-01-31]. <https://uloz.to/!CM6zAi6z/biofot-doc>.
Ekvipartiční teorém
Z kinetické teorie plynů plyne, že střední kinetická energie částice ideálního plynu
je přímo úměrná termodynamické teplotě plynu:

Kde značí Boltzmannovu konstantu. Vynásobením této rovnice Avogadrovou


konstantou dostáváme celkovou (kinetickou) energii 1 molu plynu:

neboť univerzální plynová konstanta je definována

Charakteristika
Ekvipartiční teorém říká, že celková energie plynu je rovnoměrne rodělena mezi všechny stupně volnosti systému.
Počet stupňů volnosti plynu tedy závisí na počtu atomů v jeho molekulách.

Monoatomické molekuly plynů vykonávají pouze translační pohyb popsatelný v prostoru osami x, y, z, tzn. mají tři
stupně volnosti ( ).
Biatomické molekuly vykonávají ještě vibrační a rotační pohyby ( ).

Obecná rovnice pro celkovou energii ideálního plynu je dána zoběcněním předešlé rovnice na libovolný počet stupňů
volnosti:

Odkazy
Zdroj

KUBATOVA, Senta. Biofot [online]. [cit. 2011-01-31]. <https://uloz.to/!CM6zAi6z/biofot-doc>.


Fázové změny
Fázové změny nebo také fázové přechody jsou děje, při kterých narušíme termodynamickou rovnováhu látky nebo
soustavy. Tímto narušení dojde ke změně fáze látky. Při této změně dojde ke skoku, při kterém se mohou změnit
fyzikální veličiny, které danou látku charakterizují.

Tání a tuhnutí
Atomy, molekuly nebo ionty upevněné v krystalové mřížce kmitají následkem tepelného pohybu. S rostoucí teplotou
stoupá jejich kinetická energie až dosáhne takové hodnoty, že částice opustí mřížku a ta se rozpadne. Při
ochlazovaní kapaliny dochází k opačnému jevu.

Vypařování a kondenzace
V uzavřené nádobě je kapalina a nad ní při jakékoliv teplotě její vlastní páry. Převládá-li pohyb molekul z kapalné
fáze do plynné, dochází k vypařování a roste tlak plynu nad kapalinou do té doby, než je stejný počet molekul v
plynné i kapalné fázi ( = nasycená pára).
Dochází-li k vypařování v celém objemu kapaliny, jedná se o var.
Náhlým ochlazením páry dojde k její kondenzaci, jsou k tomu však třeba kondenzační centra (malé částečky
prachu nebo ionty).

Sublimace a desublimace
Přechod z pevné fáze v plynnou a naopak.

Ionizace a rekombinace
Přechody plynné fáze v plazmatickou a naopak.

Odkazy
Externí odkazy

Teplo a teplota. Fáze a fázové změny [online]. [cit. 2019-02-07].


<http://artemis.osu.cz/ComLab/Web_cz/Heat_FP6_Cz/Phase.htm>.

Související články

Latentní teplo

Zdroj

PROF. MECHLOVÁ, CSC., Erika. Teplo a teplota : Fáze a fázové změny [online]. ostravská univerzita, katedra
fyziky, ©2005. Poslední revize 2005, [cit. 2019-02-07]. 
<http://artemis.osu.cz/ComLab/Web_cz/Heat_FP6_Cz/Phase.htm>.

KUBATOVA, Senta. Biofot [online]. [cit. 2011-01-31]. <https://uloz.to/!CM6zAi6z/biofot-doc>.


Latentní teplo

Latentní teplo – energie, kterou je nutno dodat pro změnu fáze látky (dodáváme teplo, které nemění teplotu látky,
ale jen její skupenství).

Odkazy
Související články

Fázové změny

Zdroj

KUBATOVA, Senta. Biofot [online]. [cit. 2011-01-31]. <https://uloz.to/!CM6zAi6z/biofot-doc>.


Gibbsův zákon fází

Fázový diagram vody

Disperzní systém je soustava obsahující alespoň dvě fáze nebo dvě složky (jedna – podíl je rozptýlena v druhé –
prostředí). Dvoufázový disperzní systém je heterogenní, protože mezi disperzními podíly existují v disperzním
prostředí jasné hranice. Systém sestávající se naopak ze dvou složek v jedné fázi je homogenní (cukr rozpuštěný ve
vodě) a jeho složky nejsou opticky rozlišitelné.

Gibbsův zákon fází:

= počet složek soustavy, = počet fází soustavy,


= počet stupnů volnosti (teplota, tlak)

Příklad:

Kapalina a její pára ( , ) má jeden stupeň


volnosti – měnit lze jen tlak nebo jen teplotu.
Pokud chceme měnit teplotu i tlak, může existovat pouze jedna fáze.
Mají-li se v rovnováze udržet fáze tři, stane se to pouze za dané teploty a tlaku (
) – v tzv. trojném bodě.
Pro vodu je trojný bod 273,16 K[1] (0,01 °C) při tlaku 610,6 Pa

Odkazy
Zdroj

KUBATOVA, Senta. Biofot [online]. [cit. 2011-01-31]. <https://uloz.to/!CM6zAi6z/biofot-doc>.

Reference

1. JIŘÍ, Beneš,, Jirák, DANIEL a Vítek, FRANTIŠEK. Základy lékařské fyziky. - vydání. Charles University in Prague,
Karolinum Press, 2015. 322 s. ISBN 9788024626451.
Analytické disperze
Disperzní systém o velikosti částic <1 nm (složky nezjistitelné opticky, pouze analyticky/chemicky).

disperzní disperzní
analytická disperze
prostředí podíl
plynný směsi plynů, parciální tlaky i objemy složek se při míchání sčítají
hlavně vodní páry ve vzduchu, relativní vlhkost vzduchu je podíl absolutní vlhkosti (kg páry v 1 m3) a maximální
plynné kapalný
vlhkosti (absolutní vlhkost při nasycení vzduchu vodními parami) – udáváme v %
pevný páry tuhé látky v plynu
plynný plyny rozpuštěné v kapalině – viz Henryho zákon
kapalný směsi mísitelných (neomezeně nebo omezeně) kapalin
pravé roztoky tuhých látek (nejčastěji ve vodě)
kapalné
S rostoucí teplotou roste rozpustnost, při ustavení rovnováhy se látka přestane rozpouštět a vzniká nasycený roztok.
pevný
Při ochlazení nasyceného roztoku vzniká buď přesycený roztok, nebo se v případě přítomnosti vhodných
kondenzačních center vyloučí pevná látka.

voda vázaná na krystalickou sůl (ta potřebuje vodu, aby vytvořila mřížku a při zahřívání a vysoušení se rozpadá na
kapalný
pevné prášek)
pevný tuhé roztoky, např. skla

Odkazy
Související články

Koloidální disperze
Roztoky

Zdroj

KUBATOVA, Senta. Biofot [online]. [cit. 2011-01-31]. <https://uloz.to/!CM6zAi6z/biofot-doc>.


Henryho zákon
Tento zákon říká, že váhové množství plynu rozpuštěné za dané teploty v kapalině je přímo úměrné tlaku plynu nad
kapalinou:

kde c = koncentrace plynu v kapalině, α = absorpční koeficient (rozpustnost plynu závisí na teplotě, se zvyšující se
teplotou rozpustnost klesá), P = tlak.

Henryho zákon má význam ve fyziologii dýchání (kesonová nemoc) – ohrožuje lidi pracující pod vyšším než
atmosférickým tlakem, kteří rychle přejdou do tlaku normálního (horníci, potápěči), protože se jim do krve uvolní
bublinky dusíku, který se tam předtím dostal difúzí.

Odkazy
Zdroj
KUBATOVA, Senta. Biofot [online]. [cit. 2011-01-31]. <https://uloz.to/!CM6zAi6z/biofot-doc>.
Koloidální disperze
Pevné látky ve vodě
Koloidní částice mají velikost 1-1000 nm a jsou tvořeny buď makromolekulami nebo micely. Částice se neustále
neuspořádaně pohybují – Brownův pohyb. Při větších velikostech částic se nárazy molekul disperzního prostředí na
částici vyrovnají, ale u koloidních částic obvykle jeden směr převládne a podle něj se pak molekula pohybuje.

Sedimentace
Částice disperzního podílu, která má větší hustotu než disperzní prostřední, postupně klesá (sedimentuje) vlivem
působení gravitačního pole. Pro sedimentační rychlost platí:

η = viskozita prostředí, r = poloměr částice, g = gravitační zrychlení

Ultracentrifugací se zvýší gravitační zrychlení a tím i sedimentační rychlost.

Dialýza
Dialýza – koloidní částice jsou (ne)propustné určitými filtry, a proto je můžeme oddělit od disperzního podílu
současně přítomného v roztoku, nebo i od disperzního prostředí

Elektrická dvojvrstva
Nejdůležitější vlastností koloidních částic ve vodě je existence jejich el. dvojvrstvy na povrchu. Každá kol. částice
nese na svém povrchu el. náboj a přiblíží-li se dvě částice k sobě, nemohou se spojit, protože se jejich souhlasné
náboje odpuzují.

Aerosoly
Kapalina nebo pevná látka v plynu:
plyn + kapalina = mlha;
plyn + pevná látka = dým, prach.
Nepříznivé jsou hlavně průmyslové dýmy a prachy znečišťující atmosféru.

Odkazy
Související články

Analytické disperze

Zdroj

KUBATOVA, Senta. Biofot [online]. [cit. 2011-01-31]. <https://uloz.to/!CM6zAi6z/biofot-doc>.


Pohyb koloidních částic v elektrickém poli
Elektroforéza je postup oddělování koloidních částic ve stejnosměrném elektrickém poli. Využívá se při tom
jejich rozdílné velikosti, náboje a rychlosti el. poli. Povrch koloidní částice má určitý kladný potenciál vůči okolí,
který je rozhodující pro její pohyb.
Volná elektroforéza – roztok s koloidními částicemi vystavíme vlivu stejnosměrného el. pole a částice se různou
rychlostí přibližují k opačně nabitým elektrodám. Oddělení částic však není dokonalé.
Elektroforéza na nosičích – např. gel, azbest, hedvábí, celulóza, škrob, papír; dokonalejší rozdělení složek

Článek neobsahuje vše, co by měl.


Můžete se přidat k jeho autorům a jej.
O vhodných změnách se lze poradit v diskusi.
Voda jako rozpouštědlo, voda v organizmu

Lomená struktura molekuly vody

Voda je nejvíce zastoupená složka v živých organismech, nejdůležitější rozpouštědlo a je také pochopitelně nezbytnou
součástí lidského těla. Většina důležitých životních procesů probíhá právě ve vodném prostředí.

Struktura molekuly vody, vlastnosti


V molekule jsou vázané dva atomy vodíku, každý po jednom elektronu, a jeden atom kyslíku se šesti elektrony.
Působením kovalentní vazby dosáhnou atomy ideální konfigurace vzácného plynu. Molekula vody je lomená a její
vazby spolu svírají úhel 104,45° (vycházíme z teorie hybridizace a přítomnosti dvou volných elektronových párů, které
nepatrně mění vazebný úhel).

Značný rozdíl elektronegativit mezi kyslíkem a vodíkem působí tak, že kyslík k sobě přitahuje elektrony, které se
účastní vazby. Proto se v molekule vytvoří parciální náboje (na kyslíku záporný, na vodících kladný), které způsobují
charakter elektrického dipólu.

Vodíkové můstky

Mezi molekulami vody se elektrostaticky vytvářejí tzv. vodíkové můstky, slabé vazebné interakce mezi opačnými
náboji sousedních molekul (energie vodíkové vazby u vody je 19 kJ•mol–1). Díky vodíkovým můstkům se tvoří
prostorová síť spojených molekul. Voda je proto dobrým polárním rozpouštědlem.

Molekuly se organizují do čtyřstěnů a takto vytváří krystalickou strukturu ledu (částečně i vody). Hustota vody je
nejvyšší při 3,98 °C a je to tzv. anomálie vody, protože u ostatních kapalin hustota stoupá s klesající teplotou.

Voda jako rozpouštědlo


Díky své nesymetričnosti rozpouští voda hydrofilní, polární látky plynné, kapalné i pevné fáze a vznikají tak vodné
roztoky. V organismu tvoří disperzní prostředí pro makromolekuly, molekuly a ionty v buňkách a napomáhá tak
vzájemné interakci. Když jsou látky rozpuštěné ve vodných roztocích ve formě iontů, roztok je elektrolyt a vede
elektrický proud (např. fyziologický roztok – 0,9 % vodný roztok NaCl). Mnohé organické látky (glycerol, ethanol,
bílkoviny) ve vodném roztoku nevedou proud, jsou obklopeny molekulami vody, ale nejsou rozštěpené na ionty.

Průběh rozpouštění
Proces rozpouštění vodou můžeme také jinak nazvat elektrolytická disociace (disociace účinkem polárního
rozpouštědla). Elektrolytické disociace dělíme na dva typy:

Disociace látek s iontovou mřížkou

Molekuly rozpouštědla, které obklopí mřížku, se orientují k povrchu mřížky. Každá molekula se orientuje opačným
nábojem k iontu v mřížce. Působením vazebných sil oslabí molekula rozpouštědla vazby v mřížce a odtrhne ionty,
které se následně uvolní do roztoku.

Disociace látek s polární vazbou

V důsledku interakce mezi polárními molekulami (např. HCl, CH3COOH) a molekulami polárního rozpouštědla dochází
ke zvýšení polarizace kovalentní vazby polárních rozpouštěných molekul. To způsobí, že molekula úplně rozštěpí.
Uvolněné ionty jsou ovšem stále pevně obaleny molekulami rozpouštědla a vytváří tzv. solvatační obal. Tomuto
procesu se také jinak říká solvatace.

Voda v organismu
Vodu v organismu rozdělujeme na volnou, která zajišťuje rozpouštění a transport látek, a vodu vázanou, která je
vázána v hydratačních obalech (např. hydrofilní koloidy). Obě dvě složky jsou v rovnováze a jejich molekuly se
navzájem neustále vyměňují. Voda je výsledným produktem při různých biochemických reakcích, jedním z příkladů je
průběh oxidace. Významnou roli hraje také osmóza, díky níž se voda v těle může pohybovat přes semipermeabilní
membrány. Jaký bude obsah vody v jednotlivých tělesných částech závisí na obsahu osmoticky aktivních částic –
osmolaritě. (nejméně vody obsahuje zubní sklovina) Voda je významná také pro termoregulaci a odvádění tepla
pocením, kvůli vysoké hodnotě výparného tepla (2,25 MJ•kg–1) při 37 °C.

Odkazy
Související články

Roztoky

Použitá literatura

BENEŠ, Jiří, Daniel JIRÁK a František VÍTEK, et al. Základy lékařské fyziky. 4. vydání. 2015. 322 s. ISBN
9788024626451.

NAVRÁTIL, Leoš a Jozef ROSINA, et al. Medicínská biofyzika. 1. vydání. Praha : Grada Publishing, 2005. 


524 s. ISBN 978-80-247-1152-2.
Turbidimetrie

Turbidimetrie a nefelometrie

Kromě absorpce může při průchodu světla vzorkem docházet k jeho rozptylování, pokud jde o disperzní nebo koloidní
soustavu. Toho se vyžívá v technikách nefelometrie (řecky nefelé = mrak), kdy se měří intenzita světla rozptýleného
pod určitým úhlem, a turbidimetrie (anglicky turbid = zakalený), měří-li se intenzita světla procházejícího vzorkem v
původním směru. Obě metody jsou vzájemně „zrcadlové“, budeme se proto dále zabývat jen turbidimetrií: lze ji
uskutečnit pomocí obvyklého fotometrického vybavení a úbytek světla rozptylem při průchodu vzorkem se dá snadno
popsat pomocí absorbance a dalších veličin obvyklých ve fotometrii.

Množství rozptýleného světla závisí na

1. koncentraci částic. V širokém rozsahu koncentrací jde přitom o závislost lineární, takže lze pracovat zcela
analogicky fotometrickým metodám s využitím Lambertova-Beerova zákona. Veličina odpovídající absorbanci se
jmenuje turbidita.
2. velikosti částic. Množství rozptýleného světla je přibližně nepřímo úměrné molekulové hmotnosti částice.
3. vlnové délce světla. Čím kratší je vlnová délka, tím větší podíl světla bude rozptýlen (Tyndallův jev), intenzita
rozptýleného světla roste približně se čtvrtou mocninou převrácené hodnoty vlnové délky. V praxi se jako
nejvýhodnější používají vlnové délky od 340 do 450 nm. Krátkovlnnější světlo bývá pohlceno bílkovinami, které
jsou v biologických vzorcích zpravidla přítomny.

Při turbidimetrických metodách bývá nejobtížnější vytvořit dostatečně stabilní suspenzi, která by byla stálá po celou
dobu měření. Proto se do reakčních směsí přidávají ochranné koloidy, nejčastěji polyetylenglykol.

Turbidimetrie a nefelometrie se používají nejčastěji v imunochemických metodách k vyhodnocování


imunoprecipitačních reakcí, kde zákal tvoří komplexy antigen-protilátka.
Krevní tlak

Tabuľka tlaku krvi

Měření krevního tlaku

Krevní tlak je tlak, kterým působí protékající krev na stěnu cévy. Hodnota tlaku krve je různá v různých částech
krevního řečiště, nejvyšší je ve velkých artériích, směrem do periferie klesá, nejnižší je pak v žilním systému. Pojmem
krevní tlak se nejčastěji myslí arteriální (tepenný) krevní tlak, což je tlak krve ve velkých artériích.

Arteriální krevní tlak se mění během srdečního cyklu, nejvyšší je v ejekční fázi = systolický tlak, nejnižší během
plnící fáze = diastolický tlak. Rozdíl mezi systolickým a diastolickým tlakem je tlaková (tepová) amplituda.

Hodnota krevního tlaku se skládá ze dvou čísel oddělených lomítkem, vyšší číslo je hodnota systolického tlaku, nižší
číslo je diastolický tlak (vyjadřuje se nejčastěji v mmHg, méně často v kPa). Např. hodnota tlaku 120/80 mmHg
znamená, že systolický tlak je 120 mmHg a diastolický tlak 80 mmHg. U zdravého dospělého člověka by hodnota
systolického tlaku neměla překročit 140 mmHg a hodnota diastolického tlaku 90 mmHg. Nízký krevní tlak se označuje
jako hypotenze, vysoký krevní tlak je hypertenze.

Systolický a diastolický tlak závisí na celkovém stavu a naladění člověka. Hodnoty ovlivňuje stres či fyzická zátěž.
Může se také měnit s věkem. Dále závisí na viskozitě krve, průsvitu kapilár nebo na elasticitě cév.

Při zátěži se minutový srdeční výdej zvyšuje čtyřikrát, tudíž se zvýší i tlak zhruba o 50 %. Hodnoty krevního tlaku
závisí také na druhu zátěže. Kdybychom vykonávali spíše dynamické pohyby, počítáme se zvyšováním systoly. Tlak
systolický i diastolický se zvyšuje u zátěže statické.

Hodnoty krevního tlaku v některých částech cirkulace

sTK (mmHg) dTK (mmHg)


aorta 120 80
levá komora 120 0
a. pulmonalis 25 8
pravá komora 25 0

sTK – systolický tlak krve, dTK – diastolický tlak krve

Měření krevního tlaku

Podrobnější informace naleznete na stránce Monitorování krevního tlaku.

1. Přímo (invazivně) – do tepny (nejčastěji a. radialis) zavedeme speciální katetr, který je napojený na manometr a
monitor. Tuto metodu měření krevního tlaku využíváme především u pacientů ve vážném stavu (hospitalizace na
JIP!).
2. Nepřímo – tonometrem s rtuťovým manometrem auskultací a. brachialis, případně s palpací a. radialis.
3. Automaticky – digitálními přístroji, které využívají metodu:
auskultační – zabudovaný mikrofon snímá klasické Korotkovovy šelesty; nevýhodou je více
technických artefaktů. A
oscilometrickou – omezeným průtokem krve osciluje tepna a synchronně s ní i manžetový tlak, který
je monitorován tlakovým senzorem. Tato metoda se používá při ambulantním monitorování krevního
tlaku holterovského typu. Výpočet tlaku je používán i v chytrých hodinkách pro monitoring krevního
tlaku.

Odkazy
Související články

Systolický krevní tlak


Diastolický krevní tlak
Střední arteriální tlak
Tlaková amplituda
Měření krevního tlaku
Monitorování krevního tlaku
Regulace krevního tlaku
Domácí monitoring krevního tlaku

Zdroj

ŠVÍGLEROVÁ, Jitka. Krevní tlak [online]. Poslední revize 18. 2. 2009, [cit. 10.11.2010]. 


<https://web.archive.org/web/20160306065550/http://wiki.lfp-studium.cz/index.php/Krevní_tlak>.
Bernoulliho rovnice
Vyjadřuje zákon zachování mechanické energie pro ustálené proudění ideální kapaliny.

kde v = rychlost proudění, P = tlak, ρ = hustota, g = gravitační zrychlení.

V místě s větším průřezem má proudící kapalina větší tlak, ale menší rychlost, zatímco v místě s menším obsahem
průřezu má menší tlak, ale větší rychlost tlak proudící kapaliny klesá
s její rostoucí rychlostí.

Odkazy
Externí odkazy

Bernoulliho rovnice (česká wikipedie)


Bernoulli's principle (anglická wikipedie)

Zdroj

KUBATOVA, Senta. Biofot [online]. [cit. 2011-01-31]. <https://uloz.to/!CM6zAi6z/biofot-doc>.


Krevní řečiště, rovnice kontinuity
Tok krve v krevním řečišti jako fyzikální jev studuje podobor biomechaniky, který se nazývá bioreologie (používá se i
tvar biorheologie). Pro vlastní popis proudění krve se někdy používá pojem hemodynamika.

Fyzikální zvlášnosti krevního řečiště


Z hlediska fyzikálního má krev a krevní řečiště několik vlastností, které činí jejich studium obtížnější. Jedná se
zejména o následující faktory:

Krev je složitou směsí, jsou v ní rozpuštěny nízkomolekulární látky i vysokomoloekulární látky a v krvi je
suspendováno značné množství formovaných elementů, zejména erytrocytů. V důsledku toho se krev chová jako
nenewtonovská kapalina.
Chemické složení krve se mění v závislosti na průchodu krevním řečištěm.
Fyzikálně chemické vlastnosti krve se mohou měnit i v závislosti na teplotě, teplota se mezi játry (místo s nejvyšší
teplotou) a akrálními částmi těla liší i za bazálních podmínek.
Krev je vodným roztokem, endoteliální výstelka cév je ovšem hydrofobní.
Existují fyziologické mechanismy zajišťující koagulaci krve při poranění nebo při zánětu.
Cirkulace je ponáněná stahy srdce, průběh tlakové vlny má proto poměrně komplikovaný tvar.
Cévní stěny jsou pružné, mechanické vlastnosti cév mohou měnit svalová vlákna v jejich stěnách.

Krev hraje důležitou roli v řadě fyziologických procesů. Jedná se zejména o následující procesy:

Látková výměna včetně výměny dýchacích plynů.


Transport hormonů.
Transpost tepla, u teplokrevných živočichů jde především o ochlazování metabolicky aktivního jádra.
Hydrodynamická opora některých orgánů. U obratlovců je tato funkce obvykle omezena.

Rozdělení cév z biomechanického hlediska


Z funkčního hlediska lze cévy rozdělit na tři typy:

cévy pružníkové
cévy rezistenční (odporové)
cévy kapacitní

Pružníkové cévy jsou anatomicky velké tepny, především aorta. Vyznačují se poměrně elastickou stěnou. Při systole,
kdy je do krevního oběhu vypuzeno větší množství krve, se cévy roztáhnou. Vlastně je tak do potenciální energie jejich
stěny absorbována část kinetické energie, kterou dodalo krvi srdce. Během diastoly dochází k pasivnímu stažení stěny
cévy a potenciální energie vyššího napjetí stěny je přeměněna zpět na kinetickou energii krve. Prakticky to znamená,
že pružnost stěny velkých tepen zajišťuje plynulejší tok krve.

Rezistenční cévy jsou anatomicky především menší tepny a zejména tepénky. Vyznačují se tím, že ve stěně mají
poměrně velké množství svaloviny (tepny muskulárního typu), která se na kontrahuje nebo naopak relaxuje dle potřeb
organismu. Tyto cévy regulují především průtok krve zbytnými orgány. Svojí činností zvyšují periferní odpor krevního
řečiště. Příkladem činnosti těchto tepen jsou změny prokrvení svalů při práci, kdy s rostoucí spotřebou se přívodné
cévy relaxují, zvyšuje se průsvit lumen a do svalu přitéká více krve. Při chladnutí periferie dochází naopak ke
kontrakci přívodných tepen, čímž se sníží ztráty tepla.

Kapacitní cévy jsou anatomicky především žíly. Vyznačují se především tím, že se v nich v každém okamžiku nachází
větší díl krve.

Z pohledu tlaků jde krevní řečičtě rozdělit na část vysokotlakou (tepny) a nízkotlakou (žíly).

Kapilární řečiště
Kapilární řečiště se vyznačuje především tím, že právě na úrovni kapilár dochází k výměně látek mezi krví a tkáněmi.
Kapilární síť je velmi bohatá, zejména pak v orgánech náročných na přívod kyslíku a živin. I když většina kapilár je
uzavřena a krev jimi neprochází, stále je průřez všech kapilár mnohem větší než průřez tepen nebo žil.

Rovnice kontinuity
Podrobnější informace naleznete na stránce Rovnice kontinuity.

Rovnice kontinuity je vlastně formulací zákona zachování hmotnosti. Vlastně jde jen o matematický zápis poznatku, že
když za daný čas to libovolného objemu vyplněného nestlačitelnou kapalinou přiteče nestlačitelná kapalina o objemu
VI, musí být objem kapaliny odteklé VO identický. Pokud budeme uvažovat diskrétní příkpad, totiž že daný objem má
konečný počet přívodních trubic ("armatur") n, ve kterých teče kapalina s konstatním průtokem Qi, lze rovnici
kontinuity zapsat ve tvaru:
Vlastně by bylo možné čas vytknout, ale to by nevedlo k užitečným výsledkům. Lépe je si uvědomit, že tvar v sumě lze
vyjádřit pomocí průřezu Si dané trubice a rychlosti vi v dané trubici. Pak získáme obvyklý tvar:

Pro vlastní hemodynamiku z tohoto plyne hodně. Představíme-li si efektivní průřezy krevním řečištěm podle jeho etáží,
pak má nejmenší průřez tepenná část, daleko větší má žilní část a zdaleka největší průřez mají kapiláry. Tomu
odpovídá i to, že v tepnách má krevní proud největší rychlost, v žilách menší a nejmenší v kapilárách.

Je ovšem třeba zdůraznit, že toto je pouze aproximace vycházející z představy ustáleného proudění ideální kapaliny
při zanedbání vlivu gravitačního pole. Při uvažování vlivu gravitačního pole přechází rovnice kontinuity v Bernoulliho
rovnici.
Viskozita

Tento článek potřebuje úpravy!


Tento článek ještě zdaleka nevypadá jako ideální článek. Je nutné v článku provést
základní úpravy jako například doplnit definici, článek rozčlenit, ozdrojovat a provázat s
dalšími články na wikiskriptech. Článek by neměl mít formu seznamu s odrážkami, ale více
se podobat kapitole v učebnici.
Návody jak postupovat při typografických a pravopisný úpravách, jak vkládat obrázky a mnohé další návody
naleznete v sekci Nápověda.

Demonstrace
viskozity

Viskozita je fyzikální veličina, která udává poměr mezi tečným napětím a změnou rychlosti v závislosti na vzdálenosti
mezi sousedními vrstvami proudící kapaliny. Jedná se o veličinu, která charakterizuje vnitřní tření kapaliny a závisí
především na přitažlivých silách mezi částicemi. Kapaliny s větší přitažlivou silou mezi částicemi mají větší viskozitu,
větší viskozita zpomaluje pohyb kapaliny nebo těles v kapalině. [1]

Pro ideální kapalinu má viskozita nulovou hodnotu. Kapaliny s nenulovou viskozitou se označují jako viskozní (vazké).
Viskozita tělesných tekutin se většinou udává v relativních jednotkách, kdy 1 = viskozita vody. Viskozita plazmy je 2 (2
krát vyšší než viskozita vody), viskozita krve je vzhledem k obsahu krvinek vyšší než u plazmy a její průměrná hodnota
je 4. Viskozita krve se zvyšuje při vyšším hematokritu nebo při poklesu rychlosti proudící krve. Z toho důvodu je
viskozita krve v kapilárách vyšší než v artériích (nižší rychlost proudění krve v kapilárách).[1]

kde Δx = vzdálenost vrstev, Δv = rozdíl rychlostí, τ = tečné napětí (F/A). [zdroj?]

Dynamická viskozita – síla tření, která vzniká mezi dvěma vrstvami proudící kapaliny, které se stýkají plochou 1 m2
a rozdíl jejich rychlostí je 1 ms−1.

Kinematická viskozita: .

Viskozita kapalin se stoupající teplotou klesá, při nižších teplotách se mohou molekuly shlukovat ve větší celky.

Velký biologický význam má viskozita krve – díky erytrocytům má krev 4,5 krát větší viskozitu než voda.

Odkazy
Související články

Laminární proudění
Turbulentní proudění

Externí odkazy

Viskozita (česká wikipedie)


Viscosity (anglická wikipedie)

Zdroj

ŠVÍGLEROVÁ, Jitka. Viskozita [online]. Poslední revize 2009-02-18, [cit. 2010-11-13]. 


<https://web.archive.org/web/20130513015042/http://wiki.lfp-studium.cz/index.php/Viskozita>.

KUBATOVA, Senta. Biofot [online]. [cit. 2011-01-31]. <https://uloz.to/1162346/biofot.doc>.

Reference

1. ŠVÍGLEROVÁ, Jitka. Viskozita [online]. Poslední revize 2009-02-18, [cit. 2010-11-13]. 


<https://web.archive.org/web/20130513015042/http://wiki.lfp-studium.cz/index.php/Viskozita>.
Hagenův-Poiseuillův zákon
Hagenův-Poiseuillův vztah (čti hagen, poazej) říká, že objemový tok (objem za čas)
tekutiny v trubici je přímo úměrný rozdílu tlaků na začátku a na konci trubice

Konstanta úměrnosti vyjadřuje odpor. Zahrnuje v sobě viskozitu tekutiny


, čtvrtou mocninu poloměru trubice a délku trubice
podle vztahu:

Kombinací těchto dvou rovnic do jedné získáme Hagenův-Poiseuillův vztah v plném tvaru:

Hagenův-Poiseuillův vztah můžeme přirovnat k Ohmovu zákonu v elektřině: rozdíl tlaků odpovídá elektrickému
napětí, objemový tok odpovídá elektrickému proudu a odpor trubice elektrickému odporu.

Závislost odporu (a tedy i objemového toku ) na


vysoké mocnině poloměru má praktický význam při regulaci toku krve cévou – i malé
zúžení arterioly vede k rychlému snížení průtoku krve (viz dále).

Podrobnější informace naleznete na stránce Hagenův-Poiseuillův zákon/Odvození.

Příklad využití ve fyziologii


Podívejme se na aplikaci tohoto vzorce v ledvině.

je množství krve, které proteče ledvinou za jednotku času (průtok)


je rozdíl tlaku mezi a. renalis a v. renalis
je celkový odpor renálního krevního řečiště

Je žádoucí, aby průtok zůstal relativně nezávislý na výkyvech krevního tlaku v těle. Na
druhé straně ale závisí na tlaku krve. Zvýší-li se
(vlivem změny krevního tlaku), pak by měl následovat vzestup . Vzestup tlaku
v cévách ledviny ale reflexně vyvolá jejich vazokonstrikci, tedy zmenšení poloměru cévy
. Menší poloměr znamená výrazné zvýšení (čtvrtá mocnina) odporu . Nárůst
odporu (jmenovatel) tedy vykompenzuje nárůst tlaku (čitatel), čímž se průtok prakticky nezmění. Tento mechanizmus
se nazývá myogenní autoregulace.

Odkazy
Související články

Stokesův zákon
Viskozita
Newtonovská kapalina

Použitá literatura

CHMELÍK, František. Skripta k předmětu Fyzika I [online]. [cit. 2010-06-18]. 


<https://material.karlov.mff.cuni.cz/people/hajek/skripta/>.

KITTNAR, Otomar, et al. Lékařská fyziologie. 1. vydání. Praha : Grada, 2003. 790 s. ISBN 978-80-247-3068-4.


Stokesův zákon

Při sedimentaci je částice vystavena působení tíhové síly (Fg = mg = V ρčástice g) a vztlakové síly podle
Archimédova zákona (Fvz = V ρkapalina g). Když je hustota částice větší, než hustota kapaliny, částice začne klesat.

Proti jejímu pohybu působí odporová síla, daná Stokesovým vztahem:

,
kde r = poloměr částice, η = dynamická viskozita prostředí, v = rychlost částice.

Odkazy
Související články
Měření viskozity

Zdroj

KUBATOVA, Senta. Biofot [online]. [cit. 2011-01-31]. <https://uloz.to/!CM6zAi6z/biofot-doc>.


Laminární proudění

Laminární (neboli proudnicové) proudění je způsob pohybu viskózních kapalin, při kterém jsou dráhy jednotlivých
částic kapaliny navzájem rovnoběžné. Částice se pohybují ve vzájemně rovnoběžných vrstvách, aniž by se navzájem
mísily. Při pohybu viskózní kapaliny v tenké trubici, např. krve v cévě, se jednotlivé vrstvy tekutiny nepohybují stejnou
rychlostí. Nejrychleji se pohybuje osová vrstva tekutiny, směrem ke stěně cévy se pohyb jednotlivých vrstev
zpomaluje.

Laminární proudění zůstává zachováno až do tzv. kritické rychlosti, kdy se mění na proudění turbulentní.
Pravděpodobnost přechodu laminárního proudění na turbulentní je ovlivněno kromě rychlosti proudění ještě
průměrem trubice (cévy), viskozity a hustoty kapaliny. Tato pravděpodobnost se vyjadřuje pomocí Reynoldsova čísla:

Re – Reynoldsovo číslo,
ρ – hustota kapaliny,
R – průměr trubice,
V – rychlost proudění kapaliny,
η – viskozita.

Čím vyšší je Reynoldsovo číslo, tím vyšší je pravděpodobnost vzniku turbulentního proudění.

Laminární proudění, na rozdíl od turbulentního, není doprovázeno zvukovými fenomény.

Laminární proudění – modré šipky označují směr pohybu jednotlivých vrstev kapaliny.

Odkazy
Související články
Turbulentní proudění
Hagen-Poiseuillův zákon
Reynoldsovo číslo

Zdroj

ŠVÍGLEROVÁ, Jitka. Laminární proudění [online]. Poslední revize 19. 2. 2009, [cit. 9. 11. 2010]. 


<https://web.archive.org/web/20160306065550/http://wiki.lfp-studium.cz/index.php/Laminární_proudění>.
Turbulentní proudění

Turbulentní proudění je způsob pohybu viskózních kapalin (např. krve), při kterém jednotlivé částice kapaliny
přecházejí různými vrstvami kapaliny, čímž dochází k promíchávání těchto vrstev. Částice kapaliny vykonávají při
proudění kromě posouvání i složitý vlastní pohyb, který vede ke vzniku vírů a je doprovázen zvukovými fenomény (na
rozdíl od proudění laminárního). Rychlosti jednotlivých částic kapaliny se nepravidelně mění, tzn. částice již nemají ve
všech místech neměnnou rychlost, proudění tedy není stacionární.

Pravděpodobnost vzniku turbulentního proudění tekutiny v tenké trubici (např. krve v krevní cévě) je dáno rychlostí
proudění kapaliny, průměrem trubice (cévy), viskozitou a hustotou kapaliny. Tato pravděpodobnost se vyjadřuje
pomocí Reynoldsova čísla:

Re − Reynoldsovo číslo
ρ − hustota kapaliny
R − poloměr trubice (cévy)
v − rychlost proudění kapaliny
η − viskozita

Změna laminárního proudění (před


zúžením cévy) v turbulentní (za
zúžením cévy)

Čím vyšší je Reynoldsovo číslo, tím vyšší je pravděpodobnost vzniku turbulentního proudění. Turbulence se často
objevují při anémii vzhledem ke snížené viskozitě krve. Zvukových fenoménů doprovázejících turbulentní proudění se
využívá při měření krevního tlaku − zúžení artérie pomocí nafouknuté manžety zrychluje proud krve v místě
konstrikce, což vede k turbulenci a vzniku zvuků, tzv. Korotkovových fenoménů.

Odkazy
Související články
Laminární proudění
Hagen-Poiseuillův zákon
Reynoldsovo číslo

Zdroj

ŠVÍGLEROVÁ, Jitka. Turbulentní proudění [online]. Poslední revize 2009-02-18, [cit. 2010-11-14]. 


<https://web.archive.org/web/20160501161936/http://wiki.lfp-
studium.cz:80/index.php/Turbulentn%C3%AD_proud%C4%9Bn%C3%AD>.
Reynoldsovo číslo

Reynoldsovo číslo je bezrozměrné číslo charakterizující chování proudící kapaliny. Podle jeho hodnoty lze usuzovat
na to, zda bude proudění laminární nebo turbulentní.

Vypočte se podle vztahu[1]:

kde ρ je hustota kapaliny o viskozitě η proudí trubicí o průměru d rychlostí v. Pro různé typy potrubí a různé typy
kapalin se stanovují experimentálně kritické hodnoty Reynoldsova čísla. Je-li hodnota Reynoldsova čísla nižší než
kritická hodnota odpovídající danému uspořádání, je proudění laminární. Je-li hodnota Reynoldsova čísla vyšší, je
proudění turbulentní.[1] V okolí kritické hodnoty, která se v rovných hladkých cévách typicky pohybuje kolem 1000, je
přechodná oblast mezi zjevně laminárním a zjevně turbulentním prouděním.[2]

Odkazy
Související články
Laminární proudění
Turbulentní proudění

Reference

1. NAVRÁTIL, Leoš a Jozef ROSINA, et al. Medicínská biofyzika. 1. vydání. Praha : Grada, 2005. 524 s. ISBN 80-
247-1152-4.
2. GUYTON, Arthur C a John E HALL. Textbook of Medical Physiology. 11. vydání. Elsevier, 2006. 0 s. 11; ISBN
978-0-7216-0240-0.
Měření viskozity
K měření viskozity se používají přístroje založené na několika principech, tedy na měření veličin, jejichž hodnota v
daném fyzikálním systému souvisí s viskozitou měřené kapaliny.

Rotační viskozimetry
Principem společným všem typům rotačních viskozimetrů je měření momentu síly, který musí překonávat rotující
těleso ponořené do kapaliny. Za ideálních podmínek pro velikost tohoto momentu platí:

, kde M je moment síly, ω je úhlová frekvence rotujícího tělesa, η dynamická viskozita a k je konstanta přístroje
zahrnující především jeho geometrii. Pochopitelně že rotace tělesa musí být tak pomalá, aby nedocházelo k
turbulentnímu proudění. Obvyklou geometrií je válcová nádoba, do které je ponořen rotující válec. Velkou výhodou
rotačních viskozimetrů je to, že jsou v principu schopny měřit i viskozitu nenewtonovských kapalin.

Kapilární viskozimetry
Kapilární nebo též výtokové viskozimetry jsou založeny na měření objemového průtoku měřené kapaliny trubicí
definovaných rozměrů. Výchozím principem je Hagen-Poiseuillova rovnice pro svisle umístěnou kapiláru kruhového
průřezu o poloměru r a délce l, kterou za čas τ proteče kapalina o objemu V:

Dále pro rozdíl tlaků Δp platí při pomalém proudění vzorec známý již ze základní školy:

Měření se obvykle používá tak, že se srovnává čas průroku referenční (známé) kapaliny a kapaliny měřené. Pro poměr
jejich dynamických viskozit platí po dosazení do výše uvedených rovnic:

Jednoduchou úpravou dostaneme:

Zlomek na pravé straně je vlastně poměrem kinematických viskozit, tedy:

Pomocí většiny kapilárních viskozimetrů tedy měříme kinematickou viskozitu, protože hnací silou toku přes kapiláru je
síla gravitační. Toto lze do jisté míry korigovat použitím vnější síly, která bude podstatně větší než síla gravitační.

Konkrétních tehnických realizací existuje celá řada, např.:

Fordův pohárek je nejjednodušší typ sloužící k orientačnímu měření obvykle technických olejů. Vlastně jde jen o
standardizovaný model původní „plechovky s dírou ve dně“. Tento viskozimetr měří kinematickou viskozitu.
Ostwaldův viskozimet je vlastně skleněnou trubicí tvaru písmene U, ve které je ryskami přesně vyznačen
objem. Tento viskozimetr měří kinematickou viskozitu.
Ubbelohdeho viskozimetr je poněkud kompliovanější konstrukce než Ostwaldův viskozimetr. Tento viskozimetr
měří kinematickou viskozitu.
Mariotteova láhev je vlastně uzavřená nádoba s vodorovnou kapilárou u dna. Dominující hnací silou je tlak
přiváděný nad hladinu kapaliny. Mariotteova láhev tedy měří dynamickou viskozitu.

Kapilární viskozimetry nelze použít k měření nenewtonovských kapalin, protože nenewtonovská kapalina proudící
kapilárou obecně nemá parabolický profil rychlostí. To znamená, že v Hagen-Poiseuillův vztah nebude v uvedeném
tvaru platit.

Tělískové viskozimetry
Tělískové viskozimetry jsou založeny na měření rychlosti pádu nebo naopak vystoupání k hladině zkušebního tělesa.
Odporovou sílu působící na těleso obtékané kapalinou popisuje Stokesův zákon, který má pro obvykle používané
kulové tělísko o poloměru r, které se pohybuje rychlostí v, tvar:

Obecně se pro daný přístroj sestaví pohybová rovnice zkušebního tělíska, ve které vedle odporové síly figuruje i síla
gravitační a vztlaková. Pro zkušební kuličku o poloměru r a hustotě ρ a po doplnění předpokladu, že kulička již padá
konstantní rychlostí, bude mít výsledný vztah podobu:

K vlastnímu měření se obvykle použije měření doby t, za kterou tělísko urazí danou vzdálenost. Po zvážení tohoto a po
vytknutí všech fixních parametrů do jediné konstanty k lze psát:

Příkladem technické realizace je viskozimetr Höpplerův.

Tělískové viskozimetry lze použít pouze k měření newtonovských kapalin. Dalším omezením je, že je třeba, aby byla
kapalina průhledná.

Další typy viskozimetrů


Vibrační viskozimetry jsou založeny na studiu šíření nízkofrekvenčního vlnění s poměně vysokou amplitudou v
kapalině.
Ultrazvukové viskozimetry jsou v hrubém principu podobné vibračním, liší se především tím, že jde o
jednoznačně akustické měření na vysokých frekveních a nízkých amplitudách.
Plovákové viskozimetry jsou založeny na studiu unášení plováku proudící kapalinou.

Odkazy
Související články

Viskozita
Difuze
Difuze je samovolný proces pronikání částic jedné látky do druhé se snahou o rovnoměrné prostoupení do celého
objemu.

Základní popis
Difuze nastává z důvodu neuspořádaného tepelného pohybu částic. Pohyb částic je v zásadě náhodný, ale látky
mají tendenci přecházet z prostředí o vyšší koncentraci do prostředí s nižší koncentrací. Přirozenou vlastností
látek je, že pokud se její částice mohou pohybovat, tak se rozptylují do celého prostoru, a postupně ve všech jeho
částech vyrovnají koncentraci. Říkáme, že látky difundují. Během difuze se nespotřebovává energie.

Difuze

Rychlost šíření částic je ovlivněna velikostí částic, teplotou i vlastnostmi prostředí. Matematicky popisují difuzi
Fickovy zákony.

Difuze je děj spontánní, nevratný a tepelně aktivovaný. Tuto skutečnost objasňuje Einsteinova-Stokesova rovnice,
která popisuje závislost difúzního koeficientu na teplotě T, dynamické viskozitě kapaliny η a velikosti difundujících
částic R.

Difuze umožňuje pohyb látek uvnitř buněk a tím látkovou výměnu. V živých organismech hrají významnou roli další
faktory, např. elektrický náboj částic nebo transport přes polopropustné membrány.

Rozdělení
1. Prostá difuze
2. Facilitovaná difuze
3. Difuze iontovými kanály
Akvaporiny

Specifickým případem difuze je osmóza

Difuze v různých skupenstvích


Plynné prostředí

V plynném prostředí dochází k mnohem rychlejší difuzi, než v jiném prostředí. Částice plynu mají nejvyšší
kinetickou energii. Příkladem tohoto procesu je velmi rychlé rozšíření vůně po celé místnosti.

Kapalné prostředí

Částice kapalné látky mají nižší kinetickou energii, než částice plynu. Z toho důvodu v ní dochází k pomalejší difuzi,
než je tomu u plynu. Příkladem je uvolňování látek z čajového sáčku po jeho zalití horkou vodou.

Pevná látka

Difuze v pevném skupenství je obtížnější, časově náročná a je jediným možným způsobem přenosu látky. Záleží však
také na druhu materiálu. Příkladem je spojení dvou měděných drátků cínem.

Difuze v biologických systémech


Přestup látek skrz polopropustnou
membránu

Difuze je jedním z nejzákladnějších jevů odehrávajících se v živých organismech. Pro transport látek přes buněčnou
membránu je v lidském těle využitelná difuze plynná a kapalná.

Pro organismy je nezbytně nutné stálé složení tělních tekutin. Jedním z nejdůležitějších faktorů pro udržení
homeostázy, je transport přes membránu. Buněčná membrána je selektivně permeabilní (semipermeabilní)
bariéra. Jejím úkolem je zachování osmotické a iontové rovnováhy mezi intracelulárním a extracelulárním
prostředím. Buněčná membrána funguje jako bariéra, která reguluje průchod částic mezi intra- a extracelulárním
prostředí. Nejsnáze pronikají membránou malé nepolární molekuly jako je kyslík a oxid uhličitý, které se velmi
dobře rozpouštějí, a proto velmi rychle difundují. Tato rychlá difuze je důležitá pro výměnu plynů v alveolech a
tkáních. Lipofilní látky prostupují přes fosfolipidovou dvojvrstvu membrány přímo. Rychlost difúze je přímo úměrná
jejich rozpustnosti v tucích. [1]

Elektroneutrální polární molekuly, pokud jsou dostatečné malé (např. voda) poměrně snadno difundují (ne vždy
procesem prosté difuze). Hydrofilní látky (např. voda a ionty) jsou v lipidovém prostředí biologické membrány
nerozpustné, proto mohou difundovat pouze pomocí transmembránových přenašečů zabudovaných do membrány. To
má samozřejmě význam v možnosti regulace vstupu těchto látek. [1]

Příklady difuze v organismu


Difuze patří k jednomu z nejdůležitějších fyzikálních dějů, které umožňují pohyb látek uvnitř organismu.

Jedním z velmi důležitých dějů je vznik akčního potenciálu. Ten je způsobený přestupem iontů přes buněčnou
membránu.

Podrobnější informace naleznete na stránce Klidový membránový potenciál.


Podrobnější informace naleznete na stránce Akční potenciál (fyziologie).

Dalším příkladem si můžeme uvést nutnost podávání pacientovi izotonického roztoku (roztok s totožnou koncentrací
jako krevní plazma). V případě podání čisté vody by došlo k difúzi vody do lidských buněk, které by se zvětšovaly a
přílišným prodlužováním expozice by mohly prasknout. Naopak při dodání pacientovi příliš koncentrovaného roztoku
by došlo k vysávání vody z buněk - to by vedlo ke scvrkávání buněk. Pokud by byla doba expozice dlouhá, buňky by
byly natolik koncentrovány, že by došlo také ke smrti buněk.

Podrobnější informace naleznete na stránce Osmóza.

Odkazy
Související články

Transmembránový transport
Iontové kanály
Aktivní transport
Symport
Antiport
Pasivní transport
Difuze
Prostá difuze
Facilitovaná difuze
Filtrace
Osmóza
Průnik léčiva přes membrány

Reference
1. ŠVÍGLEROVÁ, Jitka. Difúze [online]. Poslední revize 2009-02-19, [cit. 2010-11-13]. 
<https://web.archive.org/web/20160306065550/http://wiki.lfp-studium.cz/index.php/Difúze>.
Použitá literatura

1. LEOŠ NAVRÁTIL, ROSINA JOZEF A KOLEKTIV, Medicínská biofyzika [online]. [cit. 2014-16-11].
<https://www.grada.cz/medicinska-biofyzika-3633/>
1. Fickův zákon
Difuzní tok
1. Fickův zákon určuje hustotu a směr difuzního toku j – proto si nejdříve definujeme, co to je: Jedná se o vektorovou
veličinu, jejíž velikost nám vyjadřuje, kolik molů dané látky projde jednotkovou plochou S za jednotku času t:

Jak je vidět, jednotkou hustoty difusního toku v soustavě SI je mol.m-2.s-1.

Směr difusního toku nám pak vyjadřuje střední směr proudění částic. Je tedy jasné, že musí být veličinou vektorovou.
Tou se formálně stane, pokud hodnotu z předchozí rovnice vynásobíme jednotkovým vektorem ve směru toku.

Hustotu toku přes nějakou plochu lze vyjádřit i ze střední rychlosti částic proudících přes tuto plochu a koncentrace
částic:

1. Fickův zákon
1. Fickův zákon říká, že hustota difuzního toku j je úměrná záporně vzatému gradientu koncentrace:

Gradient lze intuitivně chápat jako matematickou operaci, která dostane jako argument skalární funkci v prostoru,
tedy skalární pole, a vrátí vektorovou funkci v prostoru, tedy vektorové pole. Gradient v daném bodě je vlastně vektor
ve směru největšího spádu. Difuzní koeficient D je konstantou charakterizující, jak snadno daná látka difunduje daným
prostředím.

Za zmínku stojí zejména fakt, že tok j je vektorovou veličinou. To ale není překvapující ani při letmém zamyšlení,
protože při difúzi není důležité jen to, kolik látky se přesune, ale i kam se přesune.

Pro jednorozměný případ a pro aproximaci gradientu malými ale konečnými změnami lze použít mnohem jednodušší
tvar:

, kde Δc je rozdíl molárních koncentrací dvou blízkých míst vzdálených od sebe Δx. jednotkou koncentračního
gradientu je tedy mol.m-4.

Difuzní koeficient
Konstantou úměrnosti v 1. Fickově zákoně je tzv. difuzní koeficient D vyjadřující počet molů látky, které za čas 1 s
projdou průřezem 1 m2 při koncentračním gradientu 1 mol/m.
Koligativní vlastnosti roztoků

Osmotický tlak představuje z


hlediska biologie člověka
nejvýznamnější koligativní vlastnost.
Obrázek demonstruje chování
erytrocytů umístěných v prostředí s
různým osmotickým tlakem.

Koligativní vlastnosti roztoků jsou takové vlastnosti, které nezávisí na rozpuštěné látce, ale pouze na její koncentraci
nebo přesněji na látkové koncentraci částic v roztoku. Jde tedy o takové vlastnosti, které nezávisí např. na
velikosti molekuly rozpuštěné látky, jejím náboji nebo jejím tvaru. Koligativní vlastnost Φ závisí na koncentraci částic c
v ideálním případě lineárně, tedy platí obecná formule:

kde k je vhodná konstanta úměrnosti závisející výhradně na vlastnostech rozpouštědla a nezávisející na vlastnostech
rozpuštěné látky.

Koncentrace částic se nemusí rovnat koncentraci rozpuštěné látky. Rovnost platí pouze v případě, že rozpouštěná
látka v rozpouštědle nedisociuje. Pokud dochází k disociaci, koncentrace částic je příslušným násobkem koncentrace
rozpuštěné látky. Například chlorid sodný při dostatečně nízké koncentraci plně disociuje, a proto je koncentrace
částic rovna dvojnásobku koncentrace chloridu sodného.

Základními koligativními vlastnostmi jsou:

snížení tenze par nad roztokem


zvýšení bodu varu roztoku (ebulioskopie)
snížení bodu tuhnutí roztoku (kryoskopie)
osmotický tlak

Snížení tenze par nad roztokem


Snížení tenze par nad roztokem popisuje 1. Raoultův zákon. Slovně jej lze formulovat:

Relativní snížení tlaku nasycené páry nad roztokem oproti čistému rozpouštědlu je rovno molárnímu zlomku
rozpuštěné látky v roztoku.

Matematicky lze pak toto tvrzení formulovat:

kde Δp je změna tlaku nasycené páry nad roztokem, p0 je tlak nasycené páry nad čistým rozpouštědlem, c je
koncentrace rozpuštěné látky a cr je koncentrace (tedy také počet molů na litr) rozpouštědla.

Zvýšení bodu varu roztoku (ebulioskopie)


Zvýšení bodu varu roztoku (ebulioskopie) popisuje 2. Raoultův zákon. Slovní formulace je následující:

Zvýšení bodu varu roztoku ve srovnání s čistým rozpouštědlem je přímo úměrné koncentraci rozpuštěné
látky.

Matematicky je toto tvrzení formulováno následovně:

kde ΔT je změna teploty varu, c koncentrace rozpuštěné látky a konstanta úměrnosti Ke se nazývá ebulioskopická
konstanta. Ebulioskopická konstanta je přímo úměrná molární hmotnosti rozpouštědla, druhé mocnině teploty varu
čistého rozpouštědla a nepřímo úměrná molárnímu skupenskému teplu varu rozpouštědla, nejedná se tedy
o empiricky stanovenou konstantu a nezávisí na charakteru rozpuštěné látky.

Snížení bodu tuhnutí roztoku (kryoskopie)


Snížení bodu tuhnutí roztoku (kryoskopie) popisuje 3. Raoultův zákon. Slovní formulace je následující:

Snížení bodu tuhnutí roztoku ve srovnání s čistým rozpouštědlem je přímo úměrné koncentraci rozpuštěné
látky.

Matematicky je toto tvrzení formulováno následovně:

kde ΔT je změna teploty tuhnutí, c koncentrace rozpuštěné látky a konstanta úměrnosti Kk se nazývá kryoskopická
konstanta. Kryoskopická konstanta je přímo úměrná molární hmotnosti rozpouštědla, druhé mocnině teploty varu
čistého rozpouštědla a nepřímo úměrná molárnímu skupenskému teplu varu rozpouštědla, nejedná se o empiricky
stanovenou konstantu a nezávisí na charakteru rozpuštěné látky.

Osmotický tlak
Podrobnější informace naleznete na stránce Osmotický tlak.

Osmotický tlak se od předešlých koligativních vlastností liší, projevuje se teprve v případě specifického uspořádání.
Tím specifickým uspořádáním je myšlena přítomnost membrány propustné pouze pro rozpouštědlo (tzv.
semipermeabilní membrána), která od sebe odděluje dvě prostředí o různé koncentraci rozpuštěných látek.
Rozpouštědlo pak bude mít tendenci přecházet z prostředí s nižší koncentrací rozpuštěné látky do prostředí s vyšší
koncentrací rozpuštěné látky.

Představme si, že semipermeabilní membrána od sebe odděluje prostředí s koncentrací rozpuštěné látky c od
prostředí obsahující čisté rozpouštědlo. Rozpouštědlo bude mít tendenci přecházet do roztoku obsahujícím
rozpuštěnou látku. Proti této tendenci můžeme zasáhnout tak, že zvýšíme tlak pouze v prostředí, do kterého přechází
rozpouštědlo. Jako osmotický tlak Π pak definujeme takovou hodnotu přetlaku v prostředí, která bude právě
postačovat k tomu, aby se zastavil přechod rozpouštědla přes semipermeabilní membránu. Hodnota osmotického
tlaku je přímo úměrná teplotě a koncentraci rozpuštěné látky:

kde konstanta úměrnosti R je univerzální plynová konstanta 8,314 J·K-1·mol-1.

Odkazy
Literatura
HRAZDIRA, Ivo a Vojtěch MORNSTEIN. Lékařská biofyzika a přístrojová technika. 1. vydání. Brno : Neptun, 
2001. 396 s. ISBN 80-902896-1-4.

MALIJEVSKÁ, Ivona, Anatol MALIJEVSKÝ a Josef NOVÁK. Záhady, klíče, zajímavosti očima fyzikální chemie. 
1. vydání. Praha : Vydavatelství VŠCHT, 2004. 264 s. ISBN 80-7080-535-8.

NAVRÁTIL, Leoš a Jozef ROSINA, et al. Lékařská biofyzika. 1. vydání. Manus, 2001 (1. dotisk). 357 s. ISBN 80-
902318-5-3.

Související články

Osmotický tlak
Raoultovy zákony

Externí odkazy
článek Colligative properties na anglické Wikipedii
článek Osmotic pressure na anglické Wikipedii
článek Raoult's law na anglické Wikipedii
Raoultovy zákony

První Raoultův zákon

Δp = rozdíl tlaků, p0 = původní tlak, n2 = počet částic rozpuštěné látky, n1 = počet částic rozpouštědla.

Druhý Raoultův zákon


Rozpuštěním látky v rozpouštědle se zvýší jeho bod varu.

Ke = ebulioskopická konstanta rozpouštědla, cm = koncentrace rozpuštěné látky.

Třetí Raoultův zákon


Rozpuštěním látky v rozpouštědle dojde ke snížení jeho bodu tuhnutí.

Kr je kryoskopická konstanta rozpouštědla.

Odkazy
Externí odkazy

Raoultův zákon

Zdroj
KUBATOVA, Senta. Biofot [online]. [cit. 2011-01-31]. <https://uloz.to/!CM6zAi6z/biofot-doc>.
Osmotický tlak

Osmotický tlak π můžeme definovat jako přetlak, který by musel působit na roztok oddělený od čistého rozpouštědla
semipermeabilní membránou, aby se hladiny vyrovnaly.

Osmotický tlak lze vypočítat ze vzorce π = i • c • R • T

kde c je molární koncentrace


R molární plynová konstanta
T termodynamická teplota
i vyjadřuje počet osmoticky účinných částic (pro silné elektrolyty; u slabých elektrolytů je i rovno 1)

Jednotkou osmotického tlaku je Pascal.

Osmóza
Osmóza je děj, který je charakterizován průnikem rozpouštědla (nejčastěji vody) přes semipermeabilní membránu
buňky, přičemž extracelulární a intracelulární prostředí buňky není v rovnovážném stavu z hlediska chemické
koncentrace látek rozpuštěných v rozpouštědle.

Podrobnější informace naleznete na stránce Osmóza.

Buněčná membrána

je semipermeabilní, zajišťuje zprostředkovávání látek potřebných pro buňku – metabolicky / informačně

Podrobnější informace naleznete na stránce Buněčná membrána.

Historie

1855 – Nägeli – objev existence membrány, která chrání buňku a popsání její semipermeability
chemické složení popsáno dříve než ultrastruktura – pomocí izolovaní buněčných membrán hemolýzou erytrocytů
1925 – Gartner a Grendel – buněčná membrána tvořena z bimolekulární vrstvy lipidů s vysokým obsahem fosfolipidů - hydrofilní a
hydrofobní části fosfolipidů, proteinové molekuly – vaznost na hydrofilní části těchto molekul
1935 – Dawson a Danielli – model buněčné membrány
50’s – Robertson – zabýval se ultrastrukturou membrán – zjistil, že se liší jednotlivé membrány (buněčná membrána, biomembrány organel,
karyolema) hlavně v tloušťce

tenčí a méně kompaktní membrány – tvořené fosfolipidy s nenasycenými vazbami v molekule fosfolipidu
ultrastruktura je u všech v zásadě stejná
transmisní elektronový mikroskop – membrány se jeví jako trojvrstevné (trojité / dvojitě konturované)
domníval se, že densní vrstvy jsou proteinové a střední vrstva lipidová
membrány buňky nazval – unit membranes - oddělují jednotlivé kompartmenta

1972 – Singer a Nicholson – nový model buněčné membrány

model tekuté mozaiky neboli dvojrozměrné kapaliny


složky membrány kódované genomem → ale výsledná podoba daná matricovým způsobem → množení membrán dle existující membrány
(maternální původ)

Stavba buněčné membrány

Základem je bimolekulární vrstva fosfolipidů:

vnější list – hraničí s extracelulárním prostorem

obsahuje molekuly fosfatidylcholinu nebo lecitinu a sfingomyelinu nebo sfingolecitinu

vnitřní list – hraničí s intracelulárním prostorem (k cytoplazmě)

obsahuje molekuly fosfatidyletanolaminu neboli kefalinu + fosfatidilinositolu + fosfatidilserinu neboli


serinkefalinu

- může dojít k rotaci fosfolipidů – v rámci listu / k překlopení fosfolipidu z jednoho listu na druhý (účinkem enzymu
skramblázy)

Molekuly fosfolipidů - odvozeny od triacylglycerolů, v buněčné membráně jsou jejich dlouhé apolární hydrofobní
řetězce do středu buněčné membrány a jejich hydrofilní části tvoří povrchy

Molekuly cholesterolu - vmezeřeny mezi fosfolipidy, kumulují se spolu s transmembránovými úseky molekul
proteinu nebo glykolipidů, tím omezují jejich laterální difuzi (tj. plutí listem membrány) vytvářejí lipidové rafty –
funkční mikrodomény
Molekuly glykolipidů – zejména ve vnějším listu

Proteiny – cca 50 % hmotnosti membrány (zastoupení se liší v jednotlivých membránách)

→ Důsledek – asymetrie buněčné membrány

Pohyb látek v organismu


absorpce
distribuce
metabolizmu
exkrece

Přechod látek přes buněčné membrány

Aktivní transport Pasivní transport


transport pomocí nosičů prostá a usnadněná difúze
pinocytóza prostup membránovými póry
osmóza
filtrace
ultrafiltrace

Zařazení osmotického děje z hlediska jeho funkčního využití v medicíně

Hypotonický roztok.

Hypertonický roztok.

Isotonický roztok.

Celkové

1. Intravaskulární – podávání celé dávky léku přímo do krevního oběhu, nejrychlejší účinek.

intravenózně – do žíly
intraarteriálně – do arterie
intrakardiálně – do srdce

1. Extravaskulární - podávání látek do tkáně; látky se vstřebávají do krevního oběhu v místě aplikace.

intramuskulární – nitrosvalové
sublinguální – podjazykové
intradermální – do kůže
subkutánní – podkožní
perorální – ústní
rektální – do konečníku
intraperitoneální – do dutiny břišní
inhalační – vdechováním

Lokální

Látka se podává lokálně na povrchu kůže, sliznice nebo tělesné dutiny, kde dochází k jejímu vstřebání, vzhledem k její
chemické a fyzikální podstatě, čehož následkem je lokální účinek dané látky.

Průběh osmózy z hlediska variability prostředí

Hypotonické prostředí:

extracelulární prostředí – menší koncentrace rozpuštěné látky v rozpouštědle


intracelulární prostředí – větší koncentrace rozpuštěné látky v rozpouštědle

Hypertonické prostředí:

extracelulární prostředí – větší koncentrace rozpuštěné látky v rozpouštědle


intracelulární prostředí – menší koncentrace rozpuštěné látky v rozpouštědle

Isotonické prostředí:

extracelulární / intracelulární prostředí – stejná koncentrace rozpuštěných látek

Osmotický tlak
Osmotický tlak π můžeme definovat jako přetlak, který by musel působit na roztok oddělený od čistého rozpouštědla
semipermeabilní membránou, aby se hladiny vyrovnaly.

Osmotický tlak lze vypočítat ze vzorce π = i • c • R • T

kde c je molární koncentrace


R molární plynová konstanta
T termodynamická teplota
i vyjadřuje počet osmoticky účinných částic (pro silné elektrolyty; u slabých elektrolytů je i rovno 1)

Jednotkou osmotického tlaku je Pascal.

Rozdílný osmotický tlak

Chování erytrocytů v
hypertonickém, isotonickém a
hypotonickém prostředí.

Rozdíl osmotického tlaku v intracelulárním a extracelulárním prostoru je v živých organismech spouštěčem osmózy.
Pokud bude osmotický tlak vyšší v okolním prostředí než v buňce, začne voda prostupovat přes semipermeabilní
plazmatickou membránu ven z buňky. Hovoříme o tzv. plazmolýze.

Opačným případem je plazmoptýza. Ta nastává, pokud je v buňce vyšší osmotický tlak než v jejím okolí. Do buňky se
začne přes její cytoplazmatickou membránu dostávat voda, což může vést až k prasknutí buňky. U erytrocytů se tento
děj nazývá hemolýza a můžete ji vidět na přiloženém obrázku zcela vpravo.

U živých organismů by mělo být extracelulární a intracelulární prostředí isotonické, jinak jsou buňky poškozovány
nadměrným přísunem nebo úbytkem vody.

Onkotický tlak
Prostup látek semipermeabilní
membránou.

Onkotický tlak neboli koloidně-osmotický tlak je osmotický tlak vyvolanými roztoky s obsahem částic s velkou
molekulovou hmotností (např. proteiny).

Osmolarita a osmolalita
Pro snazší porovnávání koncentrací rozpuštěných částic v praxi zavádíme pojmy osmolarita a osmolalita.

Osmolaritu definujeme jako celkovou látkovou koncentraci osmoticky aktivních částic v mol/l (vztahuje se k objemu
roztoku).

Osmolalita je pak celková látková koncentrace osmoticky aktivních částic v mol/kg (vztahuje se k hmotnosti roztoku).
Krevní plazma má osmolalitu přibližně 300 mmol/kg a s touto hodnotou srovnáváme osmolalitu jiných roztoků
nejčastěji.

Shrnutí a souvislosti mezi osmózou a osmotickým tlakem


Osmóza

Rozpouštědlo má tendenci pronikat přes polopropustné membrány do míst, kde je koncentrace osmoticky aktivních
látek vyšší, a ředit je.

Pokud je koncentrace solutu uvnitř buňky vyšší než v okolí buňky, bude voda osmoticky vstupovat dovnitř a
buňka nabobtná.

Způsoby, jak se buňka vyhýbá zbobtnání:

1. živočišná buňka udržuje nízkou vnitřní koncentraci solutu aktivním odčerpáváním iontů
2. rostlinná buňka je před nabobtnáním a prasknutím chráněna tuhou stěnou
3. prvok periodicky vystřikuje vodu nahromaděnou v buňce

Při dostatečně velkém rozdílu koncentrace rozpuštěné látky buňka praskne.

Ve výsledku jsou tedy roztoky na obou stranách membrány stejně koncentrované. Osmotický tlak je jedna ze
základních sil, které ovlivňují živé buňky, protože cytoplazmatická membrána je polopropustná.

Osmotický tlak

Tlak toku rozpouštědla pronikajícího přes semipermeabilní (polopropustnou) membránu do roztoku, ve kterém je vyšší
koncentrace rozpuštěných molekul nebo iontů.

π = i • c • R • T → závisí na teplotě a koncentraci roztoku

i je rovno počtu osmoticky účinných částic, c je molární koncentrace, R molární plynová konstanta, T absolutní
teplota

Tok rozpouštědla membránou: J = k • S • (π1- π2)

π1 a π2 jsou osmotické tlaky roztoků oddělených membránou, k koeficient propustnosti

Odkazy
Související články

Osmóza
Osmolarita
Osmolalita
Použitá literatura

NAVRÁTIL, Leoš a Jozef ROSINA, et al. Medicínská biofyzika. 1 (dotisk 2013) vydání. Praha : Grada Publishing, 
2005. 524 s. ISBN 978-80-247-1152-2.

VAJNER, Luděk, et al. Lékařská histologie I : Obecná cytologie. 1. vydání. 2010. 0 s. ISBN 978-80-246-1860-9.

VOKURKA, Martin a Jan HUGO, et al. Velký lékařský slovník. 9. vydání. Praha : Maxdorf, 2009. 1159 s. ISBN
978-80-7345-202-5.

Osmotický tlak. [online]. [s.a.]. [Cit.: 2013-11-23]. Dostupné na: http://147.33.74.135/knihy/uid_es-


001/hesla/osmoticky_tlak.html
Velký lékařský slovník. [online]. [s.a.]. [Cit.: 2013-11-23]. Dostupné na: http://lekarske.slovniky.cz/
Jevy na rozhraní fází
Povrchové napětí

Povrchové napětí

Uvnitř kapaliny jsou všechny molekuly obklopeny stejnými molekulami a ty na sebe navzájem ze všech stran stejně
působí soudržnými silami. Na rozhraní fází (tj. na hladině kapaliny) se kapalina snaží kvůli výslednici sil směřujících
dovnitř zaujmout co nejmenší povrch. Povrchové napětí je tečná síla k povrchu kapaliny, která s rostoucí teplotou
klesá. Její jednotkou je N/m. Lze ho také definovat jako energii nutnou ke zvětšení povrchu kapaliny o 1m2. Látky
rozpuštěné v kapalině, které snižují povrchové napětí kapalin, se nazývají povrchově aktivní. Povrchové napětí má
význam např. při dýchání (vlhké stěny alveolů).

Adsorpce
Na fázových rozhraních je vždy větší koncentrace látek rozpuštěných v kapalině, protože se tak snižuje její povrchové
napětí -> tento děj se nazývá adsorpce. Proti tomuto ději působí difúze a výsledkem je ustanovení tzv. adsorpční
rovnováhy vyjádřené Gibbsovou adsorpční rovnicí (izotermou).

Odkazy
Zdroj

KUBATOVA, Senta. Biofot [online]. [cit. 2011-01-31]. <https://uloz.to/!CM6zAi6z/biofot-doc>.

JIŘÍ, Beneš,, Jirák, DANIEL a Vítek, FRANTIŠEK. Základy lékařské fyziky. - vydání. Charles University in Prague,
Karolinum Press, 2015. 322 s. ISBN 9788024626451.
Vratné a nevratné děje
Průběh termodynamického procesu může být buď vratný nebo nevratný (reverzibilní × ireverzibilní).

Proběhne-li v dané soustavě rovnovážný děj v jednom směru a pak ve směru opačném a soustava se dostane zpět do
původního stavu, aniž by nastaly v okolních tělesech změny,. Takový proces nazýváme jako vratný děj. Skutečné děje
se ideálnímu vratnému pouze přibližují a to jen v případě, že probíhají dostatečně pomalu.

Všechny skutečné děje jsou nevratné. V jednom směru probíhají samovolně bez vnějšího působení a k tomu, aby děj
probíhal v opačném směru, je nutno dodat energii z vnějšího zdroje.

Odkazy
Zdroj

KUBATOVA, Senta. Biofot [online]. [cit. 2011-01-31]. <https://uloz.to/!CM6zAi6z/biofot-doc>.

Článek neobsahuje vše, co by měl.


Můžete se přidat k jeho autorům a jej.
O vhodných změnách se lze poradit v diskusi.
Práce a teplo
Mechanická práce je mírou předávání mechanické energie z jednoho tělesa na druhé.
Tepelné množství ΔQ = množství energie tepelného pohybu molekul předávaného z jednoho tělesa na druhé
výměnou tepla.

m = hmotnost, C = měrné teplo, Δ T = změna teploty

Teplo systému = součet kinetických energií jeho neustále se pohybujících částic


Práce = makrofyzikální forma předávání energie ze systému práci konajícího do systému, který ji získává

Práce se může spotřebovat na zvýšení obsahu jakékoliv energie, zatímco pokud teplo nepřevedeme na práci, může
pouze zvýšit vnitřní energii systému.

Proto nelze tyto dvě veličiny zaměňovat, i když obě vyjadřujeme ve stejných jednotkách (J).

Odkazy
Zdroj
KUBATOVA, Senta. Biofot [online]. [cit. 2011-01-31]. <https://uloz.to/!CM6zAi6z/biofot-doc>.

Článek neobsahuje vše, co by měl.


Můžete se přidat k jeho autorům a jej.
O vhodných změnách se lze poradit v diskusi.
Měrná tepelná kapacita

Měrná tepelná kapacita je množství tepla potřebné k ohřátí 1 kg látky o 1 K (při vztažení na 1mol látky = molární
teplo).

Odkazy
Zdroj

KUBATOVA, Senta. Biofot [online]. [cit. 2011-01-31]. <https://uloz.to/!CM6zAi6z/biofot-doc>.

Článek neobsahuje vše, co by měl.


Můžete se přidat k jeho autorům a jej.
O vhodných změnách se lze poradit v diskusi.
Transport tepla
Teplo se prostředím šíří:

1. vedením (kondukcí);
2. prouděním (konvekcí);
3. tepelným zářením.

Vždy se šíří z místa o vyšší teplotě do místa o teplotě nižší.

Tepelná energie se může šířit pouze v prostředí vyplněném látkou (kromě záření, které má elmag. povahu), protože
příčinou šíření tepla je neustálý pohyb částic hmoty, které si vzájemnými srážkami předávají kinetickou energii,
dokud nedojde k vyrovnání teplot.

Odkazy
Související články

Termoregulace
Tepelná pohoda organismu
Tepelné ztráty organismu

Zdroj

KUBATOVA, Senta. Biofot [online]. [cit. 2011-01-31]. <https://uloz.to/1162346/biofot.doc>.


Entalpie

Entalpie (zastarale "tepelný obsah") je veličina vyjadřující tepelnou energii uloženou v dané látce. Jedná se o
stavovou funkci termodynamické soustavy. Je určená vztahem pro soustavu s neměnným počtem částic, s vnitřní
energií , tlakem a s objemem
soustavy. Matematicky je definována:

Motivace
Vychází z 1. termodynamického zákona: systém může konat práci jen tehdy, poklesne-li jeho vnitřní energie
nebo je-li mu dodáno teplo :

V této znaménkové konvenci je práce kladná, koná-li ji systém na okolí (např. expanze
plynu).

Zvyšuje-li systém svůj objem proti vnějšímu tlaku


, koná při tom mechanickou práci:

Vzhledem k předchozí rovnici můžeme psát, že

Proto byla definována funkce (entalpie, tepelný obsah):

Jednotkou entalpie v systému SI je joule (J).

Je možné také zavést měrnou entalpii vztahem

kde značí hmotnost. Jednotkou měrné entalpie je poté v systému SI joule na kilogram
látky (J·kg-1).

Vztah k vnitřní energii


Vnitřní energie je vhodná pro vyjadřováni energetických změn dějů probíhajících za
stálého objemu, entalpie se s výhodou používá pro výpočet energetických změn za stálého tlaku. Jelikož většina
chemických procesů probíhá za stálého tlaku, je entalpie v chemické termodynamice častěji používanou veličinou než
vnitřní energie.

Pakliže bychom znali hodnoty entalpií reagujících složek v chemické reakci, byl by výpočet jednoduchý. Problém však
nastává v tom, že nedovedeme určit absolutní hodnoty entalpií. Vždy se totiž zjišťuje jen změna této funkce, ke které
dochází, když látka přechází z jednoho stavu do druhého (z výchozího do konečného). Proto se zavádí v chemické
termodynamice stupnice relativních hodnot entalpií. Nulová hodnota entalpie je entalpie prvků v základním stavu, v
jejich stabilní formě – při tlaku 101,325 kPa a při teplotě 298,15 K (25 °C). Pouze v tomto stavu lze jejich entalpie
změřit.

Význam entalpie
Zavedením entalpie se značně zjednoduší termodynamické vztahy pro izobarické děje. Jestliže soustava přijímá teplo a
koná objemovou práci, bude rovnice pro výpočet změny vnitřní energie:

v diferenciálním tvaru:

Jde-li o práci reverzibilní, pak označuje tlak soustavy. Je-li tento tlak konstantní (při
izobarickém ději), můžeme rovnici napsat ve tvaru:

Pro izobarický děj tedy platí, že teplo přijaté soustavou se rovná zvýšení entalpie soustavy.
Její význam je také při posuzování procesů v tepelných strojích, ve kterých je látka přiváděna a odváděna při stálém
tlaku. Jestliže látka koná ve stroji při expanzi práci bez přívodu a odvodu tepla, je tato práce při jednom cyklu úměrná
rozdílu entalpie látky (počáteční a konečný stav).

Reakční teplo
Reakční teplo je množství tepla, které soustava s okolím vymění při reakci.
Reakce probíhá za konstantního tlaku v rozsahu 1 molu základních reakčních přeměn. Jak již víme, při izobarických
dějích vyjadřuje výměnu tepla mezi soustavou a okolím právě entalpie.

značí entalpi na začátku (reaktanty), značí


entalpi na konci (produkty).

jedná se o endotermickou reakci, soustava přijmá teplo z okolí


jedná se o exotermickou reakci, soustava odovzdává teplo do okolí

Grafické vyjádření :

Rozdělení
Reakční teplo můžeme rozdělit dle toho, zda při reakci vznikne z prvků sloučenina, nebo zdali se sloučenina spálí v
nadbytku kyslíku. Výpočet reakčních tepel je umožněn díky 2. termodynamickému zákonu.

Standardní slučovací teplo

[ kJ.mol-1]. Je reakčním teplem reakce, při kterém vznikne 1 mol


sloučeniny z prvků. Podmínkou vzniku je standardní stav prvků i produktů, to je teplota 298,15 K, tlak 101,325 kPa.
Standardní slučovací tepla prvků jsou vždy nulová.

Standardní spalné teplo

[kJ.mol−1]. Je reakčním teplem reakce, při kterém je 1 mol látky spálen.


Opět platí, že prvky a produkty musí být ve standardním stavu. Spalná tepla prvků jsou nenulová.

Odkazy
Související články

Gibbsova funkce

Externí odkazy
Entalpie (česká wikipedie)
Enthalpy (anglická wikipedie)

Zdroj

KUBATOVA, Senta. Biofot [online]. [cit. 2011-01-31]. <https://uloz.to/!CM6zAi6z/biofot-doc>.


MECHLOVÁ, Erika a Karel KOŠŤÁL. Výkladový slovník fyziky. 1. vydání. Praha : Prometheus, 1999. 
588 s. ISBN 80-719-6151-5.

FISHER, Oldřich, et al. Fyzikální chemie. 1. vydání. Praha : SPN, 1984. 

ČELEDA, Jiří, et al. Kurs základů chemie. 1. vydání. Praha : SPN, 1968. 

MAREČEK, Aleš a Jaroslav HONZA. Chemie pro čtyřletá gymnázia. 1. díl. 3. vydání. Brno : Aleš Mareček, 
2013. ISBN 80-7182-055-5.
Entropie

Charakteristika
I. termodynamická věta kvantitativně popisuje možnost přeměny tepla v práci, avšak nedefinuje podmínky této
přeměny. Je zřejmé, že není možné trvale získávat práci (kruhovými ději) pomocí tepla odebíraného jednomu
tepelnému zásobníku o určité teplotě, aniž bychom část tepla neodevzdali druhému zásobníku o teplotě nižší. Vztah
pro účinnost η tepelného stroje, který kruhovým reverzibilním dějem odebírá teplo Q1 z teplejšího zásobníku o teplotě
T1 a předává jeho část Q2 zásobníku chladnějšímu o teplotě T2<T1, přičemž koná práci, odvodil S. Carnot jako η =
(Q1 − Q2) / Q1 = (T1 − T2) / T1. Tento vztah určuje účinnost všech možných reverzibilních dějů pracujících při
daném teplotním rozdílu. Účinnost tepelného stroje je tedy vždy menší než 1. Nejznámější slovní formulace II.
termodynamické věty, tzv. Kelvinova-Planckova formulace, říká: Nelze sestrojit cyklický tepelný stroj tak, aby v
průběhu celého cyklu pouze odebral teplejší lázni teplo a veškeré je změnil v práci.

Část tepla, která přešla do chladnějšího zásobníku, snížila svou schopnost se měnit v práci. Toto teplo může být
převedeno v práci pouze použitím dalšího chladnějšího zásobníku atd. Určité množství tepla je tedy při vyšší teplotě z
hlediska možnosti přeměny v práci cennější než totéž množství při teplotě nižší. V souvislosti s převodem tepla ze
systému o vyšší teplotě do systému o nižší teplotě hovoříme o tzv. degradaci energie.

Termodynamická entropie
Termodynamická funkce charakterizující míru degradace energie je entropie. Entropie S je extenzivní stavová
veličina. Při vratných dějích je malá změna entropie určena vztahem:

kde ΔS je změna entropie, Q je množství tepla dodané systému a T absolutní teplota systému. Ačkoli teplo závisí
(podobně jako práce) na ději, (integrační) cestě ze stavu počátečního do stavu koncového, entropie je stavovou
veličinou (podobně jako vnitřní energie) a její změna nezávisí na ději, jakým soustava prošla, jen na počátečním a
koncovém stavu. Jednotkou entropie v SI je J. K-1.

Při adiabatickém vratném ději, kdy nedochází k výměně tepla s okolím, bude tedy ΔS = 0. Při libovolném kruhovém
ději, kdy se systém dostane do původního stavu, bude výsledná změna entropie také nulová. Při nevratných procesech
v izolovaných systémech je vždy ΔS ≥ 0, což je vlastně matematické vyjádření 2. termodynamické věty: Při
samovolném nevratném ději entropie systému roste. To osvětluje její název - z řeckého ἐν τροπή (v přeměně) - udává
směr, kterým probíhají termodynamické děje samovolně. Například při (nevratné) adiabatické expanzi plynu do vakua
jeho entropie vzroste. Z toho vyplývá, že reverzibilní procesy jsou určitou hranicí procesů, které mohou v přírodě
probíhat.

Výpočet entropie přes integrál

Termodynamickou entropii lze také vypočítat pomocí integrálu:

Entropie v praktickém životě


Entropii lze chápat jako míru neuspořádanosti systému. To umožňuje její Boltzmannova mikroskopická definice:

kde k=1,38.10-23 J/K je Boltzmannova konstanta a W tzv. fyzikální pravděpodobnost stavu soustavy, což je počet
mikroskopických možností, jak tento makrostav uskutečnit. Čím je tedy entropie vyšší, tím více bude systém
neuspořádán, bude se jevit jako chaotický a naopak. V každodenním životě se s tímto pojmem téměř nesetkáme a
přece je jeho nedílnou součástí.

Mějme pytel se stejnými modrými a červenými kuličkami. Je vysoce pravděpodobné, že kuličky v


tomto pytli nebudou pravidelně uspořádány podle barvy – systém bude mít vysokou entropii.
Tento stav však můžeme našim zásahem a vykonáním jisté práce změnit a kuličky uspořádat, tím
získáme systém s nízkou entropií.

Entropie je, ku příkladu, zřetelná i ve složení krevních elementů. Když se zaměříme na průtok
krevních partikulí cévou, zjistíme že rozmístění jednotlivých buněk v plazmě je zcela náhodné,
pomineme-li biochemické reakce a potřeby organismu. To má zásadní vliv při hemostáze. Kdyby
měl systém nízkou entropii, mohlo by se stát, že by v místě poranění cévy nebyl zrovna přítomný
dostatek krevních destiček.

Řadu podobných příkladů najdeme v mnoha dalších oblastech všedního života.


Odkazy
Související články

Entalpie
Entropie (česká wikipedie)
Entropy (anglická wikipedie)

Zdroj

HRAZDIRA, Ivo, et al. Biofyzika Učebnice pro lékařské fakulty. 1. vydání. Praha : Avicenum, 1990. 318 s. ISBN
80-201-0046-6.

ANONYM,. Druhý termodynamický zákon [online]. [cit. 2017-05-02]. 


<https://physics.mff.cuni.cz/kfpp/skripta/kurz_fyziky_pro_DS/display.php/molekul/4_3>.
Volná energie

Volná energie neboli Helmholtzova funkce je definována vztahem

Celková vnitřní energie se tak skládá z volné energie


(kterou lze při izotermickém reverzibilním ději přeměnit v práci) a vázané energie
, která je izotermicky neužitečná.

Související články
Gibbsova energie

Odkazy
Zdroj

KUBATOVA, Senta. Biofot [online]. [cit. 2011-01-31]. <https://uloz.to/!CM6zAi6z/biofot-doc>.

Externí odkazy

MFF UK - Kurzy fyziky: Volná energie a volná entalpie


Chemický potenciál
Chemický potenciál je termodynamická veličina, která vyjadřuje změnu energie spojenou se změnou složení
chemické soustavy.

Při konstantním tlaku a teplotě lze chemický potenciál -té složky definovat následovně:

kde vyjadřuje Gibbsovu energii soustavy a počet


molů (molekul) -té složky.

Chemický potenciál vyjadřuje tendenci látky vstupovat do chemických reakcí (jde tedy o míru tzv. afinity dané látky).
O tom, jaké reakce budou v systému probíhat (a jak rychle), rozhoduje nejenom chemický potenciál, ale i množství
látek v systému. Pomocí chemického potenciálu lze také definovat aktivitu složky v soustavě.

Odkazy
Související články
Potenciál
Gibbsova volná energie

Zdroj

KUBATOVA, Senta. Biofot [online]. [cit. 2011-01-31]. <https://uloz.to/!CM6zAi6z/biofot-doc>.


Měření teploty

Termometrií se určuje teplota – objektivní míra tepelného stavu látky. Dle SI je základní veličinou
termodynamická teplota, jejíž jednotkou je Kelvin.

K měření teploty se používaly či používají následující teplotní stupnice:

termodynamická teplotní stupnice;


Celsiova teplotní stupnice;
Fahrenheitova teplotní stupnice;
Réaumurova teplotní stupnice;
Rankineho teplotní stupnice.

Centrální tělesnou teplotu (TT) měříme intrakavitálně, nejčastěji per rectum.

Periferní TT měříme nejčastěji na dorsu nohy a dále hodnotíme teplotní diferenci.

Rozdíl mezi centrální a periferní TT > 2 °C svědčí pro hypovolemii nebo zvýšenou α–mimetickou aktivitu, rozdíl >
8 °C svědčí pro šokovou cirkulaci, těžkou hypovolemii.

Indikací ke kontinuálnímu měření TT jsou:

intrakraniální hypertenze;
thiopentalové kóma;
pacienti s hemodynamickou nestabilitou;
pacienti s náročným ventilačním režimem;
pacienti s maligní hypertermií.

Způsob měření

Teplota se dá měřit pouze nepřímo na základě známých fyzikálních jevů za různých teplot. Termometrie se proto
provádí několika způsoby, založenými většinou na objemové roztažnosti kapalin nebo délkové roztažnosti
pevných látek za různé teploty. Nejpřesnější je intrakavitální měření, tj. rektálně, vaginálně, orálně
(aurikulární měření nelze považovat za přesné).

Z praktického hlediska naměření vyšší axilární teploty s vysokou pravděpodobností predikuje vyšší hodnotu rektální,
ale normální axilární teplota nevylučuje vyšší hodnotu rektální. Při pochybnostech o axilární teplotě je proto nutno
pacienta přeměřit rektálně.

Kapalinové teploměry

Rtuťové teploměry

Jedná se o nejrozšířenější používané teploměry vůbec. Bývají většinou rtuťové, i když jsou postupně kvůli toxicitě
rtuti vytlačovány. Skládají se z rtuťového rezervoáru s kapilárou a stupnice. S rostoucí teplotou rtuť mění svůj
objem a šplhá v kapiláře. Používají se dvě základní modifikace:

maximální teploměr – zaznamenává nejvyšší naměřenou hodnotu. Vlivem zúžení kapiláry nad rezervoárem
zůstává po použití rtuť na maximálním bodě a je nutno ji do rezervoáru „sklepat“. Doba ustálení konečné teploty
je několik minut.
rychloběžka – měří okamžitou teplotu a její hodnota se ustálí rychleji

Dalším druhem kapalinového teploměru je teploměr lihový, měřící v rozsahu od −110 °C do 70 °C.

Citlivost těchto teploměrů roste s objemem rezervoáru a menším poloměrem kapiláry. Tyto teploměry měří s přesností
na desetiny stupně.

Bezdotykové teploměry
Každé těleso o určité teplotě podle Stefan-Boltzmannova zákona vyzařuje tepelné záření. Infračervený bezdotykový
teploměr' někdy též pyrometr je založen na principu měření množství takto vyzářené energie v infračerveném
spektru. Protože Stefan-Boltzmannův zákon platí pro černá tělesa, zavádí se pro reálná tělesa veličina
emisivita. Emisivita je poměr mezi vyzařováním reálného tělesa a černého tělesa při stejné teplotě. U
reálných těles je navíc nutno přihlédnout k jejich průhlednosti a odrazivosti'. U průhledných materiálů se
přesnosti dosahuje spektrální filtrací, např. sklo se chová neprůhledně pro vlnovou délku 5 µm. U odrazivých
materiálů dochází k měření nejen vlastního, ale i odraženého záření.

Konstrukce se skládá z optické soustavy (čoček, optických vláken, spektrálních filtrů), která určuje průměr měřené
oblasti v předepsané vzdálenosti. Pro zaměření měřené oblasti se používají světelné nebo laserové zaměřovače.
Paprsky jsou soustředěny do detektoru, kterým bývá fotovoltaický článek nebo fotorezistor. Vzniká tak elektrický
signál, který je zesílen a různě zobrazen.

Výhodou je bezkontaktnost, možnost měření na větší vzdálenosti, rychlá odpověď a zanedbatelné ovlivnění
měřeného objektu.

Kovové odporové teploměry

Tento teploměr je založený na změně elektrického odporu kovu se změnou teploty. Tato závislost může být vyjádřena
vztahem:

Rt = R0·(1 + α·t),

kde Rt je odpor při teplotě t, R0 je odpor při nulové teplotě a α (K-1) je teplotní součinitel odporu

Výhodou tohoto typu teploměru je linearita měření ve velkém rozsahu teplot a lehké vyhodnocení.
Nejpoužívanější je platinový teploměr, který změří teplotu v rozsahu od -100 °C do 440 °C, přičemž měří s přesností
na tisíciny stupňů. Tento typ teploměru je používaný nejmíň v technické praxi a často bývá součástí složitějších
měřících zařízení (např. jako snímač na měření referenční teploty termočlánkových sond).

Termočlánek

Termoelektrické články měří teplotu na základě termoelektrického jevu. Ten funguje na principu, že v uzavřeném
elektrickém obvodu dvou vodičů z různých kovů, kdy má každý různou teplotu, teče elektrický proud. Pokud tento
obvod rozpojíme, jsme schopni měřit hodnoty termonapětí, které jsou dány rozdílem teploty mezi spoji. Pro praktické
užití termočlánku se jeden vodič dá do prostředí s referenční teplotou (v praxi teplota místnosti, cca 25 °C) a druhý
se vloží do prostředí, kde chceme teplotu změřit. Voltmetrem se poté měří hodnota termonapětí mezi spoji, s přesností
na setiny stupňů Celsia.

Zmíněné termonapětí je kvadratickou funkcí teploty. V případě jeho využití v medicíně, kde je rozsah teplot jen od
20 °C do 50 °C, můžeme danou lineární závislost vyjádřit pomocí vztahu:

UAB = k·(tA – tB)

kde UAB je termonapětí mezi referenčním a měřícím bodem termočlánku, tA a tB jsou teploty těchto bodů; k je
kalibrační konstanta závisící na typu termočlánku. V medicínských aplikacích jsou nejpoužívanější hlavně termočlánky
měď-konstantan (kalibrační konstanta 40 μV/K) nebo mangan-konstantan. Přesnost měření teploty pomocí
termočlánku závisí na citlivosti voltmetru měřícího termonapětí a na přesnosti měření teploty studeného konce
termočlánku.

Výhodou termočlánků je miniaturizace, proto se v lékařství používají jako invazivní měřiče teploty, např. pro účely
hypertermie.

Termistor

Měření teploty termistorem je založeno na měření elektrického odporu, kdy s rostoucí teplotou hustota volných
elektronů v polovodiči prudce stoupá. Tím klesá elektrický odpor.

Daná závislost může být vyjádřena rovnicí:

T = B·(ln R − lnA)

kde B a A jsou materiálové konstanty, R je odpor a T je absolutní teplota polovodiče.

Měření je velmi přesné, řádově v mK. Čidla pro termistory jsou většinou invazivní jehly, kdy samotný termistor je ve
špičce této jehly.

Fosforová termometrie

Fosforová termometrie slouží k měření teploty povrchů pokrytých vrstvou fosforu. Fosfor po excitaci díky
luminiscenci emituje světlo. Jeho jasnost, barva a doba doznívání jsou závislé na teplotě. Nejčastěji se využívá
závislost doby doznívání po excitaci na teplotě. Využívá se ve vláknových invazivních termometrech.
LC teploměry

Jednorázový teplotní proužek

LC (liquid crystal) teploměry fungují na bázi tekutých krystalů, které mění barvu v závislosti na teplotě. Rozlišovací
schopnost může být až 0,1° C. V podobě jednorázových teplotních proužků je lze využít k měření tělesné teploty na
čele.

Odkazy
Související články
Článek o měření teploty v Katalogu metod v biofyzice
Sledování fyziologických funkcí
Termografie
Měření a hodnocení tělesné teploty
Chyby měření fyzikálních veličin, relativní chyba

Externí odkazy

Phosphor thermometry (anglická wikipedie)


Liquid crystal thermometer (anglická wikipedie)

Zdroj

KUBATOVA, Senta. Biofot [online]. [cit. 2011-01-31]. <https://uloz.to/!CM6zAi6z/biofot-doc>.

NAVRÁTIL, Leoš a Jozef ROSINA, et al. Medicínská biofyzika. 1. vydání. Praha : Grada, 2005. 524 s. s. 68-
72. ISBN 80-247-1152-4.

HAVRÁNEK, Jiří: Ostatní monitoring.

BARRON, W. R. Principles of Infrared Thermometry [online]. [cit. 2012-12-28]. 


<https://www.omega.com/temperature/Z/pdf/z059-062.pdf>.
Kalorimetrie
Kalorimetrie je experimentální vědní obor zabývající se měřením tepla. Zkoumá fyzikální, chemické a biologické
pochody, ale i statické systémy, na které nahlíží z termofyzikálního hlediska. V lékařství nachází kalorimetrie široké
uplatnění. Budeme-li vycházet z předpokladu, že veškerá energie je v tělě nakonec přeměněna v teplo, můžeme
kalorimetrickými hodnotami měřit energetické požadavky organismu a určovat energetické hodnoty potravin.

Jednotky
Původní jednotkou fyziologické kalorické hodnoty živin, popř. spalného tepla byla kalorie. V současné době je však
postupně nahrazována jouly. Kalorie (značka cal) udává množství energie zvyšující teplotu 1 g vody ze 14,5 na
15,5 °C. Přesná teplota je v definici uvedená proto, že měrná tepelná kapacita, ze které definice také vychází, závisí i
na teplotě, a bez tohoto údaje by jednotka nebyla určena přesně. Kalorie však představuje velmi malé množství
energie, proto jsou často používány její násobky, kilokalorie (značka kcal). Jelikož měrná tepelná kapacita vody je
4185 J·kg−1·K−1 platí, že 1 cal = 4,185 J (a naopak: 1 J = 0,2389 cal) Q=mcΔT

Kalorimetrické měřící přístroje


Nejzákladnější klasifikace rozlišuje kalorimetry podle podmínek, za kterých pracují na adiabatické a izotermní. V
adiabatických systémech je veškeré spalné teplo využito k ohřátí obsahu kalorimetru a měří se teplotní změna obsahu.
V izotermních zařízeních se teplota po celou dobu experimentu nemění. Vzniklé teplo je odváděno a v další fázi např.
působí fázovou přeměnu čisté látky.

Měření energetického obsahu potravin


Pro účely změření energetického obsahu nutričních substrátů (sacharidy, tuky, bílkoviny) je nejvhodnější adiabatický
bombový kalorimetr. Vzorek se umístí do hermeticky uzavřené tlakové nádobky, zvané kalorimetrická bomba, a
napojí se na něj zápalné drátky. Následně je výbušně zapálen. Okolo celé kalorimetrické bomby je nádržka naplněná
vodou. Voda je ohřívána a přídatný mixér distribuuje teplo rovnoměrně v prostoru. Účelem zkoumání je zjišťovat
velikost teplotního nárůstu vody, čili velikost spalného tepla daného vzorku.

V lidském těle se energie uvolněná z potravy ukládá jako chemická energie (nejčastěji v ATP, GTP) a v případě
potřeby je převedena na jiné formy energie. Z danných makroergických vazeb je energie uvolňována oxidací. Tento
proces je významný pro měření energetických hodnot potravin. Nezáleží na tom, zda je energie uvolňována postupně,
nebo proběhne rychlé spálení v některém z kalorimetrických měřících přístrojů. Musíme však brát v úvahu, že lidské
tělo není stoprocentně účinný stroj, a tudíž nedokáže využít všechno spalné teplo. Hodnoty spalných tepel
naměřených v kalorimetru a pravé fyziologické kalorické hodnoty (množství energie, které se v organismu skutečně
uvolní a využije) se proto nepatrně liší (viz tabulka).

Spalné teplo Fyziologická kalorická hodnota


Sacharidy 17,2 17,2
Tuky 39,1 38,9
Bílkoviny 23,4 17,2

Tyto běžně udávané fyziologické kalorické hodnoty jsou však pouze orientační a u jednotlivých druhů nutričních
substrátů se mohou lišit. Sacharidy jsou spalovány za vzniku oxidu uhličitého a vody. Energetický obsah jednotlivých
sloučenin závisí na struktuře dané látky (jednoduché sacharidy/ maltodextriny/ polysacharidy). Např. při spalování
glukózy se uvolňuje pouze 15,7 kJ/g. Oxidace tuků také vede ke vzniku oxidu uhličitého a vody, ale energetický
potenciál tuků je využíván obtížněji než u sacharidů, jelikož se jedná především o složitější molekuly, jejichž rozklad
není v organismu úplný. Pro přesné určení energetických hodnot musíme brát v úvahu rozdíly způsobené odlišnou
strukturou mastných kyselin. MK se středně dlouhým řetězcem (8 uhlíků) uvolňují okolo 36 kJ/g, zatímco MK s
dlouhým řetězcem mohou uvolnit až 40,2 kJ/g. Při spalování proteinů se z aminokyselin uvolňuje jestě dusík, který je
vylučován ledvinami jako součást močoviny. Spolu s atomy dusíku je z těla vyloučena i část vodíkatých iontů a další
látky. Energetický potenciál proteinů závisí na obsahu dusíku, jelikož dusík není energeticky využitelný. Jeho
koncentrace v jednotlivých proteinových substrátech potravin kolísá přibližně od 15 % do 19 %. Energetický potenciál
bílkovin ovlivňují také mnohé další faktory, např. podíl bílkovin na celkovém energetickém příjmu nebo fyzická
aktivita pacienta.

Měření energetického výdeje


Kalorimetricky měříme objektivní aktuální energetickou potřebu pacienta dvěma způsoby, přímou a nepřímou
kalorimetrií. Měření přímou kalorimetrií je u člověka prakticky téměř neproveditelné. Přímý kalorimetr funguje na
stejném principu jako jednoduchý adiabatický kalorimetr. Sledovaný živočich je umístěn do tepelně izolovaného
prostoru, kolem kterého je nádoba s chladícím médiem (nejčastěji voda). Měříme změnu teploty chladícího média a
zároveň snímáme příjem kyslíku, výdej oxidu uhličitého a vyloučení dusíku močí a stolicí. Kvůli technickým obtížím
přímé kalorimetrie je využívám zjednodušený způsob, čili kalorimetrie nepřímá. Základním principem této metody je
měření spotřeby nutričních substrátů a výměny plynů v daném čase a určení respiračního koeficientu.

Respirační koeficient
RQ = VCO2 / VO2

Respirační koeficient je poměr mezi vydaným CO2 a spotřebovaným O2. Jeho hodnoty závisejí mimo jiné na
proporciální oxidaci jednotlivých nutričních substrátů. Hlavně změny trendu hodnot při změnách složení výživy mohou
vést k interpretaci změn v utilizaci jednotlivých substrátů. RQ pro běžné jídlo se pohybuje okolo 0,85. Hodnoty RQ při
oxidaci základních nutričních komponent a RQ některých metabolických dějů jsou uvedeny v tabulce:

Substrát nebo metabolický děj RQ


sacharidy
1
glykogen
tuky 0,7
proteiny 0,9
glukogeneze 0,4
lipolýza 0,7
lipogeneze 2,75

Ze základních nutričních substrátů mají nejvyšší RQ sacharidy. Molekuly mastných kyselin tuků obsahují málo kyslíku
vůči celkovému uhlíku, a proto je na jejich oxidaci potřeba více kyslíku přijatého dýcháním. Z metabolických dějů
ovlivňují nejvýrazněji hodnoty RQ děje, které navozují lipogenezi. Proto hodnota RQ > 1 indikuje lipogenezi a
nadměrnou nabídku glukózy, hodnota < 0,7 je ukazatelem neschopnosti oxidovat glukózu a hladovění, tedy lipolýzu a
glukoneogenezi. V případě bílkovin je situace složitější, jelikož jejich katabolismus probíhá neúplným metabolismem.
Nejprve tedy zjišťujeme RQ pro množství metabolizovaných bílkovin, ale pro další zkoumání pracujeme s tzv.
nebílkovinným RQ. Korekce na nebílkovinný RQ spočívá v tom, že 1 gram dusíku v moči se rovná takovému množství
bílkovin, na jejichž oxidaci je třeba 5,92 l kyslíku a vznikne 4,75 l oxidu uhličitého.

Technické provedení kalorimetrie (teoreticky)


Nejprve měříme spotřebu O2 a produkci CO2 a zároveň snímáme podíl dusíku v moči a stolici. V nemocnici se takové
měření provádí 24 hodin, pro základní měření však postačí i 30 minut. Naměřené hodnoty uvolněného dusíku
použijeme pro výpočet množství spotřebovaného kyslíku na oxidaci proteinů (xO2 * 5,92) a množství uvolněného CO2
(xCO2 * 4,75). Zbývající množství celkové naměřené spotřeby O2 a vyloučeného CO2 připadá na oxidaci sacharidů a
tuků. V další fázi vypočteme nebílkovinný RQ podle vzorce RQ = VCO2 / VO2. Dané hodnotě RQ odpovídá vždy
tabulková hodnota pro množství sacharidů a tuků. Celkové množství využitých sacharidů a tuků se pak vypočítá
vynásobením tabulkových údajů množstvím kyslíku potřebného na oxidaci sacharidů a tuků. Současně lze z
vyloučeného dusíku vypočítat množství metabolizovaných bílkovin. Vynásobíme-li množství každého nutričního
substrátu jeho fyziologickou kalorickou hodnotou, získáme celkové množství tepla uvolněného při oxidaci živin.

Technické provedení nepřímé kalorimetrie (prakticky)


V praxi se používají automatické kalorimetrické přístroje. Přístroj vypočítává respirační kvocient a na jeho základě
kalkuluje aktuální metabolickou potřebu nemocného a poměr utilizovaných živin (v procentech i v g/den). Po zadání
odpadu dusíku v moči a základních antropometrických ukazatelů přístroj vypočítá tělesný povrch a podle Fleischovy
rovnice vypočítá bazální metabolickou potřebu. Dále nám poskytne např. údaje o procentuelním rozdílu mezi BMI a
aktuální metabolickou potřebou, nebo nebílkovinný RQ. Testy mohou být provedeny diluční metodou pomocí kanopy, s
obličejovou maskou nebo s volitelným příslušenstvím u pacientů připojených na umělou plicní ventilaci na jednotkách
intenzivní péče.

Využití
Metodou nepřímé kalorimetrie se stanovují např. nutriční diety, nebo tréninkové dávky pro aktivní sportovce. V
klinické praxi má tato metoda široké využití. Používá se pro zhodnocení, diagnostiku a monitorování terapie.
Umožňuje individuálně modelovat denní dávky potravin pro pacienty a předejít tak podvýživě, či překrmování
(overfeeding). Zpravidla dostávají pacienti 110% naměřené aktuální denní energetické potřeby (AEE).

Normální hodnoty metabolické spotřeby: sacharidy 50 %, proteiny 15 %, lipidy 35 %.

Větší spotřeba proteinů identifikuje patologický stav:

> 25 % bílkovin → katabolismus


> 30 % bílkovin → "katabolický" debakl

Při poklesu zásob dusíku v těle v souvislosti s deplecí proteinů ze 100 % na 70 % hrozí riziko tzv. dusíkové
smrti. V intervalu této 30% ztráty dochází postupně ke snížení hmoty příčně pruhovaného svalstva, svaloviny srdce a
svalstva hladkého. Při progresi se snižují hladiny viscerálních proteinů a proteinů transportních. Postižení imunitních
reakcí se odráží ve funkci lymfocytů, polymorfonukleárů, poklesu tvorby protilátek a APP. Je postiženo hojení ran.
Dochází k alteraci orgánových funkcí – GIT, jater, srdce.

Odkazy
Související články

Základní nutriční ukazatelé

Externí odkazy

Kalorimetrie (česká wikipedie)


Calorimetry (anglická wikipedie)

Zdroj
KUBATOVA, Senta. Biofot [online]. [cit. 2011-01-31]. <https://uloz.to/!CM6zAi6z/biofot-doc>.

HAVRÁNEK, Jiří: Ostatní monitoring.

INSTITUT GALENUS

| url = http://galenus.cz/clanky/vyziva/bioenergetika-kaloricka-hodnota-potravin

Vysoká škola chemicko-technologická v Praze: Návody laboratoře

| url = https://www.vscht.cz/

MIROSLAV PETR: prezentace Energometrie a kalorimetrie


L.NAVRÁTIL, J. ROSINA: Medicínská biofyzika
Tepelné ztráty organismu
Z hlediska tepelných změn v organismu můžeme organismy dělit na dvě hlavní skupiny studenokrevné
(poikilotermní) a teplokrevné (homoiotermní). Studenokrevní živočichové nedokáží udržovat stálou tělesnou
teplotu. Teplota jejich těla se mění v závislosti na teplotě vnějšího prostředí. Teplokrevní živočichové udržují stálou
tělesnou teplotu regulačními mechanismy při látkové výměně. Mechanismy, které se podílejí na udržení stálé teploty i
při výkyvech teplot okolí nazýváme souhrnně termoregulačními mechanismy. Lidský organismus patří do skupiny
homoiotermních. Teplo v organismu vzniká přeměnou chemické energie. Podmínkou udržení stále teploty v organismu
je rovnost tepla vzniklého metabolickými procesy a tepla odváděného do okolí. To se děje převážně regulací
rychlosti odvádění tepla – teplo se odvádí hlavně kůží a plícemi, uvnitř těla tepelné výměny zprostředkovává
proudění krve. Taktéž je krví zajištěn přenos tepla z vnitřku organismu na povrch. Teplo v organismu vzniká převážně
jako vedlejší produkt přeměny jiných forem energie, pouze výjimečně vzniká cíleně, např. v případě vyvolání
chladového třesu jako mechanismu termoregulace. Aby se toto převážně odpadní teplo v organismu neakumulovalo a
tím se nezvyšovala jeho teplota, je organismus schopen do jisté míry jeho odvod do okolí regulovat.

Mechanismy, kterými dochází k tepelným ztrátám z organismu rozdělujeme na přímé a nepřímé.

Přímé:

vyzařování (radiace)
vedení (kondukce)
proudění (konvekce)

Nepřímé:

odpařování z plic
pocení (evaporace)

Regulace teploty organismu


Regulační mechanizmy pro snížení teploty organismu:

vasodilatace
znatelné pocení
omezení produkce tepla

Regulační mechanismy pro zvýšení teploty organizmu:

vasokonstrikce
zvýšení produkce tepla chladovým třesem
zvýšením metabolismu
"husí kůže", reakce vegetativního nervstva - pro člověka nemá velký význam, spíše relikt od předků

Pro regulaci odvodu tepla je nejefektivnější regulace množství krve, která jde z jádra na povrch, kde se ochlazuje.
Míra prokrvení kůže může kolísat mezi 0–30% minutového srdečního výdaje. Samotné řízení teploty organismu je
regulováno z hypotalamu. Čidla v hypotalamu registrují teplotu krve, která jím protéká a tím zjištují teplotu vnitřních
orgánů. Pro periferní oblasti jsou čidla zabudována přímo v kůži, spíš v hlubších částech.

Záření (sálání, radiace)

Každé těleso o teplotě větší než absolutní nula (0 K) vyzařuje do okolí elektromagnetické záření, které má vlnové
délky v ultrafialovém spektru pro vysoké teploty, ve viditelném pro nižší a v infračerveném pro teplotu, ve které
žijeme. Stavu absolutní nuly nejde prakticky dosáhnout, musel by ustat pohyb molekul. Můžeme tedy říci, že
elektromagnetické záření vyzařuje každé těleso, tedy i každý organismus, neustále. Množství tělesem vyzářené
energie je podle Stefen - Boltzmanova zákona přímo úměrné čtvrté mocnině absolutní teploty tělesa. Vzhledem k
tomu, že tento zákon platí pro všechny tělesa, celková vyzářená energie se rovná rozdílu čtvrté mocniny povrchové
teploty tělesa (organismu) a objektů v jeho okolí. Tepelné záření je jediným "bezkontaktním" způsobem tepelné
výměny, díky jeho elektromagnetické povaze. Množství vyzářené energie tedy závisí jak na teplotě organismu, tak na
teplotě v okolním prostředí. Proto je celkový význam tepelných ztrát vyzařováním jiný v extrémních klimatických
podmínkách a v podmínkách, ve kterých žijeme. V našem klimatickém pásmu (mírný pás) je vyzařování velice důležité,
představuje až 60 % tepelných ztrát organismu[1].

Vedení (kondukce)
Při vedení přechází teplo z míst o vyšší teplotě do míst, kde je teplota nižší. Přechází však pouze teplo, tedy kinetická
energie kmitavého pohybu molekul, nikoliv hmota. Tento děj se uskutečňuje pouze přímým stykem. Přechod je
formulován rovnicí:

Q=(λ*τ*S*Δt)/d

kde Q (J) je množství takto předané energie za τ (s) čas, Δt (K) je rozdíl jejich teplot dvou míst, které si mezi sebou
předávájí teplo, d(m) jejich vzdálenost a S(m²) plocha přes kterou se přenos uskutečňuje. λ je koeficient tepelné
vodivosti, který říká, jakou má daná látka schopnost předat teplo.

Srovnání některých koeficientů tepelné vodivosti:

Látka λ (W·m-1·K-1) při 25 °C


diamant 895-2300
železo 80,2
voda 0,61
krev 0,52
mozek 0,51
tuk 0,21
ovčí vlna 0,04
polystyren 0,03
vzduch 0,03

Z uvedených vyplývá, že mezi dobře vodivé patří kovy a kapaliny. Naopak dobrými izolanty jsou plyny, polystyren
nebo v lidském těle tuková tkáň. Mezi dobré izolanty tedy patří i vzduch coby plyn, avšak to platí za podmínek, že je
stálý. Naopak hodně tepla je odváděno, pokud okolní vzduch proudí nebo je-li tělo v mokrém prostředí. Za běžných
podmínek člověk ztrácí prouděním asi 15 % své tepelné energie. To je však průměrná hodnota, která se liší například
pro děti, které mají mnohem větší poměr

povrch těla : objem těla [1].

Proudění (konvekce)
Vedení tepla prouděním je možné pouze u kapalin a plynů, nikoliv u pevných látek. Proudění je spojeno s přenosem
energie (=tepla) i látky samotné. Tím je tedy spjato s vedením, kdy je nejdříve teplo předáno tělem okolo něj do
okolního vzduchu pomocí vedení a následně je pomocí proudění tento vzduch odvát pryč.

Proces proudění je charakterizován rovnicí Q= α x S x Δt x τ

Ta udává množství tepla, co je za čas τ(s), odvedena proudením z povchu tělesa o S (m2) do prostředí o teplotě nižší o
Δt (K). α je koeficient přestupu tepla rozhraním. Ten záleží na mnohých faktorech - např: tlak, teplota, vlhkosti
vzduchu nebo rychlosti proudění.

Vypařování vody (evaporace)


Díky vysoké hodnotě skupenského tepla vypařování vody (2,4 MJ/kg) je vypařování vody pro organismus významné. Za
normálních podmínek tvoří až 25 % tepelných ztrát. K vypařování dochází dvěma způsoby – dýcháním a pocením.
Pocení dělíme na neznatelné a znatelné. Neznatelné pocení probíhá procesem přímé difuze molekul vody z
epitelových buněk v pokožce ven z těla. Na neznatelném pocení se nepodílejí potní žlázy. Organismus tento typ pocení
nemůže regulovat. Množství vypařené vody (odevzdaného tepla) závisí na fyzikálních vlastnostech vnějšího prostředí
(teplota vzduchu, vlhkost vzduchu...) Je-li nasycení vodními parami v okolí tzv. rosného bodu (maximální nasycení)
pocení probíhat nemůže. V průměru ztratí organismus neznatelným pocením až 660 ml vody za den. Znatelné pocení
je energeticky významnější. Probíhá pomocí potních žláz. Znatelné pocení je regulováno organismem, ale jeho
účinnost je ovlivněna fyzikálními vlastnostmi okolního prostředí. Znatelné pocení je základním mechanismem odvodu
tepla z organismu v případech, kdy vysoká okolní teplota neumožňuje uplatnění jiných mechanismů. Vypařování se
zrychluje se zvyšující tělesnou teplotou, zpomaluje při nasycení vzduchu vodními parami nebo při absenci
proudění vzduchu okolo těla. Pod 19 °C je výdej tepla snížen díky minimálnímu prokrvení kůže. 19–31 °C
prokrvení kůže dokáže zajistit rovnováhu mezi produkovaným a odváděným teplem. Nad 31 °C přistupuje ke
způsobům záření a proudění a vedení tepla ještě vypařování. Pokud je teplota okolí vyšší než teplota těla, jediným
možným mechanismem ochlazení je právě evaporace[2].
Odkazy
Související články

Termoregulace

Použitá literatura

TROJAN, Stanislav, et al. Lékařská fyziologie. 4. vydání. Praha : Grada, 2003. 772 s. ISBN 80-247-0512-5.

NAVRÁTIL, Leoš a Jozef ROSINA, et al. Medicínská biofyzika. 1. vydání. Praha : Grada, 2005. 524 s. s. 435-
436. ISBN 80-247-1152-4.

ETH, Eth. eth [online]. [cit. 2013-01-12]. <https://www.itis.ethz.ch/virtual-population/tissenerovaného seznamu


z 18.6.2018>.

Zdroj

ETH, Eth. eth [online]. [cit. 2013-01-12]. <https://www.itis.ethz.ch/virtual-population/tissenerovaného seznamu


z 18.6.2018>.

KUBATOVA, Senta. Biofot [online]. [cit. 2011-01-31]. <https://uloz.to/!CM6zAi6z/biofot-doc>.

Reference

1. NAVRÁTIL, Leoš a Jozef ROSINA, et al. Medicínská biofyzika. 1. vydání. Praha : Grada, 2005. 524 s. s. 435-
436. ISBN 80-247-1152-4.
2. TROJAN, Stanislav, et al. Lékařská fyziologie. 4. vydání. Praha : Grada, 2003. 772 s. s. 427. ISBN 80-247-
0512-5.
Termodynamické věty

I. termodynamický zákon
První termodynamický zákon vychází ze zákona zachování energie; určuje přírůstek vnitřní energie soustavy.

Pro danou soustavu ho můžeme formulovat jako:

kde je celkový přírůstek vnitřní energie dané soustavy,


je teplo odevzdané systému okolím a je celková vykonaná práce, kterou okolí
vykonalo na systému.

V diferenciálním tvaru:

U rovnice v diferenciálním tvaru se nejedná o hodnoty změněné za dobu trvání celého děje, ale za nekonečně krátký
časový úsek.

Podrobnější informace naleznete na stránce 1. termodynamický zákon.

II. termodynamický zákon


Teplo nemůže při styku dvou těles různých teplot samovolně přecházet z tělesa chladnějšího na teplejší, tzn. nelze
sestrojit periodicky pracující tepelný stroj (perpetuum mobile druhého druhu), který by trvale konal práci pouze tím,
že by ochlazoval jedno těleso a k žádné další změně v okolí by nedocházelo (viz entropie).

III. termodynamický zákon


Při absolutní nulové teplotě (T = 0 K) je entropie čisté látky pevného nebo kapalného skupenství rovna nule. Čistou
pevnou látku nelze konečným pochodem ochladit na absolutní nulovou teplotu.

Tzv. nultý termodynamický zákon


Jsou-li dvě a více těles v termodynamické rovnováze s dalším tělesem, jsou všechna tato tělesa v rovnováze.

Odkazy
Související články

Termodynamický systém
Termodynamická rovnováha
Volná energie
Entalpie
Entropie

Zdroj

KUBATOVA, Senta. Biofot [online]. [cit. 2011-01-31]. <https://uloz.to/!CM6zAi6z/biofot-doc>.

MARŠÁK, Zlatěk. Termodynamika a statistická fyzika. 3. vyd. Praha: Vydavatelství ČVUT, 1995, s. 23-25. ISBN
80-01-01401-0.
Aktivní transport

Typy symportu

Uniport a antiport

Aktivní transport je přenos látek přes buněčnou membránu, který je narozdíl od pasivního transportu spjat se
spotřebou energie. Díky dodané energii, která vzniká nejčastěji štěpením ATP, je možné vykonávat tento transport i
proti směru koncentračního gradientu (koncentračního spádu).

Aktivní transport umožňují specializované integrální membránové proteiny zabudované v buněčné membráně:

Iontové pumpy – iontové kanály vybavené enzymem ATPáza.

Přenašečové proteiny vybavené enzymem ATPáza.

Rozlišujeme dva typy aktivního transportu:

Primární aktivní transport

Je zapotřebí přítomnost volné energie. Přenáší se pouze jedna částice (např. na Na+/K+ ATPáza, která současně
čerpá sodík z buňky a draslík do buňky).[1]

Primární aktivní transportéry se dají klasifikovat na základě způsobu získávání potřebné energie:

Přenašeče (transportéry) poháněné hydrolýzou ATP – vyskytují se ve všech doménách organismů .

Přenašeče poháněné dekarboxylací – vyskytují se v prokaryontních organismech.

Přenašeče poháněné přenosem methylové skupiny – vyskytují se u archebakterií.

Přenašeče poháněné oxidoreduktázou: zdroj energie je oxidace redukovaného substrátu zprostředkovaná


tokem elektronů – vyskytují se ve všech doménách organismů.

Přenašeče poháněné světelnou energií- vyskytují se u archebakterií.

Sekundární aktivní transport

Jako zdroj energie je využito spřažení s přenosem jiné látky ve směru koncentračního gradientu. Energie uložená
v gradientu, který následuje pasivně přenášená částice, je využita k přenosu druhé částice proti směru
koncentračního spádu.
Gradient pro pasivní přestup druhé látky je vytvořen primárně aktivním transportním mechanismem na jiném místě
membrány (např. resorpce glukózy proti gradientu a sodíku ve směru gradientu v tenkém střevě). Pro sekundárně
aktivní transport se rovněž používá termín kotransport.[1]

Podle počtu přenášených částic rozlišujeme:

uniport – je přenášena pouze jedna molekula nebo iont,


kotransport - přenášeny dvě nebo více molekul nebo iontů.

Kotransport dále dělíme podle vzájemného směru přenášených částic:

symport – částice jsou přenášeny stejným směrem,


antiport – částice jsou přenášeny opačným směrem.

Typickým příkladem aktivního transportu je sodíko-draslíková pumpa (alternativně Na+/K+ ATPáza), která udržuje
koncentrací rozdíl sodíku a draslíku mezi intracelulárním a extracelulárním prostředím tím, že vyčerpává sodík z
buňky a naopak vychytává draslík. Tyto rozdíly jsou poměrně výrazné: koncentrace sodíku je 140 mmol/l
extracelulárně a 10 mmol/l intracelulárně, koncentrace draslíku je 5 mmol/l extracelulárně a 165 mmol/l
intracelulárně. Vlastně jde o antiport sodíku a draslíku, přesněji tří molekul sodíku a dvou molekul draslíku. Protože
se sodík i draslík pohybují proti koncentračnímu gradientu, je třeba zdroj energie; v případě sodno-draselné pumpy je
zdrojem energie ATP.

Odkazy
Související články

Aktivní transport
Symport
Antiport
Pasivní transport
Difuze
Prostá difuze
Facilitovaná difuze
Filtrace
Osmóza
Průnik léčiva přes membrány
Donnanova rovnováha/příklad

Reference

1. ŠVÍGLEROVÁ, Jitka. Aktivní transport [online]. Poslední revize 18. 2. 2009, [cit. 13. 11. 2010]. 
<https://web.archive.org/web/20160416224212/http://wiki.lfp-studium.cz/index.php/Aktivn%C3%AD_transport>.

Použitá literatura
KODÍČEK, M. a V. KARPENKO. Biofysikální chemie. 1. vydání. Praha : Academia, 2000. ISBN 80-200-0791-1.
Pasivní transport

Článek ke kontrole
Žádá se kontrola tohoto článku
učitelem.
Navržený učitel: Petr Heřman

Pasivní transport je přenos látek přes buněčnou membránu, který probíhá samovolně prostřednictvím kanálů a
přenašečových proteinů. Na rozdíl od aktivního transportu tento děj nespotřebovává žádnou chemickou energii (ATP).
Pasivní transport závisí na propustnosti buněčné membrány, která závisí na uspořádání dvojité vrstvy fosfolipidů a
vmezeřených bílkovin. Základními typy pasivního transportu jsou prostá difuze, usnadněná difuze a osmóza. Při
usnadněné difuzi musí látky na rozdíl od prosté difuze použít přenašeče či kanály.

Gradient elektrochemického potenciálu


Proces pasivního transportu je řízen gradientem koncentrace a membránovým potenciálem. Tyto síly se navzájem
skládají v celkovou sílu, která se nazývá gradient elektrochemického potenciálu. Je to rozdíl elektrochemických
potenciálů na vnější a vnitřní straně membrány, to je hnací sílou pro pohyb iontů přes membránu.

Přenašečové proteiny
Pro přenos solutu (přenášená chemická látka) z vnějšku buňky dovnitř rozeznáváme 2 mechanismy prostupu přes
proteiny.

1. Přenašečový protein využívá pro přenos solutu svých konformačních změn, které mu umožňují přenos malých ve
vodě rozpustných molekul.
2. Kanálový protein je hydrofilní pór, který pro přenos specifických anorganických iontů nepotřebuje změnit svou
konformaci. Proto je tento způsob přenosu rychlejší než typ předchozí. Protein může být svou konformací otevřen
či uzavřen.

Při transportu přenašečovými proteiny mohou transportované molekuly procházet samostatně. Tento proces
nazýváme uniport. Nebo molekuly mohou procházet souběžně což je kotransport. Ten dělíme na symport a antiport.
Při přenosu symportem prostupují molekuly stejným směrem a při přenosu antiportem prostupují směrem opačným.
Tyto procesy fungují i při aktivním transportu.

Penetrující integrované proteiny

Iontové kanály
Buňka pro přenos iontů přes svou membránu používá také iontové kanály. Ty jsou převážně ovládané navázáním
acetylcholinu na vazebné místo. Jen velmi málo kanálů se otvírá bez působení acetylcholinu. Kanály jsou dále ovládané
napětím, mechanicky či chemicky. Iontový kanál je vysoce selektivní, což zapříčiňují záporně nabité řetězce
aminokyselin. Proto kanál propouští jen kladně nabité ionty například K+, Na+. Iontové kanály také pracují na dvou
základních principech - selektivita iontů a uzavíratelnost kanálů. Selektivita iontů znamená, že kanály přenášejí
jeden iont či umožňují kotransport několika iontů. Uzavíratelnost udává, že kanály jsou někdy otevřeny či uzavřeny.
Vždy to však závisí na stálém poměru otevřených a zavřených kanálů v membráně buňky. Více informací na stránce
iontové kanály.

Odkazy
Související články

Aktivní transport
Symport
Antiport
Pasivní transport
Difuze
Prostá difuze
Facilitovaná difuze
Filtrace
Osmóza
Průnik léčiva přes membrány
Donnanova rovnováha/příklad

Zdroj

ŠVÍGLEROVÁ, Jitka. Pasivní transport [online]. Poslední revize 18. 2. 2009, [cit. 12.11.2010]. 


<https://web.archive.org/web/20160306065550/http://wiki.lfp-studium.cz/index.php/Pasivní_transport>.

KODÍČEK, M. a V. KARPENKO. Biofysikální chemie. 1. vydání. Praha : Academia, 2000. ISBN 80-200-0791-1.

VAJNER, Luděk, Jiří UHLÍK a Václava KONRÁDOVÁ. Lékařská histologie I. 1. vydání. Praha : Nakladatelství
Karolinum, 2012. ISBN 978-80-246-1860-9.
Kapalinové teploměry

Rtuťový teploměr

Úvod
Teploměry využívající tepelnou roztažitelnost kapaliny (rtuti). Kapilára (je z ní vyčerpán vzduch) se na dolním
konci rozšiřuje ve válcovou nádobku naplněnou rtutí.

Rtuťovými teploměry lze měřit teplotu od −39 do +250 °C. Pro nižší teploty se používají teploměry naplněné
obarveným toluenem nebo etylalkoholem.

Lékařský teploměr
Slouží k měření tělesné teploty (rozsah 35–42 °C), hladina rtuti zde zůstává na nejvyšší naměřené hodnotě a po použití
je nutné ji sklepat.

Odkazy
Zdroj

KUBATOVA, Senta. Biofot [online]. [cit. 2011-01-31]. <https://uloz.to/!CM6zAi6z/biofot-doc>.


Měření teploty

Termometrií se určuje teplota – objektivní míra tepelného stavu látky. Dle SI je základní veličinou
termodynamická teplota, jejíž jednotkou je Kelvin.

K měření teploty se používaly či používají následující teplotní stupnice:

termodynamická teplotní stupnice;


Celsiova teplotní stupnice;
Fahrenheitova teplotní stupnice;
Réaumurova teplotní stupnice;
Rankineho teplotní stupnice.

Centrální tělesnou teplotu (TT) měříme intrakavitálně, nejčastěji per rectum.

Periferní TT měříme nejčastěji na dorsu nohy a dále hodnotíme teplotní diferenci.

Rozdíl mezi centrální a periferní TT > 2 °C svědčí pro hypovolemii nebo zvýšenou α–mimetickou aktivitu, rozdíl >
8 °C svědčí pro šokovou cirkulaci, těžkou hypovolemii.

Indikací ke kontinuálnímu měření TT jsou:

intrakraniální hypertenze;
thiopentalové kóma;
pacienti s hemodynamickou nestabilitou;
pacienti s náročným ventilačním režimem;
pacienti s maligní hypertermií.

Způsob měření

Teplota se dá měřit pouze nepřímo na základě známých fyzikálních jevů za různých teplot. Termometrie se proto
provádí několika způsoby, založenými většinou na objemové roztažnosti kapalin nebo délkové roztažnosti
pevných látek za různé teploty. Nejpřesnější je intrakavitální měření, tj. rektálně, vaginálně, orálně
(aurikulární měření nelze považovat za přesné).

Z praktického hlediska naměření vyšší axilární teploty s vysokou pravděpodobností predikuje vyšší hodnotu rektální,
ale normální axilární teplota nevylučuje vyšší hodnotu rektální. Při pochybnostech o axilární teplotě je proto nutno
pacienta přeměřit rektálně.

Kapalinové teploměry

Rtuťové teploměry

Jedná se o nejrozšířenější používané teploměry vůbec. Bývají většinou rtuťové, i když jsou postupně kvůli toxicitě
rtuti vytlačovány. Skládají se z rtuťového rezervoáru s kapilárou a stupnice. S rostoucí teplotou rtuť mění svůj
objem a šplhá v kapiláře. Používají se dvě základní modifikace:

maximální teploměr – zaznamenává nejvyšší naměřenou hodnotu. Vlivem zúžení kapiláry nad rezervoárem
zůstává po použití rtuť na maximálním bodě a je nutno ji do rezervoáru „sklepat“. Doba ustálení konečné teploty
je několik minut.
rychloběžka – měří okamžitou teplotu a její hodnota se ustálí rychleji

Dalším druhem kapalinového teploměru je teploměr lihový, měřící v rozsahu od −110 °C do 70 °C.

Citlivost těchto teploměrů roste s objemem rezervoáru a menším poloměrem kapiláry. Tyto teploměry měří s přesností
na desetiny stupně.

Bezdotykové teploměry
Každé těleso o určité teplotě podle Stefan-Boltzmannova zákona vyzařuje tepelné záření. Infračervený bezdotykový
teploměr' někdy též pyrometr je založen na principu měření množství takto vyzářené energie v infračerveném
spektru. Protože Stefan-Boltzmannův zákon platí pro černá tělesa, zavádí se pro reálná tělesa veličina
emisivita. Emisivita je poměr mezi vyzařováním reálného tělesa a černého tělesa při stejné teplotě. U
reálných těles je navíc nutno přihlédnout k jejich průhlednosti a odrazivosti'. U průhledných materiálů se
přesnosti dosahuje spektrální filtrací, např. sklo se chová neprůhledně pro vlnovou délku 5 µm. U odrazivých
materiálů dochází k měření nejen vlastního, ale i odraženého záření.

Konstrukce se skládá z optické soustavy (čoček, optických vláken, spektrálních filtrů), která určuje průměr měřené
oblasti v předepsané vzdálenosti. Pro zaměření měřené oblasti se používají světelné nebo laserové zaměřovače.
Paprsky jsou soustředěny do detektoru, kterým bývá fotovoltaický článek nebo fotorezistor. Vzniká tak elektrický
signál, který je zesílen a různě zobrazen.

Výhodou je bezkontaktnost, možnost měření na větší vzdálenosti, rychlá odpověď a zanedbatelné ovlivnění
měřeného objektu.

Kovové odporové teploměry

Tento teploměr je založený na změně elektrického odporu kovu se změnou teploty. Tato závislost může být vyjádřena
vztahem:

Rt = R0·(1 + α·t),

kde Rt je odpor při teplotě t, R0 je odpor při nulové teplotě a α (K-1) je teplotní součinitel odporu

Výhodou tohoto typu teploměru je linearita měření ve velkém rozsahu teplot a lehké vyhodnocení.
Nejpoužívanější je platinový teploměr, který změří teplotu v rozsahu od -100 °C do 440 °C, přičemž měří s přesností
na tisíciny stupňů. Tento typ teploměru je používaný nejmíň v technické praxi a často bývá součástí složitějších
měřících zařízení (např. jako snímač na měření referenční teploty termočlánkových sond).

Termočlánek

Termoelektrické články měří teplotu na základě termoelektrického jevu. Ten funguje na principu, že v uzavřeném
elektrickém obvodu dvou vodičů z různých kovů, kdy má každý různou teplotu, teče elektrický proud. Pokud tento
obvod rozpojíme, jsme schopni měřit hodnoty termonapětí, které jsou dány rozdílem teploty mezi spoji. Pro praktické
užití termočlánku se jeden vodič dá do prostředí s referenční teplotou (v praxi teplota místnosti, cca 25 °C) a druhý
se vloží do prostředí, kde chceme teplotu změřit. Voltmetrem se poté měří hodnota termonapětí mezi spoji, s přesností
na setiny stupňů Celsia.

Zmíněné termonapětí je kvadratickou funkcí teploty. V případě jeho využití v medicíně, kde je rozsah teplot jen od
20 °C do 50 °C, můžeme danou lineární závislost vyjádřit pomocí vztahu:

UAB = k·(tA – tB)

kde UAB je termonapětí mezi referenčním a měřícím bodem termočlánku, tA a tB jsou teploty těchto bodů; k je
kalibrační konstanta závisící na typu termočlánku. V medicínských aplikacích jsou nejpoužívanější hlavně termočlánky
měď-konstantan (kalibrační konstanta 40 μV/K) nebo mangan-konstantan. Přesnost měření teploty pomocí
termočlánku závisí na citlivosti voltmetru měřícího termonapětí a na přesnosti měření teploty studeného konce
termočlánku.

Výhodou termočlánků je miniaturizace, proto se v lékařství používají jako invazivní měřiče teploty, např. pro účely
hypertermie.

Termistor

Měření teploty termistorem je založeno na měření elektrického odporu, kdy s rostoucí teplotou hustota volných
elektronů v polovodiči prudce stoupá. Tím klesá elektrický odpor.

Daná závislost může být vyjádřena rovnicí:

T = B·(ln R − lnA)

kde B a A jsou materiálové konstanty, R je odpor a T je absolutní teplota polovodiče.

Měření je velmi přesné, řádově v mK. Čidla pro termistory jsou většinou invazivní jehly, kdy samotný termistor je ve
špičce této jehly.

Fosforová termometrie

Fosforová termometrie slouží k měření teploty povrchů pokrytých vrstvou fosforu. Fosfor po excitaci díky
luminiscenci emituje světlo. Jeho jasnost, barva a doba doznívání jsou závislé na teplotě. Nejčastěji se využívá
závislost doby doznívání po excitaci na teplotě. Využívá se ve vláknových invazivních termometrech.
LC teploměry

Jednorázový teplotní proužek

LC (liquid crystal) teploměry fungují na bázi tekutých krystalů, které mění barvu v závislosti na teplotě. Rozlišovací
schopnost může být až 0,1° C. V podobě jednorázových teplotních proužků je lze využít k měření tělesné teploty na
čele.

Odkazy
Související články
Článek o měření teploty v Katalogu metod v biofyzice
Sledování fyziologických funkcí
Termografie
Měření a hodnocení tělesné teploty
Chyby měření fyzikálních veličin, relativní chyba

Externí odkazy

Phosphor thermometry (anglická wikipedie)


Liquid crystal thermometer (anglická wikipedie)

Zdroj

KUBATOVA, Senta. Biofot [online]. [cit. 2011-01-31]. <https://uloz.to/!CM6zAi6z/biofot-doc>.

NAVRÁTIL, Leoš a Jozef ROSINA, et al. Medicínská biofyzika. 1. vydání. Praha : Grada, 2005. 524 s. s. 68-
72. ISBN 80-247-1152-4.

HAVRÁNEK, Jiří: Ostatní monitoring.

BARRON, W. R. Principles of Infrared Thermometry [online]. [cit. 2012-12-28]. 


<https://www.omega.com/temperature/Z/pdf/z059-062.pdf>.
Tepelná zařízení
Termostat
Zařízení udržující uvnitř určitého prostoru stálou volitelnou teplotu. V praxi může mít podobu uzavřeného objemu
s dvojitými stěnami, které jsou vyplněny izolací. Uvnitř tohoto objemu je regulací udržována konstantní teplota.

Sterilizátor a autoklávy
Jsou to přístroje ke sterilizaci nástrojů a textilu pomocí tepla.

Autokláva

Nádoba s přetlakem vodní páry a teplotou 120 °C.

Vodní lázně
Nádoby s el. topným tělesem a míchadlem, řízené na stálou teplotu. Podobné termostatům.

Temperované operační stoly


Umělé podchlazení při chirurgických zákrocích. Pod deskou stolu je potrubí s vodou, která tělesnou teplotu pacienta
udržuje na stálé zvolené teplotě.

Chladící zařízení
Zařízení, kde se teplota uměle snižuje značně pod teplotu okolí.

Kompresorový princip chlazení – stačuje vhodný plyn (čpavek), který pak v kondenzátoru kapalní, a ve
výparníku uvnitř lednice se kapalina odpařuje, plyn se rozpíná a ochlazuje okolí.
Absorpční princip – čpavek se pohlcuje ve vodě a mimo ledničku se z vody opět vypuzuje.

Odkazy
Související články

Dezinfekce
Sterilizace (hygiena)

Zdroj

KUBATOVA, Senta. Biofot [online]. [cit. 2011-01-31]. <https://uloz.to/1162346/biofot.doc>.


Coulombův zákon a permitivita prostředí

Coulombův zákon popisuje sílu F, kterou na sebe působí dva bodové náboje Q1 a Q2 ležící ve vzdálenosti r. Tato síla
je odpudivá, pokud mají náboje stejné znaménko, v případě nábojů s odlišným znaménkem náboje je tato síla
přitažlivá. Matematicky je Coulombův zákon vyjádřen následovně:

Konstanta ε se nazývá permitivita. Permitivita vakua se značí ε0 a jejíí velikost je 8,854·10-12 Fm-1.

Slovně lze Coulombův zákon formulovat tak, že síla, kterou na sebe působí dvě elektricky nabité částice, je přímo
úměrná součinu jejich nábojů a nepřímo úměrná kvadrátu jejich vzdálenosti.

Při vložení libovolné látky do elektrického pole začne na elektricky nabité částice v této látce obsažené působit síla.
Pokud je látka vodičem, tedy pokud obsahuje volné nosiče náboje, doputují tyto až na povrch tělesa. Mezi náboji na
povrchu tělesa se objeví elektrické pole, které působí proti vnějšímu elektrickému poli. V případě ideálního vodiče se
pohyb nábojů zastaví až v situaci, kdy takto indukované elektrické pole dosáhne stejné velikosti jako vnější elektrické
pole, ovšem opačné orientace. Tento jev v praxi znamená mimo jiné to, že v dutině ze všech stran obklopené vodivým
prostředím bude v ustáleném stavu nulové elektrické pole. Tomuto uspořádání se říká Faradayova klec a v praxi se
využívá vždy, když potřebujeme dokonale odstínit vnější elektrická pole.

Vzájemné působení nábojů

Pokud je do vnějšího elektrického pole vloženo ideální dielektrikum, tedy látku bez volných nosičů elektrického
náboje, je situace poněkud komplikovanější. Dielektrikum sice neobsahuje volné nosiče náboje, ovšem jeho molekuly
mohou být polární již ze své podstaty (např. řada organických polymerů) nebo se mohou částečně polarizovat
"deformací" elektronového obalu vlivem vnějšího elektrického pole (např. vzácné plyny). V obou případech získávají
částice dielektrika dipólový moment orientovaný proti vnějšímu elektrickému poli. Makroskopicky se dipólové
momenty jednotlivých částic dielektrika projeví jako vektor dipólové polarizace P, který má v případě
izotropního[pozn. 1] dielektrika směr opačný směru intenzity vnějšího elektrického pole E. V případě, že se dielektrikum
chová lineárně, což je za nepříliš velkých vnějších intenzit obvykle splněno, je hodnota polarizace násobkem intenzity
elektrického pole. Protože je polarizace vnějším projevem existence dipólů v dielektriku orientovaných proti vnějšímu
elektrickému poli, vlastně zeslabuje vnější elektrické pole a uvnitř dielektrika bude intenzita elektrického pole nižší.
Číselně se tato změna zachytí změnou permitivity prostředí ε. Tabelována je obvykle relativní permitivita εr, ze které
se permitivita vypočítá jednoduše tak, že se vynásobí permitivitou vakua:

Hodnota relativní permitivity je ve všech prostředích větší než jedna, příklady několika hodnot jsou uvedeny v
následující tabulce:

látka εr
vzduch 1,00054
papír 3,5
sklo 7,6
voda 80

Vysoká hodnota permitivity vody je jednou z příčin toho, že voda je velmi dobré rozpouštědlo iontových krystalů. V
prostředí s relativní permitivitou 80 jsou opačně nabité ionty vázány silou zhruba 80× nižší než ve vzduchu, snáze
tedy opouštějí krystal.

V případě, že je dielektrikum vloženo do střídavého elektrického pole, chová se podstatně složitěji, například
permitivita závisí na frekvenci a při matematickém popisu je nutné přejít ke komplexním číslům. Na druhé straně
chování permitivity je do jisté míry určeno chemickým složením prostředí, takže ze znalosti tzv. dielektrického spektra
(též impedančního spektra − podle způsobu měření) lze právě na chemické složení neznámého prostředí usuzovat.
V medicíně je takový přístup teoretickým základem bioimpedanční analýzy používané např. při měření hydratace
(víceméně experimentální postup) nebo při odhadu množství tělesného tuku.
Odkazy
Poznámky pod čarou

1. Izotropní znamená, že má ve všech směrech stejné vlastnosti. Tato zdánlivě přirozená vlastnost je porušena např.
u krystalů. Z každodenní zkušenosti se jako anizotropní (tedy ne-izotropní) chová například každá silnice, zatímco
podélně je jízda obvykle možná, ve směru na osu silnice kolmém je velmi problematická.

Zdroje

KUBATOVA, Senta. Biofot [online]. [cit. 2011-01-31]. <https://uloz.to/!CM6zAi6z/biofot-doc>.

SEDLÁK, Bedřich a Ivan ŠTOLL. Elektřina a magnetismus. 1. vydání. Praha : Academia a Karolinum, 


1993. ISBN 80-200-0172-7.

SVOBODA, Emanuel, et al. Přehled středoškolské fyziky. 3. vydání. Praha : Pometheus, 1996. ISBN 80-7196-


116-7.

Externí odkazy

Český překlad kurzu Elektřina a magnetizmus z MIT: [1]


Potenciál bodového náboje a na rozhraní fází
Elektrický potenciál je fyzikální veličina charakterizující energetické poměry elektricky nabité částice nacházející se v
elektrickém poli. Obvykle se značí φ a jednotkou je volt [V]. Značka U se obvykle používá pro rozdíl dvou potenciálů
nazývaný elektrické napětí. Dlužno poznamenat, že v medicínských aplikacích se toto fyzikální a technické dělení
striktně nerozlišuje, takže se např. běžně hovoří o akčním potenciálu, byť jde vlastně o „akční napětí“. Protože však
nehrozí záměna nebo omyl, bylo by chybou pokoušet se ve fyziologii o násilné zavedení fyzikálně korektního
názvosloví.

Elektrické pole složitého


tvaru. Černé linie
(siločáry) jsou jedním z
grafických způsobů
znázornění elektrického
pole, jejich směr je v
každém bodě stejný jako
směr intenzity E.
Znázornění pomocí tzv.
izopotenciál, tedy křivek
spojujících body se
stejnou hodnotou
potenciálu φ je méně
obvyklý.

Vlastnosti konkrétního elektrického pole lze plně popsat velikostí elektrického potenciálu φ nebo intenzity
elektrického pole E. Tyto hodnoty spolu poměrně jednoduše souvisí a lze je navzájem přepočítat, každá je vhodnější
pro jiné účely. Tvar elektrického pole může být obecně složitý, ovšem v případě bodového náboje je jeho tvar velmi
jednoduchý. Protože je elektrické pole ovlivněno nejen rozložením nábojů v prostředí, ale i dielektrickými vlastnostmi
prostředí, deformuje se tvar elektrického pole na rozhraní dvou prostředí o rozdílné permitivitě.

Veličiny popisující elektrické pole


Intenzita elektrického pole

Intenzita elektrického pole E je vektorová fyzikální veličina, která má přímý vztah k tomu, jak velkou silou působí
pole na vložený bodový náboj. Její jednotkou je V·m-1 (Volt na metr), někdy se používá i N·C-1 (Newton na Coulomb).
Jejím fyzikálním významem je síla, která by působila na jednotkový kladný zkušební náboj vložený do elektrického
pole. Sílu působící na skutečný náboj Q pak lze vypočítat velmi snadno:

Elektrický potenciál

Elektrický potenciál φ je skalární fyzikální veličina, která má přímý vztah k energetickým poměrům v elektrickém
poli. Její jednotkou je Volt (V). Myšlenka stojící za zavedením elektrického potenciálu stojí na záměrném přesunu
„zkušebního“ náboje q v elektrickém poli. V každém místě elektrického pole působí na jednotkový náboj nějaká síla.
Pokud se náboj posune kolmo na tuto sílu, žádná práce se nekoná – říkáme, že se náboj pohybuje po izopotenciálové
ploše. Zajímavější je situace, kdy se náboj pohybuje alespoň částečně ve směru resp. proti směru síly vyvolané
elektrickým polem. Potom pole vykonává práci resp. vnější síly vykonávají práci, kterou lze změřit nebo vypočítat a
která se obvykle značí ΔW. Velikost této práce závisí pouze na tvaru elektrického pole a na velikosti náboje, takže lze
zavést potenciál jako velikost této práce vztaženou k velikosti náboje:

Symbol Δ znamená změnu nebo rozdíl. Při zamyšlení se nad důsledkem této definice vysvitne jeden podstatný problém
– totiž my sice může změřit nebo vypočítat rozdíl dvou potenciálů (tedy elektrické napětí), ovšem nemůžeme určit jeho
absolutní hodnotu. Nulová hodnota potenciálu se tedy obvykle volí na základě dohody například v nekonečnu nebo na
nějakém konkrétním významném tělese, např. na tělese vodivě spojeném se zemí.

Souvislost mezi potenciálem a intenzitou

Intenzita elektrického pole a elektrický potenciál spolu úzce souvisí, ovšem matematická formulace tohoto vztahu
přesahuje běžné středoškolské učivo matematiky. Uvedení má pouze ilustrativní význam pro ty, kteří mají k přírodním
vědám bližší vztah.

Intenzita se z potenciálu spočítá pomocí operátoru gradientu (třetí výraz je definice gradientu):

Operátor gradientu skalární funkce více proměnných má význam velikosti a směru největší změny. Přeloženo do
češtiny to znamená, že vektor intenzity elektrického pole je co do velikosti i do směru roven opačnému vektoru k
vektoru největší změny elektrického potenciálu.

Zajímavým a často používaným důsledkem plynoucím z vlastností gradientu je to, že siločáry protínající
izopotenciálovou plochu jsou k ní vždy kolmé.

I výpočet potenciálu z intenzity je formálně snadný, zapisuje se pomocí integrálu po (orientované) křivce C spojující
body, mezi kterými nás rozdíl potenciálů zajímá:

Pro pochopení je nutné si vzpomenout, že E je vlastně síla působící na jednotkový náboj, takže součin absolutních
hodnot E·s má význam práce, jaká by byla vykonána posunem jednotkového náboje. Křivkový integrál lze chápat tak,
že se vezme křivka spojující body, které nás zajímají, a rozdělí se na tak malé úseky, že je lze považovat za úsečky a
intenzitu elektrického pole podél této úsečky lze považovat za konstantní. Na každém takovém úseku se vypočítá
práce, kterou by vykonal jednotkový náboj průchodem:

Index i označuje dílčí úsek, E velikost intenzity elektrického pole v místě daného dílku, l délku dílku a α úhel, jaký
svírá příslušný dílek s vektorem E. Celkový integrál je pak součtem hodnot jednotlivých dílků.

Elektrický potenciál v okolí bodového náboje

Elektrické pole v okolí


záporně nabitého
náboje. Černě jsou
znázorněny siločáry,
červeně ekvipotenciální
plochy

Není těžké uhádnout, že v okolí bodového náboje bude elektrické pole kulově symetrické, takže siločáry budou
vycházet paprsčitě ze středu náboje a izopotenciály budou soustředné kulové plochy. Pokud se položí hodnota
nulového potenciálu do nekonečna, pak pro velikost potenciálu v bodu ležícím ve vzdálenosti r od centrálního
náboje Q platí jednoduchý vztah:

Tedy elektrický potenciál je nepřímo úměrný první mocnině vzdálenosti od náboje. Bodový náboj je pouze fyzikální
abstrakcí, reálně neexistuje a ani existovat nemůže, protože v jeho blízkosti by dosahoval potenciál libovolně vysokých
hodnot. Ukazuje se však, že bodový náboj je dostatečně věrnou aproximací pro nabitá kulovitá tělesa s rovnoměrně
rozloženým nábojem na povrchu nebo dostatečně malá tělesa. Tak můžeme za bodové náboje (podle charakteru
řešeného problému) považovat například elektrony vznikající ve fotonásobiči, některé anorganické ionty v roztocích
nebo třeba ionizované funkční skupiny biologicky významných makromolekul.

Odkazy
Zdroj

KUBATOVA, Senta. Biofot [online]. [cit. 2011-01-31]. <https://uloz.to/!CM6zAi6z/biofot-doc>.

SEDLÁK, Bedřich a Ivan ŠTOLL. Elektřina a magnetismus. 1. vydání. Praha : Academia a Karolinum, 


1993. ISBN 80-200-0172-7.

SVOBODA, Emanuel, et al. Přehled středoškolské fyziky. 3. vydání. Praha : Pometheus, 1996. ISBN 80-7196-


116-7.

Externí odkazy
Český překlad kurzu Elektřina a magnetizmus z MIT: [1]
Elektrodové děje/Elektrochemický potenciál
Zabývejme se tím, jak velké bude napětí mezi elektrodami elektrochemického článku. Pro jednoduchost považujme
článek za soustavu produkující vratnou elektrickou práci. Tato práce musí být rovna úbytku Gibbsovy volný energie:

Elektrická práce je dána velikostí náboje a elektrickým potenciálem


, kterým je tento náboj přenášen

S využitím Faradayova zákona můžeme převést náboj na množství elektronů


( je Faradayova konstanta rovna 96 484,56 C·mol−1).

Po dosazení dostaneme

Z termodynamiky víme, že

( je univerzální plynová konstanta rovna 8,314 41 J.mol−1.K−1).

Dosadíme-li a obdobně
, dostaneme

Z praktických důvodů je užitečné považovat elektrochemický článek za soustavu složenou ze dvou poločlánků (tj. dvou
elektrod v odpovídajících elektrolytech). Výše uvedenou rovnici můžeme rozepsat pro každý poločlánek:

Výsledné napětí mezi svorkami celého článku složeného z těchto poločlánků bude

Všimněme si, že elektrodový potenciál můžeme obecně vyjádřit jako součet dvou členů. Jeden, který jsme značili s
indexem 0, je závislý pouze na teplotě a vlastnostech elektrody. Odpovídá potenciálu, který by článek měl, pokud by
aktivita všech složek byla rovna jedné (tj. jde o standardní redukční potenciál zmíněný výše). Hodnotu tohoto členu lze
zjistit pouze experimentálně – klasicky srovnáním se výše zmíněnou standardní vodíkovou elektrodou. Standardní
redukční potenciály některých elektrod jsou v tabulce:

Standardní redukční potenciály


vybraných elektrod
Redoxní pár [V] Redoxní pár [V]
Li+/Li (s) −3,04 Co2+/Co (s) −0,28

K+/K (s) −2,92 Ni2+/Ni (s) −0,25

Na+/Na (s) −2,71 Sn2+/Sn (s) −0,14

Ca2+/Ca (s) −2,50 Pb2+/Pb (s) −0,13

Al3+/Al (s) −1,66 2 H+/H2 (g) +0,00

Mn2+/Mn (s) −1,18 Sn4+/Sn2+ +0,15

Zn2+/Zn (s) −0,76 Cu2+/Cu (s) +0,34

Cr3+/Cr (s) −0,74 Ag+/Ag (s) +0,80

Fe2+/Fe (s) −0,44 Pt+/Pt (s) +1,19

Cd2+/Cd (s) −0,40 Cl2/2 Cl- (g) +1,36

Tl+/Tl (s) −0,34 Au+/Au (s) +1,50

Druhý člen je kromě teploty a počtu vyměňovaných elektronů závislý i na aktivitách jednotlivých složek článku. Napětí
článku je v obecném případě dáno rozdílem elektrodových potenciálů pravé (+, index 1) a levé (−, index 2) elektrody.
Kdyby se měřil standardní redukční potenciál měděné elektrody, musel by být poločlánek mědi zapojen jako kladný
pól článku oproti standardní vodíkové elektrodě (SVE). Jeho napětí by bylo

U = E0red(Cu) − E0red(SVE) = +0,34 − 0 = +0,34 V

V případě Danielova článku složeného ze standardní měděné a standardní zinkové elektrody je napětí článku

U = E0red(Cu) − E0red(Zn) = +0,34 − (−0,76) = +1,1 V

Odkazy
Reference
Membránový potenciál
Membránový potenciál obecně je elektrický potenciál vnitřní strany buněčné membrány vztažený k vnějšímu
povrchu buňky.

V klidu převažuje na vnitřní straně cytoplazmatické membrány záporný náboj, proto je ve většině buněk číselná
hodnota potenciálu membrány řádově okolo −70 mV (závisí na typu buňky a daném orgánu).

V průběhu času se může membránový potenciál specificky měnit, čímž může přenášet informace.

Membrána je složená z fosfolipidů, takže není možné, aby jí volně procházela většina biologicky významných látek -->
z toho plyne nerovnoměrné rozložení iontů vně a uvnitř membrány.

Při podráždění nervové buňky dojde ke změně rozložení iontů.

Co je potenciál a napětí?
Elektrický potenciál je schopnost elektrického pole působit na jednotkový náboj. Zjednodušeně řečeno
můžeme říct, že je to míra působení elektrického pole v daném místě na elektron. Čím silnější je elektrické pole, tím
intenzivněji bude na elektron působit. Čím dále od zdroje pole je elektron, tím méně pole působí. Je definován jako
práce, kterou by bylo potřeba vykonat, abychom v elektrickém poli přesunuli jednotkový náboj z místa s
definovaným nulovým potenciálem do zkoumaného místa.

Pro lepší pochopení lze použít příklad gravitačního potenciálu Země. Je to v podstatě potenciální energie Ep tělesa o
hmotnosti 1 kg. Problémem je jenom definice bodu s nulovým potenciálem. Obvykle se pro obecné fyzikální principy používá
jako bod s nulovým potenciálem místo nekonečně vzdálené od Země (kde již zemská gravitace nepůsobí). Pro výpočet
potenciální energie však používáme jako místo s nulovým potenciálem povrch Země. Pokud použijeme druhou definici,
vidíme, že funguje tvrzení, že je gravitační potenciál roven energii, která je potřeba k přenesení tělesa o hmotnosti 1 kg z
místa o nulovém potenciálu (povrch Země) na dané místo.

Čím by byla hmotonst Země vyšší, tím vyšší gravitační potenciál by v daném místě byl (gradient potenciálu je vyjádřen
gravitačním zrychlením). Pokud definujeme bod s nulovým potenciálem jako bod v nekonečnu, vidíme, že se otočí znaménko,
protože směr pohybu je opačný.

Za místo s nulovým potenciálem je u ostatních veličin obvykle brán nekonečně vzdálený bod, u elektrického pole
většinou povrch Země. V případě elektrického potenciálu vnitřní strany membrány se však z historických souvislostí
za místo s nulovým potenciálem pokládá vnější povrch buňky.

Napětí je obecně řečeno rozdíl potenciálů dvou bodů. V případu membránového potenciálu jsou však pojmy napětí
na membráně a membránový potenciál synonyma – potenciál je určen vůči vztažnému bodu na vnějším povrchu
buňky, jehož potenciál je definovaný jako 0 V. Je tedy shodný jako napětí – rozdíl potenciálů obou stran membrány.

V závislosti na působení sil (kladný nebo záporný náboj v daném místě) se mění znaménka potenciálu. Pokud
převažuje na vnitřní straně záporný náboj, membránový potenciál bude záporný a naopak. Proto je třeba rozlišovat
zvětšování (× zmenšování) napětí (tedy zvětšování rozdílu obou potenciálů) a zvyšování (× snižování)
potenciálu (což může, ale nemusí být vykládáno jako posunování ke kladnějším hodnotám, tedy častěji snižování
rozdílu).

Podmínky vzniku a určující faktory


Pro vznik membránového potenciálu musí být splněno několik základních požadavků, mezi něž patří intaktní
semipermeabilní membrána, funkční buňka a dostatek energie.

Aktuální hodnota membránového potenciálu závisí především na:

aktuální selektivní propustnosti membrány pro různé ionty,


intra- a extracelulární koncentraci iontů, pro které je membrána propustná (tzn. transmembránovém
koncentračním gradientu),
nedifuzibilních aniontech uvnitř buňky (proteiny),
efektu Na+/K+ATPasy.

Membránový potenciál se výrazně mění i při velmi malých množstvích iontů, proto i při velkých změnách potenciálu
nelze téměř pozorovat změnu iontových koncentrací.

Rozdělení membránových potenciálů


Membránové potenciály můžeme dělit podle vlastností membrány na:

pasivní:
klidový membránový potenciál (KMP, syn. resting potential RP),
postsynaptické potenciály (PSP):
excitační (EPSP) - glutamát,
inhibiční (IPSP) - GABA;
aktivní:
Akční potenciál (AP).

Aktivní vlastnost membrány znamená, že se vzruch po membráně šíří pomocí napěťově řízených kanálů.

Klidový membránový potenciál

Podrobnější informace naleznete na stránce Klidový membránový potenciál.

KMP je potenciál na membráně všech buněk lidského těla v klidovém stavu. Je daný dynamickou rovnováhou toku
iontů dovnitř a vně buňky. Jeho hodnota je závislá především na koncentracích K+ iontů a pohybuje se okolo −70 mV.

Postsynaptické potenciály

Podrobnější informace naleznete na stránce Postsynaptické potenciály.

PSP jsou potenciály vzniklé okolo synapse následkem otevření specifických chemicky řízených iontových kanálů
působením neurotransmiteru. Jejich šíření probíhá s dekrementem – jeho síla se s přibývající vzdáleností od zdroje
podráždění snižuje.

Akční potenciál

Podrobnější informace naleznete na stránce Akční potenciál (fyziologie).

AP vzniká pouze na buňkách obsahujících napěťově řízené iontové kanály, díky nimž se změna napětí na membráně
šíří bez dekrementu.

Odkazy
Související články

Akční potenciál
Akční potenciál versus postsynaptický potenciál
Význam akčního a postsynaptického potenciálu
Goldmannova rovnice

Zdroje

TROJAN, Stanislav a Stanislav TROJAN, et al. Lékařská fyziologie. 4. vydání. Praha : Grada, 2003. 772 s. ISBN 80-
247-0512-5.
Akční potenciál (biofyzika)
Akční potenciál (AP) je napětí na membráně odlišné od klidového membránového potenciálu, jehož hlavní
charakteristikou je, že vzniká principem vše, nebo nic – na dané membráně buď vznikne, nebo nevznikne, je tedy
principiálně digitální. Dále se šíří bez dekrementu – se vzrůstající vzdáleností od místa vzniku se jeho vlastnosti
(především amplituda) nemění, přenáší se bez zeslabení signálu. Tato schopnost souvisí s faktem, že akční potenciál
může vzniknout pouze na membráně s napěťově řízenými iontovými kanály.

Vznik AP
Vznik akčního potenciálu si vysvětlujeme tak, že následkem podráždění se otevřou iontové kanály, čímž se změní
propustnost membrány pro některé ionty. Propustnost pro ionty K+ vzroste pouze málo, avšak propustnost pro ionty
Na+ vzroste 600×. To má za následek podstatně rychlejší tok kladných iontů Na+ z extracelulárního prostředí dovnitř
buňky než iontů K+ směrem opačným. Tím se vyrovná záporný náboj uvnitř buňky a potenciál stoupá od původní
záporné hodnoty klidového potenciálu k nulové hodnotě a dokonce dojde k transpolarizaci (uvnitř buňky se stává
kladným). V další fázi (klesající) se membrána stává opět propustnější pro ionty K+ než pro ionty Na+ a potenciál
membrány se vrací ke klidové hodnotě.

Snížíme-li depolarizací hodnotu klidového potenciálu pod určitou prahovou hodnotu (tzv. prahový potenciál), odpoví
nervové vlákno (obdobně i svalové buňky nebo buňka myokardu) vznikem akčního potenciálu. Rozdíl mezi prahovým a
klidovým potenciálem činí většinou 5 až 15 mV. Je-li tedy klidový potenciál např. −70 mV, může prahový potenciál
činit −60 mV.

Zákon „vše nebo nic“


Akční potenciál vznikne v excitabilní buňce v případech, je-li vyvolán podnětem, který právě stačí snížit absolutní
hodnotu klidového potenciálu na hodnotu prahového potenciálu (prahový podnět), nebo jakýmkoli silnějším podnětem
(nadprahový podnět). Fyziologové vyjadřují tuto skutečnost jako platnost zákona „vše nebo nic“. Množství iontů
prošlých membránou v průběhu akčního potenciálu je nízké, asi 3·10−12 až 4·10−12 molů na 1 cm2 plochy membrány.
Proces vzniku akčního potenciálu je spojen jen s nepatrnými energetickými nároky. Také doba, po kterou jsou
otevřeny iontové kanály, je velmi krátká (kratší než 1 ms). Proto se akční potenciál hodnotou svého maxima pouze
přiblíží rovnovážnému membránovému potenciálnímu rozdílu pro Na+, který se nestačí plně ustavit.

Po proběhnutí akčního potenciálu následuje velmi krátká doba (asi 1 ms), tzv. absolutní refrakterní perioda, po kterou
nelze nový akční potenciál vyvolat. Poté následuje relativní refrakterní perioda (trvající 10–15 ms), po kterou lze
vyvolat potenciál, ale pouze nadprahovým podnětem.

Akční potenciál

Šíření akčního potenciálu


Funkcí nervového vlákna není jen umožnit vznik akčního potenciálu, ale i jeho šíření. Při vzniku a průběhu akčního
potenciálu na podrážděném úseku nervového vlákna dochází k iontovým tokům napříč membránou. Šíření akčního
potenciálu je podmíněno mechanismem tzv. místních proudů. Tyto místní proudy vznikají mezi vzbuzeným a
nevzbuzeným místem membrány. V místě podráždění se však polarizace membrány obrací, vnější strana je nabita
záporně, vnitřní kladně. Podrážděný úsek v této chvíli je opačně nabitý než sousední. Snaha po vyrovnání náboje vede
k podélným iontovým tokům, jež způsobí v sousedním úseku depolarizaci a tím pokles potenciálu pod prahovou
hodnotu a vznik akčního potenciálu. Tak se akční potenciál šíří podél vlákna.

Rychlost šíření akčního potenciálu je pro dané nervové vlákno konstantní veličinou a závisí na struktuře a elektrických
vlastnostech buněčné membrány, vnitřního a vnějšího prostředí. U různých buněk kolísá v rozmezí 100 ± 0,1 m·s-1.
Zvětšuje se, je-li průměr vlákna větší.

V myelinizovaných nervovových vláknech je šíření akčního potenciálu rozdílné. Tato vlákna jsou obalena myelinovou
pochvou, která je přerušovaná v 1–3 mm vzdálenostech Ranvierovými zářezy. Myelin má dobré izolační vlastnosti a
nevede elektrický proud. Místní proudy proto nemohou procházet přes myelinový obal a uzavírají se mezi jednotlivými
Ranvierovými zářezy. Vzruch se šíří od jednoho zářezu k druhému (saltatorické šíření). Tvar akčního potenciálu je v
místě zářezu stejný jako u nemyelinizovaného a šíří se až desetkrát rychleji.

Akční potenciál svalových buněk


Kosterní sval

Klidový potenciál membrány kosterního svalu se ustavuje podobně jako klidový potenciál membrány nervového
vlákna. Během činnosti svalu v něm vzniká a šíří se akční potenciál, podobným způsobem a na základě podobného
mechanizmu jako u nervových buněk. Povely k činnosti kosterního svalu přichází od zakončení axonu motorického
nervu. Různé axony jsou svým zakončením připojeny na různé počty svalových vláken. Svalová vlákna ovládaná
jedním axonem tvoří tzv. motorickou jednotku. Ta pracuje, podobně jako nervové vlákno, v souladu s pravidlem „vše
nebo nic“, takže všechna vlákna jedné motorické jednotky jsou aktivována téměř současně. Různého napětí (tonusu)
svalu se dosahuje buď změnou frekvence akčních potenciálů nebo změnou počtu motorických jednotek uváděných v
činnost.

Srdeční sval

U buněk srdečního svalu je klidový membránový potenciál (během diastoly) okolo −80 až −90 mV. V průběhu systoly
vzniká akční potenciál, který, na rozdíl od potenciálu nervového vlákna, má charakteristické plató. Fáze depolarizace
je velmi rychlá, repolarizace naopak pomalá. Celý průběh akčního potenciálu a tím i refrakterní perioda je řádově
delší než u nervového vlákna a delší než u kosterního svalu. Při depolarizaci se, podobně jako u nervové buňky,
uplatní hlavně rychlý průnik sodných kationtů iontovými kanály. V oblasti plata se uplatňují odlišné mechanismy.
Jednak se zvýší propustnost membrány pro vápenaté ionty, které pak působí podobně jako sodné ionty, takže přispějí
(spolu s ionty Na+, které mohou dále pronikat spolu s nimi nespecifickým kanálem) k udržení stavu depolarizace, a
jednak se během akčního potenciálu snižuje propustnost membrány pro ionty K+, což brání repolarizaci. Teprve po
snížení zvýšené permeability pro ionty Ca2+ se zvýší permeabilita pro ionty K+ a dojde k repolarizaci a ustanovení
klidového potenciálu. V srdci jsou centra automacie, ve kterých jsou buňky schopné spontánní depolarizace.
Spontánní depolarizace je základem srdeční automacie.

Hladké svalstvo

Membrány buněk hladkého svalu mají nižší klidový potenciál (okolo −50 mV), což je způsobeno vyšší propustností pro
ionty Na+. U některé hladké svaloviny je průběh akčního potenciálu charakterizován přítomností fáze plató, podobně
jako u srdečního svalu.

Odkazy
Související články

Akční potenciál (fyziologie)


Klidový membránový potenciál

Zdroj
KYMPLOVÁ, Jaroslava. Katalog metod v biofyzice [online]. [cit. 2012-09-20]. <https://portal.lf1.cuni.cz/clanek-
793-katalog-metod-v-biofyzice>.
Vedení elektrického proudu tělem
Za pasivní elektrické vlastnosti těla je odpovědná především kombinace ohmického odporu tělesných tekutin a
kapacity buněčných membrán.

Model pasivních elektrických vlastností


Jako elektrický model pasivních elektrických vlastností tkáně lze použít paralelní zapojení kapacitoru (idealizovaného
kondenzátoru) a rezistoru, k nimž je do série připojen další rezistor.:

Využití modelu

Protože lze jen obtížně měřit stejnosměrný odpor tkání, měří se obvykle impedance při různých frekvencích a na
základě předpokládané shody s modelem se pak stejnosměrný odpor dopočítá.

Odchylky v chování modelu a organizmu

V organizmu se však uplatňují další jevy, které způsobují někdy i výrazné odchylky chování organizmu od tohoto
modelu. Tyto jevy mají příčinu jednak ve vlastnostech organizmu a jednak v účincích elektrického proudu na
organizmus.

Z vlastností organizmu se uplatňuje především to, že organizmus se chová jako vodič II. druhu, tedy že volnými nosiči
náboje jsou ionty. Průchod stejnosměrného proudu tedy vyvolává přesuny hmoty. V praxi to znamená především to, že
elektrický odpor není nezávislý na přiloženém elektrickém napětí.

Proti pohybům náboje působí i homeostatické mechanizmy, což se v praxi projeví tak, že při průchodu nízného
stejnosměrného proudu organizmem hodnota elektrického odporu pomalu klesá a ustálí se obvykle až po několika
desítkách minut.

V případě vyšších intenzit protékajícího proudu se uplatňuje působení elektrického proudu na organizmus. Obecně lze
rozdělit účinky elektrického proudu na tepelné a dráždivé. Zatímco dráždivé účinky s rostoucí frekvencí klesají až
zcela mizí, tepelné jsou přítomny i při poměrně vysokých frekvencích.

Zvláštním případem je průchod eletrického proudu o velmi vysokých frekvencích je tzv. skin efekt (povrchový jev). Při
velmi vysokých frekvencích prochází proud prakticky jen po povrchu těla, tělo pak může být bez nebezpečí
závažnějších následků připojeno i k poměrně vysokému napětí.

Odkazy
Související články

Bioimpedance
Úrazy elektrickým proudem
Účinky elektrického proudu na organizmus

Použitá literatura

HRAZDIRA, Ivo a Vojtěch MORNSTEIN. Lékařská biofyzika a přístrojová technika. 1. vydání. Brno : Neptun, 


2001. 396 s. ISBN 80-902896-1-4.
Molární vodivost
Závislost vodivosti na koncentraci iontů vyjadřuje molární vodivost:

kde κ=měrná vodivost, c=koncentrace daného iontu.

Biologické membrány mají velkou kapacitu a mají o 8 řádů vyšší odpor než protoplasmatické prostředí => špatně
vodivé pro střídavý (kvůli kapacitanci) i stejnosměrný proud.
Elektrická impedance
Elektrická impedace je rozšířením pojmu elektrický odpor na situace, kdy prostředím prochází střídavý elektrický
proud. Nejjednodušším pohledem na impedanci je ten, že se jedná o odpor kladený střídavému proudu. Jednotkou
impedance je ohm Ω, obvykle se značí písmenem Z. Je-li impedance připojena k napětí U a protéká-li jí proud I, je její
hodnota dána Ohmovým zákonem:

Impedance elektrických prvků


Základními elektrickými prvky jsou rezistor, kapacitor a induktor. Základní vlastností rezistoru je elektrický odpor,
základní vlastností kapacitoru je kapacita a základní vlastností induktoru je indukce. Jde pochopitelně o idealizované
modely, pro zdůraznění tohoto faktu se používají tyto termíny a nikoliv technické názvy odpor, kondenzátor a cívka.

Při výpočtu impedancí se obvykle nepoužívá frekvence f, ale kruhová frekvence ω určená vztahem:

Impedance rezistoru

Impedance samotného rezistoru se nazývá rezistance, značí se R. Hodnota rezistance nezávisí na frekvenci.

Impedance kapacitoru

Impedance kapacitoru se nazývá kapacitance, značí se obvykle XC. Kapacitance je nepřímo úměrná kapacitě C
kapacitoru a nepřímo úměrná frekvenci f přiloženého napětí:

Impedance induktoru

Impedance induktoru se nazývá induktance, značí se obvykle XL. Induktance je přímo úměrná indukčnosti L
induktoru a přímo úměrná frekvenci f proudu protékajícího induktorem:

Impedance sériového zapojení rezistoru a kapacitoru

Impedance nelze zcela snadno sčítat, pro impedanci Z sériového zapojení rezistoru R a kapacitoru C platí:

Impedance paralelního zapojení rezistoru a kapacitoru

Vztah pro impedanci Z paralelního zapojení rezistoru a kapacitoru má poměrně složitý tvar, stojí však za pozornost,
protože paralelní zapojení rezistoru a kapacitoru je často používaným modelem impedance tkáně:

Elektrická impedance tkání


Elektrická impedance tkání charakterizuje pasivní elektrické vlastnosti organismu a tkání. To znamená, že popisuje
chování organismu jako spotřebiče elektrické energie, je-li připojen ke zdroji střídavého elektrického napětí.

Model elektrické impedance


K modelování elektrické impedance tkání lze použít jednoduchý model spočívající v paralelním zapojení rezistoru R1 a
kapacitoru C a do série k nim připojeného rezistoru R2:
Není příliš obtížné vyjádřit závislost impedanci modelu jako funkci odporu obou rezistorů, kapacity kapacitoru a
frekvence přiloženého napětí, výsledný vztah by však vypadal poměrně složitě.

Rozborem úlohy však lze chování impedance v závislosti na frekvenci snadno odhadnout. Pro stejnosměrný proud se
chová kapacitor jako rozpojený vodič, takže výsledný model je tvořen pouze sériovým zapojením rezistorů R1 a R2.
Pro velmi vysokou frekvenci se bude kapacitor chovat téměř jako zkrat, takže přemostí rezistor R1 a na impedanci se
bude podílet pouze rezistor R2. Poněkud méně zřejmé je to, že impedance bude klesat plynule, nikde nebude mít
lokální maximum.

Kapacitor C modeluje celkovou kapacitu ve tkáni, zejména kapacitu buněčných membrán. Kapacita buněčných
membrán může být značná, plošná kapacita buněčné membrány je 1 μF · cm-2. Na kapacitě se však podílí např. i
kapacita plošných vazivových struktur, takže nelze ztotožnit kapacitu v modelu s kapacitou buněčných membrán.
Rezistor R1 modeluje elektrickou vodivost tělesných tekutin, především tekutiny extracelulární. Ovšem ani tekutina
intracelulární není zcela bez vlivu na hodnotu parametru R1. Rezistor R2 odpovídá především kožnímu odporu, ovšem
dílem na něj má vliv i vodivost tělesných tekutin.

Detailnější analýza zohledňuje např. následující jevy:

tělesné tekutiny jsou vodičem II. druhu, tedy vodivost není nezávislá na proudové hustotě a na frekvenci
polarizace makromolekul probíhá různě při různých frekvencích, tedy permitivita prostředí není nezávislá na
frekvenci
struktura tkáně je spíše než homogennímu prostředí podobná suspenzi dielektrických kuliček s vodivým jádrem
ve vodivém prostředí
tkáněmi proudí vodivá krev
vodivost buněčných membrán excitabilních buněk se mění podle jejich stavu

Modely sestavené na základně těchto předpokladů jsou však poměrně komplikované, přitom jejich užitečnost není
podstatně vyšší než užitečnost výše uvedeného modelu.

Využití elektrické impedance tkáně


Bioimpedanční analýza

Elektrická impedance tkáně je ovlivněna mimo jiné i složením a uspořádáním tkáně. Znalost o elektrické impedanci
tak lze použít ke získání informace o složení těla. Informace o impedanci se tak využívá v následujících případech:

analýza celkového tělesného tuku (bioimpedanční váha)

hodnocení dehydratace a dynamiky tělesných tekutin

Tomografické systémy

Víceméně experimentálně se používají metody, které by po rozmístění velkého počtu elektrod na těle pacienta
výpočetně rekonstruovaly rozložení vodivosti jednotlivých segmentů uvnitř těla, tzv. elektroimpedanční tomografie.
Metodika naráží na obtíže především ve dvou oblastech. První oblastí obtíží je nutnost používat relativně nízkých
proudů, tedy poměrně velká citlivost na rušení. Druhou oblastí obtíží je to, že metody matematické rekonstrukce jsou
poměrně náročné, mimo jiné z toho důvodu, že naprosto nelze použít představu nějakého "proudového paprsku". Lze
se setkat s následujícími aplikacemi:

elektroimpedanční tomografie prsu - nabízena jako alternativa nebo doplněk mammografie


elektroimpedanční tomografie hrudníku - včasná detekce edému plic u pacientů na intenzivních lůžkách

Odkazy
Související články

Elektrická impedance/NMgr
Vedení elektrického proudu tělem
Měření proudu
Měření odporu
Měření napětí

Zdroj
KUBATOVA, Senta. Biofot [online]. [cit. 2011-01-31]. <https://uloz.to/!CM6zAi6z/biofot-doc>.
Účinky různých druhů proudu na organizmus
Druh proudu elektrolytické účinky dráždivé účinky tepelné účinky
stejnosměrný + jen při náhlé změně, zapnutí nebo vypnutí –
střídavý nízkofrekvenční slabé + –
střídavý vysokofrekvenční – – +

Vodné prostředí organismu obsahuje ionty – elektrolyt. Stejnosměrný proud má tedy na organismus elektrolytické
účinky a proudové hustoty okolo 0,5 mA/cm2 mohou způsobit vážné poškození tkáně.

Střídavý nízkofrekvenční (50–500 Hz) proud má pouze slabé elektrolytické účinky, protože polarita elektrod se
periodicky střídá. Má však výrazné dráždivé účinky, které se na svalu projeví jeho záškubem (a při průchodu
srdečním svalem může způsobit jeho zástavu).

Střídavý vysokofrekvenční proud nemá elektrolytické účinky a při frekvencích nad 100 kHz už se vůbec neuplatňují
ani dráždivé účinky. Plně se zde uplatňují tepelné účinky.
Měření vodivosti roztoků
Obsahuje-li roztok rozpuštěnou látku, je jeho měrná vodivost přímo úměrná koncentraci této látky, takže z hodnot
vodivost lze vyvodit koncentraci.

R = odpor vodiče při průchodu el. proudu, ρ = měrný odpor (charak. pro materiál vodiče), l = délka vodiče
(vzdálenost elektrod v elektrolytu), q = průřez vodiče (plošná velikost elektrod)

Po změření odporu roztoku a po experimentálním určení poměru l/q můžeme vypočítat ze vztahu měrnou vodivost κ.

Odkazy
Související články

Měření proudu
Měření odporu
Měření napětí

Zdroj

KUBATOVA, Senta. Biofot [online]. [cit. 2011-01-31]. <https://uloz.to/!CM6zAi6z/biofot-doc>.


Osciloskop
Osciloskop je elektronický měřící přístroj zobrazující průběhu napětí v čase.

Osciloskopy podle technického provedení dělíme na analogové, digitální a softwarové.

Princip analogového osciloskopu

Schéma analogového osciloskopu

V principu se osciloskop skládá ze tří částí:

obrazovka (obrazová elektronka)


vertikální zesilovač
generátor časové základny

K těmto částem je pochopitelně třeba přidat zdroje vysokého napětí potřebného k dostatečnému urychlení elektronů v
obrazovce a dostatečně tvrdých proudových zdrojů pro žhavení.

Obrazová elektronka

Pro princip analogového osciloskopu je klíčová obrazovka s elektrostatickým vychylováním, jejíž princip je znázorněn
na obrázku. Zhruba lze princip shrnout do konstatování, že obrazovka funguje tak, že jsou v jedné její části emitovány
elektrony, ty jsou zaostřeny, cíleně vychylovány a dopadají na světélkující látku. Z toho je patrné, že podmínkou
fungování obrazovky je vakuum uvnitř obrazovky. Dále je třeba poznamenat, že právě elektrostatickým vychylováním
se tato obrazovka dosti podstatně liší od obrazovek televizních, které k vychylování elektronů používají magnetického
pole cívek.

První částí obrazovky, která je na obrázku i při obvyklém použití obrazovky nejdále od nás, je elektronové dělo nebo
též katodová trubice. Dokonce anglické pojmenování (cathode ray tube) dalo základ označení monitorů CRT.
Elektronové dělo začíná přímo žhavenou katodou, tedy v principu drátkem z materiálu odolného vysoké teplotě. Při
jeho zahřívání se do okolí uvolňují elektrony (termoemise elektronů), jejich množství závisí na teplotě a tedy na
žhavicím proudu. Katoda je zároveň nositelem záporného potenciálu vůči dále umístěné anodě. Mezi anodou a
katodou tedy vzniká elektrické pole, které přitahuje elektrony k anodě. Protože na elektrony v elektrickém poli působí
síla, jsou samozřejmě urychlovány. Mezi katodou a anodou jsou umístěny další elektrody se záporným potenciálem,
jejichž úkolem je fokusace elektronového paprsku. Vlastní anoda je uzpůsobena tak, aby elektronový paprsek alespoň
částečně prolétl a mohl pokračovat dále.

Za elektronovým dělem se nachází systém vychylovacích elektrod. V principu jde o dva páry destiček, jeden pár je
umístěn horizontálně a druhý pár vertikálně. Podle polarity a velikosti napětí připojeného je elektronový paprsek
vychýlen doleva či doprava v případě horizontálních vychylovacích destiček (jejichž plošky jsou svislé!), respektive
nahoru či dolů v případě vertikálních vychylovacích destiček. Úhel, jakého lze dosáhnout, se obvykle pohybuje kolem
15°, takže k tomu, aby bylo stínítko dostatečně velké, musí být obrazovka poměrně dlouhá.

Poslední částí obrazovky je stínítko pokryté luminoforem. To je látka, která po dopadu elektronů přejde do
excitovaného stavu a do základního stavu se vrací vyzařováním fotonů ve viditelné oblasti. Stínítko bývá doplněno
filtry, protože při provozu obrazovky vzniká i rentgenové záření.

pozn. 1: Magnetické vychylování sice dosahuje mnohem vyšších úhlů, ale má hned dvě nevýhody, díky kterým nelze použít v osciloskopu. První
nevýhodou je, že výchylka elektronového paprsku závisí na proudu procházejícím vychylovací cívkou nelineárně, takže by bylo třeba do
osciloskopu tuto nelinearitu obtížně zavádět. Druhou nevýhodou je nárůst impedance vychylovacích cívek s frekvencí. Již při poměrně nízkých
frekvencích dosahuje takových hodnot, že přestává být prakticky realizovatelným vhodný zesilovač.

pozn. 2: Vlastnosti luminoforu mohou být různé dle aplikací. U televizoru nebo u běžného osciloskopu se požaduje, aby luminofor velmi rychle
vyhasl. V jiných aplikacích se naopak požaduje, aby svítil co nejdéle. Takovými aplikacemi byly např. radary (požadavek, aby luminofor svítil tak
dlouho, aby operátor zaregistroval cíl) nebo paměťové osciloskopy zaznamenávající jednorázové děje (požadavek, aby stopa na stínítku svítila tak
dlouho, dokud nebude vyhodnocena nebo vyfotografována).

pozn. 3: U černobílých televizorů bylo žádoucí bílé světlo, u osciloskopů se stalo standardem zelené. Barevné obrazovky se v principu neliší, jen je
jejich systém "ztrojený" pro červenou, zelenou a modrou barvu. Vnímaná barva pak vznikne kombinací různých intenzit jednotlivých složek.

Vertikální zesilovač
Vertikální zesilovač slouží k zesílení měřeného napětí natolik, aby bylo schopno po přivedení na vertikální vychylovací
destičky dostatečně vychýlit elektronový paprsek v obrazovce. Přirozenými požadavky jsou vysoký vstupní odpor (aby
co nejméně zatížil měřený obvod) a široká frekvenční charakteristika (aby co nejméně zkresloval rychlé děje).
Nastavitelným zesílením se přepíná napěťový rozsah.

Většina osciloskopů má vstupní zesilovače dva, nazývají se kanál A a B. Přepínáním je pak umožněno zobrazit kanál A,
kanál B nebo oba průběhy současně.

pozn. 1: K zobrazení dvou průběhů se používají dva režimy. V režimu ALT, který je vhodnější pro signály s vyšší frekvencí, se střídavě zobrazují
kanály A a B. V režimu CHOP, ve kterém se s vysokou frekvencí přepíná mezi kanály A a B je vhodnější pro signály s nižší frekvencí.

pozn. 2: Dalším obvyklým režimem je režim XY, při kterém je signál z jednoho kanálu přiveden na horizontální a z druhého na vertikální
vychylovací destičky. Toto uspořádání slouží např. k měření frekvence srovnáním s etalonem pomocí Lissajouseových obrazců.

Časová základna

Časová základna slouží k řízení pohybu elektronového paprsku zleva doprava (z pohledu obsluhy) při běžném provozu.
Vlastně nejde o nic jiného než o generátor napěťových pulsů pilovitého tvaru. Při pomalejším vzrůstu napětí se
elektronový paprsek pohybuje a zanechává stopu. Při rychlejším pokesu, tzv. zpětném běhu, je technicky zajištěno, že
se elektronový paprsek přeruší. Nastavením frekvence časové základny se mění časové rozlišení osciloskopu.

Začátek pilového pulsu může být určen několika způsoby. Nejjednodušší je volně běžící časová základna, při které
jsou generovány pilové pulsy s pevnou frekvencí. To ovšem v obecném případě může vést k tomu, že se obraz na
stínítku pohybuje. Řešením je buď jemná korekce frekvence časové základy (časová lupa), nebo použití spouštěné
časové základy. Ve druhém případě je pilový puls vygenerován jen za daných podmínek, obvykle že napětí na vstupu
dosáhne jisté hodnoty.

pozn.: Většina osciloskopů má možnost ještě tzv. externího spouštění. Tedy že se do osciloskopu přivede další signál sloužící výhradně k řízení
časové základny.

Digitální osciloskopy
Digitální osciloskop se v principu skládá z několika částí:

zesilovač
A/D převodník
vestavěný počítač
LCD displej

Zesilovač slouží především k zesílení příliš slabých signálů a k nastavování rozsahu. Požadavky jsou obvyklé, tedy co
největší vstupní odpor a co nejlepší frekvenční charakteristika.

Analogově digitální převodník slouží k převodu napětí na výstupu vstupního zesilovače do číslicové podoby. Je
požadováno, aby převáděl co nejrychleji, aby bylo možné sledovat dostatečně rychlé děje, tedy aby byla co nejvyšší
vzorovací frekvence (počet měření za sekundu). Na straně druhé je požadováno, aby byla co nejmenší diskretizační
chyba, tj. aby bylo napětí převedeno na co největší počet číslic. Bohužel při technické realizaci jsou tyto požadavky do
jisté míry protichůdné.

Vestavěný počítač provádí analýzu měřeného signálu a jeho zobrazení na LCD displeji. Starší a levnější typy
digitálních osciloskopů umožňují jen jednoduchou analýzu spočívající nanejvýš ve změření napětí (někdy i
maximálního, efetivního a středního) a frekvence. Moderní typy umožňují provádět poměrně pokročilou analýzu
signálu (Fourierova analýza, výkonové spektrum), vestavěný počítač pak umožňuje bezproblémový export naměřených
dat do osobního počítače k další analýze nebo třeba připojení osciloskopu na sběrnici (např. GPIB), která umožňuje
řídit složitější měření z jednoho počítače.

Softwarový osciloskop
Softwarový osciloskop je založen na stejném principu jako osciloskop digitální. Liší se především tím, že využívá
komponenty běžného počítače, na kterém je spuštěn. Jako vstup používá např. vstup na zvukové kartě (po lehké
modifikaci), který obsahuje zesilovač i A/D převodník. Výhodou je nízká pořizovací cena, existují i freeware nebo open
sourcce osciloskopy. Nevýhodou je především to, že zvukové karty mají fixní a ve srovnání s osciloskopy velmi nízkou
vzorkovací frekvenci. Další nevýhodou je omezený napěťový rozsah vstupu zvukové karty. Problémem i to, že vstupy
zvukových karet obsahují nízkofrekvenční filtr, takže příliš pomalé děje nebudou změřeny. Nakonec může být
problémem i náchylnost ke zničení.

Měřící karta
Měřící karta je kompromisem mezi digitálním a softwarovým osciloskopem. Vlastně jde o zesilovač a A/D převodník s
nezbytnou elektronikou, který lze podle typu buď zabudovat do počítače nebo připojit k USB. Ani měřící karta
nedosahuje kvalit digitálního osciloskopu[zdroj?], ale zejména pokud jde o úlohy s nižšími frekvencemi (např. většina
měření fyziologických dějů), pak jde o řešení poměrně levné a přitom dostatečně kvalitní.

Odkazy
Externí odkazy

Osciloskop (česká wikipedie)


Oscilloscope (anglická wikipedie)
Princip osciloskopu na webu MFF UK
Podrobný popis osciloskopu na stránkách Integrované střední školy
KUBATOVA, Senta. Biofot [online]. [cit. 2011-01-31]. <http://uloz.to/1162346/biofot.doc>.
Měření proudu

Ampérmetr

Měření pomocí ampérmetru


Měřidla elektrického proudu se zapojují sériově do té části obvodu, ve které má být proud změřen. Rozsah
ampérmetru zvětšíme n-krát, předřadíme-li mu v obvodu paralelně odpor (n-1)krát menší, než je jeho vlastní.

Kompenzační metoda
Měření můžeme též provést kompenzační metodou: místo měřidla zapojíme určitý známý odpor, který nám zmenší
napětí, které pomocí kompenzátoru změříme a ze vztahu R=U/I vypočítáme hodnotu proudu. Velmi malé proudy
měříme pomocí galvanometru (citlivost až 10-10A).

Odkazy
Související články

Měření napětí
Měření odporu
Měření vodivosti roztoků

Externí odkazy
Měření elektrického proudu
Ampérmetr (česká wikipedie)

Zdroj

KUBATOVA, Senta. Biofot [online]. [cit. 2011-01-31]. <https://uloz.to/!CM6zAi6z/biofot-doc>.

Článek neobsahuje vše, co by měl.


Můžete se přidat k jeho autorům a jej.
O vhodných změnách se lze poradit v diskusi.
Měření napětí

Voltmetr

Elektrické napětí U je definováno jako rozdíl elektrických potenciálů mezi dvěma body vodiče. Podle průběhu v čase
rozlišujeme napětí na stejnosměrné (rozdíl potenciálů nemění svou orientaci) nebo střídavé (polarita je periodickou
funkcí času).

Pro měření napětí používáme voltmetry (1V = 1J/1C) které k měřenému obvodu připojujeme paralelně. Ideální
voltmetr by měl mít nekonečný vnitřní odpor. Při měření střídavého napětí musíme rozlišovat mezi okamžitou,
maximální, efektivní a střední hodnotou napětí.

Odkazy
Související články
Měření proudu
Měření odporu
Měření vodivosti roztoků

Zdroj

KUBATOVA, Senta. Biofot [online]. [cit. 2011-01-31]. <https://uloz.to/!CM6zAi6z/biofot-doc>.


Měření odporu

Různé druhy rezistorů,


tedy elektrotechnických
součástek vyznačujících
se především přesně
definovaným
elektrickým odporem.

Elektrický odpor je vlastností konkrétního vodivého tělesa. Pro jednoduchost předpokládejme, že je takové těleso
vybaveno dvěma vodivými svorkami a nikudy jinudy do něj elektrický proud nevstupuje. Mezi svorkami bude
elektrické napětí U a přívodním vodičem bude protékat elektrický proud I. Elektrický odpor lze zavést například
pomocí Ohmova zákona (i integrálním tvaru) jako konstantu úměrnosti mezi přiloženým napětím U a protékajícím
proudem I:

Jednotkou elektrického odporu je Ohm [Ω], často se používají násobky kΩ a MΩ. Převrácená hodnota elektrického
odporu se nazývá elektrická vodivost G, její jednotkou je Siemens [S].

Elektrický odpor i elektrická vodivost jsou konstanty charakterizují konkrétní těleso, vedle materiálu jsou ovlivněny i
jeho geometrickou konfigurací. K charakterizaci konkrétního materiálu se používají veličiny:

měrná elektrická vodivost (konduktivita) γ [S·m-1]


měrný elektrický odpor (rezistivita) ρ [Ω·m]

Měření elektrického odporu


Ohmova metoda

Elektrický odpor se obvykle měří přímo podle Ohmova zákona (jeho tzv. integrální formy), tedy tak, že se tělesem
nechá protékat elektrický proud a změří se úbytek napětí, ke kterému dojde na tělese. Odpor se pak spočítá podle
vztahu:

Elektrický proud měříme ampérmetrem a úbytek elektrického napětí voltmetrem. Protože je obvykle nutné měřit
napětí i proud současně, jsou možná dvě uspořádání:

A. Zapojení vhodné pro měření malých odporů B. Zapojení vhodné pro měření velkých odporů

Obě uspořádání jsou však svým způsobem problematická. V případě zapojení A je na voltmetru skutečné napětí na
měřeném odporu, ovšem ampérmetr ukazuje součet proudů procházejících měřeným odporem i voltmetrem. Toto
uspořádání je vhodné tam, kde je proud procházející měřeným odporem podstatně vyšší než proud procházející
voltmetrem, tedy pokud bude měřený odpor poměrně malý. Při známé hodnotě odporu voltmetru RV lze provést
korekci a neznámý odpor vypočítat podle vztahu:

V zapojení B je sice přesně změřen proud protékající odporem, ovšem změřený úbytek napětí je navýšen o úbytek
napětí na ampérmetru. Toto uspořádání bude vhodné tam, kde je úbytek napětí na měřeném odporu podstatně vyšší
než úbytek napětí na ampérmetru, tedy při poměrně vysokých hodnotách měřeného odporu. Při známé hodnotě
vnitřního odporu ampérmetru lze provést korekci:

Substituční metoda
Substituční metoda měření elektrického odporu je realizována tak, že ke zdroji stálého elektrického napětí se připojí
neznámý odpor a změří se protékající elektrický proud. Posléze se připojí ocejchovaný proměnlivý odpor (obvykle
odporová dekáda) a jeho hodnota se zkusmo nastavuje tak, aby obvodem procházel stejný proud, jako když byl to
obvodu připojen neznámý odpor. Prochází-li stejný proud, je hodnota na odporové dekádě rovna neznámému odporu.
Neznámý odpor se tak substituuje známým odporem tak dlouho, až se podaří vybrat odpor, kterým protéká stejný
proud.

Analogicky je možné provádět měření se zdrojem stálého elektrického proudu. Jedinou modifikací by bylo, že bychom
měřili úbytky napětí.

Další metody

K měření elektrického odporu se v některých případech používají i další metody. Jedná se zejména o metodu
srovnávací a metodu můstkovou.

Odkazy
Související články
Měření proudu
Měření napětí
Měření impedance
Měření vodivosti roztoků

Zdroj

KUBATOVA, Senta. Biofot [online]. [cit. 2011-01-31]. <https://uloz.to/!CM6zAi6z/biofot-doc>.

Literatura
HAASZ, Vladimír a Miloš SEDLÁČEK. Elektrická měření: Přístroje a metody. 1. vydání. Praha : Vydavatelství
ČVUT, 1998. 327 s. ISBN 80-01-01717-6.

Externí odkazy

učebnice Praktická elektronika na WikiKnihách


Použití elektřiny v diagnostice
K diagnostice lze použít měření aktivních i pasivních elektrických vlastností lidského těla. V praxi zdaleka převažuje
využití aktivních elektrických vlastností lidského těla, tedy toho, že se některé tkáně chovají jako zdroj elektrického
napětí.

Přehled elektrodiagnostických metod


Metody využívající aktivních elektrických vlastností
Elektrokardiografie (EKG) – registrace potenciálů vznikajících v důsledku činnosti myokardu
Elektroencefalografie (EEG) – registruje potenciály vznikající v mozku, snímá se z kůže na hlavě
Elektrokortikografie (ECoG) – signály se snímají přímo z povrchu obnažené mozkové kůry při mozkových
operacích
Elektromyografie (EMG) – potenciály kosterního svalstva; buď z elektrod zavedených do svalu nebo z
povrchu kůže nad příslušným svalem
Elektroretinografie (ERG) – snímání akčních potenciálů oční sítnice pomocí elektrod přiloženým na
rohovku a spánek pacienta
měření elektrické aktivity GIT
měření elektrické aktivity sfinkterů

Metody využívajících pasivních elektrických vlastností


měření kožního odporu v polygrafu ("detektor lži")
bioimpedanční měření množství tělesného tuku
bioimpedanční měření hydratace
bioimpedanční tomografie

Odkazy
Zdroj

KUBATOVA, Senta. Biofot [online]. [cit. 2011-01-31]. <https://uloz.to/!CM6zAi6z/biofot-doc>.


Elektrokardiografie

Elektrokardiografie (EKG) je základní vyšetřovací metoda v kardiologii. Jejím principem je snímání elektrické
srdeční aktivity a v podobě elektrokardiogramu (časový záznam EKG křivek) umožňuje její hodnocení. EKG
vyšetření je většinou neinvazivní. Pomocí elektrod umístěných na kůži, ale i na stěně jícnu či přímo v srdci, měříme
napětí (rozdíl elektrických potenciálů) jako projev šíření akčního potenciálu myokardem.

Protože je elektrická aktivita srdce podmínkou mechanické, má EKG důležitou diagnostickou roli u řady srdečních
chorob. Navíc s jeho pomocí dokážeme odhalit i poruchy extrakardiálních příčin (např. poruchy činnosti štítné žlázy,
iontové dysbalance), přesto samotné vyšetření EKG nemá takovou výpovědní hodnotu a vždy je důležité posuzovat
celkový klinický obraz.

V rámci zefektivnění prevence kardiovaskulárních chorob se do praxe zavádí také telekardiologie, díky které lze
pacienty s kardiovaskulárními problémy dlouhodobě a efektivně sledovat.

Princip snímání EKG

Během šíření akčního potenciálu myokardem vznikají v oblastech rozhraní rozdílného potenciálu místní elektrické
proudy, to vede ke vzniku elektromagnetického pole. Tělesné tekutiny fungují jako dobré vodiče, čímž lze snímat
změny srdečních potenciálů i z povrchu těla.

W. Einthoven
(1860 – 1927)

Snímání těchto potenciálů zavedl na počátku 20. století holandský fyziolog Willem Einthoven. Byly to dnešní 3
standardní bipolární končetinové svody (I, II, III) tvořící tzv. Einthovenův trojúhelník, v jehož pomyslném těžišti
leží srdce. Principem těchto svodů je zapojení vždy dvou aktivních elektrod, jejichž polarita je předem dána. Jednotlivé
svody poté zaznamenávají rozdíl potenciálů mezi elektrodami.

Dnes je měření EKG zdokonaleno přidáním dalších svodů. Jsou to unipolární svody, které vznikají spojením aktivní
elektrody s indiferentní elektrodou – Wilsonovou svorkou. Tak získáme další končetinové (VR, VL, VF) a hrudní (V1–
V6) svody. Záznam z končetinových unipolárních svodů lze zesílit, pokud odpojíme aktivní elektrodu od nulové svorky,
poté měříme potenciál mezi odpojenou a dvěma zbylými elektrodami. Získáme tak tzv. zvýšené (augmented)
končetinové pseudo-unipolární Goldbergerovy svody (aVR, aVL, aVF). Nejběžnější EKG záznam je tedy v
současnosti 12svodový. Za speciálních okolností je možné použít i další svody.

Podrobnější informace naleznete na stránce Elektrokardiografie/Fyzika.


Video v angličtině, základy EKG.

12svodové EKG
svod zapojení svodu

levá horní pravá horní


I
končetina končetina
bipolární
končetinové
svody levá dolní pravá horní
II + končetina - končetina
Einthovenovy
levá dolní levá horní
III
končetina končetina

aVR zvýšený svod na pravé horní končetině


unipolární
končetinové
svody aVL zvýšený svod na levé horní končetině

Goldbergerovy

aVF zvýšený svod na levé dolní končetině

V1 IV. mezižebří, parasternálně vpravo

V2 IV. mezižebří, parasternálně vlevo

V3 mezi V2 a V4

V. mezižebří (u žen lepíme na linii


V4 dolního úponu prsu), levá
medioklavikulární čára

V5 V. mezižebří, levá přední axilární čára

unipolární
hrudní svody

Wilsonovy
V6 V. mezižebří, levá střední axilární čára

Barva končetinových elektrod:

červená – pravé předloktí,


žlutá – levé předloktí,
zelená – levý bérec,
černá – pravý bérec (uzemnění).

Správné umístění elektrod je velice důležité, protože umožňuje správně odečítat výslednou EKG křivku vzhledem k
umístění srdce jako celku i jednotlivých srdečních oddílů.

EKG křivka

Šíření depolarizace myokardem se zobrazuje rozdílně vzhledem k tomu, který svod potenciál snímá, to znamená ve
kterém směru a ve které rovině je umístěn vzhledem k srdci. Dráha šíření potenciálu v srdci má typický charakter a
vytváří tak typické výchylky – vlny, kmity a linie, které odpovídají určité fázi elektrického srdečního cyklu.

P vlna – depolarizace síní,


PQ interval – převod vzruchu ze síní na komory,
QRS komplex – depolarizace komor,
kmit Q = negativní kmit předcházející kmitu R,
kmit R = každý pozitivní kmit QRS komplexu,
kmit S = negativní kmit následující za kmitem R,
T vlna – repolarizace komor,
U vlna – není konstantní, její původ není úplně jasný.

Akční potenciál myokardu normálně vzniká spontánní depolarizací v sinoatriálním uzlu, odkud se šíří na svalovinu
síní. Okamžitý výsledný vektor má díky tenké stěně síní relativně malou amplitudu a směřuje doleva dolů. Na EKG
záznamu se projeví jako vlna P. Pomalým vedením v atrioventrikulárním uzlu dochází ke zbrzdění postupu
depolarizace ze síní na komory kvůli oddělení systoly síní od systoly komor. Na EKG tomu odpovídá izoelektrická
linie. Celkový převod depolarizace ze síní na komory značí PQ interval.

Následuje depolarizace komor, tvořící na EKG záznamu QRS komplex. Vzruch postupuje skrze Hisův svazek a
Tawarova raménka na svalovinu mezikomorového septa, kde se depolarizace šíří zleva doprava, čímž okamžitý
vektor směřuje doprava dolů. Na EKG se píše buď negativní kmit Q, či pozitivní kmit R, záleží na svodu. Další
postup vzruchu se šíří k srdečnímu hrotu, okamžitý vektor směřuje doleva dolů, vytváří střední část QRS komplexu,
ve většině svodů kmit R. Odtud se depolarizace přes Purkyňova vlákna rozšíří na pracovní myokard obou komor a to
směrem od endokardu k epikardu, v okamžitém vektoru se projeví především mohutnější svalovina levé komory,
směřuje tedy doleva, a na závěr doleva nahoru při depolarizaci bazální části levé komory, a tím dokončuje komorový
QRS komplex.

Po skončení depolarizace je elektrická srdeční aktivita chvíli nulová, svalová vlákna se nachází ve fázi plató, kdy se
žádné elektrické proudy myokardem nešíří. Na EKG záznamu se projeví jako izoelektrická linie – ST úsek. Po tomto
okamžiku začíná komorová repolarizace, která postupuje opačně a to od epikardu k endokardu. Výsledný
sumační vektor je ale stejný jako při depolarizaci, a to kvůli tomu, že repolarizace je elektricky opačný děj. Vytváří se
tedy vlna T. Někdy lze na EKG záznamu zaznamenat vlnu U, jejíž původ není přesně znám, pravděpodobně jde o
projev repolarizace papilárních svalů.

Elektrokardiogram
EKG se snímá na milimetrový papír. Pro správně odečtení hodnot musíme mít stanovený:

tzv. cejch 1mV (odpovídá amplitudě) obvykle = 10 mm,


rychlost posunu papíru buď 50 mm/s (1 mm → 0,02 s) nebo 25 mm (1 mm → 0,04 s).

Vyhodnocení EKG záznamu se skládá z deseti bodů – tzv. DESATERO:

1. srdeční akce

2. srdeční rytmus

3. srdeční frekvence

4. vlna P

5. PQ interval

6. QRS komplex

7. ST úsek

8. vlna T

9. QT interval

10. osa srdeční

Podrobnější informace naleznete na stránce Popis EKG.

Hodnocení EKG záznamů nám umožňuje odhalovat arytmie, jako projevy poruch tvorby, či vedení vzruchu.
Významnou roli hraje také při zjišťování ischemických změn, lokalizace i stádia infarktu myokardu. Změny na EKG
nacházíme buď ve všech svodech, nebo jen v jednom, či skupině svodů, které spolu vzhledem k anatomii srdce
souvisejí. Např. zmíněnou lokalizaci infarktu myokardu (IM) dokážeme určit podle znalosti jeho cévního zásobení a
projekce srdečních oddílů na povrch těla v souvislosti právě se směrem snímání jednotlivých svodů.

lokalizace změn (IM) zaznamenávající svody


laterální I, aVL, V5, V6
diafragmatický (spodní) II, III, aVF
anteroseptální V1–V3
celá přední stěna V1–V6
dutinový aVR

Příklady EKG
Zde jsou ukázky EKG pacientů z interního oddělení:

Věk pacienta: 89 Věk pacienta: 88 Věk pacienta: 56 Věk pacienta: 84 Věk pacienta: 60 Věk pacienta: 83
let let let let let let

Věk pacienta: 38 Věk pacienta: 84


let let

Další EKG svody


Pro vyšetření všech srdečních oddílů tradiční dvanáctisvodové EKG plně nestačí, úžíváme další přídatné svody např.
pro hodnocení elektrické aktivity pravé síně a komory a za dalších speciálních situací. Jsou to unipolární svody, z nichž
nejčastěji se používají tzv. etážové svody, které se označují apostrofem. Pomocí jícnových svodů můžeme sondou
dosáhnout na oblast levé síně i zadní stěny levé komory, umístění sondy se stanovuje v cm od kraje zubů, nebo vstupu
do nosu.

svod zapojení svodu

V7 zadní axilární čára vlevo v úrovni V6

V8 skapulární čára vlevo v úrovni V6

unipolární hrudní svody V9 paravertebrální čára vlevo v úrovni V6

VE vlevo od processus xiphoideus

V3R - V6R vpravo zrcadlově k V3-V6

V1´– V6´ o 1 mezižebří výše než V1-V6

unipolární hrudní svody - etážové


V1´´– V6´´ o 2 mezižebří výše než V1-V6

jícnové svody E/Oe např. 37,5 cm (levá síň)

Další používané svody – Nehbův trojúhelník (bipolární D, A, I); Frankův korigovaný ortogonální systém (7
elektrod); Korigovaný ortogonální systém McFee-Parungao (9 svodů).

Odkazy
Související články
Skorovací systémy (pediatrie)
Popis EKG
Procvičování EKG
Elektrokardiografie/Fyzika

Externí odkazy
Elektrokardiografie (česká wikipedie)
Kardioblog – EKG pro pokročilé
1400 stranová EKG kniha (TECHmED)
Úvod do EKG – prof. Jan Malík

Zdroj

HAVRÁNEK, Jiří: Monitoring v intenzivní péči.

Použitá literatura
VOKROUHLICKÝ, L a J KVASNIČKA. Základy elektrografie. 1. vydání. Praha. 1984. 

TROJAN, Stanislav, et al. Lékařská fyziologie. 4. vydání. Praha : Grada, 2003. 772 s. ISBN 80-247-0512-5.


Elektroterapie
Způsoby využívané v elektroterapii jsou:

Galvanoterapie
Iontoforéza
Elektroforéza
Magnetoterapie

Stejnosměrný proud nízkofrekvenční

Diadynamické proudy
TENS (transkutánní elektrická neurostimulace)
Träbertovy proudy

Střídavý proud středofrekvenční

Interferenční proudy

Střídavý proud vysokofrekvenční

Krátkovlnná diatermie
Ultrakrátkovlnná diatermie
Mikrovlnná diatermie

Odkazy
Zdroj

Návod na praktickou úlohu - Elektrodiagnostika a elektroléčba

Související články

Elektrostimulační metody
Použití elektřiny v terapii
Vlastnosti zvuku
Zvuk je mechanické (akustické) vlnění šířící se pružným prostředím. Je vymezeno konvenčním frekvenčním rozsahem
lidského ucha, tedy 16 Hz–20 kHz. Akustické vlnění s nižší frekvencí se nazývá infrazvuk, s vyšší pak ultrazvuk.

Fyzikální akustika se zabývá zákonitostmi šíření akustického vlnění včetně jejich matematického popisu a
technických aplikací. Velmi hrubě lze říci, že fyzikální akustika se zabývá chováním, které lze popsat pomocí údajů
jako je frekvence nebo intenzita.

Fyziologická akustika se naproti tomu zabývá spíše sluchem a řečí z fyzikálního hlediska, ovšem se zohledněním
vlastností sluchových receptorů a fyziologickými účinky. Fyziologická akustika se zabývá např. studiem prahu
citlivosti, prahu bolesti či vnímáním barvy tónů, zavádí nové veličiny jako je zejm. hladina intenzity.

Fyzikální vlastnosti
Frekvence
Frekvence udává počet kmitů za sekundu. Ve slyšitelné oblasti udává výšku tónu. Její převrácenou hodnotou je
perioda, udává trvání jednoho kmitu.

Rychlost šíření zvukové vlny

Pro rychlost šíření akustického vlnění platí v obecném prostředí obecný vztah:

kde K je modul objemové pružnosti postředí a ρ je hustota postředí.

Pro rychlost šíření akustikého vlněn v plynech platí:

kde κ je Poissonova konstanta, ρ je hustota plynu, p je tlak plynu

Pro ilustraci, rychlost zvuku se pohybuje od cca 330 m/s ve vzduchu (silně teplotně závislé), přes 1450 m/s v tuku a
1570 m/s v krvi až po 4080 m/s v lebečních kostech.

Vlnová délka

Vlna se šíří prostředím konečnou rychlostí. Vlnová délka je vlastně vzdálenost dvou maxim. S rychlostí vlny a s
frekvencí je spojena vztahem:

Někdy se používá převrácená hodnota vlnové délky, vlnočet (vyjadřující, kolik vln se vejde do délky 1 m), ale v
akustice to není obvyklé.

Akustická rychlost

Protože je zvuk mechanickým kmitáním částic kolem rovnovážné polohy, kmitají tyto s nějakou rychlostí v.

Akustický tlak
Šířením vlny v prostředí vznikají místa s naředěním resp. nahloučením částic. Makroskopicky se to projevuje
kolísáním tlaku. Velikost akustického tlaku závisí mimo jiné na hustotě prostředí, rychlosti vlny a frekvenci vlnění.

S akustickou rychlostí v je akustický tlak p spojen veličinou akustická impedance Z, která je charakteristikou
prostředí:

Akustický tlak se ve výsledku pochopitelně superponuje na atmosférický tlak.

Maximální akustický tlak pMAX je obtížně měřitelný, proto se v praxi používá, podobně jako např. v elektrotechnice,
hodota tlaku efektivního. Zavedení efektivní hodnoty souvisí s přenášenou energií a matematické odvození, i když není
složité, přesahuje potřeby studentů medicíny. Pro harmonické vlnění platí:

Ještě jednou je třeba zdůraznit, že toto platí jen pro harmonické průběhy. V případě neharmonického průběhu záleží
na konkrétním měřidle, někdy může být hodnota správná, ale zejména u starších nebo špatně navržených přístrojů
může být silně zavádějící.
Pro další potřeby se stanovuje prahový akustický tlak p0 jako do jisté míry konvencí stanovená hodnota akustického
tlaku, jakou ještě může lidské ucho při použití čistého tónu o frekvenci 1 kHz ještě zaregistrovat:

Intenzita vlnění
Intenzita vlnění (zvuku) je definována jako množství energie, které projde jednotkovou plochou kolmou na směr šíření
na jednotku času.

Podobně jako u akustického tlaku, je i u intenzity vlnění zaváděna prahová intenzita I0 jako nejnižší ještě
registrovatelná intenzita čistého tónu o frekvenci 1 kHz:

Hladina intenzity zvuku

Protože intenzita běžných zvuků kolísá v rozsahu několika dekád, zavádí se logaritmická škála, která prahové
intenzitě přiřadí nulovou hodnotu:

Hladina intenzity vychází z poměru dvou stejných veličin a proto je v SI bezrozměrná. Pro označení takto
definovaných hladin (nejen v akustice, ale i např. v elektrotechnice) se používá jednotka decibel (dB). Jednotka je
pojmenovaná po skotském vědci a vynálezci prvního prakticky použitelného telefonu A.G.Bellovi, předpona deci- pak
značí vynásobení desítkou.

Protože je intenzita vlnění přímo úměrná druhé mocnině akustického tlaku, platí současně:

Fyziologické vlastnosti
Akustické spektrum
V obecném případě není akustický signál tvořen jedním tónem o harmonickém („sinusovém“) průběhu, ale průběh má
složitější tvar. Pokud je tento průběh periodický, vnímáme jej jako čistý tón. Čistý tón rozložit na součet případně
různě posunutých sinusových průběhů s frekvencemi f0, 2f0=f1, 3f0f2,... (Fourierův teorém). Vynesení amplitud těchto
sinusovek za sebe představuje amplitudové spektrum akustického signálu. Takovéto spektrum se nazývá diskrétní,
protože je tvořeno jen diskrétními body. Základní frekvence f0 určuje výšku tónu. Další frekvence f1,... se nazývají
vyšší harmonické, určují barvu tónu.

Pokud zvuk není periodický, tedy pokud jde o jednorázový děj typu exploze nebo souhlásky nebo pokud jde o šum, lze
na něj pohlížet jako na periodický děj s nekonečně velkou periodou a tedy nekonečně malou frekvencí. Spektrum
takového neperiodického signálu je pak spojité.

Hlasitost

Protože vztah mezi intenzitou vlnění a intenzitou počitku závisí mimo jiné i na frekvenci, je třeba hodnotit pro každou
frekvenci (výšku) tónu vztah individuálně. K porovnání se používají veličiny hlasitost a hladina hlastitosti, které
kvantifikují případné rozdíly ve vnímané intenzitě.

Hladina hlasitosti LN je definována pomocí srovnání s hladinou intenzity při frekvenci 1 kHz. Tón na libovolné
frekvenci má právě takovou hladinu hlasitosti, jakou má hladinu intenzity zvuku tón o frekvenci 1 kHz, který vnímáme
jako stejně silný. Jednotkou hladiny hlasitosti je fón (Ph).

Hlasitost N byla definována z toho důvodu, že hladina hlasitosti nekvantifikuje příliš dobře rozdíly ve vnímání.
Referenčním bodem je tón o hladině intenzity 40 dB při frekvenci 1 kHz, kterému je přiřazena hodnota 1. Jednotkou
hlasitosti je son. Hlasitost lze určit z hladiny hlasitosti podle následujícího vztahu:

Ve skutečnosti závisí vnímaná intenzita i na tom, zda je použit skutečně čistý tón. Při měření hlasitosti zvuků, které
nemají sinusový průběh (jednobodové spektrum), je třeba dále zohlednit frekvenční závislost jednotlivých složek.
Běžným přístupem je zapojení frekvenčního filtru, který v měřeném signálu přesně definovaným způsobem potlačí
nevnímané složky a omezí vliv jednotlivých frekvencí.
Odkazy
Zdroje

KUBATOVA, Senta. Biofot [online]. [cit. 2011-01-31]. <https://uloz.to/!CM6zAi6z/biofot-doc>.

HRAZDIRA, Ivo a Vojtěch MORNSTEIN. Lékařská biofyzika a přístrojová technika. 1. vydání. Brno : Neptun, 


2001. 396 s. ISBN 80-902896-1-4.

NAVRÁTIL, Leoš a Jozef ROSINA, et al. Lékařská biofyzika. 1. vydání. Manus, 2001 (1. dotisk). 357 s. ISBN 80-
902318-5-3.

SCHAUER, Pavel. Vybrané statě z akustiky [online]. Ústav fyziky, Fakulta stavební, VUT Brno, [cit. 2013-08-
20]. <http://fyzika.fce.vutbr.cz/doc/vyuka_schauer/vybrane_state_z_akustiky.pdf>.

Související články

Audiometrie (biofyzika) • Audiometrie (fyziologie)


Sluch • Klasifikace sluchových poruch • Vyšetření sluchu
Hluk • Hluková zátěž
Akustický odpor
Akustická impedance (velmi nepřesně někdy nazývaná jako akustický odpor) je fyzikální veličina, která popisuje
akustické vlastnosti prostředí. Vypočítá se jako poměr efektivní hodnoty akustického tlaku (pef) a efektivní hodnoty
akustické rychlosti, tedy rychlosti kmitavého pohybu částic prostředí vyvolané zvukovou vlnou vef. Jedná se o
zobecněný pojem, používá se proto i k vykládání jevu při jiných vlnových délkách než jsou ty akustické (např.
ultrazvukové,…)

(Pa·s·m-1)

Efektivní hodnoty akustického tlaku a rychlosti vlnění lze vypočítat z maximálních hodnot podle vztahu:

resp.

Na rozhraní dvou prostředí o různé akustické impedanci může docházet k lomu a odrazu procházejícího akustického
vlnění. Právě toho využívá ultrazvuková diagnostika.

jiný výpočet: Z = ρ. c (Pa.s/m), kde ρ – hustota látky, c – fázová rychlost šíření v dané látce

Odkazy

Související články
Ultrazvuk
Vlastnosti zvuku
Dopplerův jev
Dopplerův jev představuje změnu detekované frekvence vlnění, jsou-li zdroj a detektor ve vzájemném pohybu.
Fyzikální podstatou Dopplerova jevu je skládání rychlosti vlnění s rychlostí vzájemného pohybu zdroje a detektoru. Jev
se uplatňuje pro libovolné vlnění, tedy zejména akustické i elektromagnetické. Poprvé byl popsán Christianem
Dopplerem jako posuv spektrálních čar u rotujících dvojhvězd, kde se spektrum hvězdy pohybující se směrem k nám
posouvalo směrem k modrému konci a spektrum hvězdy pohybující se od nás k červenému konci spektra. Pro
medicínu je zdaleka tím nejdůležitějším Dopplerův jev při odrazu ultrazvuku od pohybujících se částic, zejména pak
červených krvinek.

Typickou situací použítí v medicíně je stojící pozorovatel, tedy pevně umístěný detektor, a pohybující se zdroj, tedy
tkáň odrážející na něj dopadající vlnění. Pro vlnovou délku detekovaného vlnění potom platí:

Znaménko se určuje podle toho, zda se zdroj zvuku vzdaluje (+) nebo přibližuje (-) k detektoru, λ0 je vlnová délka
vlnění, které opouští zdroj, f0=původní frekvence tohoto vlnění vlnění a vzdr je rychlost zdroje.

Základním použitím je zjišťování toku krve. Typickým spojení barevně kódované informace o toku krve do
ultrazvukového snímku v B módu, tzv. duplexní sono. V angiologii se využívají i tužkové průtokoměry, které pouze
detekují pohyb a signalizují jej pomocí zvukového výstupu.

Uspořádání je v tomto případě poněkud složitější. Vlnění je vysláno sondou a dopadá na tkáň, která funguje jako
"detektor". Ve tkáni se již s posunutou frekvencí odrazí a jako vlnění od pohybujícího se zdroje se vrací zpět k sondě,
která funguje současně i jako detektor. Tedy dochází ke dvěma posuvům frekvence. Pro rozdíl mezi detekovanou a
vyslanou frekvencí platí vztah:

kde v je rychlost pohybu tkáně, c je rychlost vlnění a α je úhel, který svírá vektor rychlosti pohybu tkáně s osou sondy.

Související články

Dopplerovská ultrasonografie v medicíně • Dopplerovská ultrasonografie


Dopplerovská zobrazení
Dopplerovská echokardiografie • Transkraniální dopplerovská ultrasonografie • Fetální Dopplerometrie •
Dopplerovský průtokoměr

Použitá literatura

BENEŠ, Jiří, Daniel JIRÁK a František VÍTEK, et al. Základy lékařské fyziky. 4. vydání. 2015. 322 s. ISBN
9788024626451.
Hlasitost

Dané intenzitě zvuku o určité frekvenci přiřazujeme subjektivně hlasitost.

Pro kvantitativní vyjádření hlasitosti se používá jednotka fón (Ph), která byla určena měřením křivek stejné hladiny
hlasitosti (izofóny) u zdravých osob. Každá křivka odpovídá stejnému sluchovému vjemu při různých frekvencích.
Počet fónu byl každé křivce přiřazen podle počtu dB při frekvenci 1 kHz.

Hladinu hlasitosti tedy u libovolného tónu určíme tak, že intenzitu referenčního tónu (f = 1 kHz) měníme tak dlouho,
dokud nemá stejnou hlasitost jako určovaný tón a počet fónu určovaného tónu se rovná počtu absolutních dB
referenčního tónu.

Odkazy
Související stránky

Práh sluchu a sluchové pole

Externí odkazy

Hlasitost (česká wikipedie)

Zdroj

KUBATOVA, Senta. Biofot [online]. [cit. 2011-01-31]. <https://uloz.to/!CM6zAi6z/biofot-doc>.


Weber-Fechnerův zákon
Weber-Fechnerův zákon charakterizuje vztah mezi fyzikální intenzitou podnětu působícího na receptor a
subjektivním vjemem intenzity (počitkem), které toto působení vyvolá. Pojmenován je po německém lékaři E. H.
Weberovi (1795–1878) a německém psychologovi G. Th. Fechnerovi (1801–1887).

Weber-Fechnerův zákon lze matematicky popsat například následujícím vztahem:

Tento vztah lze upravit vyřešením příslušné diferenciální rovnice na známou formuli:

S – intenzita subjektivního vjemu;


k – konstanta;
I – fyzikální intenzita podnětu působícího na receptor;
I0 – prahová intenzita, tedy absolutně nejnižší možná intenzita, jakou je schopný jedinec vnímat.

K názorné interpretací matematického popisu pomůže, když si diferenciál v první rovnici nahradíme malou změnou Δ:

Vztah lze pak chápat tak, že subjektivně vnímaná malá změna počitku je přímo úměrná malé změně intenzity podnětu
a nepřímo úměrná intenzitě počitku. To znamená, že například:

Přidáme-li ke třem gramům závaží jeden gram, bude subjektivně vnímaná změna hmotnosti poměrně dobře patrná;
přidáme-li ale jeden gram ke třem kilogramům, bude již subjektivně vnímaná změna hmotnosti nepatrná.

Důvodem pro takovéto chování receptorů je zřejmě především to, že je umožněn současně vysoký rozsah
vnímaných fyzikálních intenzit a vysoká rozlišovací schopnost alespoň při nízkých intenzitách.

Článek neobsahuje vše, co by měl.


Můžete se přidat k jeho autorům a jej.
O vhodných změnách se lze poradit v diskusi.
Práh sluchu a sluchové pole

Sluchové pole - Graf

Křivky hlasitosti

Práh slyšitelnosti na vysokých


frekvencích stoupá s věkem

Sluchové pole
Sluchové pole (nebo oblast slyšitelnosti) je rozsah všech zvuků, které dokáže lidské ucho vnímat. Vnímání zvuku je u
člověka omezeno slyšitelnými frekvencemi (přibližně 16–20 000 Hz). U každé frekvence je odlišný rozdíl intenzit, jež
slyšíme. Lidský sluchový orgán je nejcitlivější v oblasti frekvencí 1–5 kHz.

Tvar sluchového pole

Zdola je sluchové pole vymezeno křivkou sluchového prahu, tedy nejmenší intenzitou tónu, kterou je pozorovatel
schopen při dané frekvenci vnímat. Shora je omezeno křivkou prahu bolesti. Zvuky nad tímto prahem vyvolávají
bolestivý vjem a mohou vést k poškození samotného sluchového orgánu. Vnímání zvuků je individuální a rozdíly
můžeme najít i mezi zdravými jedinci. Tvar sluchového pole se také mění s věkem.

Intenzita zvuku I

Intenzita zvuku je energie zvukového vlnění, která projde za dobu 1 s plochou 1 m2 orientovanou kolmo na směr
šíření zvuku.

P je výkon zvukového vlnění, S je obsah plochy, kterou vlnění prochází. Jednotkou je tedy W•m−2.

Hladina intenzity zvuku L

Rozdíl v intenzitě zvuku mezi slabým zvukem, který již můžeme slyšet a silným, který nám působí bolest, při frekvenci
1 kHz je v řádech bilionů (10–12 W•m−2 a 1 W•m−2). Zavedla se proto hladina intenzity zvuku v relativních
jednotkách, v belech (B) nebo decibelech (dB). 1 B odpovídá poměru intenzit zvuku 1:10. Zvýšíme-li tedy hladinu
intenzity zvuku o 1 dB, zvýší se intenzita zvuku asi o 26 %, což je nejmenší rozdíl, který je zdravé ucho schopno
postřehnout.

Intenzitám zvuku I registrovaným lidským sluchovým orgánem v rozmezí 10–12 W•m−2 až 10 W•m−2 přiřazujeme
hladiny intenzity zvuku L v rozmezí 0 dB až 130 dB.

Hladina intenzity v belech je určena dekadickým logaritmem poměru intenzity, jejíž hladinu určujeme, k určité
intenzitě I0, kterou bereme za základ (nulovou hladinu).

Izofóny

Izofóna je křivka stejné hlasitosti. Spojuje zvuky příslušné frekvence a intenzity, které vnímáme jako stejně silné.
Nulová izofóna je práh slyšení, izofóna na hladině 120 dB je práh bolesti.

Fón (jednotka) a Hladina hlasitosti

Fón (Ph) je jednotkou hladiny hlasitosti. Hladinou hlasitosti je nazývána subjektivní hlasitost, která je vnímána
sluchem.

Hladina hlasitosti je definována tak, že hladina hlasitosti 1 fón je při kmitočtu 1 kHz stejně velká jako jednotka hladiny
zvuku 1 dB. Pro zvuky ostatních kmitočtů jsou hladiny hlasitosti definovány subjektivním porovnáváním s hladinou
hlasitosti referenčního tónu. [1]

Práh sluchu
Práh sluchu je úroveň hlasitosti, pod kterou není lidské ucho schopno zaznamenat jakýkoli zvuk. [2]

Práh sluchu je závislý na stavu sluchového aparátu daného člověka. Obecně je definován jako smluvní vztažná
hodnota hladiny intenzity zvuku (při frekvenci 1 kHz) akustického tlaku vůči tlaku atmosférickému (p0=2.105 Pa=200
kPa). [3] Pro dospělé má práh sluchu referenční úroveň hlasitosti velikost 0 decibelů (dB).

Závislost prahu sluchu na frekvenci zvuku

Práh sluchu je odlišný pro různé frekvence zvuku. Standardizovaný práh sluchu pro frekvenci 1 000 Hz je 0 dB. U
nižších a vyšších frekvencí než 1 000 Hz je pak viditelný posun prahu sluchu. Pro 30 Hz se pohybuje kolem 60 dB.
Naopak největší citlivost lidské ucho vykazuje při frekvencích okolo 3 500–4 000 Hz, což odpovídá rezonanční
frekvenci zvukovodu. Těmto frekvencím odpovídá práh sluchu −12 dB. [4]

Závislost prahu sluchu na věku

S rostoucím věkem se rozvíjí tzv. sensorineurální (percepční) sluchová vada. Tato porucha je způsobena absencí nebo
poškozením drobných vláskových buněk v hlemýždi, které pak již neposílají signály do mozku, což má za následek
posunu prahu sluchu. [5] [6] (viz graf "Práh slyšitelnosti na vysokých frekvencích stoupá s věkem").

Odkazy
Související stránky

Hlasitost
Klasifikace sluchových poruch
Sluch
Ucho

Reference
1. UNKNOWN,. Wikipedia [online]. [cit. 2013-12-18]. <https://cs.wikipedia.org/wiki/F%C3%B3n_(jednotka)>.
2. GREENFACTS,. Glossary of audiology terms [online]. [cit. 2013-11-29]. 
<http://ec.europa.eu/health/opinions/en/hearing-loss-personal-music-player-mp3/glossary/ghi/hearing-
threshold.htm>.
3. UNKNOWN,. Wikipedia [online]. [cit. 2013-11-29]. 
<https://en.wikipedia.org/wiki/Absolute_threshold_of_hearing>.
4. NAVE, R.. Threshold of Hearing [online]. [cit. 2013-11-29]. <http://hyperphysics.phy-
astr.gsu.edu/hbase/Sound/earcrv.html>.
5. AUDIONIKA,. O sluchu a sluchových vadách [online]. [cit. 2013-18 - 12]. <http://www.audionika.cz/stranka/o-
sluchu-a-sluchovych-vadach>.
6. UNKNOWN,. Age and Shift in Hearing Threshold [online]. [cit. 2013-10-10]. 
<https://www.engineeringtoolbox.com/age-shift-in-threshold-d_1474.html>.

Zdroje
GREENFACTS,. Glossary of audiology terms [online]. [cit. 2013-11-29]. 
<http://ec.europa.eu/health/opinions/en/hearing-loss-personal-music-player-mp3/glossary/ghi/hearing-
threshold.htm>.
UNKNOWN,. Wikipedia [online]. [cit. 2013-11-29]. 
<https://en.wikipedia.org/wiki/Absolute_threshold_of_hearing>.

NAVE, R.. Threshold of Hearing [online]. [cit. 2013-11-29]. <http://hyperphysics.phy-


astr.gsu.edu/hbase/Sound/earcrv.html>.

UNKNOWN,. Age and Shift in Hearing Threshold [online]. [cit. 2013-10-10]. 


<https://www.engineeringtoolbox.com/age-shift-in-threshold-d_1474.html>.

Použitá literatura
NAVRÁTIL, Leoš a Jozef ROSINA. Medicínská biofyzika. Praha: Grada Publishing, 2005. ISBN 978-80-247-1152-
2, str. 282.
Sluch
Úvod do problematiky

Střední ucho

Sluchové kůstky

Sluch je jedním z pěti smyslů, založený na percepci (zpracování a vedení) akustických (zvukových) podnětů. Jeho
podstatou je transformace mechanických zvukových vln na elektrické akční potenciály.
Zvuk je mechanické vlnění vyvolané nějakým vnějším činitelem, vyskytující se ve formě podélné vlny na základě
střídavého zahuštění a zředění molekul v okolí původce podnětu (tj. kolísání akustického tlaku). K zobrazení
zvukové vlny užíváme jejích vlastností: amplitudy a vlnové délky.
Amplituda je maximální výchylka akustického tlaku. Udává sílu zvuku (přímo úměrně): čím vyšší amplituda, tím
silnější zvuk.
Vlnová délka je úsek vlny, který se periodicky opakuje. Je nepřímo úměrná frekvenci. Udává výšku tónu
(nepřímo úměrně). Čím větší frekvence, tím vyšší tón. (Čím větší vlnová délka, tím nižší tón.)

Zevní ucho
Zvuková vlna je zachycena ušním boltcem a beze změny vedena zevním zvukovodem až k bubínku, který funguje
jako rezonátor.

Střední ucho
Popis

Ve středoušní dutině se nachází 3 sluchové kůstky, vzájemně kloubně spojené: kladívko (malleus), kovadlinka
(incus) a třmínek (stapes), dále svaly m. tensor tympani (úpon na kladívko, inervace z N. V. ) a m. stapedius (úpon
na stupátko třmínku, inervace z N. VII.). Kladívko je připojeno k bubínku, kloubně spojeno s kovadlinkou. Kovadlinka
je spojena s třmínkem, jehož stupátko přiklápí a odklápí fenestra ovalis do prostoru vnitřního ucha, konkrétně scala
vestibuli hlemýždě. Vnitřní ucho vyúsťuje kochleární scala tympani zase zpět do středouší druhým okénkem, fenestra
rotunda, které leží těsně pod fenestra ovalis.

Princip

Sluchové kůstky působí jako pákový systém, jenž přenáší vibrace bubínku na pohyby stupátka třmínku proti
fenestra ovalis. Jelikož přechodem ze středního do vnitřního ucha se změní prostředí, kudy vlna prochází, z plynného
(ve středouší) na kapalné (perilymfa a endolymfa v hlemýždi vnitřního ucha), přičemž kapalné prostředí vyvíjí zvukové
vlně mnohem větší odpor, je třeba zvýšit tlak, jímž zvuková vlna vtrhne do vnitřního ucha. To se děje mechanickou
úpravou vzduchu:

poměrem plochy bubínku (velká plocha) a fenestra ovalis (malinká ploška);


nerovnoramenností pák sluchových kůstek (kladívko je delší než výběžek kovadlinky).

Následkem toho se tlak zvukové vlny zvýší (asi 22×) a amplituta sníží.

Tympanický reflex

Jedná se o zvýšení tuhosti a odporu převodního systému ve středoušní dutině (a tím snížení vedení zvuku) při
současném stahu m. tensor tympani a m. stapedius; vzniká jako ochranný reflex na základě hlasitých zvuků o nízké
frekvenci, např. výbuchu. Chrání sluchové receptory před nadměrnou stimulací příliš silnými zvukovými vlnami.

Vnitřní ucho
Popis

Cortiho orgán

Ve vnitřním uchu se jako sluchově relevantní (nachází se tu ještě vestibulární aparát, tomu se ale nebudeme věnovat)
vyskytuje kostěný hlemýžď, zvnitřku kopírovaný hlemýžděm membranózním (stočená membranózní trubice uvnitř
stočené kostěné trubice), vyplněný tekutinou.

Hlemýžď (kochlea) je tvořen dvěma a půl závity stočené trubice, podélně rozdělené na tři části: scala vestibuli,
ductus cochlearis a scala tympani. Scala vestibuli začíná za fenestra ovalis a v apexu hlemýždě kontinuálně
přechází ve scala tympani, jež ústí do fenestra rotundum. Vnitřní komoru tvoří ductus cochlearis, jenž v apexu
hlemýždě slepě končí. Ductus cochlearis je od scala vestibuli oddělen Reissnerovou a od scala tympani bazilární
membránou. V ductus cochlearis jsou na bazilární membránu posazeny buňky Cortiho orgánu, kromě jiných také
vnější (laterálně) a vnitřní (mediálně) vláskové buňky, senzory akustického podnětu. Vnější buňky jsou zanořeny do
tektoriální membrány, kryjící Cortiho orgán svrchu. Vnitřní vláskové buňky nejsou s tektoriální membránou pevně
spojeny.

Princip
Zvuková vlna přicházející z fenestra ovalis rozvlní tekutinu ve scala vestibuli. Jelikož je scala vestibuli kryta (vůči
ductus cochlearis) poddajou Reissnerovou membránou, rozvlní i tu a následně i membránu bazilární. Tímto způsobem
vlna postupuje hlemýžděm dál- nazýváme ji postupnou vlnou. Kvůli vlastnostem bazilární membrány se rychlost
postupu se vzdáleností od fenestra ovalis snižuje, amplituda vlny ale naopak roste, až dosáhne v určitém místě
maxima, aby pak rychle opadla. Místo, kde zvuková vlna dosáhne maximálního vyklenutí membrán (maximální
amplitudy), závisí na frekvenci této zvukové vlny. Čím vyšší frekvence, tím dříve (tzn. tím blíže k fenestra ovalis)
dosáhne maxima. Takovéto místo dosažení maximální amplitudy je pro jednotlivé frekvence charakteristické; principu,
v němž je každé frekvenci přiřazeno určité místo maxima amplitudy, se říká princip tonotopie. Znamená to, že z
určitého konkrétního místa vedou do mozku aferentní nervová vlákna zvukovou informaci o jediné frekvenci.
Analyzátorem frekvencí zvukových vln je bazilární membrána- ta určí, ve kterém místě vlna dosáhne maxima.

Vláskové buňky

V místě dosažení maximální amplitudy se bazilární membrána vyklene také maximálně. Svým vyklenutím posouvá vůči
sobě tektoriální membránu a změní směr stereocilií vnějších vláskových buněk (VB), které jsou do tektoriální
membrány zanořeny. Tím se vnější VB depolarizuje: otevírá kationtové kanály a dochází k influxu draselných, sodných
a vápenatých kationtů. Současně s tím se vnější VB zkracuje. Následuje ohnutí cilií v protisměru, buňka se
hyperpolarizuje a prodlouží na původní délku. Tímto způsobem působí vnější VB jako kochleární zesilovač zvuku
(zvyšuje amplitudu) – zesílená vlna přechází k vnitřním VB. Ty mají za úkol převést zpracované mechanické vlnění za
pomoci chemických procesů na elektrickou energii – akční potenciál. Tato transformace, kterou vnitřní VB provádí,
se nazývá transdukce. Princip depolarizace je zde stejný: stereocilie vnitřních VB drážděny tekutinou mění směr,
otevírají iontové kanály, a tím se depolarizují. Následkem toho uvolňují transmitery, tím vznikne na zakončeních
přilehlých aferentních vláken nervový impuls, který je veden do vyšších etáží (do mozku). Vnější VB jsou obklopeny
eferentními zakončeními a jejich funkcí je modulace (zesílení) zvukové vlny. Vnitřní VB jsou obklopeny aferentními
zakončeními bipolárních neuronů z ganglion spirale a jejich funkcí je transdukovat mechanickou energii na energii
elektrickou.

Souhrn
1. rozkmitání bazilární membrány;
2. posun tektoriální membrány vůči membráně bazilární;
3. změna směru stereocilií vnějších VB;
4. otevření iontových kanálů a zkrácení vnější VB = depolarizace vnějších VB;
5. zesílení zvuku vnějšími VB;
6. dráždění stereocilií vnitřních VB pohybem tekutiny ductus cochlearis;
7. změna směru stereocilií vnitřních VB;
8. otevření iontových kanálů a uvolňování transmiteru = depolarizace vnitřních VB;
9. na synapsi s aferentním vláknem vznik elektrického impulsu.

Odkazy
Související články

Nervus vestibulocochlearis
Vývoj sluchového a rovnovážného ústrojí
Klasifikace sluchových poruch
Práh sluchu a sluchové pole
Vyšetření sluchu. Rozdělení základních poruch podle audiogramu

Externí odkazy

Sluch (česká wikipedie)


Hearing (sense) (anglická wikipedie)

Použitá literatura
Silbernagl, S., Despopoulos, A. : Atlas fyziologie člověka. Grada Publishing, a. s., Praha 2004, 6. vyd.
Trojan, S. a kolektiv: Lékařská fyziologie. Grada Publishing, a. s., Praha 2003, 4. vyd.
Ganong W.F.: Přehled lékařské fyziologie. Galén, Praha 2005. 20. vyd.
Audiometrie (fyziologie)
Tónová audiometrie je elektroakustická vyšetřovací metoda sluchu. Je jednou z metod vyšetření sluchu, kdy se
pomocí tónového generátoru testuje citlivost sluchu na jednotlivé tóny. Provádí se přístrojem - audiometrem, který
generuje tóny určitého kmitočtu a intenzity.

Průběh audiometrického vyšetření

Průběh audiometrického
vyšetření

Při vlastním měření je vyšetřovaný umístěn do zvukotěsné kabiny a vzniklý tón je vyslán do zkoumaného ucha
vzdušným nebo kostním sluchadlem. Vyšetřující, většinou audiologická sestra, postupně zesiluje intenzitu čistého tónu
a v okamžiku, kdy vyšetřovaný signalizuje, že tón slyší, si vyšetřující zaznamená hladinu intenzity zvuku, pak přejde
k dalšímu tónu. Audiometrické vyšetření určuje prahy jednotlivých tónů kvantitativně, jedná se však o subjektivní
pocit pacienta.

Po vyčerpání všech měřených frekvencí se totéž opakuje i na druhém uchu. Výsledek vyšetření se zapisuje do
audiogramu. Pokud je to pro diagnostiku nezbytné, provádí se i vyšetření kostního vedení zvuku, kdy je na processus
mastoideus přiložen vibrátor. Přístroj je kalibrován podle sluchového prahu vzdušného a kostního vedení normálně
slyšících lidí. Audiometry je nutné pravidelně kalibrovat, dle zákona je tomu tak jednou za 2 roky a kalibraci provádí
akreditované metrologické ústavy.

Sluch se měří v rozmezí od nízkých frekvencí, tj. 250 Hz, do vysokých frekvencí 10 000 Hz a od –10 do 100 dB. Počet
decibelů je dán desetinásobným logaritmem poměru intenzity vyšetřovaného tónu k intenzitě sluchového prahu. Výše
popsaná metoda nám umožňuje objektivizovat, kvalifikovat a lokalizovat ztrátu sluchu. Porovnáním vzdušného a
kostního vedení, můžeme rozlišit vady sluchu způsobené poruchou vedení sluchu do vnitřního ucha, tzv. poruchy
převodní – konduktivní, a poruchy způsobené poškozením vnitřního ucha, poruchy percepční.

Hodnocení výsledků vyšetření

Normální audiogram

Ztráta sluchu je často popisována takto:

za normální sluch se považuje sluch do 15 dB ztrát,


percepční porucha se projeví ztrátou symetrickou ve vedení kostním i vzdušném,
převodní nedoslýchavost se projeví ztrátou ve vedení vzdušném, zatímco vedení kostní je normální.

Míra ztráty sluchu se hodnotí jako:

1. Mírně těžká ztráta sluchu = 25–40 dB,


2. Středně těžká ztráta sluchu = 41–65 dB,
3. Těžká ztráta sluchu = 66–90 dB,
4. Velmi těžká porucha včetně hluchoty = 90 a více dB.

Způsob zaznamenání vyšetřovaného sluchu byl mezinárodním audiologickým kongresem sjednocen tak, že audiogram
je nezávisle na tom, kde, kdy a jak vznikl, srozumitelný všem.
Odkazy
Související články

Vyšetření sluchu. Rozdělení základních poruch podle audiogramu


Vyšetření sluchu • Otoakustické emise
Práh sluchu a sluchové pole
Nedoslýchavost

Použitá literatura

BROHM, František. Praktická audiometrie. 1. vydání. Avicenum, 1971. 

HYBÁŠEK, Ivan. Ušní, nosní krční lékařství. 1. vydání. Galén, 1999. ISBN 8072620177.

TICHÝ, Stanislav, et al. Otorinolaryngologie. 1. vydání. Avicenum, 1983. 

cs.Wikipedie. Audiometrie [online]. [cit. 22. 10. 2010, 10:41 UTC]. <https://cs.wikipedia.org/w/index.php?


title=Audiometrie&oldid=5658717>.
Ultrazvuk

Příklad USG vyšetření –


Achondrogeneze

Ultrazvuk je periodická změna tlaku v prostředí s frekvencí vyšší než horní hranice lidského sluchu, tedy vyšší než 20
kHz.

Ultrazvukové kmity lze vytvořit třemi typy generátorů:

1. mechanickými (malé ladičky, píšťaly: malá frekvence a výkon),


2. magnetostrikčními (kmity kolem železné tyčinky v magnetickém poli elektromagnetu, který je napájen
střídavým proudem: velký výkon, ale frekvence jen do 100 kHz; použití v zubním lékařství a chirurgii),
3. piezoelektrickými (destička z křemene je připojena k elektrodám se střídavým napětím, a tak kmitá se stejnou
frekvencí jako napětí a mění energii elektrickou na mechanickou, která rozkmitá okolní prostředí: použití pro
diagnostické i terapeutické účely).

Vlastnosti
Vysoká frekvence způsobuje velmi krátkou vlnovou délku ultrazvukových vln, ve kterých ale dochází při vyšších
intenzitách k tlakovým změnám v řádech MPa (nebezpečí poškození buněčných jader). Na rozhraních různých tkání (s
různou rychlostí šíření zvuku) dochází k částečné změně směru šíření vln a jejich odrazu (analogicky k zákonu o lomu
světla) → použití v diagnostice.

. 100 R = procento odražené energie dopadající vlny, z1 a z2 = akustické impedance dvou prostředí.

Účinky
Podrobnější informace naleznete na stránce Účinky ultrazvuku.

Tepelné účinky – energie vlny je přímo úměrná její f2. Značná absorpce je na rozhraní tkání s různou akustickou
impedancí (měkká tkáň X kost = periostální bolest).
Mechanické – průchod ultrazvukové vlny prostředím má za následek lokální tlakové změny (MPa/mm).
Fyzikálně-chemické – ultrazvuk má účinky disperzní, což znamená, že s jeho pomocí připravujeme jemné
suspenze, emulze, pěny apod., a také koagulační – slouží např. k čištění plynů.
Biologické – do intenzity 3 W/cm2 mají ultrazvuky spíše biopozitivní účinky: zrychlení metabolické výměny
apod., nad 3 W/cm2 mají za následek ireverzibilní morfologické změny – rozbití buněčného jádra, tepelná
koagulace bílkovin.

Diagnostické použití ultrazvuku


Podrobnější informace naleznete na stránce Diagnostické aplikace ultrazvuku.

Diagnostické ultrazvuky pracují s frekvencemi 3–10 MHz (podkožní sondy okolo 7 MHz).

Ultrazvuky generují akustický puls, který se šíří rychlostí zvuku v daném prostředí a při dopadu se část jeho energie
odrazí. Krystal pak detekuje odrazy signálu a určí velikost echa, z časového zpoždění pak hloubku odrazu.

A-obraz: lineární záznam odrazů v závislosti na hloubce.

B-obraz: otočení A-obrazu o 90°, velikost echa v určité hloubce je úměrná sytosti bodů v řádku na obrazovce →
složením mnoha přímek vedle sebe vznikne dvojrozměrný v čase pohyblivý obraz.

M-obraz: sytost bodů lze v čase zaznamenávat na běžící papír; důležité při hodnocení pohyblivosti jednotlivých
úseků srdce.

S vyšší frekvencí vln se zvyšuje rozlišení, ale stoupá také absorpce (snižuje se hloubka zobrazení).

Terapeutické použití ultrazvuku


Podrobnější informace naleznete na stránce Terapeutické užití ultrazvuku.

Při léčbě pomocí ultrazvuku se používají nejčastěji přístroje o frekvenci 0,8–1 MHz s intenzitou 0,5–3 W/cm2 při
expozici 10 min.

Léčebné účinky: hloubkový tepelný účinek, tišení bolesti, uvolnění dlouhotrvajícího lokálního svalového napětí,
zvýšení místního krevního oběhu, zvýšení metabolismu. Používá se především u kloubních a nervových zánětů.

V zubním lékařství se kmity s malou amplitudou (setiny mm) používají k odstraňování zubního kamene.

Odkazy
Související články
Ultrazvuk/Diagnostické aplikace ultrazvuku
Dopplerovská echokardiografie
Dopplerovská ultrasonografie
Piezoelektrický jev
Ultrasonografie (2. LF UK)

Externí odkazy

Ultrazvuk (česká wikipedie)


Ultrasound (anglická wikipedie)
Janík Václav: Ultrasonografie. Multimediální podpora výuky klinických a zdravotnických oborů :: Portál 3.
lékařské fakulty UK [online] 14.3.2011, poslední aktualizace 23.11.2011 [cit. 2011-12-22] Dostupný z WWW:
<http://portal.lf3.cuni.cz/clanky.php?aid=92>. ISSN ISSN 1804-3143
echopedia.org
Rychlá sonografická vyšetření FAST a EFAST www.prolekare.cz

Zdroj

KUBATOVA, Senta. Biofot [online]. [cit. 2011-01-31]. <https://uloz.to/!CM6zAi6z/biofot-doc>.


Ultrazvuk/Diagnostické aplikace ultrazvuku

Ultrazvuk lze diagnosticky využít v řadě aplikací. Protože jde o metodu zatíženou jen minimálním rizikem a metodu
snadno dostupnou, jde o poměrně rozšířenou techniku spadající do řady oborů. Jedná se zejména o:

Ultrazvukové zobrazování − pořizování tomografických řezů na podkladě rozdílných akustických parametrů


tkání
Dopplerovské zobrazování − využití Dopplerova jevu k měření a vizualizaci pohybu nebo toku
Dopplerovský průtokoměr − měření toku krve
Ultrazvuková elastografie − vizualizace pružnosti (tuhosti) tkání
Ultrazvuková kostní denzitometrie − méně vhodná než rentgenové měření
Akustická mikroskopie − víceméně experimentální zobrazování s velmi vysokou frekvencí

K diagnostice má ultrazvuk dále nepřímý vztah jako laboratorní nástroj. V laboratořích se ultrazvuku používá
například při následujících činnostech:

čištění nástrojů
homogenizace a disperze látek
rozptyl buněk do suspenze

Ultrazvukové zobrazování
Fyzikální princip

Piezoelektrický jev

Pro diagnostické účely je obvykle využíváno generování ultrazvuku pomocí piezoelektrického jevu. K pochopení
piezoelektrického jevu je třeba vědět, že v krystalech mohou být jednak elektrické náboje volně pohyblivé, těm se říká
volné nosiče náboje, a jednak pevně vázané v krystalové mřížce, těm se říká vázané náboje. Dojde-li k pružné
deformaci krystalu, změní se krystalová struktura a tím i vzájemná poloha vázaných nosičů náboje. Vzájemným
posuvem nábojů se od sebe mohou oddálit "těžiště" kladných a záporných nábojů v krystalu se objevuje nenulové
elektrické pole. To pak lze změřit na povrchu krystalu jako elektrické napětí. Děj může probíhat i opačně, totiž pokud
se krystal z vhodného materiálu umístí do dostatečně intenzivního elektrického pole, dojde k jeho měřitelné
deformaci. Tento jev se někdy nazývá obrácený piezoelektrický jev nebo elektrostrikční jev. Pokud je vnější elektrické
pole časově proměnné, krystal mění svůj tvar se stejnou periodou a sám se tak stává zdrojem mechanického vlnění.
Vedle některých krystalů může piezoelektrický jev nastávat i u některých keramických materiálů.

Vznik signálu a šíření ve tkáni

Jeden piezoelektrický prvek nazývaný ultrazvukový měnič může být současně zdrojem i detektorem ultrazvukového
vlnění. U běžných ultrazvukových přístrojů je uspořádání takové, že měnič po dobu několika milisekund generuje
ultrazvukové vlnění s frekvencí obvykle z rozsahu 3 až 10 MHz, poté se stává detektorem a zachycuje odražené vlnění.

Vlastní akustické vlnění, které je vysíláno do tkání, se šíří jako podélné vlnění. Na rozhraní dvou prostředí s rozdílnou
akustickou impedancí se částečně odráží zpět ke zdroji a částečně prochází. Odražený signál, tzv. echo, je zachycen
měničem a převeden na elektrický signál, kterému se pro jeho vysokou frekvenci obvykle říká radiofrekvenční signál.

Protože je známa doba, která uplynula mezi vysláním akustického signálu do tkáně, lze určit, v jaké hloubce došlo k
odrazu. K tomu je ovšem třeba znát, s jakou rychlostí se šíří ultrazvuk ve tkáních. Tato rychlost se v měkkých tkáních
pohybuje kolem 1540 ms-1.

Frekvence ultrazvuku je velmi důležitý faktor. S rostoucí frekvencí klesá vlnová délka ultrazvuku a tak lze principiálně
vidět i větší detaily a získat kvalitnější obraz. Na druhou stranu s frekvencí roste i disipace energie, takže při použití
příliš vysokých frekvencí pro vyšetření orgánů uložených v hloubce by docházelo k neúměrnému ohřevu kůže a
povrchově uložených orgánů. Z toho důvodu se k vyšetření orgánů uložených např. v břišní dutině užívají frekvence v
jednotkách MHz, k vyšetření štítné žlázy a povrchově uložených lymfatických uzlin a cév frekvence kolem 10 MHz.
Ultrazvukové vyšetření s frekvencí vyšší je pak ne zcela rutinním výkonem např. v dermatologii (až několik desítek
MHz - akustický mikroskop).

Speckle
Speckle představují fenomén, který je pro ultrasonografii charakteristický. Ultrazvuková vlna ve skutečnosti
prostupuje tkání poměrně složitým způsobem, protože drobné tkáňové struktury mají rozměry srovnatelné s vlnovou
délkou prostupujícího ultrazvuku. Tak dochází k nezanedbatelnému rozptylu vlnění. Protože jde o vlnění
monochromatické, rozptýlené vlny interferují jak mezi sebou, tak i s vlnou užitečnou. Výsledný signál je tak "zarušen"
interferenčními vzorci.

Přístup ke specklím není zcela jednoznačný. Na jedné straně snižují přehlednost řezu tkání pro neškolené oko, na
straně druhé nesou alespoň část informace o struktuře tkáně. V odborném písemnictví lze proto nalézt jak práce,
které se pokoušejí speckle eliminovant potlačit tak, že má upravený snímek charakter "anatomického" řezu, tak práce,
které se pokoušejí speckle využít k počítačové podpoře diagnostiky.

Ultrazvukové sondy
Obvykle nepostačuje, aby na sondě byl pouze jeden měnič, proto jsou na sondě umístěny měniče ve větším počtu.
Podle tvaru sondy a uspořádání rozlišujeme několik typů sond. Základními sondami jsou sondy lineární, konvexní a
sektorová, pro speciální aplikace se používá např. sonda tužková, cirkulární nebo sonda typu array.

Lineární sonda

V lineární sondě jsou měniče uspořádány v jedné řadě, podkladem je úsečka. Výsledný obraz má tvar obdélníku.
Lineární sondy se typicky používají k vyšetření povrchových orgánů, proto jsou obvykle konstruovány na vyšší
frekvence.

Konvexní sonda

V konvexní sondě jsou měniče opět v řadě, ovšem jejich podklad je mírně konvexní. Výsledný obraz má tvar výseče z
mezikruží s poměrně malým vrcholovým úhlem. Jde o nejběžnější typ sond používaných k vyšetření orgánů dutiny
břišní.

Sektorová sonda

Sektorová sonda je uzpůsobena k tomu, aby přes poměrně malé okno (kontakt sondy s tělem pacienta) zobrazila
poměrně široký řez tkání. Technicky toto může být řešeno dvojím způsobem:

1. Sonda s rotačním měničem představuje starší řešení. Měnič rychle rotuje a tak je ultrazvukové vlnění vysíláno
z poměrně malého bodu širokým směrem.
2. Sonda s elektronickým vychylováním představuje moderní řešení. Na sondě je několik málo měničů, vychýlení
je zajištěno přesně definovaným fázovým posunem jimi generovaných vln, které vede ke konstruktivní
interferenci právě v žádoucím směru.

Obraz ze sektorové sondy vypadá podobně jako obraz ze sondy konvexní, má však mnohem větší vrcholový úhel.
Sektorových sond se používá zejména v echokardiografii a v gynekologii.

Tužková sonda

Tužková sonda obsahuje jeden jediný měnič. Používá se obvykle jako součást přenosného ultrazvukového
průtokoměru. Výsledkem měření pochopitelně není obraz ale křivka nebo akustický signál.

Cirkulární sonda

V cirkulární sondě jsou měniče uspořádány tak, aby byl pořízen velmi široký až kruhový řez tkání v rovině kolmé na
osu sondy. Používá se především k transrektálními vyšetření prostaty, může být však natolik malá, že lze provádět i
intravaskulární ultrazvukové vyšetření např. ateromových plátů.

Sonda typu array

Sonda typu array je jednou z cest, jak získat 3D obraz. Měniče jsou rozmístěny v matici (array) a jejich součinností
jsou získávána data pro celý objem pod sondou.

Módy zobrazení

A mód

A mód (Amplitude) je jednorozměrné zobrazení, typicky jde o signál pouze z jednoho měniče. Jednotlivé odrazy
registrované ultrazvukovou sondou jsou zobrazeny na monitoru jako impulzy na časové ose. Amplituda impulzů
odpovídá intenzitě odražených ultrazvukových vln. Aby nepůsobil rušivě výraznější útlum závislý na hloubce, je ještě
radiofrekvenční signál zesílen tzv. TGC zesilovačem (Time Gain Control), jehož zesílení roste s časem uplynulým od
vyslání echa.

A mód má v současnosti jen omezené využití, např. při biometrii oka. A mód je ovšem východiskem všech dalších
metod, protože ty si lze představit tak, že jde o jednotlivé "paprsky" v A módu, jehož hodnoty grafu jsou převedeny na
stupně šedi.

B mód

B mód (Brightness) je základem tomografických zobrazení. V zásadě existují dva typy zobrazení - statické a
dynamické.

Statické zobrazení

Statické zobrazení je historicky starší, technicky snazší a již delší dobu opuštěné díky dostupnosti elektroniky a
výpočetní techniky umožňující realizovat rutinní dynamické zobrazení. Měření bylo prováděno pomocí jednoho
měniče, který byl posouván po těle pacienta. Postupnou sumací jednotlivých měření byl získán obraz.

Dynamické zobrazení

Při dynamickém zobrazení je v jedné sondě řada měničů, které pracují v součinnosti a jejichž echa jsou
vyhodnocována tak, že výsledný obraz vidíme jako obraz pořízený v reálném čase. Dnes se jedná o jednoznačně
nejobvyklejší způsob zobrazení.

M mód

M mód (Motion) slouží k vyšetření pohybu anatomických struktur, zejména srdce. V principu nejde o nic jiného než o
to, že se v pravidelných časových intervalech pořizuje jednorozměrný záznam. Naměřená echa jsou pak zakódována
do stupňů šedi a zobrazena pod sebou podle času.

3D zobrazení

Trojrozměrný snímek lze získat v principu dvěma způsoby, trojrozměrnou sondou a rekonstrukcí.

Trojrozměrné sondy

V principu se trojrozměrná sonda neliší běžné od sondy dvojrozměrné, pouze měniče nejsou uspořádány v řadě ale v
matici. Taková sonda pak umožňuje nasnímat data z celého objemu v poměrně krátké době, takže je dokonce možné
zobrazení v reálném čase, někdy se označuje jako 4D zobrazení. Nevýhodou je, že sondy jsou poměrně neohrabané, je
třeba zaručit dobrý kontakt pacientova těla s poměrně velkou plochou.

Rekonstrukční přístupy

Trojrozměrný obraz lze získat i matematickým zpracováním snímků pořízených pomocí sondy. První pokusy spočívaly
v tom, že se sonda připevnila s rámu s řízeným posunem a veškeré zpracování spočívalo pouze v synchronizaci posunu
a záznamu obrazů do paměti. Flexibilnější byl systém čidel snímajících polohu sondy v prostoru. Na základě řady řezů
doplněných o informaci o poloze sondy se pak dopočítaly hodnoty jednotlivých voxelů. Moderní přístroje jsou ještě
sofistikovanější, vyšetřující lékař pouze volně "projede" sondou přes oblast zájmu a ze získaných řezů je matematicky
určena nejprve jejich pozice a poté je rekonstruován trojrozměrný obraz.

Metody zobrazení rychlosti toku krve – dopplerovské metody


Dopplerovské metody vycházejí ze vztahu změny frekvence. Cílem těchto metod je měření rychlosti pohybujících se
struktur.

Spojité dopplerovské zobrazení – průtokoměry


Při tomto měření musíme použít odděleného vysílače a přijímače ultrazvukového vlnění. Vysílač s jedním měničem
spojitě generuje akustický signál a nelze ho proto přepínat do režimu přijímače. Průtokoměry mají akustický výstup,
neboť změna frekvence je pro rychlosti porudění krve v lidském těla ve slyšitelné oblasti. Průtokoměry zobrazují
rychlost toku krve v závislosti na čase. Nevýhodou této metody je, že nelze zobrazit uspořádání a umístění
sledovaných cév. Proto v případě dvou překrývajících se cév nevíme, ve které z nich rychlost krve měříme.

Pulzní dopplerovské metody

Tyto metody používáme v kombinaci s odrazovými metodami (B). Používáme sondy, které mohou pracovat v různých
režimech, mluvíme pak o duplexním měření. Sledujeme časový i frekvenční posun odraženého vlnění. Na monitoru
pak vidíme jakou rychlost měříme a kde ji měříme.

Duplexní metoda je kombinací dvojrozměrného dynamického zobrazení a pulzní dopplerovské metody měření
rychlosti. Dvojrozměrné dynamické zobrazení nám poskytuje informaci o morfologii sledované oblasti (i o
morfologii cév). Pulzní dopplerovské měření poskytuje záznam rychlostního spektra toku krve v cévě.

Barevná duplexní ultrasonografie − Obraz je složen z barevné a černobílé části. Černobílá část poskytuje
morfologickou informaci o odrazivosti, barevná část informaci o pohybu ve sledovaném řezu (pohyb toku krve).
Tok krve od sondy je zobrazen modře, tok k sondě červeně. Jas barvy pak udává rychlost toku. Čas k získání
barevného obrazu je větší než čas potřebný k získání obrazu černobílého. Proto je obrazová frekvence barevných
obrazů menší než obrazová frekvence černobílého obrazu.

Odkazy
Související články
Dopplerovská echokardiografie
Ultrazvuk

Příklad USG vyšetření –


Achondrogeneze

Ultrazvuk je periodická změna tlaku v prostředí s frekvencí vyšší než horní hranice lidského sluchu, tedy vyšší než 20
kHz.

Ultrazvukové kmity lze vytvořit třemi typy generátorů:

1. mechanickými (malé ladičky, píšťaly: malá frekvence a výkon),


2. magnetostrikčními (kmity kolem železné tyčinky v magnetickém poli elektromagnetu, který je napájen
střídavým proudem: velký výkon, ale frekvence jen do 100 kHz; použití v zubním lékařství a chirurgii),
3. piezoelektrickými (destička z křemene je připojena k elektrodám se střídavým napětím, a tak kmitá se stejnou
frekvencí jako napětí a mění energii elektrickou na mechanickou, která rozkmitá okolní prostředí: použití pro
diagnostické i terapeutické účely).

Vlastnosti
Vysoká frekvence způsobuje velmi krátkou vlnovou délku ultrazvukových vln, ve kterých ale dochází při vyšších
intenzitách k tlakovým změnám v řádech MPa (nebezpečí poškození buněčných jader). Na rozhraních různých tkání (s
různou rychlostí šíření zvuku) dochází k částečné změně směru šíření vln a jejich odrazu (analogicky k zákonu o lomu
světla) → použití v diagnostice.

. 100 R = procento odražené energie dopadající vlny, z1 a z2 = akustické impedance dvou prostředí.

Účinky
Podrobnější informace naleznete na stránce Účinky ultrazvuku.

Tepelné účinky – energie vlny je přímo úměrná její f2. Značná absorpce je na rozhraní tkání s různou akustickou
impedancí (měkká tkáň X kost = periostální bolest).
Mechanické – průchod ultrazvukové vlny prostředím má za následek lokální tlakové změny (MPa/mm).
Fyzikálně-chemické – ultrazvuk má účinky disperzní, což znamená, že s jeho pomocí připravujeme jemné
suspenze, emulze, pěny apod., a také koagulační – slouží např. k čištění plynů.
Biologické – do intenzity 3 W/cm2 mají ultrazvuky spíše biopozitivní účinky: zrychlení metabolické výměny
apod., nad 3 W/cm2 mají za následek ireverzibilní morfologické změny – rozbití buněčného jádra, tepelná
koagulace bílkovin.

Diagnostické použití ultrazvuku


Podrobnější informace naleznete na stránce Diagnostické aplikace ultrazvuku.

Diagnostické ultrazvuky pracují s frekvencemi 3–10 MHz (podkožní sondy okolo 7 MHz).

Ultrazvuky generují akustický puls, který se šíří rychlostí zvuku v daném prostředí a při dopadu se část jeho energie
odrazí. Krystal pak detekuje odrazy signálu a určí velikost echa, z časového zpoždění pak hloubku odrazu.

A-obraz: lineární záznam odrazů v závislosti na hloubce.

B-obraz: otočení A-obrazu o 90°, velikost echa v určité hloubce je úměrná sytosti bodů v řádku na obrazovce →
složením mnoha přímek vedle sebe vznikne dvojrozměrný v čase pohyblivý obraz.

M-obraz: sytost bodů lze v čase zaznamenávat na běžící papír; důležité při hodnocení pohyblivosti jednotlivých
úseků srdce.

S vyšší frekvencí vln se zvyšuje rozlišení, ale stoupá také absorpce (snižuje se hloubka zobrazení).

Terapeutické použití ultrazvuku


Podrobnější informace naleznete na stránce Terapeutické užití ultrazvuku.

Při léčbě pomocí ultrazvuku se používají nejčastěji přístroje o frekvenci 0,8–1 MHz s intenzitou 0,5–3 W/cm2 při
expozici 10 min.

Léčebné účinky: hloubkový tepelný účinek, tišení bolesti, uvolnění dlouhotrvajícího lokálního svalového napětí,
zvýšení místního krevního oběhu, zvýšení metabolismu. Používá se především u kloubních a nervových zánětů.

V zubním lékařství se kmity s malou amplitudou (setiny mm) používají k odstraňování zubního kamene.

Odkazy
Související články
Ultrazvuk/Diagnostické aplikace ultrazvuku
Dopplerovská echokardiografie
Dopplerovská ultrasonografie
Piezoelektrický jev
Ultrasonografie (2. LF UK)

Externí odkazy

Ultrazvuk (česká wikipedie)


Ultrasound (anglická wikipedie)
Janík Václav: Ultrasonografie. Multimediální podpora výuky klinických a zdravotnických oborů :: Portál 3.
lékařské fakulty UK [online] 14.3.2011, poslední aktualizace 23.11.2011 [cit. 2011-12-22] Dostupný z WWW:
<http://portal.lf3.cuni.cz/clanky.php?aid=92>. ISSN ISSN 1804-3143
echopedia.org
Rychlá sonografická vyšetření FAST a EFAST www.prolekare.cz

Zdroj

KUBATOVA, Senta. Biofot [online]. [cit. 2011-01-31]. <https://uloz.to/!CM6zAi6z/biofot-doc>.


Ultrafialové záření (biofyzika)

Ultrafialové zaření (UV) je elektromagnetické vlnění o vlnové délce kratší než má viditelné světlo, a delší než má
rentgenové záření – tedy asi 400–10 nm.

UV záření dělíme do 3 pásem podle vlnových délek:

pásmo blízké neboli UV-A, jehož vlnové délky se pohybují v rozmezí 320-380 nm
pásmo střední neboli UV-B, ve kterém se nacházejí vlnové délky s rozsahem 280-320 nm
pásmo vzdálené neboli UV-C, kde jsou vlnové délky kratší než 280 nm[1] a označuje se též jako germicidní pásmo,
neboť má největší baktericidní účinky.

Zdroje UV záření

Umělé zdroje jsou např. tělesa zahřátá na vysokou teplotu, výbojky a lampy naplněné parami rtuti (výbojky musí být z
křemenného skla, protože obyčejné sklo UV záření pohlcuje). Přirozeným zdrojem je Slunce a součástí jeho záření je
ultrafialové světlo všech 3 pásem. Krátkovlnnou část UV-C absorbuje ozonová vrstva, k povrchu se tak dostává jen UV-
A a UV-B, kdy záření blízkého pásma tvoří 95% a záření středního pásma 5%[1] z celku (UV-C již neuvažujeme).
Zatažená obloha přitom úměrně snižuje jeho intenzitu.

Účinky a využití UV záření

Škodlivě působí na zrakový orgán, ve kterém může zapříčinit zánět spojivek a rohovky, je proto důležité chránit se
vhodnými brýlemi. Vlnové délky kratší než 300 nm[1] při působení na kůži vyvolávají tvorbu vitaminu D z jeho
prekurzorů (UV záření bylo využíváno jako prevence křivice). Ultrafialové záření se na kůži částečně rozptýlí nebo jej
odrazí stratum corneum a částečně se v závislosti na vlnové délce buď absorbuje v epidermis (UV-B) nebo se dostává
až k dermis (UV-A)[1]. Na ozáření UV paprsky reaguje kůže zarudnutím (erytémem) a následným vytvořením pigmentu
melaninu v pigmentových buňkách. Melanin poté brání pronikání UV-A záření do hlubších vrstev kůže. Vysoké dávky
ultrafialového záření mohou v kůži vyvolat degenerativní změny (nádory kůže). Jeho dezinfekční účinky jsou
využívány např. v operačních sálech, JIP nebo v laboratořích (germicidní lampy, což jsou rtuťové výbojky). V biochemii
se využívají fotoluminiscenční účinky UV záření k detekci některých organických látek.

Odkazy
Související články
Ultrafialové záření (hygiena)
Biologické účinky UV záření, ochrana zdraví

Použitá literatura

JIŘÍ, Beneš, Kymplová JAROSLAVA a Vítek FRANTIŠEK. Základy fyziky pro lékařské a zdravotnické obory : pro
studium i praxi. - vydání. Grada Publishing, a.s., 2015. 236 s. ISBN 9788024747125.

Reference
1. JIŘÍ, Beneš, Kymplová JAROSLAVA a Vítek FRANTIŠEK. Základy fyziky pro lékařské a zdravotnické obory : pro
studium i praxi. - vydání. Grada Publishing, a.s., 2015. 236 s. ISBN 9788024747125.
Infračervené záření

Charakteristika
Infračervené záření je elektromagnetické vlnění o vlnové délce delší než 760nm.
Zdrojem jsou tělesa zahřátá na vyšší teplotu.
Při pohlcování tohoto záření se těleso zahřívá (tepelná výměna).
Viditelné světlo
Viditelné světlo, jinak také označované pouze jako světlo, je elektromagnetické záření o frekvenci 3,9 x 1014 Hz do
7,9 x 1014 Hz, což ve vakuu odpovídá vlnovým délkám v rozmezí 380–740 nm. Tato část elektromagnetického spektra
má tu vlastnost, že při dopadu na fotoreceptory lidského oka (tyčinky a čípky) vyvolává zrakový vjem. Jedná se o
záření Slunce (hvězdy), které při průchodu atmosférou dopadne na zemský povrch. V celém elektromagnetickém
vlnění se světlo nachází mezi infračerveným (vlnová délka větší než 740 nm) a ultrafialovým zářením (vlnová délka
kratší než 380 nm). Světlo má duální charakter, můžeme ho popisovat jako vlnu, ale i jako částici. Fyzikální obor,
který se zabývá zkoumáním vlastností viditelného světla se nazývá optika.

Rychlost světla
Rychlost světla (světelná rychlost) ve vakuu je 299 792 458 m/s, zaokrouhleně se uvádí 3×108 m/s (300 000 km/s).
Fyzikální jednotka se značí písmenem c. Ve vzduchu je jeho rychlost jen zanedbatelně nižší než ve vakuu, ale v jiných
prostředích je vždy nižší, je závislá na indexu lomu. Například ve skle činí zhruba 200 000 km/s a ve vodě 225 000
m/s.

viditelné světlo

Barevné spektrum
Viditelné světlo se skládá z několika barev, které nazýváme spektrálními. Jednotlivé barvy jsou typické pro určitou
vlnovou délku a vzájemně do sebe přecházejí. Nejkratší vlnovou délku, ale nejvyšší frekvenci, má fialová (380 až 430
nm), s nárůstem vlnové délky a poklesem frekvence následují modrá (430 až 500 nm), azurová (500 až 520 nm),
zelená (520 až 565 nm), žlutá (565 až 590 nm), oranžová (590 až 625 nm) a červená (625 až 740 nm). Lidské oko je
nejcitlivější na vlnové délky okolo 555 nm (zelená). Objekt vidíme v takové barvě, jakou vlnovou délku jeho povrch
odráží, ostatní barvy jsou předmětem absorpovány (pohlceny). Vedle spektrálních barev existují i barvy nespektrální,
které vznikají jako vjem smíšení několika barev. Mezi tyto barvy patří šedá, bílá, černá, ale i například růžová nebo
tyrkysová.

Viditelné spektrum, vlnová délka v nanometrech

Šíření
Šíření světla probíhá na základě Huygensova principu pomocí vlnoploch, vliv na tento jev mají i vlastnosti samotného
prostředí, ve kterém k šíření dochází. V opticky homogenním prostředí mají vlnoplochy v blízkosti zdroje světla kulový
tvar, ve velké vzdálenosti by je šlo považovat za roviny. Paprsky, představují myšlené čáry, které ukazují směr šíření
světla, a jsou vždy kolmé na vlnoplochu, se zde šíří přímočaře. Šíření světla je také

Světelné zdroje

Zdrojem světla můžeme nazvat každé těleso, ve kterém světlo vzniká a je vyzařováno do okolí (Slunce, žárovka,
plamen atd.). V případě, že velikost samotného zdroje je oproti vzdálenosti, ze které ho pozorujeme zanedbatelná,
nazveme ho bodovým zdrojem světla (laser). Přirozené zdroje nelze považovat za bodové, protože se světlo šíří
najednou z několika různých bodů.

Optické prostředí

Optickým prostředím nazveme každé prostředí , ve kterém dochází k šíření světla. Můžeme ho rozdělit na průhledné
(nedochází zde k rozptylu světla), průsvitné (světlo se šíří, ale dochází k částečnému rozptylu) a neprůhledné (světlo
je na povrchu pohlcováno nebo odráženo). Z pohledu optiky dochází ještě k dělení na prostředí homogenní (v celém
svém objemu má stejné vlastnosti), izotropní (vlastnosti nejsou závislé na směru šíření - sklo) a anizotropní
(vlastnosti závisí na směru šíření světla - krystal). Průhledné sklo dělíme na čiré (světlo prochází beze změny) a
barevné (pohltí některé vlnové délky).
Odkazy
Související články

Barevnost látek
Umělé osvětlení
Infračervené záření
Ultrafialové záření

Zdroj

Odkazy

WIKIPEDIE Otevřená encyklopedie. Světlo [online]. [cit. 2019-01-20]. <https://cs.wikipedia.org/wiki/Sv%C4%9Btlo>.

Gymnázium Ladislava Jaroše Holešov. Úvod do optiky [online]. [cit. 2019-01-20]. 


<http://www.gymhol.cz/projekt/fyzika/01_uvod/01.htm>.

Techmania Science Center / EDUPORTÁL. Světlo [online]. [cit. 2019-01-20]. 


<https://edu.techmania.cz/cs/encyklopedie/fyzika/svetlo>.

Encyklopedie fyziky. Šíření světla [online]. [cit. 2019-01-20]. <http://fyzika.jreichl.com/main.article/view/435-sireni-


svetla>.

FYZIKA 007. Šíření světla [online]. [cit. 2019-01-20]. <http://www.fyzika007.cz/optika/sireni-svetla>.

Literatura

Beneš, Jiří, Jirák, Daniel a Vítek, František. . Základy lékařské fyziky. 4. vydání vydání. 2015. ISBN 978-80-246-2645.

Beneš Jiří, Kymplová Jaroslava, Vítek František. . Základy fyziky pro lékařské a zdravotnické obory pro studium a
praxi. 1. vydání vydání. 2015. ISBN 978-80-247-4712-5.
[zdroj?]

Článek neobsahuje vše, co by měl.


Můžete se přidat k jeho autorům a jej.
O vhodných změnách se lze poradit v diskusi.
Index lomu světla

Lom světla

Rozklad světla různých vlnových


délek na hranolu ilustruje závislost
indexu lomu na vlnové délce

Index lomu světla, obvykle značený n, je poměr rychlosti světla ve dvou optických prostředích. Je to bezrozměrná
veličina charakteristická pro danou dvojici prostředí a pro danou frekvenci. Podle toho, s čím porovnáváme rychlost
světla v daném prostředí, rozlišujeme absolutní a relativní index lomu.

Absolutní a relativní index lomu


Absolutní index lomu

Absolutní index lomu je definovaný jako poměr rychlosti šíření světla ve vakuu c a rychlosti v v daném prostředí:

Absolutní index lomu je charakteristikou konkrétního prostředí, jedná se o materiálovou konstantu. U řady pevných
látek je tabelován. Protože index lomu je závislý na vlnové délce procházejícího záření, dochází na optickém hranolu k
rozkladu světla.

Protože je v každém prostředí rychlost světla nižší než rychlost světla ve vakuu, je absolutní index lomu vždy vyšší než
jedna.

Relativní index lomu

Relativní index lomu je definovaný jako poměr rychlosti šíření světla ve dvou optických prostředích v1 a v2:

Zatímco absolutní index lomu je materiálovou konstantou, relativní index lomu je závislý na uspořádání, obvykle tedy
charakterizuje vlastnosti rozhraní dvou optických prostředí. Relativní index lomu na rozhraní prostředí označených
čísly 1 a 2 a označovaný obvykle n12 úzce souvisí s absolutními indexy lomu těchto prostředí n1 a n2, vlastně je dán
jejich poměrem (v opačném pořadí!). Vztah se odvodí následujícím způsobem:

Lom světla
Dopadá-li světlo na rozhraní dvou různých optických prostředí, dochází k jeho odrazu (paprsek se vrací do prostředí,
ze kterého přišel pod stejným úhlem a ve stejné rovině) a lomu (paprsek se lomí do druhého prostředí). Při lomu
světla paprsek dopadající pod úhlem α od kolmice na optické rozhraní mění svůj směr a láme se pod úhlem β se
změněnou rychlostí, charakteristickou pro druhé prostředí.

Dle Snellova zákona platí:


Prostředí o menším indexu lomu se označuje jako prostředí opticky řidší, prostředí o indexu lomu vyšším se označuje
jako prostředí opticky hustší.

Pokud je úhel dopadu α větší než úhel β, hovoříme o lomu ke kolmici. Tato situace nastává v případě, že světlo
vstupuje z prostředí opticky řidšího do prostředí opticky hustšího.

V případě, že je úhel β větší než úhel α, hovoříme o lomu od kolmice. Zcela analogicky toto nastává, pokud světlo
postupuje z prostředí opticky hustšího do prostředí opticky řidšího.

Totální odraz
Dochází-li k lomu od kolmice, může při určitém úhlu dopadu nastat situace, že procházející paprsek bude kolmý na
kolmici, tedy měl by procházet právě rovinou rozhraní. Tento úhel nazýváme mezní úhel. Pro paprsky, které při
dopadu na rozhraní svírají s kolmicí úhel větší než mezní, se rozhraní chová jako zrcadlo a dochází k totálnímu
odrazu (synonymně se používá tvar úplný odraz). Velikost mezního úhlu φ závisí pouze na indexu lomu:

Měření indexu lomu


Přístroj určený k měření indexu lomu se nazývá refraktometr. Refraktometr je obvykle založen na měření mezního
úhlu (Abbého refraktometr pro kapaliny, Pulfrichův refraktometr pro pevné látky). Index lomu lze měřit i přímo
měřením lomu např. laserového paprsku při průchodu daným prostředím pomocí goniometru (zařízení pro přesné
měření úhlů), ovšem je třeba použít geometricky přesně definovaný kus měřeného materiálu.

V medicíně má význam především měření indexu lomu kapalin. Platí totiž, že index lomu roztoku je závislý na
koncentraci roztoku, takže měření indexu lomu lze využít jako kvantitativní analytickou metodu. Měření indexu lomu
moči může sloužit jako metoda stanovení relativní hustoty. Podobně měření indexu lomu séra je metoda používaná k
rychlému stanovení celkové bílkoviny.

Odkazy
Související články

Polarizace světla
Polarimetrie
Viditelné světlo
Odraz světla
Refraktometrie

Zdroj

KUBATOVA, Senta. Biofot [online]. [cit. 2011-01-31]. <https://uloz.to/!CM6zAi6z/biofot-doc>.


Endoskop

Flexibilního endoskopu

Endoskop je optický přístroj pro zobrazování vnitřních dutin.

Podle technického provedení se endoskopy dělí na:

endoskopická zrcátka;
rigidní endoskopy (tubusové);
flexibilní endoskopy;
fibroskopy – CCD čip umístěn proximálně;
videoendoskopy – CCD videočip umístěn distálně.

Flexibilní endoskop může být založen na vedení světla ve svazku optických vláken. Pokud má optické vlákno ve
vnější vrstvě index lomu menší (řidší prostředí) než je index lomu uvnitř vlákna, tak ve vláknu dochází k úplnému
odrazu světla. Použitím mnoha takových vláken lze pak přenést obraz z dutiny až k pozorovateli.

Diagnostické uplatnění
Jedná se o endoskopická vyšetření dutin:

Gastroenterologie (gastroskopie, kolo(no)skopie).


Pneumologie (thorakoskopie, bronchoskopie).
Urologie (uretheroskopie).
ORL (laryngoskopie).
Ortopedie (arthroskopie).
Gynekologie (kolposkopie, uteroskopie).

Terapeutické použití
Zástava krvácení.
Odstranění polypů.
Extrakce žlučových kamenů.
Celá řada laparoskopických operací.

Odkazy
Související články

Endoskopie
Gastroskopie
Kolonoskopie

Externí odkazy

Endoskop (česká wikipedie)


Endoscopy (anglická wikipedie)

Endoskopie – Doc.RNDr. Roman Kubínek, CSc.


Disperse světla

Disperse světla na optickém


hranolu

Disperzí (světla) nazýváme jevy, které vznikají v důsledku závislosti indexu lomu na vlnové délce, tedy jevy při
kterých dochází během průchodu světla látkou k rozložení bílého světla na jednotlivé barvy. Příčinou disperze je
závislost fázové rychlosti šíření světelné vlny na její vlnové délce (v = f (λ)) při průchodu hmotným prostředím.

Disperze světla je rozklad elektromagnetického záření na složky v různých vlnových délkách.

c = fázová rychlost ve vakuu, ε = permitivita vakua, μ = magnetická permeabilita vakua

Tento vztah ukazuje, že rychlost světla je ve vakuu pro všechny vlnové délky stejná a ve vakuu disperze světla
nenastává.

Disperse látek – veličina určující rychlost změny indexu lomu (n =c / v) v závislosti na vlnové délce (derivace indexu
lomu podle vlnové délky). V průhledných prostředích roste index lomu s klesající λ. Je-li tato veličina < 0 = normální
disperze, > 0 = anormální disperze.

Rozdělení

Normální disperze,
Chromatická disperze,
Anomální disperze.

Barvy duhy v porovnání s barevným


spektrem

Nejčastější disperze je duha. Sluneční světlo dopadá na padající kapku deště, lomí se paprsek do kapky a část světla
se odráží od vnitřního povrchu kapky a následovně se láme ven. První lom způsobí rozklad světla na spektrum.
Druhý lom zvětší. Světlo se láme ve velkém počtu. Červená vychází z kapek výše a fialová níže. Pokud je Slunce výš
nad obzorem, duha není vidět.

Duha 2 řádu má úhel 53° a obrácené pořadí barev.

Odkazy
Související články

Difrakce světla
Index lomu světla

Externí odkazy
Disperze světla (encyklopedie fyziky)

Zdroj
KUBATOVA, Senta. Biofot [online]. [cit. 2011-01-31]. <https://uloz.to/!CM6zAi6z/biofot-doc>.
Pružný a nepružný rozptyl
Při průchodu světla zředěným plynem dochází k jeho rozptylu na molekulách plynu.

Pružný rozptyl – nastává na částicích výrazně menších než vlnová délka světla, rozptýlené světlo si zachovává
původní vlnovou délku,

Nepružný rozptyl – na částicích srovnatelných s vlnovou délkou světla, rozptýlené světlo má v různých směrech
různou vlnovou délku.

Intenzita rozptýleného světla je nepřímo úměrná čtvrté mocnině jeho vlnové délky, tzn. obloha se nám jeví jako
modrá, protože krátkovlnné záření má největší intenzitu.

Obsahuje-li atmosféra více větších částic (prach), rozptylují se všechny složky světla stejně a obloha je bílá.

Při svítání je dráha slunečních paprsků nejdelší a je tedy rozptylem ovlivněna nejvíce. V této chvíli atmosférou nejlépe
(přímo) prochází červené světlo.

Odkazy
Související články

Elektromagnetické spektrum
Viditelné světlo
Infračervené záření
Ultrafialové záření (biofyzika)

Použitá literatura

KUBATOVA, Senta. Biofot [online]. [cit. 2011-01-31]. <https://uloz.to/!CM6zAi6z/biofot-doc>.


Polarizace světla
Světlo je elektromagnetické vlnění, tedy jde o kmitání vektoru intenzity elektrického pole E a k němu kolmého vektoru
indukce magnetického pole B. Oba tyto vektory leží v rovině kolmé ke směru šíření vlnění (ve směru paprsku) a spolu
s vektorem ve směru šíření (tzv. Poyntingův vektor) tvoří pravotočivou soustavu. Z toho důvodu stačí uvažovat pouze
jeden vektorů E a B, obvykle se volí intenzita elektrického pole E.

Představme si, že svazkem světla povedeme rovinu kolmou ke směru šíření. V každém bodě této roviny bude mít
vektor E jiný a v zásadě zcela nahodilý směr. Takovéto záření se nazývá nepolarizované. Polarizované světlo se od
nepolarizovaného liší tím, že vektory E mají v myšlené rovině protínající svazek světla stejný směr.

Je-li směr vektoru E v konkrétní rovině neměnný, hovoříme o lineární polarizaci. Pokud se ale směr vektoru E v čase
mění, bude opisovat elipsu. V tomto obecném případě hovoříme o eliptické polarizaci. Snadno lze nahlédnout, že
lineární polarizace je speciálním případem eliptické polarizace. Jiným speciálním případem eliptické polarizace je
kruhová polarizace, při které vektor E opisuje kružnici.

Zdroje polarizovaného světla


Polarizované světlo lze získat následujícími způsoby:

1. odrazem – Při odrazu světla dochází k částečné polarizaci, tedy k tomu, že jeden směr vektorů E převažuje. Při
jednom konkrétním úhlu (Brewsterův úhel) však může dojít k polarizaci úplné. Brewsterův úhel závisí na indexu
lomu prostředí a z toho důvodu je závislý i na vlnové délce.
2. lomem – Při lomu světla dochází, podobně jako při odrazu, k částečné polarizaci. Ovšem při lomu světla se jedná
vždy o polarizaci neúplnou.
3. dvojlomem – Některé krystaly (např. islandský vápenec) jsou anizotropní, tedy jejich optické vlastnosti (index
lomu) jsou závislé na směru procházejícího paprsku. Když na takový krystal dopadne paprsek světla, dojde k lomu
na dva paprsky (dvojlomu). Jeden paprsek, tzv. řádný paprsek, se řídí zákonem lomu, tedy prochází jako kdyby
index lomu nezávisel na směru. Druhý paprsek, tzv. mimořádný paprsek, se zákonem lomu v základním tvaru
neřídí, index lomu, který v zákonu lomu vystupuje, zde totiž není konstantní, závisí na úhlu, pod jakým mimořádný
paprsek prochází prostředí.Oba tyto paprsky jsou lineárně polarizované a jejich vektory E jsou na sebe kolmé.
4. polaroidem – Polaroid (polarizační filtr) je představitelem tzv. absorpčních polarizátorů. Základem jsou
rovnoběžně uspořádané podlouhlé, v praxi např. molekuly herapatitu (perjodid síranu chininového). Mechanisticky
si to můžeme představit jako hustý plaňkový plot. Pokud prochází takovýmto prostředím světlo, jehož vektor E je
rovnoběžný s dlouhými osami molekul, částečně prochází. Pokud ale prochází světlo, jehož vektor E není s dlouhými
osami molekul rovnoběžný, je takové světlo plně absorbováno.

Měření optické aktivity


Více informací naleznete v článku Polarimetrie

Optická aktivita je schopnost některých látek otáčet rovinu polarizovaného světla (tedy otáčet vektor E). V případě
roztoků pak úhel, o který je rovina otočena, závislý i na koncentraci roztoku.

Z hlediska principu lze hovořit o dvou částech přístroje k měření optické aktivity (polarimetru):

1. Polarizátor – Generátor polarizované světla, který je umístěn před kyvetou s měřeným roztokem.
2. Analyzátor – Zařízení, které umožňuje vizualizovat rovinu polarizace. V praxi používáme dva polaroidy, kdy
pootáčením analyzujícího zjistíme, že v světlo prochází jen v některých jeho polohách.
Odkazy
Související články

Index lomu světla


Polarimetrie
Viditelné světlo

Zdroj

KUBATOVA, Senta. Biofot [online]. [cit. 2011-01-31]. <https://uloz.to/!CM6zAi6z/biofot-doc>.


Interference světla
Interference světla znamená vzájemné ovlivňování, střetávání nebo prolínání světelných vln. Tímto jevem je potvrzena
vlnová podstata světla a jeho vlastnosti se tedy podobají mechanickému vlnění.

Vznik
Světelnou interferenci pozorujeme zejména u záření monochromatického. Důležité je především to, že vlny se v
některých místech střetu navzájem zesilují a v jiných zeslabují, dochází tak ke změnám intenzity světla. Skládáním
světelných vln vzniká interferenční obrazec (struktura), který je pozorovatelný pouze za předpokladu, že obě vlnění
jsou vzájemně koherentní. Jako koherentní nazveme vlny tehdy, jestliže mají stejnou frekvenci a jejich fázový rozdíl se
v daném místě nemění (je konstantní). U běžných zdrojů světla (Slunce, žárovka, plamen) nelze interferenci
pozorovat, protože se jedná o nekoherentní vlnění. Světlo, které vyzařují tyto zdroje, je výsledkem chaotických vln o
různé frekvenci.

Základní vlastnosti

Interference dvou vlnění

Pro světelné vlnění platí:

λ0 = vlnová délka světla ve vakuu


c = rychlost světla ve vakuu
n = index lomu prostředí
λ = vlnová délka světla v prostředí o indexu lomu n
v = fázová rychlost šíření světla

Výsledek světelné interference závisí na celkovém dráhovém rozdílu:

interferenční maximum nastává, když je dráhový rozdíl roven sudému počtu půlvln

interferenční minimum nastává, když je dráhový rozdíl roven lichému počtu půlvln

Využití
Příkladem praktického využití interference světelného vlnění jsou interferometry, které detekují intenzitu dvou vln.
Interferenci světla můžeme také použít při výrobě antireflexních vrstev na povrchu čoček (fotoaparát, kamera).

Newtonova skla
Newtonova skla jsou tvořena skleněnou deskou a ploskovypuklou čočkou o velkém poloměru křivosti. Při dopadu
monochromatického světla na optickou soustavu dochází k interferenci odraženého světla a vzniku interferenčního
obrazce, který má podobu světlých a tmavých kroužků (Newtonovy kroužky). Použijeme-li bílé světlo, budou vzniklé
proužky duhových barev.
Energie a intenzita světla
radiometrické veličiny (pro elmag.záření) jednotky rad.vel. fotometrické veličiny (pro viditelné světlo) jednotky fotom.vel.
zářivost (Ie) W/sr svítivost (I) kandela (cd)
zářivý tok (Φe) W světelný tok (Φ) lumen (lm)
intenzita záření (Ee) W/m2 osvětlení (E) lux (lx)

Zářivá energie se šíří do všech směrů.

Zářivý tok je výkon přenášený zářením, odpovídá zářivé energii, kterou vyzáří zdroj za jednotku času.
Světelný tok vyjadřuje intenzitu zrakového vjemu, který vyvolává energie světelného záření procházející
prostorem za jednotku času (závisí na vlnové délce použitého světla).

Intenzita záření = zářivý tok procházející příčným průřezem plochy zdroje (v m2).
Osvětlení = světelný tok dopadající na jednotku plochu.

Zářivost = podíl zářivého toku a velikosti prostorového úhlu, do kterého je tento tok vyzařován.
Svítivost = světelný tok vyslaný bodovým zdrojem do jednotkového prostorového úhlu.
Jas = kolmá svítivost plochy dělená její velikostí [cd/m2].

Světelná účinnost záření:

[lm/W]
(Vzhledem k různé citlivost oka na různé vlnové délky je tato veličina fcí vlnové délky.)

Lambertův zákon: zákon vyjadřující pokles svítivosti plošného zdroje s rostoucím úhlem φ.

Iφ = IN. cos φ, kde IN je svítivost ve směru kolmém, Iφ svítivost ve směru odchýleném od normály o úhel φ.
Světelný zdroj splňující tento zákon se nazývá kosinový zářič.

Odkazy
Související články

Sdružené osvětlení, oslnění | Umělé osvětlení | Denní osvětlení

Zdroj

KUBATOVA, Senta. Biofot [online]. [cit. 2011-01-31]. <https://uloz.to/!CM6zAi6z/biofot-doc>.


Difrakce světla

Difrakce na dvou štěrbinách

Jev odchýlení světla od přímočarého směru šíření, které není způsobeno odrazem či lomem. Nastává na překážkách,
které mají velikost srovnatelnou s vlnovou délkou světla.

Důležitý je průchod světla štěrbinou, která je podle Huygensova principu zdrojem elementárních vln.

Optická mřížka je tvořena velkým počtem rovnoběžných vrypů na skle (až 1000 na 1 mm), kdy neporušená místa
představují štěrbiny a vrypy mezery mezi nimi.

d · sin α = k · λ
d = mřížková konstanta (vzdálenost štěrbin), α = úhel vycházejících paprsků, k = řád ohybového maxima, λ =
vlnová délka světla

Z rovnice vyplývá, že při průchodu bílého světla vznikne interferencí ohybové (mřížkové) spektrum, kdy je nejvíce
odchýleno červené a nejméně fialové světlo.

Odkazy
Související články

Disperse světla
Optická mřížka
Kapitola Monochromátor v článku Spektrofotometr

Zdroj

KUBÁTOVÁ, Senta. Difrakce (ohyb) světla [online]. [cit. 2010-09-03]. <https://uloz.to/!CM6zAi6z/biofot-doc>.


Zobrazení lomem, čočková rovnice, optická
mohutnost
K zobrazení lomem se nejvíce využívá čoček. Čočka je nejjednodušší optická soustava. Je tvořena průhledným
prostředím (nejčastěji sklem, někdy plastem) ohraničeným dvěma lámavými plochami (obvykle kulovými).

Existují dva typy čoček:

spojky : rovnoběžné paprsky se protínají v obrazovém ohnisku ohnisko je


skutečné (f > 0);
rozptylky : rovnoběžné paprsky jsou v obrazovém ohnisku rozbíhavé; protínají se při zpětném prodloužení v
předmětovém ohnisku ohnisko je zdánlivé (f < 0).

Čočka může být:

tenká : tloušťka čočky je zanedbatelná vůči její ohniskové vzdálenosti;


tlustá : tloušťka čočky není zanedbatelná vůči její ohniskové vzdálenosti.

Čočku nazýváme tenkou, je-li její tloušťka malá ve srovnání s poloměry jejích kulových povrchů. V homogenním
prostředí platí: .

Čočková rovnice
Tlustá čočka

n2 : index lomu materiálu čočky; n1 : index lomu okolního prostředí;


r1 a r2 : poloměry křivosti.

Tenká čočka

a : předmětová vzdálenost,
a‘ : obrazová vzdálenost,
f : ohnisková vzdálenost.

Tato rovnice je platná pro spojku i rozptylku. Při dosazování hodnot a, a‘ se řídíme znaménkovou konvencí.

a > 0 : předmět se nachází v prostoru předmětovém (před čočkou);


a < 0 : předmět se nachází v prostoru obrazovém (za čočkou);
a‘ > 0 : obraz se nachází v prostoru obrazovém; je skutečný;
a‘ < 0 : obraz se nachází v prostoru předmětovém; je neskutečný.

Optická mohutnost
Optická mohutnost je veličina vyjadřující lámavou schopnost čočky.

Výpočet optické mohutnosti

φ : optická mohutnost,
f : ohnisková vzdálenost.

Jednotka: dioptrie (D) = m-1 (f musí být uvedena v metrech!). 1 D je optická mohutnost čočky s ohniskovou
vzdáleností 1 m. Pro spojky platí φ > 0 a pro rozptylky φ < 0.

Zobrazení spojkou

(Pozn.: Vzdálenost předmětu (x) od čočky: jaký je obraz)

x > 2f : skutečný, převrácený a zmenšený;


x = 2f : skutečný, převrácený, stejně velký jako předmět;
x < 2f : skutečný, převrácený, zvětšený;
x = f : obraz je v nekonečnu;
x < f : zdánlivý, přímý, zvětšený.
Při zobrazení rozptylkou nezáleží na jeho vzdálenosti od čočky, obraz je vždy zdánlivý, přímý a zmenšený.

Odkazy
Použitá literatura

SVOBODA, Emanuel. Přehled středoškolské fyziky. - vydání. Prometheus, 2005. 531 s. ISBN 9788071963073.

JIŘÍ, Beneš,, Jirák, DANIEL a Vítek, FRANTIŠEK. Základy lékařské fyziky. - vydání. Charles University in Prague,
Karolinum Press, 2015. 322 s. ISBN 9788024626451.
Zvětšení lupy
Lupa – spojná čočka s ohniskovou vzdáleností 1–10 cm. Vzniká zdánlivý, zvětšený, přímý obraz. Pro zvětšení Z platí:

kde δ = 0,25 m (konvenční zraková vzdálenost) a f = ohnisková vzdálenost.

Odkazy
související články
Vady zraku
Oko

Zdroj

KUBATOVA, Senta. Biofot [online]. [cit. 2011-01-31]. <https://uloz.to/!CM6zAi6z/biofot-doc>.


Optický mikroskop
Optický (nebo také světelný) mikroskop je zařízení, které používáme k pozorování předmětů, které nejsme schopni
spatřit pouhým okem, tj. předmětů, které mají velikost menší než 0,2 mm. Jeho rozlišovací schopnost (schopnost
odlišit od sebe dva body) je tedy 0,2 mm - 0,2 μm. Pro menší předměty musíme použít mikroskop elektronový, který
rozliší předměty až do velikosti 0,1 nm. Tedy předměty o tři řády menší, než zvládá rozlišit mikroskop světelný.
Obecně tedy můžeme říci, že kde končí rozlišovací schopnost oka, začíná rozlišovací schopnost světelného
mikroskopu, a kde končí rozlišovací schopnost světelného mikroskopu, tam začíná rozlišovací schopnost mikroskopu
elektronového.

Schéma optického
mikroskopu.

Historie mikroskopu
Již ve starověku byly používány různé čočky na pozorování předmětů, které byly příliš malé na pozorování pouhým
okem. První velký objev se ovšem datuje do 17. století, kdy Anton van Leeuwenhoek vynalezl přístroj, který se skládal
z šroubku, kovového plátku, drážky na upevnění předmětu a sférické čočky . Tento přístroj se dnešnímu mikroskopu
nepodobal, je ovšem až obdivuhodné, že jím bylo možné dosáhnout neuvěřitelného zvětšení až 250-500x. K dalšímu
přelomovému objevu dospěl Robert Hook, který jako první začal pracovat se světlem v mikroskopii. Vzhledem k velké
poptávce byl mikroskop postupně zdokonalován a měnil design, až se změnil v mikroskop, který známe dnes.

Schéma zobrazení v optickém


mikroskopu.

Optická soustava mikroskopu


Optická soustava mikroskopu se skládá ze tří spojných soustav - z objektivu, okuláru a kondenzoru (osvětlovací
soustavy).

Objektiv
Nejdůležitější částí optického mikroskopu je objektiv. Jedná se o soustavu obvykle několika čoček, které dohromady
fungují jako spojná čočka. Předmět, který v mikroskopu pozorujeme musí být umístěn ve vzdálenosti mezi f a 2f, tak
bude obraz skutečný, zvětšený a převrácený. V praxi má objektiv malou ohniskovou vzdálenost (v řádu milimetrů),
proto poskytuje větší zvětšení než okulár, jehož ohnisková vzdálenost se volí větší (v řádu centimetrů).
Objektiv určuje zvětšení a kvalitu výsledného obrazu (čím kvalitnější obraz požadujeme, tím kvalitnější objektiv
musíme použít). Zvětšení mikroskopu můžeme měnit, a to otáčením revolvérové hlavice s různými objektivy, které
jsou většinou barevně označeny pro rychlou orientaci poskytovaného zvětšení.

Části mikroskopu

cesta paprsků

Okulár

Okulár, jak již název napovídá, je část mikroskopu, do které pozorovatel hledí (z latinského oculus = oko). Podobně
jako objektiv se také skládá ze soustavy čoček, kterých je ale oproti objektivu méně. Tyto čočky společně opět fungují
jako spojná čočka a poskytují zdánlivý, zvětšený a převracený obraz. Okulár poskytuje zvětšení (10 – 20x), avšak
nezlepšuje rozlišovací schopnost mikroskopu.

Kondenzor

Kromě objektivu a okuláru nalezneme v mikroskopu ještě další opticky důležitý systém, kondenzor. Jedná se o
soustavu čoček nebo zrcadel, které soustřeďují paprsky na pozorovaný objekt a zajišťují tak jeho osvětlení.

Ostatní části mikroskopu


Běžný mikroskop sestává jednak z optické soustavy, ve které vzniká obraz a druhak z mechanické části. Tu tvoří stativ
a stolek, na který se umístí preparát a pomocí makro-šroubu zkoumaný objekt posouváme do vzdálenosti f - 2f
objektivu, kde se preparát začne zaostřovat. K jemnému doostření slouží mikro-škroub, který se nachází těsně u
makro-šroubu. Pro pohyb preparátu ve vodorovné rovině slouží další 2 šrouby (obvykle na opačné straně než
makro/mikro-šroub).

Zobrazovací metody
Zobrazovacích metod ve světelné mikroskopii je spousta. Preparáty jsou obvykle naneseny na průhledném sklíčku,
které je prosvětlováno konkrétním světlem, a my potom můžeme vidět dané preparáty v okuláru. Mezi nejčastější
metody spadá metoda světelného pole a metoda temného pole.

Metoda světelného pole je metoda, kdy dochází k prosvícení preparátu, a my ho vnímáme jako objekt s tmavými
obrysy, který se vyskytuje ve světlém poli (tuto metodu nazýváme mikroskopií v procházejícím světle). Může docházet
i k odražení paprsků, které pak vstupují do objektivu. Pak tuto metodu nazýváme mikroskopií v odraženém světle.

Metoda temného pole je svým způsobem opakem výše uvedené metody. Předmět pozorujeme jako světlý předmět ve
tmavém poli. Důvod tmavého podkladu je ten, že paprsky, které preparát osvětlují, nevstupují do objektivu. Paprsky se
však na preparátu lámou a do objektivu jde světlo různě nakloněné, díky lomu na preparátu.

Další metodou, která je velmi používaná, zvláště v pozorování biologických preparátů, je metoda fázového
kontrastu. Struktury, které chceme v preparátu pozorovat, se zpravidla liší svým indexem lomu od okolí. My potom
vidíme změnu intenzity světla a tím jsme schopni pozorovat jednotlivé struktury.

Zobrazovacích metod samozřejmě existuje daleko více, o některých dalších si můžete přečíst zde: Mikroskopické
metody.

Zvětšení mikroskopu
Poté, co světlo projde skrz objektiv a následně okulár, dostáváme zdánlivý, zvětšený a převrácený obraz. Celkové
zvětšení mikroskopu se vypočítá jako součin příčného zvětšení objektivu a úhlového zvětšení okuláru.

Maximální zvětšení optického mikroskopu ovšem nemůže přesáhnout 2000. větší zvětšení je totiž považováno za
zvětšení prázdné, protože předmět sice vidíme větší, ale nejsme schopni rozlišit detaily a předmět se nám
"rozmazává"

Zvětšení mikroskopu:

Z ...................... Zvětšení mikroskopu

......... Zvětšení poskytované objektivem a okulárem

.................... Optický interval (= vzdálenost ohnisek objektivu a okuláru)

........... Ohnisková vzdálenost objektivu a okuláru (v mm)

250 ................... Konvenční zraková vzdálenost lidského oka (v mm)

Numerická apertura:

Tímto termínem se rozumí schopnost objektivu zachytit co nejširší úhel paprsků. Je to jedna ze základních
charakteristik objektivu a její hodnota je na něm přímo uvedena.

A ..................... Numerická apertura


n .................... Index lomu
a .................... Polovina otvorového úhlu kužele světla

Rozlišovací schopnost mikroskopu:

Je schopnost mikroskopu odlišit od sebe 2 body. Závisí na vlnové délce použitého světla, indexu lomu prostředí
(možné využití imersního oleje, který zvýší index lomu) a samozřejmě na vlastnostech objektivu. Obecně platí, že není
možné rozlišit od sebe 2 body bližší než polovina vlnové délky použitého záření.

a .................... Rozlišovací schopnost


λ .................... Vlnová délka
A .................... Numerická apertura
n .................... Index lomu
a .................... Polovina otvorového úhlu kužele světla

Odkazy
Související články

Konfokální mikroskop
Elektronový mikroskop
Mikroskopie skenovací sondou
Princip zobrazení optickým mikroskopem

Externí odkazy

Mikroskop (česká wikipedie)


Mikroskopické metody

Zdroj
Navrátil, L., Rosina, J. a kol.: Medicínská biofyzika. Praha: GRADA, 2010. 241-256s

Práce s mikroskopem, mikroskopické preparáty. [cit. 2018-12-02]. Dostupné z:


https://is.muni.cz/th/tupwu/PastrnkovaPrilohy2.pdf
Elektronový mikroskop

Elektronový mikroskop je v principu (elektronovou) obdobou optického (fotonového) mikroskopu. Optické čočky
jsou nahrazeny elektromagnetickými čočkami a místo fotonů jsou ke zkoumání objektu použity elektrony. Rozlišovací
schopnost a maximální použitelné zvětšení optického mikroskopu jsou omezeny rozsahem vlnových délek viditelného
světla. Platí, že nejmenší vzdálenost dvou objektů, které lze v pod mikroskopem ještě rozpoznat, je polovina vlnové
délky použitého světelného záření. Fyzikální mez rozlišovací schopnosti optického mikroskopu je tak necelých 200 nm
a maximální užitečné zvětšení mikroskopu s kvalitní optikou a imerzním objektivem nepřesáhne 1500×. Vlnové délky
urychlených elektronů jsou o mnoho řádů menší než vlnové délky fotonů viditelného světla. Proto má elektronový
mikroskop mnohem vyšší rozlišovací schopnost a může tak dosáhnout mnohem vyššího zvětšení (až 1 000 000×).

Podrobnější informace naleznete na stránce Mez rozlišení mikroskopu.

Přehled typů mikroskopů

Vlnová délka elektronu je při urychlovacím napětí 10 kV pouze 0,0123 nm.

(kinetická energie: mv2/2 = eU)


p = m·v = hybnost, h = Planckova konstanta, m = hmotnost elektronu, e = náboj elektronu, U = urychlovací
napětí

Funkci čoček v elektronovém mikroskopu zastávají vhodně tvarovaná elektromagnetická pole. Pozorovaný předmět je
umístěn ve vakuu a „prosvětlujeme“ ho svazkem elektronů, který se průchodem rozptýlí a dopadne na stínítko.

Typy mikroskopů
TEM transmisní elektronový mikroskop – nepohyblivý svazek elektronů, detekce elektronů prošlých vzorkem
(TE) na fluorescenčním stínítku nebo detektorem.
REM rastrovací elektronový mikroskop – pohyblivý svazek, zobrazení povrchu vzorku pomocí odražených
sekundárních elektronů.
SPM mikroskopie skenující (rastrující) sondou (Scanning Probe Microscopy) je soubor metod určených ke
zjišťování struktury povrchu s rozlišením na úrovni velikosti atomu.
AFM mikroskopie atomárních sil je založena na mapování rozložení atomárních sil na povrchu vzorku. Tyto
síly jsou mapovány přiblížením hrotu k povrchu, čímž vzniká přitažlivá nebo odpudivá síla, která způsobí
ohnutí nosníku, na němž je upevněn hrot. Toto ohnutí je snímáno laserovým snímačem. Výhodou metody
AFM je možnost studovat jak nevodivé, tak i vodivé vzorky.
STM skenující tunelovací mikroskopie je jedna z metod SPM. Její princip je založen na kvantové fyzice. Mezi
hrotem elektrody a zkoumaným vzorkem teče proud díky tunelovém jevu i když se hrot vzorku přímo
nedotýká. Při pohybu nad vzorkem se mění vzdálenost hrotu tak, aby tunelový proud zůstával stejný. Jako
jedna z mála metod je schopna poskytnout až atomární rozlišení, přičemž je zároveň vcelku jednoduchá.
Oproti ostatním metodám (transmisní elektronová mikroskopie, autoemisní iontová mikroskopie) nevyžaduje
náročnou přípravu vzorku. Na druhou stranu poskytuje informace jen o povrchu.
SNOM rastrovací optický mikroskop blízkého pole (scanning near-field optical microscope).[1]

Odkazy
Související články

Optický mikroskop
Konfokální mikroskop
Mikroskopie skenovací sondou
Kontrastování v elektronové mikroskopii
Mez rozlišení mikroskopu

Externí odkazy

Pavel Janda: Mikroskopické a analytické techniky – mikroskopie rastrovací sondou


Mikroskop atomárních sil použit jako atomární tužka umožňující psaní jednotlivými atomy
Nové možnosti zobrazování jednotlivých atomů pomocí rastrovacích mikroskopů

Reference

1. Jan Valenta, Spektroskopie jednotlivých molekul v blízkém optickém poli, Vesmír 74, 236, 1995/4 dostupne online

Použitá literatura

JELÍNEK, Pavel, Prokop HAPALA a Vladimír CHÁB. Rastrovací a tunelová mikroskopie : Jitro a poledne
kouzelníků. Vesmír [online]. 2010, roč. 89, no. 5, s. 290-294, dostupné také z
<http://casopis.vesmir.cz/clanek/rastrovaci-tunelova-mikroskopie>. ISSN 1214-4029. 
Oko (biofyzika)

Schéma lidského oka

Oko je smyslový orgán zraku, skládá se z světločivné vrstvy obsahující světlocitlivé buňky a optického systému, který
usměrňuje paprsky tak, aby dopadaly na sítnici.

Oko má přibližně tvar koule o poloměru 12 mm.

Světlolomný systém oka


Podrobnější informace naleznete na stránce Světlolomný systém oka.

Světlo před svým dopadem na fotoreceptory prochází postupně následujícími částmi oka:

1. Rohovka – index lomu 1,377;


2. přední komora oční – vyplněná očním mokem o indexu lomu 1,336;
3. duhovka, resp. otvor v duhovce zvaný zornička;
4. zadní komora oční – mezi zadní plochou duhovky a ciliárním aparátem, n = 1,336;
5. čočka – n = 1,42, která může prostřednictvím ciliárního aparátu měnit zakřivení a tím i optickou mohutnost;
6. sklivec;
7. sítnice - člověk má tzv. inverzní typ sítnice, tedy fotoreceptory jsou až na straně odvrácené od čočky.

Optická mohutnost zdravého oka jako optické soustavy je pro předmět v nekonečnu, při akomodaci se zvětšuje.

Světlocitlivé buňky
Podrobnější informace naleznete na stránce Světlocitlivé buňky a jejich funkce.

Světlocitlivé buňky sítnice jsou buňky vytvářející nervovou stimulaci na základě absorpce fotonu přicházejícího na
sítnici. Tyto buňky jsou dvojího typu: tyčinky a čípky.
Čípky jsou citlivé na světlo různé barvy, čili různé vlnové délky, různé intenzity a různé sytosti barev. Jsou prvními
neurony sítnice. Zajišťují fotopické vidění, jsou zodpovědné za zrakovou ostrost. Nacházejí se v nejhojnějším počtu v
centrální jamce (fovea centralis), což je malá jamka ve žluté skvrně. Směrem k periferii sítnice jejich hustota postupně
klesá. Celkově nacházíme na sítnici 6 milionů čípků. Rozlišujeme 3 typy čípků, které je možné rozlišit pouze podle
pigmentu v cytoplasmě, nikoliv podle tvaru buňky.
Tyčinky jsou světlocitlivé buňky reagující na nižší intenzitu osvětlení než čípky, ale nejsou schopny rozeznávat barvy.
Zajišťují skotopické vidění.

Oblasti vidění
Fotopické (denní) vidění

Barevné vidění;
zajištěné hlavně čípky;
vnímaný jas >102 cd/m2;
rychlá adaptace na světlo (20–60 s);
maximální citlivost pro vlnovou délku 555 nm;
vnímané vlnové délky 400 až 750 nm.

Skotopické (noční) vidění

Vnímání pouze jasu (změny intenzity);


zajištěno pouze tyčinkami;
vnímaný jas 10-3 cd/m2;
adaptace dvoufázová, plná adaptace po 40 až 60 minutách;
maximum pro vlnové délky se pohybuje okolo 500 nm.

Mezopické (soumračné) vidění

Jas mezi oběma předchozími hodnotami;


vidění zajišťují jak tyčinky, tak čípky;
spektrální citlivost oka se liší od spektrální citlivosti při fotopickém vidění.

Spektrální citlivost lidského oka

Spektrální citlivost čípků

Lidské oko je schopno vnímat pouze malou část elektromagnetického záření. Při běžné intenzitě osvětlení je sítnice
citlivá v oblasti záření o vlnové délce od 380 nm do 760 nm (oblast viditelného světla elektromagnetické spektra).
Tato oblast se také kryje s jedním z pásem propustnosti zemské atmosféry. Dalším z důvodů, proč lidské oko nejvíc
vnímá právě v této oblasti je fakt, že odpovídá maximu spektrálního vyzařování Slunce. Z grafu spektrální
citlivosti lidského oka vidíme, že lidské oko je citlivé i na červené světlo vlnové délky např. 760 nm. Aby však byl
dosažen zrakový vjem stejné intenzity jako pro záření světla o vlnové délce 550 nm, musí být světelný tok ze stejné
plochy 10 000x větší.

Světlocitlivé buňky lidského oka

Stavba čípku

V lidské sítnici je asi 6 milionů čípků. Existují tři funkční typy, obsahující různé druhy fotopsinu. Každý z nich totiž
má poněkud jiný jodopsin (specializace na zelenou barvu modrou a červenou barvu). Čípky potřebují ke své práci
poměrně hodně velké osvětlení, ale na druhou stranu zajišťují preciznější vidění než tyčinky. [1]

Veličiny důležité k určení spektrální citlivosti oka


Jestliže z určitého zdroje vychází elektromagnetické záření, pak E(t)/S se nazývá zářivý tok Φe, jednotka − Watt (W).
Výkon zářivé energie, zhodnocený podle velikosti světelného vjemu, který vyvolá se nazývá světelný tok Φ, jednotka
− lumen (lm). Φ/Φe nazýváme světelnou účinností záření. Normalizovaná funkce světelné účinnosti pro různé
vlnové délky se nazývá relativní světelnou účinností záření její 3 exempláře vidíme v grafu výše. Maximum
světelné účinnosti je 680 lm/W. Jinak řečeno: monochromatické záření o vlnové délce 550 nm při výkonu 1 W je rovno
světelnému toku 680 lm.

Adaptace oka na intenzitu světla


Adaptací rozumíme schopnost zraku přizpůsobit se různým hladinám osvětlení.

Dopadá-li světelný tok na těleso, je jeho povrch osvětlen. Tuto vlastnost tělesa charakterizuje veličina osvětlenost.
Hlavní jednotou osvětlenosti je lux (lx). Plocha o obsahu 1 m² má osvětlenost 1 lx, dopadá-li na ni rovnoměrně
světelný tok 1 lumen. Zdravé lidské oko je schopno registrovat předmět, jehož osvětlenost je aspoň 2 nlx. Na tuto
osvětlenost reagují pouze tyčinky, čípky až na větší. Osvětlenost předmětu za jasného slunného dne je asi 0,1 Mlx.
Doporučená hodnota pro čtení je 100 lx, pro jemné mechanické operace a rýsování 200 lx, pro osvětlení chodeb 20 lx.

Citlivost sítnice

Citlivost sítnice je velmi vysoká, asi 10⁴ krát větší než citlivost fotografické emulze, není však všude stejná. Největší je
v okolí průsečíku optické osy oka, kde leží tzv. žlutá skvrna. Je známo, že při přechodu ze světla do tmy lze
rozeznávat jednotlivé předměty s dostatečnou citlivostí až po určité době (asi 10-20 min, max za 45 min). Za tuto dobu
se oko adaptuje na tmu. Dříve jsou rozeznávány předměty, jejichž obraz vzniká v periferních oblastech sítnice. Při
přechodu ze tmy do světla potřebuje oko rovněž určitou dobu na adaptaci, tato doba je však podstatně kratší. Po
prudkém osvětlení jsou oči oslněny, ale díky rychlé reakci zornic se rychle přizpůsobují (mióza, mydriáza).

Funkce tyčinek a čípků

Čípky a tyčinky mají relativně nezávislé vlastnosti. Při velkých intenzitách zajišťují vidění čípky, při nízkých
intenzitách se tyčinky stanou citlivějšími než čípky. Všechny barvy jsou registrovány jak čípky, tak tyčinkami, na
červenou barvu jsou však citlivé pouze čípky.

Rozdíly v rychlosti adaptace oka na tmu po předchozím osvětlení vysvětluje funkce tyčinek. Tyčinky obsahují
rodopsin-tzv. zrakový purpur, složený z proteinu opsinu a z retinalu, což je aldehyd vitamínu A. Působením světla se
rodopsin rozkládá na tyto složky a mění svoji barvu na žlutou. Reakce je reverzibilní a velmi rychlá. Při příliš silném
osvětlení se však retinal mění na retinol a jeho barva na bílou, tato reakce je reverzibilní pomalým procesem.
Regenerace rodopsinu tedy může probíhat pomalou nebo rychlou cestou.

Hemeralopie (šeroslepost) je snížená adaptační schopnost. Může být dědičná nebo vzniknout např. při avitaminóze
A. Nevytváří se dostatečné množství rodopsinu, což se projeví poruchou vidění za šera, šeroslepostí.

Vady oka
Podrobnější informace naleznete na stránce Refrakční vady oka.

Refrakční vady oka jsou způsobeny špatnými vlastnostmi jeho lomivých ploch. Rovnoběžné paprsky vstupující do oka
nejsou koncentrovány na sítnici. Neleží na ní tedy obrazové ohnisko optické soustavy oka. Tento jev nemusí být
způsoben pouze nesprávným lomem světelných paprsků. Častěji jsou problémy způsobené osovými vadami oka –
lomivá prostředí jsou v pořádku, ale oko má různou délku. Osové i refrakční vady se projevují stejně a mohou se i
kombinovat. Obecně platí, že větší refrakční vady způsobují menší vedlejší obtíže. Obraz je zamlžený, nebo rozmazaný
a oko nedokáže tuto vadu korigovat. Jinak je tomu u menších refrakčních vad. Ty je oko schopno do určité míry
vykompenzovat, toto úsilí vede k svalovému a nervovému vyčerpání. To má za následek únavu a slzení očí, bolesti
hlavy a další projevy. U oka bez refrakční vady se paprsky procházející optickou soustavou protínají v ohnisku na
ploše sítnice, takové oko se nazývá emetropické. Velice často se setkáváme s okem ametropickým, kdy se paprsky na
sítnici neprotínají. Rozlišujeme vady sférické a asférické. Sférické vady se korigují sférickými čočkami. Patří mezi ně
myopie – Krátkozrakost a hypermetropie – Dalekozrakost. Myopie – oko fyziologicky příliš dlouhé. Ostrý obraz se
promítá před sítnicí. Pacient vidí špatně na dálku, ale dobře na blízko. Hypermetropie – oko fyziologicky příliš
krátké. Ostrý obraz by se promítal za sítnicí, na sítnici je obraz neostrý. Asférické vady korigujeme asférickými, tzv.
torickými, čočkami. Nejčastější asférická porucha je astigmatismus, kdy pacient vidí neostře na blízko i do dálky.
Vyskytuje se při něm asymetrie optické mohutnosti rohovky, případně vzácně čočky.

Rohovka má ve dvou na sebe kolmých osách rozdílné zakřivení.

Mezi oční defekty se řadí i presbyopie (vetchozrakost). Zde se ovšem jedná o přirozený proces ztráty pružnosti
optického aparátu oka a tedy snížení akomodačních schopností ve vyšším věku.
Korekce vad oka
Podrobnější informace naleznete na stránce Způsoby korekce refrakčních vad.

Vady oka lze korigovat čočkami (brýle, kontaktní čočky) nebo laserovou operací, cílem je navození emetropie. Na
korekci myopie se používají rozptylky. Ideální optická mohutnost rozptylky je taková, aby pacient viděl ostře do
nekonečna (v praxi 5 - 6 metrů). Na korekci hypermetropie se používají spojky, volí se taková optická mohutnost, aby
pacient přečetl text ve vzdálenosti 25 cm. Astigmatismus se koriguje cylindrickými nebo torickými skly, v případě
nepravidelného astigmatismu je korekce obtížná. U Presbyopie může pacient používat jednoohniskové brýle na více
vzdáleností nebo bifokální (a multifokální) skla.

Odkazy
Související články
Vady oka
Princip vidění
Optický systém oka
Oko (histologie)
Biochemie procesu vidění
Poleptání rohovky a spojivky

Zdroj

KUBATOVA, Senta. Biofot [online]. [cit. 2011-01-31]. <https://uloz.to/!CM6zAi6z/biofot-doc>.

SVOBODA, Emanuel a Karel BARTUŠKA, et al. Přehled středoškolské fyziky. 4. vydání. Praha : Prometheus, 


2006. 531 s. ISBN 80-7196-307-0.

NAVRÁTIL, Leoš a Jozef ROSINA, et al. Medicínská biofyzika. 1. vydání. Praha : Grada, 2005. 524 s. ISBN 80-
247-1152-4.

PRAMACOM-HT, spol. s r. o. infrared.cz : Spektrální citlivost lidského oka [online]. [cit. 2012-12-28]. 


<http://www.infrared.cz/domains/infrared.cz/cz/>.

PROCHÁZKOVÁ, Simona. Korekce refrakčních vad [online]. Brno, 2006, dostupné také z


<https://is.muni.cz/th/bn7cw/Bakalarka.doc?so=nx>. 

Reference
1. JUNQUEIRA, Luiz Carlos Uchôa a José CARNEIRO. Basic histology : text & atlas. 11. vydání. New York : 
McGraw-Hill, c2005. ISBN 0071440917.

Externí odkazy

Kymplová Jaroslava: Oko a oční vady. Multimediální podpora výuky klinických a zdravotnických oborů :: Portál 1.
lékařské fakulty Karlovy Univerzity v Praze [online] 19.2.2008, poslední aktualizace 19.2.2008 [cit. 2011-12-22]
Dostupný z WWW: <https://portal.lf1.cuni.cz/clanek-810-oko-a-ocni-vady>. ISSN 1803-6619
Oko (biofyzika)/Vady oka
Fyziologický stav – optický systém oka je schopný vytvořit na sítnici ostrý obraz předmětu.

K tomu je třeba splnit dvě podmínky:

Obrazem bodu je bod

Porucha této podmínky = astigmatismus. Astigmatismus je způsoben asymetrií optické mohutnosti oka
(příčinou je nejčastěji rohovka, která je nepravidelně zakřivená), tudíž se paprsky neprotnou v jednom ohnisku,
ale ve dvou úsečkách.

Obraz vznikne přesně na sítnici (u zdravého, emetropického oka)

Vzdálený bod – vzdálenost, ve které oko vidí ostře bez akomodace (normálně je to nekonečno).
Pokud je vzdálený bod ve vzdálenosti konečné, podle umístění vzdáleného bodu rozeznáváme dva typy sférických
ametropií:

Krátkozrakost (myopie)

Myopie.

Vzdálený bod je v konečné vzdálenosti před okem, tzn. takové oko vidí ostře jen předměty ve vzdálenosti
menší, než je vzdálený bod, protože u větších vzdáleností vzniká obraz předmětu před sítnicí (optická
mohutnost je příliš velká). Silná myopie je nejrozšířenější riziko pro odchlípení sítnice.

Dalekozrakost (hypermetropie)

Hypermetropie.

Vzdálený bod je v konečné vzdálenosti za okem, obraz předmětů umístěných v nekonečnu vzniká za sítnicí
(optická mohutnost je příliš malá).

Oko mění svou optickou mohutnost pomocí zakřivení čočky. Mírou zakřivení je tzv. blízký bod = vzdálenost, kterou
oko vidí ostře při maximální akomodaci čočky. Schopnost akomodace se snižuje s věkem. Prodlouží-li se vzdálenost
blízkého bodu nad konvenční zrakovou vzdálenost (d = 25cm), projeví se tento stav únavou oka při čtení –
presbyopie.

Korekce vad se provádí pomocí brýlí nebo kontaktních čoček.

Korekce:

myopie – rozptylky
hypermetropie – spojky
presbyopie – spojky
astigmatismus lze korigovat pouze pravidelný – pomocí cylindrických skel – snažíme se, aby obě ohniska
splynula.
Odkazy
Související články

Astigmatismus
Oko (biofyzika)
Oko (biofyzika)/Princip vidění
Okohybné svaly
Oko (histologie)

Externí odkazy

Oční vady - rady, informace


Kymplová Jaroslava: Oko a oční vady. Multimediální podpora výuky klinických a zdravotnických oborů :: Portál 1.
lékařské fakulty Karlovy Univerzity v Praze [online] 19.2.2008, poslední aktualizace 19.2.2008 [cit. 2011-12-22]
Dostupný z WWW: <https://portal.lf1.cuni.cz/clanek-810-oko-a-ocni-vady>. ISSN 1803-6619
Oko (biofyzika)/Princip vidění

Sítnice pravého oka

Oko je spojná optická soustava s měnitelnou ohniskovou vzdáleností. Vzniklý obraz je zmenšený, převrácený a
skutečný.

při vstupu paprsku do oka projde světlo rohovkou a očním mokem


měnitelný otvor v duhovce řídí intenzitu dopadajícího světla
po průchodu čočkou (největší index lomu ze všech částí oka) se světlo šíří sklivcem (nejmenší index lomu)
obraz vytvořený čočkou vzniká na sítnici, ve které jsou dva druhy buněk:

1. tyčinky (vnímají intenzitu světla)


2. čípky (vnímají barvy)

Zde jsou zakončení zrakového nervu, které počitky vedou do mozku.

pro orientaci očního lékaře na sítnici slouží dva hlavní body:

1. žlutá skvrna (macula lutea) – místo největší citlivosti oka


2. slepá skvrna (discus nervi optici; papilla nervi optici) – místo vstupu zrakového nervu do oka, tzn.
místo, kde nejsou tyčinky ani čípky

vznik ostrého obrazu umožňuje akomodační schopnost čočky, která je držena ciliárním svalem, jež mění podle
potřeby zakřivení optických ploch (při velké vzdálenosti menší zakřivení a naopak):

při pohledu do dalekého bodu (nekonečno) je akomodace oka minimální


při pohledu do blízkého bodu (až 15 cm) je akomodace oka maximální

Konvenční zraková vzdálenost = 25 cm.

Odkazy
Související články

Biochemie procesu vidění


Trichromatické vidění
Oko (biofyzika)
Oko (biofyzika)/Vady oka
Okohybné svaly
Oko (histologie)
Odchlípení sítnice

Zdroj

Holešov. Gymnázium Ladislava Jaroše [online]. [cit. 23.05.2009]. 


<http://www.gymhol.cz/projekt/fyzika/07_soustavy/07_soustavy.htm>.

Externí odkazy

Kymplová Jaroslava: Biofyzika vidění. Multimediální podpora výuky klinických a zdravotnických oborů :: Portál 1.
lékařské fakulty Karlovy Univerzity v Praze [online] 19.2.2008, poslední aktualizace 19.2.2008 [cit. 2011-12-22]
Dostupný z WWW: <https://portal.lf1.cuni.cz/clanek-809-biofyzika-videni>. ISSN 1803-6619
Laser (biofyzika)

Symbol laseru

Laser je optický přístroj emitující vysoce koherentní světlo, tedy světlo, které má společnou vlnovou délku, fázi i
směr šíření. Název je zkratkou anglického označení Light Amplification by Stimulated Emission of Radiation.

Historie laseru

Laser

Předchůdcem laseru byl maser, zařízení které pracuje na stejném principu, avšak generuje mikrovlnné záření. První
maser sestavil Charles Townes, J. P. Gordon a H. J. Zeiger v roce 1953. Tento prototyp nebyl schopný fungovat
nepřetržitě.

V roce 1960 Theodore H. Maimanv USA poprvé předvedl funkční laser. Jako aktivní prostředí použil krystal rubínu s
využitím tří energetických hladin; laser mohl pracovat pouze v pulsním režimu.

V roce 1964 obdrželi Charles Townes, Nikolaj Basov and Alexandr Prochorov společně Nobelovu cenu za fyziku za „za
zásadní výzkum v oboru kvantové elektroniky, který vedl ke konstrukci oscilátorů a zesilovačů založených na principu
maserů a laserů“.

Laserové paprsky

Princip laseru

Obsahuje-li látka částice v excitovaných stavech, pak ozáření látky fotony s energií rovnou rozdílu energetických
hladin mezi excitovaným a základním stavem, vyvolá přechod excitovaných částic do nižšího energetického stavu,
spojený s emisí záření se stejnou vlnovou délkou, fází i směrem šíření, jako mělo záření, které emisi vyvolalo. Částice,
nejčastěji foton, do excitovaného elektronu narazí velmi prudce a proto primární foton o energii h. ν pokračuje dále a
k němu se přidává foton generovaný přechodem elektronu, také o energii h. ν. Od tohoto okamžiku pokračují dva
fotony. Ty srazí do základního stavu další 2 excitované elektrony, budou pokračovat 4 fotony, ty srazí další 4
elektrony, bude pokračovat 8 fotonů … Proces takto popsaný se nazývá stimulovanou emisí.

Elektronové obaly atomů a molekul existují pouze v určitých kvantových stavech (energetických hladinách Ei). Při
přechodu mezi dvěma energetickými stavy vyzáří atom (molekula) záření s frekvencí:

,
kde fnm = frekvence emitovaného kvanta záření, En = vyšší energetická hladina, Em = nižší energetická hladina,
h = Planckova konstanta.

V základním stavu mají atomy nejnižší energii, kdy mohou pouze absorbovat elektromagnetické záření. Vlivem
vnějšího záření se atomy mohou dostávat do excitovaných stavů, ze kterých mohou do stavu základního přecházet
spontánně (na sobě nezávislé přechody, nekoherentní záření) nebo vynuceně (vlivem vnějšího elektromagnetického
záření, emitované záření má frekvenci stejnou jako vnější záření).

Typy laserů
Lasery dělíme podle toho, jaké médium se v nich nachazí.

lasery s plynným médiem - (např. Argonový laser, Excimer laser)

lasery s kapalným médiem - (lasery s kapalným krevním barvivem)

lasery s pevným médiem - (rubínový, skelný...)

Účinky

tepelné, nárazové, tvorba volných radikálů (závislé na vlnové délce záření, délce pulzu, intenzitě záření a
vlastnostech biologického objektu)

Použití
sdělovací technika, páteřní počítačové sítě (zdroj světla pro optická vlákna)
medicína (oftalmologie, dermatologie, chirurgie)
urychlování částic (hlavně elektronů – bodový zdroj rentgenového záření)
laserová ablace (ablace = odebírání)

Odkazy
Související články

Laser (hygiena)
Laser

Zdroj

KUBATOVA, Senta. Biofot [online]. [cit. 2011-01-31]. <https://uloz.to/!CM6zAi6z/biofot-doc>.

MACH, Petr. Výkonové součástky a technologie - Předmět ČVUT - A1B13VST, AD1B13VST

Použitá literatura

ČERNOHORSKÝ, P. Jandera: Atomová spektroskopie, Univerzita Pardubice, Pardubice (1997)

http://fyzika.jreichl.com/
Rentgenové záření

Rentgenové záření objevil roku 1895 v podstatě náhodou Wilhelm Conrad Röntgen.

Rentgenové paprsky jsou elektromagnetické ionizující záření s vlnovou délkou 10 nm–1 pm (10−8–10−12 m).
Vzhledem ke kvantové dualitě je můžeme nahlížet též jako fotony s energií 5–200 keV [1], dostačující k vyražení
elektronu z atomového obalu (ionizaci).

Zdroje RTG záření


Přirozenými zdroji RTG je záření hvězd (např. slunce), ale i dalších kosmických zdrojů.
Umělým zdrojem RTG záření je například rentgenová lampa – rentgenka.

Typy RTG záření


Původ RTG záření je v elektronovém obalu. Podle vlnové délky můžeme rozlišit 2 typy RTG záření, měkké (s
větší vlnovou délkou λ= 10−8−10−10m) a tvrdé (λ= 10−10−10−12m). Používané zdroje RTG produkují dva typy záření
(podle odlišného rozložení energie ve spektru):

Spektrum rentgenového záření:


modře spojité, červeně
charakteristické

Brzdné rentgenové záření

Elektron rychle letící (při napětí 100 kV je jejich rychlost cca 165.000m/s) od katody k anodě se při nárazu do anody
dostává do silného elektrostatického pole, kde dochází k zakřivení jeho dráhy a ke zbrždění. Kinetická energie, kterou
elektron ztratil, je vyzářena ve formě fotonu RTG záření. Při tomto procesu jsou vyzařovány fotony o různých vlnových
délkách. Čím více se elektron přiblíží k jádru a čím větší je jeho energie, tím větší je energie vznikajícího kvanta RTG
záření. Energie brzdného rentgenového záření závisí na protonovém čísle anody a na rychlosti elektronů (tedy na
velikosti napětí mezi elektrodami rentgenky). Toto záření se vyznačuje širokým spojitým energetickým spektrem,
protože rychlost elektronů emitovaných katodou není jednotná. Brzdné RTG záření vytváří spojité spektrum.
Elektrony ale mohou být urychleny i jiným způsobem než pouhým vystavením velmi vysokému napětí – v
urychlovačích částic jako je např. lineární urychlovač, betatron nebo mikrotron se dosahuje výrazně vyšších energií
než u rentgenky a vznikající RTG záření je podstatně tvrdší.

Brzdné záření se využívá v lékařské diagnostice a v radioterapii, příp. v průmyslu v defektoskopii.

Charakteristické rentgenové záření

Charakteristické RTG záření se liší podle materiálu, ze kterého je zhotovena anoda. Elektrony dopadající na
anodu (většinou wolframovou) předávají svoji energii elektronům v atomech anody, tyto elektrony jsou excitovány
(vyraženy do vyšší energetické hladiny), nebo úplně ionizovány (vytrženy z obalu). Pokud byl elektron pouze
excitován, následně se vrací zpět do původního základního stavu, pokud byl "vyražen", potom se jeho místo zaplní
elektronem z jedné z energeticky bohatších hladin vzdálenějších od jádra. Při obou variantách sestupu elektronu se
uvolní značné množství energie ve formě RTG záření. Energie fotonu záření je rovna energetickému rozdílu mezi
elektronovými hladinami, mezi kterými došlo k přesunutí elektronu. Rozdíl energie mezi jednotlivými hladinami je
stále stejný, proto vzniká RTG záření pouze o určitých vlnových délkách – odtud plyne název charakteristické záření,
protože onen energetický rozdíl, který určuje vlnovou délku záření, závisí na materiálu, ze kterého je anoda vyrobena.
Získáme tedy RTG záření charakteristické pro určitý konkrétní prvek (materiál anody); jeho energie je tím vyšší, čím
vyšší je protonové číslo prvku tvořícího anodu. Vzniklé záření tvoří tzv. diskrétní – čárové spektrum.
Charakteristické rentgenové záření se využívá v analytické chemii.

Výsledné záření reálného zdroje RTG je součtem brzdného a charakteristického záření.

Vlastnosti rentgenového záření


(uplatňující se v diagnostice)

Schopnost pronikat látkami − tato schopnost závisí na vlastnostech absorbující hmoty a na energii záření
(energie je tím větší, čím je vlnová délka kratší);
Diferencovaná absorpce − schopnost různých látek pohlcovat rentgenové paprsky závisí například na
protonovém čísle prvků absorbující tkáně, tloušťce vrstvy (kosti − Ca, P → velká absorpce);
Fotochemické účinky − způsobuje zčernání fotografické desky nebo filmu (záleží na intenzitě záření);
Luminiscenční účinky − vznik viditelného záření při dopadu na některé materiály;
Přímočaré šíření ze zdroje − šíří se do prostoru na všechny strany a intenzita ubývá se čtvercem vzdálenosti;
Rozptyl záření − negativní vlastnost pro diagnostiku, snižuje kontrast, při interakci fotonu s elektronem dojde k
vychýlení paprsku a snížení energie (zvětší se vlnová délka);
Ionizační účinky − negativní vlastnost, může mít škodlivé biologické účinky.

Rentgenka

Rentgenka

Jedná se o vakuovou trubici obsahující 2 elektrody: katodu a anodu. Nejčastěji bývají obě zhotoveny z
wolframu, pro mammografické vyšetření se používá molybdenová anoda (měkčí RTG záření). Pro snížení radiační
dávky a vyšší ostrost při zobrazování se stíní (nejčastěji olovem). Na elektrody je přiváděno velmi vysoké napětí, což
vede k urychlení elektronů.

Katoda

Katoda má tvar spirály. Při rozžhavení katody dojde k emisi elektronů a vzniká elektronový mrak. Hustota mraku je
dána žhavícím proudem katody. Po připojení vysokého stejnosměrného napětí začnou záporně nabité elektrony
vylétávat z oblaku směrem k anodě (+ současné urychlování silným elektrickým polem). Při zbrzdění na anodě vzniká
RTG záření pouze z 1 % pohybové energie elektronů.

Anoda

Anoda může být pevná nebo rotační. Při dopadu elektronů na anodu dochází totiž k jejímu zahřívání a proto je nutné
chlazení. Pro rentgenky s nízkým výkonem stačí chlazení vzduchem. Rentgenka s vysokým výkonem má uvnitř dutinku
pro chladící kapalinu.

Princip

U rentgenky lze nezávisle na sobě regulovat napětí mezi katodou a anodou (anodové napětí) a intenzitu žhavícího
proudu katody (katodový proud).

Na katodovém proudu závisí intenzita záření. Čím vyšší bude katodový proud, tím vyšší bude intenzita záření.
Na anodovém napětí závisí tvrdost, penetrace, absorpce a vlnová délka záření. S rostoucím anodovým napětím
poroste tvrdost a penetrace záření a naopak bude klesat absorpce a vlnová délka. Čím je potenciál mezi katodou a
anodou větší, tím větší je urychlení elektronů a tím kratší vlnovou délku má vznikající RTG záření.

Parametry záření
energie fotonů − je přimo uměrná napětí mezi katodou a anodou;
intenzita záření − souvisí se žhavícím proudem katody;
používaná napětí − 20–80 kV, pro klasické zobrazování asi 70 kV, pro mamografii 20–30 kV;
energie záření − při vyšší energii získáme nižší kvalitu zobrazení, při nižší energii získáme vyšší kvalitu
zobrazení (kontrast), ale zároveň docílíme větší radiační zátěže pacienta;

Detekce záření
film − při skiagrafii, jedná se o fotochemický proces;
fluorescenční stínítko − při skiaskopii;
digitálně − při CT, stimulace fosforové vrstvy na záznamovém médiu, následná luminiscence detekována
digitálně;
CCD − přímá digitalizace, při viziografii;
pevné velkoplošné detektory pro digitalizaci, při přímé radiografii (DR).

Comptonův jev
Jedná se o pružný rozptyl fotonů RTG záření na volných elektronech, přičemž vlnová délka rozptýleného záření je větší
než vlnová délka záření dopadajícího. Frekvence rozptýleného záření je menší než frekvence záření dopadajícího.

Tento jev poprvé pozoroval A. Compton v roce 1923 při dopadu RTG záření na uhlíkovou destičku, na níž pak dochází k
rozptylu a také ke změně frekvence záření.

Comptonův posun je rozdíl mezi vlnovou délkou záření dopadajícího a vlnovou délkou záření rozptýleného.
Comptonův jev dokazuje částicovou povahu RTG záření. Rentgenové záření má však i vlnové vlastnosti, důkazem
je jeho schopnost polarizace a difrakce.

Využití v medicíně

Rentgenové paprsky mohou procházet lidskými tkáněmi a v důsledku svého průchodu vytvářet stíny
podobné obrazu struktur v těle (př.: kosti, některé orgány a v neposlední řadě různé patologické stavy).
Rentgenový snímek je nepohyblivý obraz, zjednodušeně řečeno jde o rentgenovou fotografii. Jedním z nejznámějších
přístrojů využívající rentgenové záření v medicíně je beze sporu CT (Výpočetní tomografie). Charakteristickým rysem
rentgenového záření je, že má mnohem vyšší energii než viditelné světlo, a ta je právě z části pohlcena během
průchodu lidským tělem. Absorbovaná energie rentgenového záření může mít i různé biologické účinky uvnitř tkáně a
jejímu množství se říká dávka záření. Velmi velké dávky záření se používají v radiační onkologii nebo při terapii k
zastavení množení nádorových buněk. Ovšem dávka záření v průběhu zobrazovacích metod je velmi malá. K vytvoření
potřebné kvality obrazu při diagnostických zobrazovacích metodách se totiž používá co nejmenší možná dávka záření.

Další využití:

Rentgenová diagnostika − kosti mající převahu vápníku pohlcují rentgenové záření více než svaly a tkáně
(převážně voda). Kosti se pak jeví na RTG snímku světlejší (negativ);
Rentgenová strukturní analýza;
Rentgenová defektoskopie;
V archeologii;
Rentgenová astronomie;
Skiagrafie;
Skiaskopie;
Angiografie;

Odkazy
Související články
Výpočetní tomografie
Skiagrafie
Ionizace

Použitá Literatura
SVOBODA, Emanuel, et al. Přehled středoškolské fyziky. 4. vydání. Praha : Prometheus, 2010. ISBN
9788071963073.

NAVRÁTIL, Leoš a Jozef ROSINA, et al. Medicínská biofyzika. 1. vydání. Praha : Grada, 2005. 524 s. ISBN 80-
247-1152-4.

TARÁBEK, Pavol a Petra ČERVINKOVÁ, et al. Odmaturuj! z fyziky. 2. vydání. Brno : Didaktis, 2004. ISBN


8073580586.

Reference
1. ULLMANN, Vojtěch. Jaderná fyzika, radiační fyzika, radioisotopy: Rentgenová diagnostika. [online]. [cit. 2012-
02-13]. Dostupné z: http://astronuklfyzika.cz/JadRadMetody.htm#2

Externí odkazy

ULLMANN, Vojtěch. Jaderná fyzika, radiační fyzika, radioisotopy: Rentgenová diagnostika. [online]. [cit. 2012-
02-13]. Dostupné z: http://astronuklfyzika.cz/JadRadMetody.htm#2
Přispěvatelé Wikipedie, Rentgenové záření [online], Wikipedie: Otevřená encyklopedie, c2015, Datum poslední
revize 17. 10. 2015, 04:20 UTC, [citováno 30. 11. 2015] <https://cs.wikipedia.org/w/index.php?
title=Rentgenov%C3%A9_z%C3%A1%C5%99en%C3%AD&oldid=12970975>
Šprindrich Jan: Radiologické zobrazovací metody. Multimediální podpora výuky klinických a zdravotnických
oborů :: Portál 3. lékařské fakulty UK [online] 6.2.2011, poslední aktualizace 6.2.2011 [cit. 2011-12-22] Dostupný
z WWW: <http://portal.lf3.cuni.cz/clanky.php?aid=71>. ISSN ISSN 1804-3143
Šprindrich Jan: Rtg dynamické metody a kontrastní látky. Multimediální podpora výuky klinických a
zdravotnických oborů :: Portál 3. lékařské fakulty UK [online] 6.3.2011, poslední aktualizace 16.3.2011 [cit.
2011-12-22] Dostupný z WWW: <http://portal.lf3.cuni.cz/clanky.php?aid=85>. ISSN ISSN 1804-3143

Zdroj
CHUDÁČEK, Z.: Radiodiagnostika, 1. vydání Vydavateľstvo Osveta, 1993, 440 s.
Rtg záření - mechanismus vzniku
Rentgenové záření je elektromagnetické vlnění, pro které jsou charakteristické krátké vlnové délky a vysoká energie.
Jako základní zdroj rentgenového záření slouží rentgenka (dříve se používaly iontové lampy). Základem mechanismu
vzniku rentgenového záření je interakce elektronů uvolněných termoemisí z katody s atomy anody. Při dopadu
elektronů na anodu se přes 99 % jejich kinetické energie přemění na teplo, zbylá energie se přemění v rentgenové
záření. Rentgenové záření vzniká v rentgence dvěma způsoby – jako brzdné a charakteristické záření.

Rentgenové záření je součástí


elektromagnetického spektra o kratších vlnových
délkách než je viditelné spektrum. Různé
aplikace vyžadují různé části rentgenového
spektra.

Stavba a funkce rentgenky


Jedná se o evakuovanou skleněnou trubici, tzv. Coolidgeovu lampu, v níž jsou zabudovány elektrody – katoda a
anoda. Žhavená wolframová katoda má tvar spirály a je umístěna v záporně nabité molybdenové misce. Hlavní
funkcí této misky je odpuzování a soustřeďování (fokusace) uvolněných elektronů na plošku na anodě. Anoda je
vyrobena obvykle rovněž z wolframu, který se vyznačuje vysokým atomovým číslem, vysokým bodem tání a dobrou
tepelnou vodivostí. Vysoký bod tání zabraňuje roztavení anody, tepelná vodivost zajišťuje odvod tepla vzniklého při
dopadu elektronů. Pří mamografii se můžeme setkat s molybdenovou anodou.

Vznik rentgenového záření

Katoda je žhavená ze žhavícího transformátoru. Elektrony se termoemisí z katody uvolňují a vytvářejí různě hustý
mrak. Vlivem vysokého napětí vloženého mezi elektrody (desítky až stovky kV) jsou elektrony přitahovány a
urychlovány k anodě. Prudce dopadají na tzv. dopadové ohnisko, které je skloněno a svírá s výstupním okénkem
rentgenky úhel přibližně 19°. Díky tomuto sklonu má ploška, na kterou elektrony dopadají, tvar úzkého obdélníku a
nazývá se termické ohnisko. Projekcí termického ohniska ve směru centrálního paprsku je ohnisko optické. Na
velikosti optického ohniska závisí ostrost rentgenového obrazu. Čím je optické ohnisko menší, tím větší je ostrost
obrazu. S ostrostí obrazu ovšem klesá zatížitelnost rentgenky. (Zatížitelnost udává příkon, který rentgenka snese po
dobu jedné sekundy.) Proto se někdy používají tzv. rotační anody tvořené otáčejícím se wolframovým diskem. Dochází
ke zvětšení termického ohniska. Zatížitelnost pak závisí na průměru anody a na rychlosti jejího otáčení.

Při dopadu elektronů na anodu vzniká velké množství tepla, anodu proto musíme chladit. Rozlišujeme chlazení
vzduchové a olejové, které se někdy kombinuje s vodním.

Rentgenka

Rentgenka je mechanicky chráněna kovovým krytem, který zajišťuje i celou řadu dalších funkcí (připojení kabelů
vysokého napětí, upevnění primární clony, ochrana pracovníků před vysokým napětím (díky uzemnění) a před
ionizujícím zářením). Mezi krytem a rentgenkou je ještě vrstva izolačního oleje. V krytu rentgenky je umístěna
beryliová destička – výstupní okénko. Destičkou nemohou procházet elektrony, ale pouze využitelné rentgenové
záření.

Pronikavost rentgenových paprsků závisí na vlnové délce, a tedy na napětí mezi elektrodami (čím větší napětí, tím
kratší vlnová délka), intenzita paprsků závisí na počtu uvolněných elektronů, a tedy na teplotě žhaveného vlákna
katody.

Brzdné rentgenové záření (Bremsstrahlung)


Když se rychle letící elektron dostane do elektrostatického pole jádra atomů anody, dochází k elektromagnetické
interakci, která způsobí zakřivení dráhy a prudké snížení rychlosti elektronu. Zabrzdění elektronu způsobí, že elektron
ztratí část své kinetické energie. Tato energie se přemění na foton rentgenového záření. Fotony rentgenového záření
mají různé vlnové délky a nesou různé energie, brzdné záření má proto spojité spektrum. S rostoucí energií elektronu
roste frekvence vzniklého rentgenového záření, vlnová délka se naopak zmenšuje. Kvalita brzdného záření závisí jen
na anodovém napětí.

Spektrum rentgenového záření

Charakteristické rentgenové záření


Charakteristické rentgenové záření vzniká jinak než záření brzdné. Mají-li elektrony dopadající na anodu dostatečnou
energii, dochází k předávání této energie elektronům vnitřní slupky elektronového obalu, což má za následek jejich
přemístění (excitaci) do vyšších energetické hladiny nebo častěji úplné vyražení z atomu (ionizace). Na uvolněné místo
pak přeskakují elektrony z vyšších energetických hladin atomu (nebo i volné elektrony), čímž se atom snaží získat zpět
svoji stabilitu. Jelikož se jedná o značné energetické rozdíly mezi jednotlivými energetickými hladinami, je rozdíl
energií vyzářen ve formě fotonu elektromagnetického záření, neboli charakteristického rentgenového záření.

Atom každého prvku (v tomto případě kovu – např. vanad, měď) má své charakteristické hodnoty energetických hladin
elektronů v obalu. Každý prvek má proto typické spektrum, které se pro svoji diskrétní energii označuje jako spektrum
čárové, podle kterého může být identifikován. Mluvíme tedy o záření, které je charakteristické pro materiál, z něhož je
anoda vyrobena. Podíl charakteristického rentgenového záření na celkovém spektru rentgenových paprsků závisí na
anodovém napětí. Zvyšuje-li se na rentgence postupně napětí, je při nízkých hodnotách napětí emitováno rentgenové
záření se spojitým spektrem. Pokud toto napětí začne stoupat, jsou rentgenkou produkovány oba dva druhy
rentgenového záření a jejich spektra se skládají.

Odkazy
Související články

Použitá literatura

NAVRÁTIL, Leoš a Jozef ROSINA, et al. Medicínská biofyzika. 1. vydání. Praha : Grada, 2005. 524 s. ISBN 80-
247-1152-4.

BENEŠ, Jiří, et al. Základy lékařské biofyziky. 1. vydání. Praha : Karolinum, 2005. 196 s. ISBN 80-246-1009-4.


Absorpce rentgenového záření
Rentgenové záření je elektromagnetické záření o vlnové délce 10 – 1 pm. Rentgenové paprsky neboli paprsky X, jak
zněl jejich původní název, objevil německý vědec Wilhelm Conrad Röntgen na konci devatenáctého století. Dnes jsou
jednou z nejužívanějších metod ke zjištění rozličných defektů v těle pacienta a jen těžko bychom se bez nich dokázali
obejít. Přitom rentgen funguje na relativně jednoduchém principu absorbce záření tkáněmi. Ty se nám poté v
závislosti na jejich složení zobrazí v různě sytých odstínech šedi.

Wilhelm Conrad Röntgen

Princip absorpce rentgenového záření


Při průchodu záření absorbující látkou dochází k vzájemné interakci kvant záření s elektrony nebo celými atomy látky.
Výsledkem je snížení intenzity záření ať už částečné (na principu tzv. Comptonova jevu) nebo celkové (při fotoefektu).
Pravděpodobnost absorbce kvanta rentgenového záření roste úměrně v závislosti na čtvrté mocnině atomového čísla
Z4. Z toho vyplývá, že tvrdé tkáně jako je například kost absorbují záření ve větší míře než tkáně měkké s velkým
obsahem vody. Útlum rentgenového záření je popisován celkovým lineárním absorpčním koeficientem μ.
Množství energie absorbované v jednotce hmotností absorbátoru (= látka pohlcující záření) nazýváme dávka záření
a vyjadřujeme ji v jednotkách Gray (Gy). Dalšími používanými jednotkami jsou roentgen (R) a sievert (SV), který
zohledňuje i biologické efekty záření.

Celkový lineární absorpční koeficient μ

Můžeme ho spočítat jako součet lineárního absorpčního koeficientu pro Comptonův rozptyl a fotoefekt. Formou
fotoefektu probíhá absorbce při nízké energii rentgenového záření, Comptonův rozptyl se uplatňuje při vyšších
energiích fotonů. Obecně je jasné, že čím hustší je absorbátor, tím větší je koeficient μ a tím více je záření tlumeno.
Dále záleží koeficient μ na již zmiňované čtvrté mocnině atomového čísla absorbujícího materiálu a konečně také na
vlnové délce záření. Můžeme ho tedy vyjádřit vztahem: μ = ρ · λ3 · Z4

Mimo jiné ze vzorce odvodíme i fakt, že s rostoucí energií záření, klesá absorpční koeficient. Energie záření je totiž
nepřímo úměrná vlnové délce podle Einsteinovi rovnice.

Zajímavosti
Absorpce rentgenového záření v kosti je zhruba 16krát větší než ve svalu.
Každý rok jsme na Zemi vystaveni "přírodnímu" rentgenovému záření o hodnotě 2,5 mSv.

Odkazy
Související články
elektromagnetické záření
Rentgenové záření
Charakteristika RTG záření
Comptonův jev
Fotoelektrický jev

Zdroje
BENEŠ, Jiří, Daniel JIRÁK a František VÍTEK. Základy lékařské fyziky. 4. vydání. Praha : nakladatelství
Karolinum, 2015. ISBN 978-80-246-2645-1.

NAVRÁTIL, Leoš a Josef ROSINA. Medicínská biofyzika. 1. vydání. Praha : Grada Publishing, 2005. ISBN 80-


247-1152-4.
Kontrastní látky
Kontrastní látka slouží ke zvýšení kontrastu mezi různými tkáněmi, odlišení anatomických struktur, zobrazení a
zvýraznění patologie a k funkčnímu zobrazení (dynamické studie). Kontrastní látky se v zásadě rozlišují:

podle fyzikálního principu zobrazovací metody:


rentgenové záření (skiagrafie, skiaskopie, CT, angiografie),
ultrazvuk (kontrastní ultrazvuk, CEUS),
magnetická rezonance;
podle způsobu podání:
systémově (intravenózní, intraarteriální podání),
endoluminálně a do preformovaných prostor (GIT, žlučové cesty, peritoneální dutina, píštěle);
podle absorpce záření:
pozitivní − zvyšují absorpci záření (jodové, baryové),
negativní − snižují absorpci záření (plyny, voda, metylcelulosa).

Skiagrafie
Při skiagrafii se kontrastní látky používají jen výjimečně – např. pro označení polohy mammil, které mohou mimikovat
patologický ložiskový stín.

Skiaskopie
U valné většiny skiaskopických vyšetření je podání kontrastní látky nezbytné. Výjimku tvoří např. skiaskopie bránice.

Pro vyšetření trávící trubice se užívá baryová či jodová kontrastní látka.

Baryová kontrastní látka se nesmí použít v případě podezření na perforaci či neprůchodnost trávící
trubice.

Baryová kontrastní látka dosahuje lepšího kontrastu, kromě odlitkové náplně je možno zobrazit i náplň reliéfovou.

Reliéfové náplně dosáhneme

podáním doušku vody u vyšetření jícnu,


přidáním šumáku při vyšetření žaludku,
methylcelulosou u enteroklýzy,
insuflací vzduchu u irrigografie,

… ovšem vše až po podání baryové kontrastní látky.

Irrigografie: tumorosní stenóza sigmatu

Angiografie
Angiografii nelze bez podání kontrastní látky vůbec provádět. Obvykle se používá jodová kontrastní látka, nicméně
k zobrazení lze užít i negativní kontrastní látky (zřídka).

Angiografie: stenóza renálních tepen

Ultrazvuk
Pro ultrazvukové vyšetření obecně není zapotřebí kontrastní látky. Výjimku tvoří jednak speciální dynamické
vyšetření (CEUS – Contrast Enhanced UltraSound) např. jater, kdy se i.v. podává kontrastní látka ve formě
mikrobublinek v polymerním obalu, jednak metoda THI v dopplerovské ultrasonografii, taktéž za podání kontrastní
látky ve formě mikrobublin.

CEUS: obrovský hemangiom jater


Výpočetní tomografie (CT)
Podání kontrastní látky se u výpočetní tomografie (CT) řídí indikací vyšetření a klinickou otázkou.

Některá vyšetření lze provádět nativně (bez kontrastní látky), např. HRCT plic, CT kostí, CT mozku k vyloučení
krvácení.

Ve většině vyšetření se ale kontrastní látka podává jak i.v. (během vyšetření), tak i per os (vyšetření břicha), někdy i
do preformované dutiny (fistulografie, peritoneografie).

U výpočetní tomografie je nutné správné časování nitrožilního podání kontrastní látky – vyšetření lze pak provést v
arteriální, portální, venosní, vylučovací, či odložené fázi. Nitrožilně se podává jodová kontrastní látka. Pro přesné
načasování vyšetření v arteriální fázi (CT angiografie) se užívá metoda bolus-trackingu: během aplikace kontrastní
látky přístroj skenuje nízkodávkovým skenem zvolený objem a vyšetření spustí až po dosažení prahové hodnoty.
Perorálně se podává ředěná jodová kontrastní látka, voda (u enterochromafinních tumorů) či roztok mannitolu (u CT
enterografie).

CT břicha: rutinní CT břicha po perorální přípravě a intravenosním podání jodové kontrastní látky

Magnetická rezonance (MRI)


Nitrožilně se u magnetické rezonance (MRI) podává gadoliniová kontrastní látka – opět záleží na vyšetřované oblasti
a klinické otázce.

Nežádoucí účinky kontrastních látek


Jodová kontrastní látka:

anafylaxe, bronchospasmus, hypotenze, vazovagální reakce, nevolnost, zvracení, erytém,


kontrastní nefropatie:
největší část jodové kontrastní látky se vylučuje ledvinami, pouze zlomek žlučí
důležitá je prevence – hydratace,
u renální insuficience pečlivě zvážit indikaci, dop. max množství podané kontrastní látky,
vyšší riziko u diabetiků.
Biguanidy: riziko laktátové acidózy, doporučeno dočasně vysadit.
Nekorigovaná hypertyreóza: kontrastní látka může vyvolat tyreotoxickou krizi.
Před scintigrafií štítné žlázy nepodávat jodovou kontrastní látku (nutnost jodové karence).

Metodický list intravaskulárního podání jódových kontrastních látek

Dle metodického listu podání jodové kontrastní látky Radiologické společnosti ČLS JEP je doporučena u rizikových
pacientů (polyvalentní alergie, astma bronchiale, alergie na jodovou kontrastní látku) premedikace:

Prednison tbl: 40 mg 12−18 hodin před aplikací jodové k.l. a 20 mg 6−9 hodin před aplikací jodové k.l.,
v akutním případě, kdy není možné pacienta řádně připravit: kortikoidy a antihistaminikum i.v.,
u závažných případů alergie je doporučeno premedikovat po dobu 24−48 hodin ve spolupráci s anesteziologem,
anesteziolog by měl být dostupný při vyšetření s aplikací jodové k.l. u rizikových pacientů.

U rizikových pacientů je doporučeno zvážit provedení alternativního vyšetření (např. ultrazvuk, MR).

Gadoliniové kontrastní látky

Nefrogenní systémová fibróza: opožděná vzácná, ale velmi těžká komplikace.

Odkazy
Související články

Příprava k vyšetření diagnostickými zobrazovacími metodami


Principy diagnostických zobrazovacích metod

Externí odkazy

Metodický list intravaskulárního podání jódových kontrastních látek na stránkách Radiologické společnosti ČLS
JEP
Maximální množství jodové kontrastní látky, které lze jednorázově podat − kalkulačka
Obrázky k tématu na atlas.mudr.org
Irrigografie: reliéfová náplň tlustého střeva
Fistulografie
RTG jícnu a žaludku
Kontrastní ultrazvuk (CEUS)
Angiografie
CT fistulografie
Comptonův rozptyl rentgenového záření, clony

Comptonův rozptyl rengenového záření


Podrobnější informace naleznete na stránce Comptonův rozptyl.

Principiálně se Comptonův rozptyl rentgenového záření nijak výrazně neliší od rozptylu záření gama. Nakonec prvně
byl tento objev pozorován právě u rentgenového záření a to již v první polovině 20. století. Obecně je principem
Comptonova rozptylu vyražení elektronu fotonem. Aby mohla reakce proběhnout spotřebuje se pouze určité
množství energie fotonu. To je zásadní rozdíl mezi Comptonovým rozptylem a fotoelektrickým jevem, kdy se na
vyražení elektronu z atomového obalu spotřebuje veškerá energie záření. Fotoefekt tudíž může u daného fotonu
proběhnout pouze jednou, zatímco Comptonův rozptyl jednoduše pokračuje do té doby, dokud má foton dostatečnou
energii na vyražení elektronu. Každou srážkou určitou energii ztrácí a zároveň je "rozptýlen" - pokračuje v pohybu po
jiné dráze nezávislé na dráze původní. Proto se také tento jev označuje jako Comptonův rozptyl.

Ilustrace Comptonova rozptylu

Clony rentgenového záření


Clony jsou zařízení pro eliminaci ionizačních účinků rentgenového záření. Dělí se na primární a sekundární:
a) Primární clona

Slouží k zúžení (regulaci) svazku rentgenového záření.


Snižuje dávku ionizačního záření u pacienta.
Je složena z několika párů olověných lamel a upevněna na výstupním okénku rentgenky.

b) Sekundární clona

Zachycuje sekundární záření vznikající interakcí rentgenového záření s tkáněmi v těle pacienta (projde jen tehdy,
má-li stejný směr s primárním zářením).
Je složena rovněž z olověných lamel umístěných však blíže k pacientovi.

Odkazy
Související články

Comptonův jev - co dokazuje, přínos


Comptonův jev - v čem spočívá
Comptonův rozptyl
Skiaskopie
Skiagrafie

Zdroje

BENEŠ, Jiří, Daniel JIRÁK a František VÍTEK. Základy lékařské fyziky. 4. vydání. Praha : nakladatelství
Karolinum, 2015. ISBN 978-80-246-2645-1.

NAVRÁTIL, Leoš a Josef ROSINA. Medicínská biofyzika. 1. vydání. Praha : Grada Publishing, 2005. ISBN 80-


247-1152-4.
Použití rentgenového záření v diagnostice

Rentgenové záření je elektromagnetické záření o vlnové délce 10 – 1 pm. Rentgenové paprsky neboli paprsky X, jak
zněl jejich původní název, objevil německý vědec Wilhelm Conrad Röntgen na konci devatenáctého století. Dnes jsou
jednou z nejužívanějších metod ke zjištění rozličných defektů v těle pacienta a jen těžko bychom se bez nich dokázali
obejít. Kromě všeobecně známého "rentgenování" za účelem zjištění zlomeniny či jiného poškození v těle pacienta, se
rentgen také používá k účelům terapeutistickým.

Použití rentgenového záření v diagnostice


Zobrazení rentgenem funguje na principu různého tlumení záření tkáněmi, přičemž rozhodující je jejich biochemické
složení. Nejvíce rentgenové záření absorbují kosti, proto je na snímku vidíme v bílých barvách. Naopak vodnaté
orgány jako například ledviny se nám zobrazí v tmavých odstínech šedi. Z tohoto důvodu se při vyšetření orgánů s
velkým obsahem vody musejí použít kontrastní látky. Podle způsobu vytváření rentgenového obrazu rozlišujeme 2
základní metody: Skiaskopii a skiagrafii. Navíc byla v druhé polovině dvacátého století vyvinuta tzv. výpočetní
tomografie, jež nám umožňuje prohlížet jednotlivé řezy tkání.

Rentgen ruky paní Röntgenové

Skiaskopie

1. PŘESMĚRUJ Skiaskopie

Skiagrafie

1. PŘESMĚRUJ Skiagrafie

citlivost přídatné komponenty velikost snímku prostorová lokalizace ložiska dávka ionizujícího záření
skiaskopie malá zesilovač štítového obrazu celý děj ano vysoká
skiagrafie velká (podrobnosti) zesilovací folie pouze výsek děje ne nízká

Výpočetní tomografie

1. PŘESMĚRUJ Výpočetní tomografie a Hounsfieldovy jednotky

Samotné rentgenové vyšetření

Abychom při vyšetření dosáhli co nejlepšího výsledného obrazu, je důležité použít správný typ projekce a vhodné
napětí záření. Obecně platí, že čím nižší napětí tím lepší, jelikož se nám tkáně zobrazí kontrastněji (s většími rozdíly
barev). Ohledně projekcí existují 3 základní: Předozadní, zadopřední a boční. Předozadní projekci použijeme, pokud
chceme zobrazit lépe krční páteř, zadní část trupu či hlavy (části blíže ke snímači a dále ke zdroji záření). Při
zadopřední projekci je zřetelněji vidět obličej včetně čelisti, žebra atd. Konečně u boční projekce záleží na jakém boku
je inkriminované místo, jež chceme blíže vyšetřit a podle toho musíme nasměrovat zářič či pacienta.

Vedlejší účinky rentgenového záření


S ohledem na fakt, že rentgenové záření patří mezi ionizační formy záření, existují jisté vedlejší účinky, které může
jeho použití skýtat. Proto je třeba se vyvarovat využívání rentgenu, když to není nezbytně nutné. S počtem ozáření
pacienta totiž roste pravděpodobnost vzniku jeho případného poškození. To je obvykle podobného charakteru jako
újmy způsobené radiačním zářením např. při katastrofách jaderných elektráren. Může se vyskytnout poškození DNA,
popáleniny, nádorové bujení, různé degenerace a mnoho dalších zdravotních problémů. Nelze opomenout ale ani
ozáření, kterému je denně vystaven zdravotnický personál. Pro jeho ochranu jsou v místnostech s rentgenem
instalovány olověné dveře, baryová omítka a existuje mnoho dalších ochranných opatření včetně osobních dozimetrů,
kterými si pracovníci pravidelně kontrolují, jestli opravdu nebyli vystaveni ozáření.

Zajímavosti

Ze statistik vyplývá, že CT přístroj je největším zdrojem radiační zátěže u obyvatel vyspělých zemí.
Každé 2 hodiny v letadle v letové hladině pro zaoceánské lety - 11 000 m.n.m. mají stejné radiační účinky jako 1
rentgen plic.
V České republice se každoročně provede na 12 milionů rentgenů a 500 000 vyšetření CT.
Angiografická vyšetření jsou větší zátěží, než ostatní rentgenová vyšetření.
Dávka radiačního záření při vyšetření CT je až o 2 řády vyšší naž při prostém rentgenu.

CT (mSv) RTG (mSv)


hrudník 5,8 0,02
břicho 5,3 1,06
hlava 1,5 0,1

Tabulka srovnávající radiační účinky CT vyšetření oproti vyšetření RTG

Odkazy
Související články
Rentgenové záření
Ionizující záření
RTG terapie
Skiaskopie
Skiagrafie

Zdroje

BENEŠ, Jiří, Daniel JIRÁK a František VÍTEK. Základy lékařské fyziky. 4. vydání. Praha : nakladatelství
Karolinum, 2015. ISBN 978-80-246-2645-1.

NAVRÁTIL, Leoš a Josef ROSINA. Medicínská biofyzika. 1. vydání. Praha : Grada Publishing, 2005. ISBN 80-


247-1152-4.
Výpočetní tomografie (heslo)

Počítačový (výpočetní)
tomograf

Schéma CT

CT (zkr. computed tomography) je diagnostická metoda, kde měříme intenzitu úzkého svazku rentgenového
záření, které prošlo tělem pacienta různými směry.

Z naměřených hodnot počítač vypočte obraz.

Zdroj záření se postupně po malých úhlech otočí o 180° okolo pacientova těla, takže je každý řez prozářen z mnoha
směrů. Počítač poté vypočte hodnoty absorpce v určitém místě roviny prozařování a na displeji hodnoty zobrazí
určitou barvou nebo jasem.

Detektory záření jsou v CT zařízení velmi citlivé a rozlišují velký počet intenzit záření, tzn. obraz je kvalitnější.

Odkazy
Související články
Ochrana před ionizujícím zářením
Low-dose
Výpočetní tomografie

Zdroj

KUBATOVA, Senta. Biofot [online]. [cit. 2011-01-31]. <https://uloz.to/!CM6zAi6z/biofot-doc>.

Externí odkazy

Portál Radiodiagnostika
Šprindrich Jan: Výpočetní tomografie – CT. Multimediální podpora výuky klinických a zdravotnických oborů ::
Portál 3. lékařské fakulty UK [online] 6.3.2011, poslední aktualizace 23.11.2011 [cit. 2011-12-22] Dostupný z
WWW: <http://portal.lf3.cuni.cz/clanky.php?aid=82>. ISSN ISSN 1804-3143
Radioterapie
Úvod
Rentgenové záření se využívá jak k diagnostickým účelům, tak k terapii. Při radioterapii je využíváno ionizující záření,
které lze dle částic rozdělit na elektromagnetické záření nebo záření korpuskulární (částicové). Mezi
elektromagnetické záření řadíme např. záření gama či rentgenové záření. Mezi korpuskulární záření patří např.
elektrony, pozitrony, neutrony a produkty štěpení jader. Negativní účinek má zejména ta část záření, která se v tkáni
absorbuje, nikoliv ta, která projde.

Z medicínského pohledu představuje radioterapie důležitou součást péče o onkologického pacienta. Téměř
polovina onkologických pacientů absolvuje během své léčby radioterapii. Radioterapie je využívána rovněž k léčbě
některých nenádorových onemocnění.

Biofyzikální základy radioterapie


Ionizující záření se šíří přímočaře, ubývá se čtvercem vzdálenosti a zeslabuje se při průchodu látkou. Nejvíce ubývají
těžké nabité částice, nejméně fotonové záření.

Biologický účinek je ovlivňován intenzitou záření, kvalitou záření (těžké částice mají větší účinek), dobou expozice
(čím delší expozice, tím větší účinek) a prostorovým rozložením záření (čím větší ozařovaný objem tkáně, tím větší
účinek).

Jednotky určující intenzitu a vliv ionizujícího záření na organismus

Absorbovaná dávka záření

Absorbovaná dávka (D) je definována jako poměr energie ionizujícího záření E ku hmotnosti látky m, která záření
absorbovala.

Jednotkou absorbované dávky je jeden Gray (Gy).

Tato jednotka se využívá pro přímo ionizující záření. Pro nepřímo ionizující záření (= ionizaci způsobí až sekundární
částice, které vznikly při interakci záření a hmoty) se používá veličina kerma.

Kerma

Kerma (Kinetic energy released in material) popisuje vliv sekundárních částic na danou látku. Jednotkou kerma je 1
Gy.

Každé záření má jiné biologické účinky a poškozuje tkáň jiným způsobem. Aby bylo možné biologický účinek porovnat,
byla zavedena veličina dávkový ekvivalent.

Dávkový ekvivalent

Dávkový ekvivalent H je součin absorbované dávky záření D a koeficientu relativní biologické účinnosti (jakostního
činitele) Q.

Ačkoliv má dávkový ekvivalent stejné jednotky jako absorbovaná dávka záření, používá se jednotka 1 Sievert (Sv)

V radiační ochraně se používají veličiny efektivní dávka a efektivní hodnota, v nichž jsou definovány limity ozáření.

Ekvivalentní dávka

Ekvivalentní dávka je součin radiačního váhového faktoru (který pro různé částice nabývá hodnot 1–20) a střední
absorbované dávky.

Efektivní dávka

Efektivní dávka je součet součinů tkáňových váhových faktorů a ekvivalentní dávky v ozářených orgánech.

Jednotky udávající vylučování radionuklidu z organismu


Doba, za kterou dojde k přeměně právě poloviny atomů ze vzorku radioaktivního nuklidu, se nazývá fyzikální poločas
rozpadu (Tf). Lidské tělo však vylučuje radionuklid také močí, stolicí apod., proto se udává tzv. biologický poločas (Tb),
za který se vyloučí z organismu právě polovina množství daného radionuklidu. Efektivní poločas (Tef) pak určuje dobu,
za kterou klesne celková aktivita radionuklidu na polovinu.

Zdroje záření v radioterapii


V zevní radioterapii

Lineární urychlovače
Lineární urychlovače produkují vysokoenergetický svazek elektronů nebo svazek fotonů, který vzniká po
následném prudkém zabrzdění urychlených elektronů (brzdné záření s vysokou energií fotonů). Maximální dávka
záření je 1–4 cm pod povrchem kůže. Vycházející svazek fotonů je tvarován pomocí clon.

Betatrony
Betatrony jsou cyklické urychlovače částic. Lze je využít jako zdroj rentgenového nebo gama záření (při
nasměrování elektronového paprsku na kovovou desku dojde k prudkému zbrzdění a vzniku brzdného záření).

Cyklotrony
Lze jimi urychlovat protony, využívají se v protonové terapii.

V brachyterapii

V brachyterapii se používají uzavřené zářiče, které se vloží přímo do místa nádoru. Při využití přístrojů s vysokým
energetickým příkonem, kdy ozáření trvá jen několik minut, se používají vysoce aktivní zářiče (např. 192 Ir).
Permanentní aplikace využívají radioizotopů s krátkým poločasem rozpadu a nízkou energií záření.

Otevřené zářiče
Léčba pomocí otevřených zářičů využívá přirozeného metabolismu některých látek. Např. jód se vychytává ve štítné
žláze, proto lze využít radioizotopy jódu při karcinomu štítné žlázy.

Rozdělení RTG terapie podle intenzity záření


1. Povrchová radioterapie: ozařování kůže a sliznice do hloubky 1–1,5 cm, napětí 50–60 kV
2. Polohloubková radioterapie: ozařování nádorů těsně pod kůží pomocí konvenčních rentgenových přístrojů,
energie fotonů do 100 keV
3. Konvenční hloubková radioterapie: používají se rentgenové přístroje s energií fotonů 200–400 keV
4. Vysokovoltážní radioterapie: pomocí přístrojů produkujících záření s energií vyšší než 1 MeV; čím je energie
fotonů vyšší, tím více dochází při interakci záření s tkání k rozptylu

Princip léčby ionizujícím zářením v medicíně


Radioterapie využívá ionizačního záření. Cílem radioterapie je dostat maximum energie ionizujícího záření do
oblasti nádoru a současně nepoškodit okolní zdravou tkáň. Při použití záření je zasažena tkáň před i za nádorem.
Možnou dávku záření tedy určuje odolnost okolní zdravé tkáně.

Žena připravena podstoupit


radioterapii

Intenzitu biologického účinku ozáření ovlivňuje:

celková doba expozice, ale také časové rozdělení jednotlivých dávek v průběhu terapie;
prostorové rozložení ozáření – účinek se u stejných expozic zvyšuje s objemem ozářené tkáně;
kvalita záření – určuje se tzv. polotloušťkou, která udává tloušťku určitého kovu, který sníží intenzitu záření na
polovinu (v mm).

Mechanismus biologického účinku ionizujícího záření


Účinky ionizujícího záření se projevují u všech druhů živých organismů. Lze rozlišit přímý a nepřímý účinek záření
na buňku (tkáň).

Konkrétně pak při průchodu záření biologickým prostředím (tkání) dochází při absorpci záření v dané tkáni k přímé
či nepřímé ionizaci a excitaci atomů a molekul buněk organismu. Při excitaci se elektron posune na vyšší
energetickou hladinu, tj. vzdálí se od jádra. Při ionizaci elektron vyletí z atomu a atom se přemění na kladný ion a
elektron. Dochází-li k absorpci energie záření přímo v jádře, hovoříme o přímé ionizaci. Způsobí-li ionizace
radiolýzu vody za vzniku radikálů (H• a OH•) a radikály následně poškodí DNA, dochází k nepřímé ionizaci.
Radikály dále reagují s molekulami DNA, způsobují zlomy vláken a jsou tedy příčinou poškození. Závažné jsou změny
ve struktuře a biosyntéze DNA. Dále může dojít k různým mutacím (genetickým či somatickým).

Biologické účinky ionizujícího záření závisí na dávce záření, která představuje celkovou energii, kterou záření dodalo
tkáni nebo organismu. Buňky mají však jistou schopnost poškození enzymaticky opravit, což se děje jen tehdy, není-li
přísun energie do buněk příliš rychlý. Prakticky to znamená, že při určité dávce je poškození organismu menší, je-li
tkáň nebo organismus touto dávkou ozáření ozářen nikoli najednou, ale je-li dávka buď rozprostřena rovnoměrně na
delší dobu, nebo rozdělena na několik menších dávek s časovými prodlevami mezi nimi (frakcionace dávky).

Krom radikálů vznikají ještě molekulární produkty radiolýzy (H2, O2, H2O2) s životností jen okolo 10-6 s, protože se
rychle přeměňují na reakční produkty. V řetězci reakcí následujícím po radiolýze je velice důležitá přítomnost kyslíku,
ovlivňuje totiž radiosenzitivitu buněk. Tkáně s menším obsahem kyslíku mohou být více odolné. Přítomnost kyslíku
zvyšuje vznik radikálů a brání reparativním pochodům v buňce. Buňky s dostatečným zásobením kyslíkem jsou 2–3x
citlivější na záření.

Poškození buněk závisí také na rozmnožovacích schopnostech buňky. Pokud má buňka větší schopnost
k rozmnožování a čím méně je diferencovaná, tím je citlivější. Vysoce radiosenzitivní jsou proto buňky na počátku
vývoje (např. kostní dřeň, samčí gonády, epitel střeva). Radiorezistentní jsou pak málo se dělící dobře diferencované
buňky (mozek či myokard).

Základní typy radioterapie


Podle umístnění zdroje záření ve vztahu k tělu nemocného lze rozlišit několik typů radioterapie. Ve vzdálenosti desítek
centimetrů se jedná o zevní (externí) radioterapii (external beam radiation therapy, EBRT, XRT, či také
teleterapie). Pokud je zdroj záření zaveden do tělesných dutin nebo do tkání, hovoříme o brachyradioterapii (BRT,
[1]
sealed source radiation therapy).

Zevní radioterapie

U zevní radioterapie se využívá zdroj záření mimo tělo ozařovaného pacienta, zpravidla ve vzdálenosti 80-100 cm od
těla pacienta, resp. od osy rotace ozařovacího přístroje.

Využívá se brzdné záření lineárního urychlovače. Jeho svazek záření je tvarován pomocí mnoholistového kolimátoru,
který umožňuje přesné ozáření určené oblasti technikou konformní radioterapie (3D CRT) i radioterapie s
modulovanou intenzitou svazku (IMRT). Využitím těchto moderních technik je minimalizováno ozáření okolních
zdravých tkání.

K léčbě může být kromě brzdného záření využito i svazku elektronů. Výhodou lineárního urychlovače je i možnost
volby různé energie záření podle uložení nádoru. Čím vyšší energie záření, tím je větší průnik záření do hloubky. V
některých případech, např. ozáření kožních nádorů, kostí, apod. je výhodné použít terapeutický rentgenový
ozařovač.[2]

Brachyterapie

Při brachyterapii je zdroj záření umístěn do těsné blízkosti ložiska nebo přímo do původního místa nádoru, event. do
jeho lůžka. To umožňuje lokálně aplikovat ve srovnání se zevní radioterapií větší dávku v kratším čase. Hlavním
smyslem této léčby je možnost zvýšení dávky v nádoru nebo jeho lůžku bez většího ozáření okolních zdravých tkání a
orgánů. V minulosti se jako zdroj záření využívalo radium, avšak léčba byla příliš zdlouhavá. V současné době lze
léčbu provést velmi rychle a po ambulantním zákroku může jít pacient domů. Brachyterapie se nejčastěji využívá v
léčbě gynekologických nádorů (děložního čípku, dělohy, zevních rodidel). Brachyterapie se využívá i v léčbě nádorů
zažívacího traktu (jícnu, žlučových cest, konečníku), v posledních letech pak i k léčbě nádorů prostaty.[2]

Brachyterapie je dělena podle způsobu umístění radionuklidového zářiče v těle pacienta na:
Intrakavitární – aplikátor je umístěn do tělní dutiny, ze které nádor vychází;
Intraluminární aplikace – vodiče a zdroj záření je zaveden do lumen trubicového orgánu, např. u maligní
stenózy způsobené nádory plic, jícnu, žlučových cest, rekta;
Intersticiální – zdroj záření je zaveden přímo do nádorového ložiska nebo jeho lůžka;
Povrchová – speciální aplikátory formou muláží jsou umístěny na povrch postižené kůže či sliznice.[1]

Jak zevní radioterapie, tak brachyterapie se používají samostatně nebo se v indikovaných případech vzájemně
kombinují. Díky novější metodě ozařování nádorů svazkem protonů a iontů se mohou zklikvidovat i radiorezistentní,
do této doby prakticky neléčitelné typy nádorů.

Konformní radioterapie
Konformní radioterapie, resp. trojrozměrná konformní radioterapie, dnes patří k moderně vedené léčbě
radioaktivním zářením. Cílem je individuálně přizpůsobit ozařovaný objem nepravidelnému trojrozměrnému tvaru
cílového objemu nádorového ložiska. Ozařuje se cílový objem s minimálním lemem a s menším zatížením zdravých
tkání.

Tím lze zvýšit dávku v cílovém objemu, což přináší vyšší lokální kontrolu. Snižuje se riziko vzniku především
chronických ireverzibilních změn po ozáření a navíc je umožněno zvýšení dávky v cílovém objemu bez zvýšeného
rizika poškození okolních zdravých tkání.

Prostorového souhlasu léčeného objemu (např. nádor, lůžko nádoru) a plánovacího cílového objemu lze dosáhnout
kombinací několika technik konformní radioterapie:

1. tvarováním ozařovacích polí pomocí individuálních vykrývacích bloků vícelamelového kolimátoru (systém
vykrývacích lamel v hlavici lineárního urychlovače individuálně upravující tvar ozařovacího pole)
2. aplikací radioterapie s modulovanou intenzitou ozařovacího svazku nebo obrazem řízené radioterapie,
které jsou schopny vytvořit a provést ozáření přesně podle hranic cílového objemu i s konkávním průběhem;
jejich cílem je co nejpřesnější ozáření s maximálním šetřením zdravých tkání;
3. použitím trojrozměrných 3D plánovacích systémů.

V České republice existují dva druhy ozařovacích přístrojů poskytující vysoce konformní techniky radioterapie, a to
gama nůž a lineární urychlovače s mikrokolimátorem či konickými tubusy (X nůž). Chystá se zavedení dalších
systémů: CyberKnife či TomoTherapy.[1]

Leksellův gama nůž

Gamma nůž

Pomocí Leksellova gama nože je dosaženo požadovaného biologického efektu v malém cílovém objemu tkáně
aplikací jedné dávky záření. Toto záření je soustředěno do úzkých svazků paprsků, které se kříží ve společném
ohnisku. Do tohoto ohniska se umístí cíl, ve kterém má vzniknout ohraničená léze. Ta vznikne působením vysoké
radiační dávky, kterou tvoří křížící se paprsky. Dávka do okolní tkáně je minimální a v malé vzdálenosti od ohniska
představuje prakticky jednotlivý paprsek. Jinými slovy řečeno, ke koncentraci dostatečného množství energie
potřebné k vyvolání žádoucí léčebné biologické reakce v malém ohraničeném objemu tkáně dochází překřížením
úzkých svazků paprsků ionizujícího záření vedených z různých směrů ve společném ohnisku.

Záření gama nože je tvořeno kobaltovými zdroji izotopu 60Co.[3]

X nůž

X nůž je upravený lineární urychlovač se stereotaktickým systémem (mikrokolimátor upravující úzkými lamelami
tvar ozařovacího svazku). Na rozdíl od gama nože, který je možno použít pouze ke stereotaktické radiochirurgii, lze
lineární urychlovač se stereotaktickým systémem použít jak k radiochirurgii, tak i k stereotaktické radioterapii (i
extrakraniální).

Další rozvíjející se moderní technikou je kombinace stereotaktické radiochirurgie s technikou IMRT 4D-
konformní radioterapie.

Vývoj radioterapie dnes směřuje k tzv. adaptivní (dynamické) radioterapii (ART), která individuálně umožňuje
vyhodnotit a upravit změny vznikající v cílovém objemu v reálném čase (např. změnu polohy, velikosti cílového objemu
při výrazné regresi nádoru či vzhledem k nově vzniklým anatomickým změnám při váhovém úbytku pacienta aj.). Dále
umožňuje i ověřit dávku záření a případně upravit naplánované distribuce v průběhu léčby. Cílem je aplikovat
správnou dávku do správného reálného objemu (tzv. radioterapie řízená dávkou, Dose Guided Radiotherapy, DGRT).[1]

Specifické techniky radioterapie

Velkoobjemové techniky radioterapie, kdy jsou ozařovány „velké objemy velkými poli“ např. před transplantací
krvetvorné tkáně:

technika ozáření kraniospinální osy – u nádorů s neuroaxiální diseminací (např. u meduloblastomu), ozářen
celý centrální nervový systém
ozáření pokožky celého těla elektronovým svazkem (sprchou) - u lymfomů kůže
tzv. IF (involved field) radioterapie – ozáření iniciálně postižených či reziduálních uzlin
intraoperační radioterapie (IORT) – používá se jednorázové aplikace vysoké dávky záření na oblast lůžka
nádoru během chirurgického výkonu.

Protonová terapie

Protonová terapie využívá v léčbě urychlených protonů. Má vyšší biologickou účinnost, neboť protony mají vyšší
hustotu ionizace podél dráhy než fotony nebo elektrony. Využívají se kladně nabité elementární částice jádra atomu
vodíku – protonů, které mají větší hmotnost oproti elektronům. Protony jsou v cyklotronu urychleny na rychlost
rovnající se přibližně polovině rychlosti světla. Jejich energie dosahuje hodnoty až 230 MeV a umožňuje tak zničit
nádory až do hloubky 30 cm. Protony jsou poté usměrněny silným magnetickým polem do velmi úzkého paprsku a
s vysokou mírou přesnosti přeneseny do prostorově zobrazeného zhoubného nádoru. Při zbrzdění v nádorové tkáni
dojde k uvolnění energie, ionizaci a vzniku radikálů vedoucí k poškození DNA zasažené buňky.

Výhodou protonové terapie je tzv. Braggův efekt, tj. kdy se největší část energie předá v místě úplného zabrždění
protonu, v oblasti tzv. Braggova vrcholu, tu lze vhodnou energií modulovat do oblasti nádoru, kde tak dojde
k maximálnímu účinku. Svazek urychlených částic ovšem musí mít vysokou energii a být velmi přesně cílen, jedině
pak předá většinu energie výhradně nádorové tkáni. V porovnání s dosavadními ozařovacími postupy šetří zdravé
tkáně před nádorem a zcela minimálně poškozuje zdravé tkáně za nádorem. Protože je nemocný ozařován
v isocentrickém systému ze všech směrů a intenzitu paprsku lze modulovat, přináší tato metoda teoreticky výrazné
snížení nežádoucích účinků léčby. Praktickým problém je však úzký tvar Braggova píku, pro potřeby ozařování celého
objemu nádoru je nezbytná superpozice množství svazků různých energií a nakumulovaná dávka před nádorem tak ve
výsledku není tak nízká, jak by se mohlo zdát z tvaru Braggovy křivky pro jediný proton. Výhody protonové terapie
proto stále čekají na potvrzení ze strany dlouhodobých klinických studií.

Tato fyzikální vlastnost svazku protonů – nízká vstupní dávka, maximální dávka energie v požadované hloubce a
nulová výstupní dávka – umožňuje velmi plánovat rozložení dávky záření v těle pacienta.[4]

Klinická aplikace radioterapie


Radioterapie je po chirurgické léčbě nejefektivnější kurativní léčbou. Dále je používána v léčbě adjuvantní či
neoadjuvantní. Nezastupitelné místo má i v léčbě paliativní.

Kurativní (radikální) radioterapie je primární volbou léčby (např. kožní nádory, karcinomy děložního hrdla či
spinocelulární karcinomy anu). Při kurativní radioterapii je aplikována maximální dávka záření (v případě zevní
radioterapie 60–80 Gy) s přijatelnou mírou závažných komplikací. Léčba trvá zpravidla 6–8 týdnů. Obvykle je
aplikována standardní frakcionací 2,0 Gy 1x denně, 5x týdně. V radikální léčbě se kombinuje zevní radioterapie s
brachyterapií, především u gynekologických malignit (karcinom děložního čípku od klinického stadia IIb a
pokročilejší; inoperabilní endometriální karcinomy, nádory pochvy aj.).

Cílem adjuvantní radioterapie je zničit předpokládanou zbytkovou mikroskopickou chorobu. Tím se snižuje riziko
vzniku lokální či regionální recidivy onemocnění a může se zvýšit doba přežití. Nejvíce je indikována po chirurgickém
zákroku (např. parciální či totální mastektomie, resekce rekta, exstirpace glioblastomu, exstirpace spinocelulárního
karcinomu kůže, nádorů hlavy a krku, resekce sarkomů měkkých tkání, karcinomu žaludku, aj.).

Cílem neoadjuvantní radioterapie je zmenšení nádoru před základním léčebným výkonem, zpravidla před operací
(tzv. předoperační radioterapie). Tím se významně zmenšuje rozsah operačního výkonu.

Paliativní radioterapie cílí na odstranění či zmenšení symptomů nádorového onemocnění (zejména bolest, útlak,
krvácivé stavy, u gynekologických malignit aj.). Druhotným cílem je prodloužení přežití. Nejčastěji jsou paliativně
ozařována metastatická ložiska ve skeletu, mozku nebo uzlinách. Dále je paliativní radioterapie využívána při obtížích
vyplývajících z obstrukce (dušnost) či útlaku orgánů (syndrom horní duté žíly) či krvácení. Paliativní radioterapie je
obvykle aplikována v několika málo frakcích s vyšší dávkou na frakci (např. 10x3 Gy, 5x4 Gy) a jednoduchými
ozařovacími technikami (1–2 ozařovací pole). Výjimkou není ani použití jednorázového ozáření vysokou dávkou (1x6–8
Gy) u pacientů s předpokládanou krátkou dobou života. U paliativní radioterapie s dlouhodobým záměrem se aplikují
dávky vyšší, zpravidla standardní frakcionací.[1]

Nenádorová radioterapie ulevuje od obtíží způsobených nenádorovým onemocněním či zabraňuje zhoršení funkce
postiženého orgánu.

Odkazy
Související články

Protinádorová terapie

Zdroje

ZOUL, David. Biologické účinky ionizujícího záření,


http://www.cytoprostor.euweb.cz/radiobiologie/radiobiologie.pdf
Česká onkologická společnost České lékařské společnosti Jana Evangelisty Purkyně,
https://www.linkos.cz/pacient-a-rodina/lecba/jak-se-lecit/radioterapie-ozarovani/
MUDr. L. HYNKOVÁ, MUDr. H. DOLEŽALOVÁ, Ph.D., prof. MUDr. P.ŠLAMPA, CSc. Radioterapie. Učební texty
pro studenty 5. roč. LF MU Brno Klinika radiační onkologie, LF MU.
Základní princip gama nože, https://www.homolka.cz/cs-CZ/oddeleni/stereotakticka-a-radiacni-neurochirurgie-
osrn/lekselluv-gama-nuz.html
Dirk VeRELLEN, Mark De RIDDER, Nadine LINTHOUT, Koen TOURNEL, Guy SOETE & Guy STORME.
Innovations in image-guided radiotherapy. Nature Reviews Cancer 7, 949 -960 (December 2007)
NAVRÁTIL, Leoš a Jozef ROSINA. Medicínská biofyzika. Vyd. 1. Praha: Grada, 2005, 524 s. ISBN 80-247-1152-4.

Reference
1. Radioterapie - učební texty pro studenty 5. roč. LF MU Brno. Klinika radiační onkologie, LF MU. Vypracovali:
MUDr. L. Hynková, MUDr. H. Doleželová, Ph.D., prof. MUDr. P. Šlampa, CSc.
2. Společnost radiační onkologie, biologie a fyziky, https://www.srobf.cz/cz/home/
3. Základní princip gama nože,https://www.homolka.cz/cs-CZ/oddeleni/stereotakticka-a-radiacni-neurochirurgie-
osrn/lekselluv-gama-nuz.html
4. Protonová léčbahttp://www.ptc.cz/protonova-lecba/princip-lecby/
Dozimetrie
Dozimetrie ionizujícího záření je založena na základní vlastnosti tohoto záření a sice vytvářet v látce, kterou
prochází, záporné a kladné ionty (ionizovat) nebo vyvolávat v látce fyzikální jevy, které jsou měřitelné a nějakým
způsobem závislé na množství záření, kterému byla daná látka vystavena (např. změna vodivosti, teploty, barvy, vznik
termoluminiscence apod.).

Pomocí různých metod dozimetrie můžeme charakterizovat vlastnosti zdroje záření, pole záření nebo účinky záření
na látku, kterou prochází − včetně tkání a živých organismů. Veličinou, která charakterizuje zdroj záření, je aktivita
udávaná v Bq (becquerel). Pole záření lze popsat fluencí částic (počet částic dopadajících na jednotku plochy).
Základní veličinou, pomocí které charakterizujeme účinek záření na látku, kterou prochází je absorbovaná dávka.
Absorbovaná dávka je množství energie předané látce zářením, jednotkou dávky je 1 gray (Gy) s rozměrem J.kg-1.

Existuje mnoho druhů dozimetrů, které lze rozdělit na dvě základní skupiny podle toho zda podávají informaci
kontinuálně – zde patří zejména ionizační komory, Geiger-Müllerovy počítače, polovodičové detektory, scintilační
detektory, elektronické dozimetry − a nebo je v nich informace o množství sdělené energie nějakým způsobem
kumulovaná a vyhodnocena až pomocí nějakého vhodného vyhodnocovacího zařízení − to jsou dozimetry integrální −
zde patří zejména filmový, termoluminiscenční nebo fotoluminiscenční dozimetr. Tyto dozimetry jsou také nejčastěji
používané v současné době v osobní dozimetrii. Rozšiřuje se také použití elektronické osobní dozimetrie.

Ionizační komora

Ionizační komora je tvořena dvěma elektrodami (anodou a katodou), umístěnými v plynném prostředí. Za normálních
okolností (bez přítomnosti záření) systémem neprochází žádný proud − plyn mezi elektrodami je nevodivý, obvod není
uzavřen. Vnikne-li však do prostoru mezi elektrodami ionizující záření, vyráží z původně neutrálních atomů plynu
elektrony a mění je na kladné ionty. Záporné elektrony putují v elektrickém poli okamžitě ke kladné anodě, kladné
ionty se dají do pohybu k záporné katodě − obvodem začne protékat slabý elektrický proud způsobený iontovou
vodivostí ionizovaného plynu mezi elektrodami. Proud je přímo úměrný intenzitě ionizujícího záření.

Scintilátor

Ionizační komory v
dozimetru

Scintilační detektory

Scintilační detektory převádějí absorbovanou energii ionizujícího záření na energii fotonů náležejících zpravidla do
viditelné krátkovlnné nebo blízké ultrafialové oblasti spektra. Scintilační detektory patří mezi nejpoužívanější
detektory ionizujícího záření. Jejich výhoda spočívá vedle dobrých spektrometrických vlastností také v tom, že
detekční médium, scintilátor, může mít různé rozměry a téměř libovolný tvar. Přitom hmotnost scintilačních látek je
dostatečné velká, takže lze dosáhnout poměrně velké detekční účinnosti, zejména pro záření gama. Scintilační
detektor dává rovněž výstupní signál, jehož další zpracování obvykle nevyžaduje použití citlivých zesilovačů.

Polovodičové detektory

Polovodičové detektory jsou založeny na ionizačních účincích v pevných látkách. Vnikne-li ionizující částice do
vhodného polovodiče, vytváří v něm ionizací páry elektron − díra, přičemž většina primárních elektronů má tak velkou
energii, že způsobuje další nárazovou ionizaci prostředí. Dochází k lavinovitému uvolňování elektronů do vodivostního
pásu a tvorbě děr ve valenčním páse, počet uvolněných nosičů náboje tedy závisí na energii primární částice.
Přiložíme-li na tento polovodič napětí, pak vlivem elektrického pole se volné nosiče nábojů (elektrony a díry) dají do
pohybu v příslušném směru a v připojeném obvodu vznikne proudový impuls, jehož velikost závisí na energii
dopadající částice ionizujícího záření. To umožňuje využít polovodičové detektory jak pro detekci ionizujícího záření,
tak pro spektrometrická měření.

Filmové dozimetry
Filmové dozimetry jsou založeny na skutečnosti, že působením ionizujícího záření vzniká tzv. latentní obraz, který lze
vyvolávacím procesem zviditelnit a vzniklé zčernání (optická hustota), které lze měřit, je závislé na míře ozáření filmu.
Dozimetrický film je oboustranně překryt sadou filtrů a díky tomu lze určit i energii záření a směr ozáření. Pomocí
filmového dozimetru lze měřit dávku od fotonů, elektronů i neutronů. Mezi výhody filmového dozimetru patří trvalý
záznam údaje o ozáření s možností opětovné analýzy vyvolaného filmu. Nevýhodou je citlivost na světlo, vysokou
vlhkost, teplotu a některé chemikálie.

Termoluminiscenční dozimetry

Termoluminiscenční dozimetry jsou vhodné látky, v nichž ionizující záření vyvolává excitace elektronů z valenčního
do vodivostního pásu s následným záchytem v záchytných centrech. Zahřátím potom získají elektrony dostatečnou
energii k opuštění záchytného centra a k rekombinaci při současném vyzáření ultrafialového záření nebo viditelného
světla, které je detekováno pomocí fotokatody a fotonásobičů. Celková vyzářená energie je úměrná energii ionizujícího
záření pohlceného v látce. Pro výrobu termoluminiscenčních dozimetrů se používají různé druhy
termoluminiscenčních materiálů např. LiF (lithium fluorid), CaF2, MgBeO4, CaSO4(Dy), aj. s rozdílnou energetickou
závislostí a citlivostí pro různé druhy záření.Výhodou termoluminiscenčních dozimetrů je jejich vysoká citlivost,
možnost přesného měření odezvy, poměrně široká oblast lineární závislosti mezi dávkou a odezvou dozimetru,
možnost opakovaného použití dozimetru a také možnost použití látek s vlastnostmi blízkými lidské tkáni. Nevýhodou
termoluminiscenčních dozimetrů je jejich citlivost na světlo a znečištění.

Radiofotoluminiscenční dozimetry

Podstatou radiofotoluminiscenčního dozimetru je fotoluminiscence, která je založena na principu tvorby


luminiscenčních center indukovaných ionizujícím zářením v určitých látkách (př. fosfátová skla dopovaná stříbrem).
Luminiscence je vybuzena osvětlením ozářeného detektoru ultrafialovým světlem. Stejně jako u termoluminiscenčního
dozimetru je vyzářené světlo úměrné dávce ionizujícího záření absorbované v detektoru. Výhodou
radiofotoluminiscenčního dozimetru je dlouhodobá stabilita odezvy, konstantní a vysoká citlivost a nízká energetická
závislost. Nevýhodou je citlivost detektorů na světlo.

Elektronické osobní dozimetry


Elektronické osobní dozimetry nabývají na významu postupně s vývojem miniaturizace elektroniky a dostupnosti
výpočetní techniky. Zpravidla pracují na bázi Geiger-Müllerových detektorů nebo polovodičových − Si-detektorů.
Nevýhodou elektronických osobních dozimetrů je možné ovlivnění elektromagnetickým zářením. Elektronické osobní
dozimetry je možné používat autonomně nebo ve spojení s vyhodnocovacím zařízením. Všechny uvedené dozimetry v
podstatě fungují tak, že na základě kalibrace pomocí zdroje záření známých vlastností jsou kalibrovány tak, že velikost
zkoumané odezvy (např. luminiscence) je vztažena k množství působícího záření.

Radiační ochrana pracovníků se zdroji ionizujícího záření

Umístění
osobního
dozimetru

Při používání ionizujícího záření je nutné stanovit určitá pravidla tak, aby nedocházelo k nežádoucím účinkům záření
na lidské zdraví. Tyto nežádoucí účinky jsou děleny na dvě základní skupiny − deterministické a stochastické.
Deterministické účinky jsou účinky akutní a nastávají při překročení určité prahové hodnoty dávky (patří sem např.
nemoc z ozáření, popáleniny, katarakta apod.), ochrana před nimi spočívá v tom, že zabráníme tomu, aby prahová
dávka byla dosažena. Ochrana před stochastickými účinky je složitější, protože tam takovou prahovou hodnotu
nemáme (patří sem různé druhy nádorů, genetické poškození).

Jsou stanoveny limitní hodnoty, které nesmí být překročeny při používání zdrojů záření. Stanovení absorbované dávky
nám v případě živých tkání a organismů ještě nic neříká o možném účinku daného záření na tento organismus nebo
tkáň. Je nutno vzít v úvahu o jaký druh záření se jedná (alfa, beta, fotony, neutrony atd.), protože jednotlivé druhy
záření mají na tkáň odlišné účinky − liší se jejich tzv. biologická účinnost. Např. částice alfa jsou těžké nabité
částice a při průchodu hmotou předávají svou energii rychle na poměrně krátkém úseku své dráhy − říkáme, že hustě
ionizují − jinak je tomu u fotonů, které procházejí hmotou snadno a ionizují řídce − proto je také vhodné je např.
využívat v zobrazovacích metodách v medicíně − radiodiagnostice. Pokud aplikujeme na stanovenou absorbovanou
dávku v dané tkáni koeficient, který zohledňuje tyto vlastnosti ionizujícího záření a kterému říkáme radiační váhový
faktor dostaneme se k tzv. ekvivalentní dávce. Pokud chceme hodnotit možné účinky ozáření celého těla, musíme
také vzít v úvahu citlivost jednotlivých orgánů a tkání na ozáření a pravděpodobnost vzniku výše uvedených
stochastických účinků. Toto je zohledněno v tzv. tkáňovém váhovém faktoru. Pokud pro každou tkáň stanovíme
ekvivalentní dávku a vážíme ji tímto faktorem a provedeme součet přes všechny tkáně pro které je faktor stanoven
dostaneme se k tzv. efektivní dávce. Efektivní dávka v sobě tedy zahrnuje i informaci o závažnosti velikosti ozáření
živého organismu. Limitní hodnoty pro ozáření pracovníků jsou potom stanoveny v ekvivalentní dávce pro vybrané
orgány a tkáně (tím je omezen vznik deterministických účinků) nebo v efektivní dávce (snížení pravděpodobnosti
vzniku stochastických účinků na přijatelnou úroveň).

Limitní hodnota efektivní dávky pro pracovníky je daná legislativou (Atomový zákon) a je 100 mSv za 5 let a
současně nesmí překročit 50 mSv za rok. Limitní hodnota pro např. oční čočku je 150 mSv za rok, pro končetiny 500
mSv za rok. Pracovníci se zdroji ionizujícího záření (radiační pracovníci) musí být vybaveni osobními dozimetry příp.
je jejich dávka hodnocena na základě měření na pracovišti, aby mohlo být prokázáno, že stanovené limity
nepřekračují.

Odkazy
Související články

Ochrana před ionizujícím zářením

Použitá literatura

NAVRÁTIL LEOŠ, ROSINA JOSEF, ET AL,. Medicínská biofyzika. 1 (dotisk 2010) vydání. Praha : Grada
Publishing, a.s., 2005. 524 s. ISBN ISBN 978-80-247-1152-2.
Radioaktivní rozpad
Radioaktivní rozpad je jev, při kterém nestabilní atomy vyzařují svoji energii ve formě částic nebo
elektromagnetických vln. Je to děj náhodný, tudíž nejsme schopni předpovědět, jaké jádro a kdy se přemění.
Radioaktivní rozpad je určován tzv. rozpadovou konstantou, která má pro každý radioizotop jinou hodnotu. Existují
3 základní druhy rozpadu – alfa, beta a gama.

Tabulka znázorňující charakteristiku jednotlivých typů rozpadu:

charakter typ míra ionizace v


charakter prvků emitované částice pohlcení
záření spektra organismu
těžké a přirozeně často se papír, vrstva
rozpad alfa částice alfa (helium) čárové velká
radioaktivní vychyluje vzduchu
elektronové často se
rozpad beta lehké (umělé radionuklidy) spojité hliníkový plech střední
neutrinum vychyluje
rozpad
fotony vyzářené jádrem fotony nevychyluje se čárové olovo, dural slabá
gama

Průchod radioaktivního záření

Odkazy
Související články

Efektivní, fyzikální a biologický poločas

Zdroje

BENEŠ, Jiří, Daniel JIRÁK a František VÍTEK. Základy lékařské fyziky. 4. vydání. Praha : nakladatelství
Karolinum, 2015. ISBN 978-80-246-2645-1.
Efektivní, fyzikální a biologický poločas
Během radioaktivní přeměny dochází k neustálému zmenšování počtu aktivních atomů radioaktivního nuklidu, dobu
rozpadu lze vyjádřit pomocí tří veličin:

fyzikální poločas
biologický poločas
efektivní poločas

Fyzikální poločas
Fyzikální poločas (značíme Tf) charakterizuje rychlost radioaktivní přeměny jádra. Jedná se o dobu, ve které dojde k
přeměně poloviny atomů daného radioaktivního nuklidu. Čas je rozměrem dané veličiny (časové jednotky - minuty,
sekundy,hodiny,dny,roky - jsou voleny vzhledem k danému izotopu). Hodnoty daného typu poločasu jsou pro jednotlivé
radionuklidy značně odlišné. Pomocí hodnoty Tf můžeme zhotovit graf jeho rozpadové křivky.

Fyzikální poločas je definován: Tf = ln(2)/ λ = 0,693../λ

λ = přeměnová konstanta, je pro každé radioaktivní jádro charakteristická, hodnoty jsou uvedeny v tabulkách

ln(2) = hodnota logaritmu

Biologický poločas
Biologický poločas (značíme Tb) je doba, za kterou se vyloučí z organizmu polovina množství daného radionuklidu.

Efektivní poločas
Efektivní poločas (značíme Tef) definujeme jako čas, během kterého se sníží celková aktivita radionuklidu, který byl
vpraven do organismu. Vzhledem k radioaktivní přeměně a biologickému vylučování klesne na polovinu. V
podmínkách reálných je vždy kratší než poločas fyzikální.

Efektivní poločas je definován: 1/ Tef = 1/Tf + 1/Tb

Odkazy
Související články

Poločas rozpadu

Použitá literatura

BENEŠ, Jiří, et al. Základy lékařské biofyziky. 3. vydání. Praha : Karolinum, 2011. 200 s. ISBN 978-80-246-


2034-3.

NAVRÁTIL, Leoš a Jozef ROSINA, et al. Medicínská biofyzika. 1. vydání. Praha : Grada, 2005. 524 s. ISBN 80-
247-1152-4.
Rozpad alfa

Záření alfa

Rozpad alfa je případem spontánní emise těžké částice z radioaktivního jádra. Jejich přeměnou se atomy snaží
dosáhnout stabilnějšího stavu. Rozpad alfa je nejčastějším případem přeměny. Mimo rozpadu alfa dochází také k
rozpadu beta a spontánnímu štěpení jader.

S přeměnou alfa se setkáváme pouze u těžkých přirozených radionuklidů. Částice alfa je složena ze dvou protonů a
dvou neutronů, jedná se tedy o jádro prvku helia. Vzhledem k přítomnosti dvou protonů, nese částice dva kladné
elementární náboje. Energetické spektrum daného záření je čárové.

Při emisi částice dochází k přeměně mateřského jádra, protonové číslo Z se zmenší o 2, nukleonové číslo A se zmenší
o 4, uvolní se jádro helia (A = 4, Z = 2). Výsledkem přeměny je dceřiné jádro, které v Mendělejevově periodické
tabulce prvků leží o dvě místa nalevo od jádra mateřského. Kinetická energie jádra je při emisi zanedbatelná, u
emitované částice je nižší.

Při průchodu prostředím částice alfa silně ionizují i excitují, rychle ztrácejí svojí energii. Tato skutečnost určuje
omezený rozsah záření alfa. Záření alfa je málo pronikavé, pronikavost ve vzduchu je několik milimetrů, ve vodě
zlomky milimetrů. Při dopadu na kůži se absorbuje v horních vrstvách pokožky. Záření alfa je proto nebezpečné pouze
při vnitřní kontaminaci organismu. Ochranou před zářením je list papíru či oděv, nebo i tenká folie z plexiskla.

Odkazy
Související články

Záření alfa
Rozpad beta
Ionizující záření

Použitá literatura

BENEŠ, Jiří, et al. Základy lékařské biofyziky. 3. vydání. Praha : Karolinum, 2011. 200 s. ISBN 978-80-246-


2034-3.

ROSINA, Jozef a Hana KOLÁŘOVÁ. Biofyzika pro studenty zdravotnických oborů. 1. vydání. Praha : Grada, 
2006. ISBN 80-247-1383-7.

NAVRÁTIL, Leoš a Jozef ROSINA, et al. Medicínská biofyzika. 1. vydání. Praha : Grada, 2005. 524 s. ISBN 80-
247-1152-4.
Rozpad beta

Rozpad beta je izobarickou transformací jádra, z tohoto důvodu je zachován počet nukleonů. Jádro je nestabilní díky
nadbytku či nedostatku neutronů. Vyskytuje se u umělých i přirozených radionuklidů.
Při srovnání s částicemi alfa zjistíme, že částice beta jsou mnohem lehčí, proto se při stejné energii pohybují mnohem
rychleji. Při průchodu prostředím méně ionizují a excitují. K ochraně před zářením beta používáme plexisklo či tenký
hliníkový plech.

Rozlišujeme tři druhy rozpadu beta:

Přeměna β- (emise elektronů)


Přeměna β+ (emise pozitronů)
Elektronový záchyt

Přeměna β-

Přeměna β-
V dceřiném jádře dochází k nárůstu protonového čísla o 1 (tedy Z → Z+1). Mateřským jádrem je emitován elektron
(záporně nabitý) a antineutrino (antičástice neutrina, částice ze skupiny leptonů). Spektrum emitovaných elektronů je
spojité.

Přeměna β+

Přeměna β+
V dceřiném jádře dochází k poklesu protonového čísla o 1 (tedy Z → Z-1). Mateřským jádrem je emitován pozitron
(kladně nabitý) a neutrino. Spektrum emitovaných pozitronů je spojité.

Elektronový záchyt (záchyt elektronu z elektronového obalu)


Nejčastěji dochází k záchytu elektronu ze slupky K. Protonové číslo mateřského prvku se zmenšuje o 1. Po zachycení
elektronu jádrem se uvolní místo pro elektron v nejnižších energetických hladinách, dojde k jeho zaplnění elektronem
z vyšší energetické hladiny. Přeskok elektronu v elektronovém obalu je doprovázen emisí charakteristického
elektromagnetického vlnění z obalu atomu. Mateřským jádrem je rovněž emitováno neutrino.

Odkazy
Související články

Záření beta
Rozpad alfa
Ionizující záření

Použitá literatura
BENEŠ, Jiří, et al. Základy lékařské biofyziky. 3. vydání. Praha : Karolinum, 2011. 200 s. ISBN 978-80-246-
2034-3.

ROSINA, Jozef a Hana KOLÁŘOVÁ. Biofyzika pro studenty zdravotnických oborů. 1. vydání. Praha : Grada, 
2006. ISBN 80-247-1383-7.
NAVRÁTIL, Leoš a Jozef ROSINA, et al. Medicínská biofyzika. 1. vydání. Praha : Grada, 2005. 524 s. ISBN 80-
247-1152-4.
Kosmické záření, měření pozadí detektoru

Energetické spektrum kosmického záření

Kosmické záření je ionizující záření, které dopadá na Zemi z vesmíru. Je to záření s nejmenší vlnovou délkou
a největší frekvencí kmitů, které známe. Jeho potenciální energie je tudíž obrovská. Naštěstí pro člověka je z velké
části odstíněno magnetickým polem a zemskou atmosférou. Na zemi dopadá pouze v nepatrném množství, které je
slučitelné s existencí života.

Složky kosmického záření

Kosmické záření rozdělujeme na primární a sekundární. Jako primární označujeme záření, které nepřišlo do styku
se zemskou atmosférou. Jeho energie se běžně pohybuje okolo 10 GeV, ale může dosáhnout až 1018 eV. Naopak
sekundární vzniká interakcí primárního se zemskou atmosférou a obsahuje prakticky všechny elementární částice
(např. neutrony).

Hodnoty kosmického záření na zemi a mimo ní

Intenzita kosmického záření je ovlivněna 2 faktory: Nadmořskou výškou a zeměpisnou šířkou. Se stoupající výškou
jeho intenzita roste asi do výšky 20 km, pak do 50 km klesá a ve větších vzdálenostech je konstantní. Zajímavostí je,
že na pólech je intenzita kosmického záření až o 25 % vyšší, což souvisí s orientací magnetického pólu země.

Kosmické záření v medicíně

S kosmickým zářením se běžně setkáváme v nukleární medicíně, kde se podílí na chodu detektorů. Konkrétně tvoří
tzv. "pozadí", tedy složku, kterou se snažíme eliminovat stíněním. Vzhledem k vysoké energii záření však kompletní
eliminace není možná

Kosmické záření v jiných oborech

Je logické, že kosmickému záření jsou vystaveni lidé, vyskytující se mimo zemskou atmosféru. Velké potíže působí v
kosmonautice, kde se na něj musí pamatovat při konstrukci kosmických lodí. Rovněž cestující v letadlech jsou
vystaveni relativně většímu množství kosmického záření stejně tak jako záření gama a RTG. Hodnoty ozáření však
nejsou vysoké a při uměřeném používání leteckých dopravních prostředků se nemusíme bát žádných vedlejších
účinků.

Pozadí detektoru

V pozadí detektoru se uplatňují 3 složky záření - vnější záření z okolního prostoru, vnitřní radioaktivita materiálu a
elektický šum přístroje. Logicky je kosmické záření přítomno v té první.

Odkazy
Související články

Ionizující záření
Dozimetrie
RTG záření

Externí odkazy

Kosmické záření (česká wikipedie)


článek o ozáření při letu letadlem (časopis třípól)

Zdroje

BENEŠ, Jiří, Daniel JIRÁK a František VÍTEK. Základy lékařské fyziky. 4. vydání. Praha : nakladatelství
Karolinum, 2015. ISBN 978-80-246-2645-1.

ULLMANN, Vojtěch. AstroNuklFyzika [online]. [cit. 2016-01-31]. 


<http://astronuklfyzika.cz/DetekceSpektrometrie.htm>.
Záření alfa

Záření α
Záření α je korpuskulární (částicové) záření, jehož částicemi je proud jader helia 4He. Částice α vznikají při α-
rozpadu těžkých jader popsaném rovnicí:

Takto vzniklé částice mají jen několik možných hodnot energie; částice α si odnášejí rozdíl energií mezi mateřským a
dceřiným jádrem. Říkáme, že α-záření má čárové energetické spektrum, to znamená, že energie uvolněných α-
částic může nabývat jen určitých, pro daný druh jádra specifických hodnot.

Interakce záření α

Částice α má poměrně velkou hmotnost a nese 2 kladné elementární náboje. Ionizační ztráty při průchodu
absorbátorem jsou značné. Zhruba polovina energie se ztrácí ionizací a druhá polovina důsledkem excitace. I když
může být rychlost vzniklých α-částic velká, jejich dosah na vzduchu je jen krátký. Záření lze odstínit i listem papíru.
Nebezpečná je především vnitřní kontaminace, tedy vpravení α-zářiče do organismu. Může docházet k uvolnění
velkého množství energie malému objemu tkáně, což má negativní biologický účinek.

Odkazy
Související články

Záření gama
Záření beta
Ionizující záření
Nemoc z ozáření
Záření beta

Spojité energetické spektrum záření


beta

Záření, neboli přeměna β je korpuskulární (částicové) záření, jeho částicemi je proud elektronů β−, pozitronů
(antielektronů) β+ nebo tzv. elektronový záchyt.

Částice β+

vznikají při reakci zvané pozitronová přeměna, které si můžeme představit jako přeměnu jednoho nadbytečného
protonu na neutron, pozitron a neutrino. Pozitron a elektronové neutrino jsou emitovány velkou rychlostí z jádra
ven, zatímco neutron zůstává v jádře.

Protonové číslo vzniklého prvku bude o 1 menší, tedy vzniklý prvek bude odpovídat v periodické tabulce prvku o
jedno místo vlevo.

Pozitronovou přeměnu lze popsat rovnicí:

Pozitronová přeměna se vyskytuje prakticky jen u umělých nuklidů. Vzniklý pozitron velmi rychle anihiluje s
elektronem a vznikají dva fotony γ o energii 0,51 MeV.

Částice β− (elektrony) mohou vznikat β− přeměnou, kterou si můžeme zjednodušeně představit jako přeměnu
neutronu na elektron, proton a antineutrino:

Protonové číslo vzniklého prvku je o jedno vyšší (vzniklý prvek bude v periodické tabulce o jedno místo napravo).

Přeměnu částice β− lze popsat rovnicí:

Vzniklé elektrony mají spojité energetické spektrum. Maximální hodnota kinetické energie elektronu, s níž může být
emitován, je pro každý nuklid charakteristická. Záření β má poměrně malou pronikavost, lze odstínit hliníkovým
plechem. V elektrickém i magnetickém poli se záření &beta vychyluje.

Elektronový záchyt je třetím druhem záření β a můžeme si ho představit jako přeměnu, kdy se nejdříve sloučí proton
z jádra s elektronem z atomového obalu a vytvoří neutron a neutrino. Neutron zůstane v jádře atomu a poté, co dojde
k záchytu elektronu atomovým jádrem, dojde k okamžitému obsazení uvolněného místa v elektronové slupce
elektronem z vyšších vrstev elektronového obalu. Při tomto přesunu elektronu z vyšší vrstvy je vyzářeno
charakteristické záření X nebo Augerova elektronu.

Interakce záření β

Při průchodu absorbátorem připadají největší ztráty energie elektronů na ionizaci a excitaci. Dolet částic je větší než
u záření α a uplatňuje se pružný rozptyl částic. Další interakcí elektronů je tzv. brzdné záření. Jedná se o
elektromagnetické vlnění, které vzniká zabrzděním pohybujícího se elektronu v blízkosti jádra působením
coulombické interakce. Brzdným zářením se uvolní energetické kvantum ve směru jeho původní dráhy. Platí také, že
intenzita brzdného záření je přímo úměrná atomovému číslu absorbátoru a energii elektronů.

Zdroje
KUPKA, Karel a Jozef KUBINYI. Nukleární medicína. 6. vydání. 2015. 0 s. ISBN 978-8087343-54-8.

Odkazy
Související články

Záření gama
Záření alfa
Ionizující záření
Nemoc z ozáření
Elektron-pozitronové páry
Interakce ionizujícího záření
Při průchodu ionizujícího záření hmotou dochází k interakci mezi částicemi nebo fotony záření a strukturami okolních
atomů, tedy jádrem a elektronovým obalem. Samotný průběh interakce závisí na charakteru záření, jeho kinetické
energii a složení látky ve které interakce probíhá.

Interakce je hodnocena ze dvou pohledů:

z pohledu záření – změny energie, počtu částic a směru procházejícího záření;


z pohledu prostředí – přesuny subatomárních částic a na ně navazující reakce.

Podle interakce rozdělujeme ionizující záření na:

přímo ionizující – elektricky nabité částice – záření α, β- a β+, protony, jaderné fragmenty;
nepřímo ionizující – elektroneutrální záření – RTG, záření γ, neutronové záření.

Podle místa interakce dělíme na:

interagující s jádrem;
interagující s atomovým obalem.

Celkově lze tedy ionizující záření rozdělit do tří skupin:

elektromagnetické (fotonové) záření – RTG a γ záření;


nabité částice – p, α, β;
nenabité částice – neutrony.

Interakce elektromagnetického záření

Pravděpodobnost jednotlivých typů


interakce fotonového záření.

K interakci dochází v jádru a jeho elektromagnetickém poli nebo v obalu atomu. Interakce obou druhů záření (RTG a
γ) jsou si velmi podobné, liší se místem vzniku (RTG z obalu, γ z jádra) a frekvencí.

Celkově rozlišujeme šest typů interakcí fotonového záření s hmotou (viz tabulka). Podrobněji budou rozebrány jen tři
nejvýznamnější: fotoelektrický jev, Comptonův rozptyl a tvorba elektron-pozitronových párů.

absorbce pružná srážka nepružná srážka


elektronový obal fotoelektrický jev Rayleighův rozptyl Comptonův rozptyl
atomové jádro fotojaderná interakce jaderný rezonanční rozptyl
EMG pole tvorba elektron-pozitronových párů

Fotoelektrický jev

Úvod

Fotoelektrický jev

Fotoelektrický jev (fotoefekt) je jednou ze tří možných interakcí γ záření s elektronovým obalem atomu. Z těchto tří
interakcí má foton zpravidla nejslabší energii. Je to fyzikální jev, při němž jsou elektrony uvolňovány (vyzařovány,
emitovány) z látky (nejčastěji z kovu) v důsledku absorpce elektromagnetického záření látkou. Elektrony emitované z
jaderného obalu jsou pak označovány jako fotoelektrony. Jejich uvolňování se označuje jako fotoelektrická emise
(fotoemise).

Historie

Za objevitele fotoelektrického jevu je považován Heinrich Hertz, který si při svých pokusech (roku 1887), jejichž
cílem bylo experimentální prokázání existence Maxwellem předpovězených elektromagnetických vln, všiml, že ozáření
jiskřiště ultrafialovým zářením usnadňuje přeskok jiskry – tj. přenos elektrického náboje mezi elektrodami.

Roku 1899 Joseph John Thomson udělal rozhodující krok k objasnění podstaty jevu. Thomson experimentálně
identifikoval v nositelích záporného náboje unikajících z ozařovaného kovového vzorku elektrony.

Vlastní podstatu fotoelektrického jevu popsal v roce 1905 Albert Einstein (Nobelovu cenu za tento objev získal v
roce 1921).

Popis jevu

Dopad na povrch látky

Fotoelektrický jev nastává, když se celá energie kvanta záření γ předává některému elektronu z elektronového
obalu absorbujícího materiálu nebo případně volnému elektronu (např. v kovech). Část energie se spotřebuje na
uvolnění elektronu (vykonáním tzv. výstupní práce Wv) a část se přemění na kinetickou energii Ek vzniklého
fotoelektronu. Foton záření γ tímto zaniká a jeho energii přebírá fotoelektron, který ionizuje své okolí.

Einsteinova rovnice pro fotoefekt vyjadřuje zákon zachování energie.

(h je Planckova konstanta)

Atom, kterému byl vyražen elektron je v excitovaném stavu a přechází do základního stavu emisí elektromagnetického
záření o frekvenci odpovídající rozdílu energie excitovaného a základního stavu.

(Volné místo po elektronu je zaplněné jiným elektronem, který sem přeskočil z jiné slupky atomového obalu. Při tomto
přeskoku se vyzáří energie ve formě charakteristického záření. Místo charakteristického záření může dojít k
alternativnímu jevu - energie se předá některému elektronu na vyšší slupce, který se pak uvolní a vyzáří jako tzv.
Augerův elektron.)

Foton interaguje s elektronem na slupkách K, L a M. Tedy s elektrony, které leží blízko jádru atomu. Nejčastěji
probíhá na slupce K (80% pravděpodobnost).

Fofoefekt je pravděpodobnější v materiálech s vyšším protonovým číslem absorpčního materiálu (kost, kontrastní
látky).

Podle představ klasické fyziky by elektronům měla být předána kinetická energie dopadajícího elektromagnetického
vlnění. Energie elektromagnetických vln souvisí s intenzitou záření, tzn. energie vyzařovaných elektronů by měla
záviset na intenzitě dopadajícího záření. Experimenty však ukázaly, že kinetická energie vyzařovaných elektronů
je závislá na frekvenci a nikoliv na intenzitě dopadajícího záření.

Pro každý kov existuje určitá mezní frekvence f0 taková, že elektrony se uvolňují pouze při frekvenci f0 a frekvencích
vyšších. Na frekvenci použitého elektromagnetického záření závisí také energie emitovaných elektronů. Pokud je
frekvence f dopadajícího záření vyšší než mezní frekvence f0, mají fotoelektrony energii v rozmezí od nuly do určité
maximální hodnoty Emax.

Závislost pozorovaného jevu na frekvenci záření nebylo možné vysvětlit klasicky.

Druhy fotoefektu

Podle způsobu vzniku elektronů vlivem dopadajícího elektromagnetického záření můžeme rozlišit:

1. vnější fotoelektrický jev − jev probíhá na povrchu látky, elektrony se uvolňují do okolí
2. vnitřní fotoelektrický jev − uvolněné elektrony zůstávají v ní jako vodivostní elektrony (např. polovodiče, v
nichž jsou tímto způsobem uvolňovány elektrony zejména z přechodu PN)

Inverzní fotoelektrický jev

Inverzní (obrácený) fotoelektrický jev je jev, kdy na látku dopadají elektrony, které způsobují vyzařování fotonů.

Vysvětlení jevu

Závislost kinetické energie elektronu


na frekvenci dopadajícího světla

V roce 1905 Albert Einstein vyšel z Planckovy kvantové hypotézy a z představy, že elektromagnetická vlna o
frekvenci f a vlnové délce λ se chová jako soubor částic (světelných kvant), z nichž každá má svou energii a
hybnost. Tyto částice mají zvláštní vlastnosti, především se stále pohybují rychlostí světla a nelze je žádným způsobem
zastavit, zpomalit ani urychlit. Podle teorie relativity musí mít nulovou klidovou hmotnost. Tyto částice byly v roce
1926 nazvány fotony. Velikost kvanta energie závisí na frekvenci (vlnové délce) elektromagnetického záření, přičemž
platí:

Světlo při dopadu na povrch látky předává energii povrchovým elektronům zkoumané látky. K uvolnění elektronu z
vazby v atomu je potřeba tzv. ionizační energie. Tato nutná energie k uvolnění elektronu může vzniknout, jestliže je
vlnová délka světla dostatečně malá. V tom případě může frekvence a energie dosáhnout dostatečně vysoké
hodnoty. Předáním takové energie elektronům je možné překonat tzv. fotoelektrickou bariéru k uskutečnění
výstupní práce. Minimální frekvence, při níž dopadající fotony předávají elektronům výstupní energii se označuje
jako prahová frekvence. Jestliže je energie předaná elektronu větší než energie potřebná k jeho uvolnění, pak
fotoelektronu zůstane část energie jako kinetická energie.

Rovnice fotoelektrického jevu: (hf je energie dopadajícího fotonu,


hf0 je výstupní práce − minimální energie potřebná k uvolnění elektronu, Emax je maximální možná energie
uvolněného elektronu) Z této rovnice vyplývá, že energie uvolněného elektronu závisí pouze na frekvenci
dopadajícího záření, a nikoliv na intenzitě tohoto záření.

Využití
Fotoelektrický jev hraje významnou úlohu na poli biofyziky. Příkladem je uplatnění těchto jevů při radiačních
vyšetřeních pacienta. Rentgenové snímky vznikají na principu obráceného fotoelektrického jevu, kdy se povrch
ostřeluje elektrony a uvolňují se paprsky X. Různé tkáně mají jinou absorbci, proto můžeme na snímcích rozeznat
struktury. Elektron zcela pohltí foton a Rtg foton zaniká. Absorbce fotoelektrického jevu je na rozdíl od Comptonova
rozptylu, který probíhá také, žádoucí. Při Comptonově jevu zůstávají volné elektrony a foton nezaniká, dochází tedy ke
srážkám těles a mění se jejich směr a vlnová délka.

Comptonův rozptyl

Comptonův rozptyl popisuje srážku fotonu s např. elektronem za následné změny vlnové délky vzniklého fotonu.

Comptonův rozptyl

Zjednodušené schéma Comptonova


jevu
Historie

V roce 1905 zavedl Albert Einstein myšlenku korpuskulárně vlnového charakteru částic pro vysvětlení
fotoelektrického jevu. Vzhledem k tomu, že podle ní bylo možné foton pokládat zároveň za vlnění i částici, mělo by
docházet mezi ním a například elektronem k interakcím, které by svým charakterem odpovídaly pružným srážkám, při
niž dochází v rámci izolované soustavy k zachování celkové hybnosti a energie.

Schéma Comptonova experimentu

Avšak dle představ klasické fyziky by po srážce fotonu s elektronem měl být elektron rozkmitán frekvencí
dopadajícího fotonu a následně vyslat fotony opět se stejnou frekvencí.

Roku 1922 se rozhodl tuto teorii prověřit Arthur Holly Compton. Vytvořil experiment s rozptylem rentgenového záření
na volných elektronech. Bylo třeba využít dopadu záření na materiály s velmi slabě vázanými elektrony. Rentgenové
záření (λ = 0,07 nm ) dopadalo na uhlíkový terčík. Compton byl schopen zachytit zdvojené spektrální čáry: jedna
odpovídala původní vlnové délce (rozptyl na pevně vázaných elektronech), druhá měla vlnovou délku vyšší (rozptyl na
volných elektronech). Byla tak experimentálně potvrzena správnost Einsteinovy teorie a Compton roku 1927 získal
Nobelovu cenu za fyziku.

Comptonův posun

Existence druhé vlnové délky byla vyjádřena rovnicí pro Comptonův posun:

λ... vlnová délka fotonu před srážkou

λ´ … vlnová délka fotonu po srážce

φ … úhel rozptylu

h/m0c... Comptonova vlnová délka (pro elektron = 2,4262 · 10-12 m)

Dodatky k teorii

Teoreticky ke Comptonovu jevu dochází při každé srážce fotonu s elektronem, je-li však hmotnost fotonu velmi malá v
porovnání s hmotností elektronu, je tento posun minimální. Vzhledem k tomu lze Comptonův jev pozorovat pouze za
použití záření s vysokou hmotností fotonů, např. záření rentgenové nebo gama.

Demonstrace Comptonova jevu při


použití gama záření

Sekundární foton se vychyluje v intervalu 0–180° a na odchylce je závislá jeho energie. Pokud dochází ke zpětnému
rozptylu (tj. 180° úhel), má foton nejmenší energii. Sekundární foton může být schopen znovu opakovat jev, pokud má
dostatečnou energii, nebo zaniká fotoelektrickým jevem.

Využití

Comptonova jevu se využívá v mnoha vědních oborech. Jako příklad můžeme uvést zejména radioterapii (cílené
poškozování DNA např. rakovinných buněk), spektroskopii (detekce ionizujícího záření) a astronomii (Comptonova
gama observatoř).

Elektron-pozitronové páry
Ke tvorbě elektron-pozitronových párů dochází při interakci vysokoenergetického γ záření s elektronovým obalem
atomu. Je to energeticky nejvyšší možnost ze tří interakcí γ záření s obalem.
Tvorba elektron-pozitronového páru

Při energiích fotonů teoreticky nad 1,02 MeV, prakticky však mnohem vyšších, dochází k přeměně fotonu blízko
atomového jádra na pozitron a elektron. Přitom je nutné, aby se tak stalo v blízkosti atomového jádra nebo jiné
částice, která může převzít část hybnosti fotonu (jelikož hybnost pozitronu a elektronu je nižší). Samovolná přeměna
fotonu na elektron a pozitron není možná při jeho pohybu ve vakuu z důvodu zákona zachování hybnosti (součet
hybností vzniklého elektronu a pozitronu je menší než hybnost dodaná fotonem). Samotná proměna probíhá v
důsledku elektrického pole atomového jádra (čím větší náboj jádro má, tím je větší pravděpodobnost proměny).
Kinetická energie vytvořeného elektron-pozitronového páru je rozdělena mezi obě částice náhodně.

Pomocí následující rovnice lze vyjádřit energetickou bilanci daného děje:

Z uvedeného vztahu vyplývá, že energie fotonu musí být větší než energie, která představuje součet dvou klidových
hmotností elektronu (součet klidové energie elektronu a pozitronu jsou stále stejné).

Vzniklé částice ztrácejí svou energii při interakcích s okolním prostředím, tj. ionizací nebo excitací. Pozitron se však
většinou spojuje s elektronem za procesu anihilace a vyzáří tak dvě kvanta elektromagnetického záření o energii
511 keV. Tato kvanta se pohybují opačným směrem.

Interakce nabitých částic


Těžší částice, nesoucí náboj, interagují s hmotou nepružnými nárazy. Tím předávají okolí svou kinetickou energii.
Tento děj nazýváme srážkové ztráty energie. Náboj se nemění.

Interakce může proběhnout také formou tzv. radiační ztráty, kdy spolu interagují pouze elektromagnetická pole
částic. K tomu dochází často u lehkých částic, elektronů.

Částice záření nemusí předat celou svou energii najednou. Energie se v cílové struktuře projeví jako excitace buď
jádra nebo elektronů v obalu. Vždy dochází ke ztrátám energie v podobě tepla. Pokud je předaná energie dostatečně
velká, může dojít k odtržení elektronu, který se pak chová jako β- částice, jeho kinetická energie je rovna energii
předané nárazem. Toto takzvané sekundární elektronové záření je někdy označováno jako záření δ.

Těžší částice nesoucí větší náboj interagují častěji, svou energii předají okolí na krátké vzdálenosti a pak zanikají.

Podrobnější informace naleznete na stránce LET.

Interakce nenabitých částic


Neutrony, jako nejvýznamnější zástupci skupiny nenabitých částic, interagují s okolní hmotou jen na základě silných
a slabých jaderných sil.

Interakce může probíhat formou pružného a nepružného rozptylu, emisí nabité částice, radiačního
(neutronového) záchytu, nebo dojde k rozštěpení jádra.

Pružný rozptyl

Nejpravděpodobnějším typem interakce je pružný rozptyl. Dochází k němu na velmi malých jádrech, která se svou
velikostí blíží neutronu, jako například vodík. Energie, předaná neutronem, se celá přemění na kinetickou energii
zasažené částice. Atom se neexcituje. Odražený neutron pokračuje dále se zbytkem energie. Tomuto ději se říká
moderace neutronové rychlosti. Děj pokračuje dokud se neutron nezpomalí natolik, že může být absorbován
jádrem. Moderace se využívá v 235uranových jaderných reaktorech, kdy atomy vodíku v molekule vody zpomalují
rychlé neutrony, vzniklé štěpením.

Podrobnější informace naleznete na stránce Jaderný reaktor.

Nepružný rozptyl

K nepružnému rozptylu dochází na jádrech těžkých prvků. Neutron, obdobně jako při pružném rozptylu, předá část
své kinetické energie a jako zpomalený pokračuje dál. Zasažené jádro se ale excituje, část předané energie je
vyzářena v podobě γ fotonu, zbytek se změní v kinetickou energii jádra.

Emise nabité částice


Neutron má tolik energie, že při zásahu jádra vyrazí jeden nebo i několik jaderných elementů. Kinetická energie
neutronu je tedy spotřebována na vyražení protonu, α částice nebo deuteronu (jádro deuteria, jeden proton a jeden
neutron), zbytek předané energie se změní v kinetickou energii vyražené částice. Tím může dojít ke vzniku
nestabilního nuklidu a jeho dalšímu rozpadu.

Radiační záchyt

Neutron je zachycen jádrem, jeho kinetická energie je vyzářena v podobě γ fotonu.


Jaderné štěpení

Při vhodné rychlosti neutronu, v poměru k cílovému atomovému jádru, může dojít k rozštěpení jádra za vzniku
štěpných produktů, kterými jsou většinou radioaktivní izotopy. Při štěpení se z jádra uvolní tolik energie, že vzniklé
neutrony mají i vyšší energii, než ten, který způsobil štěpení. Obvykle je emitováni foton γ záření. Pokud se uvolní víc
než jeden neutron schopný štěpení, dochází k tzv. lavinovému efektu s exponenciálním nárůstem interakcí. Této
řetězové štěpné reakce se využívá u jaderných zbraní. V moderované podobě (= ne všechny vzniklé neutrony štěpí
další jádra) je základem jaderného reaktoru.

Odkazy
Související články
Ionizující záření

Použitá literatura

PODZIMEK, František. Biofyzika ionizujícího záření. 1. vydání. Hradec Králové : Vojenská lékařská akademie


Jana Evangelisty Purkyně, 1990. 119 s. ISBN 80-85109-24-7.

BENEŠ, Jiří, Pravoslav STRÁNSKÝ a František VÍTEK. Základy lékařské biofyziky. 2. vydání. Praha : 


Karolinum, 2007. 201 s. ISBN 978-80-246-1386-4.

NAVRÁTIL, Leoš a Jozef ROSINA, et al. Medicínská biofyzika. 1. vydání. Praha : Grada, 2005. 524 s. ISBN 80-
247-1152-4.
Interakce ionizujícího záření
Při průchodu ionizujícího záření hmotou dochází k interakci mezi částicemi nebo fotony záření a strukturami okolních
atomů, tedy jádrem a elektronovým obalem. Samotný průběh interakce závisí na charakteru záření, jeho kinetické
energii a složení látky ve které interakce probíhá.

Interakce je hodnocena ze dvou pohledů:

z pohledu záření – změny energie, počtu částic a směru procházejícího záření;


z pohledu prostředí – přesuny subatomárních částic a na ně navazující reakce.

Podle interakce rozdělujeme ionizující záření na:

přímo ionizující – elektricky nabité částice – záření α, β- a β+, protony, jaderné fragmenty;
nepřímo ionizující – elektroneutrální záření – RTG, záření γ, neutronové záření.

Podle místa interakce dělíme na:

interagující s jádrem;
interagující s atomovým obalem.

Celkově lze tedy ionizující záření rozdělit do tří skupin:

elektromagnetické (fotonové) záření – RTG a γ záření;


nabité částice – p, α, β;
nenabité částice – neutrony.

Interakce elektromagnetického záření

Pravděpodobnost jednotlivých typů


interakce fotonového záření.

K interakci dochází v jádru a jeho elektromagnetickém poli nebo v obalu atomu. Interakce obou druhů záření (RTG a
γ) jsou si velmi podobné, liší se místem vzniku (RTG z obalu, γ z jádra) a frekvencí.

Celkově rozlišujeme šest typů interakcí fotonového záření s hmotou (viz tabulka). Podrobněji budou rozebrány jen tři
nejvýznamnější: fotoelektrický jev, Comptonův rozptyl a tvorba elektron-pozitronových párů.

absorbce pružná srážka nepružná srážka


elektronový obal fotoelektrický jev Rayleighův rozptyl Comptonův rozptyl
atomové jádro fotojaderná interakce jaderný rezonanční rozptyl
EMG pole tvorba elektron-pozitronových párů

Fotoelektrický jev

Úvod

Fotoelektrický jev

Fotoelektrický jev (fotoefekt) je jednou ze tří možných interakcí γ záření s elektronovým obalem atomu. Z těchto tří
interakcí má foton zpravidla nejslabší energii. Je to fyzikální jev, při němž jsou elektrony uvolňovány (vyzařovány,
emitovány) z látky (nejčastěji z kovu) v důsledku absorpce elektromagnetického záření látkou. Elektrony emitované z
jaderného obalu jsou pak označovány jako fotoelektrony. Jejich uvolňování se označuje jako fotoelektrická emise
(fotoemise).

Historie

Za objevitele fotoelektrického jevu je považován Heinrich Hertz, který si při svých pokusech (roku 1887), jejichž
cílem bylo experimentální prokázání existence Maxwellem předpovězených elektromagnetických vln, všiml, že ozáření
jiskřiště ultrafialovým zářením usnadňuje přeskok jiskry – tj. přenos elektrického náboje mezi elektrodami.

Roku 1899 Joseph John Thomson udělal rozhodující krok k objasnění podstaty jevu. Thomson experimentálně
identifikoval v nositelích záporného náboje unikajících z ozařovaného kovového vzorku elektrony.

Vlastní podstatu fotoelektrického jevu popsal v roce 1905 Albert Einstein (Nobelovu cenu za tento objev získal v
roce 1921).

Popis jevu

Dopad na povrch látky

Fotoelektrický jev nastává, když se celá energie kvanta záření γ předává některému elektronu z elektronového
obalu absorbujícího materiálu nebo případně volnému elektronu (např. v kovech). Část energie se spotřebuje na
uvolnění elektronu (vykonáním tzv. výstupní práce Wv) a část se přemění na kinetickou energii Ek vzniklého
fotoelektronu. Foton záření γ tímto zaniká a jeho energii přebírá fotoelektron, který ionizuje své okolí.

Einsteinova rovnice pro fotoefekt vyjadřuje zákon zachování energie.

(h je Planckova konstanta)

Atom, kterému byl vyražen elektron je v excitovaném stavu a přechází do základního stavu emisí elektromagnetického
záření o frekvenci odpovídající rozdílu energie excitovaného a základního stavu.

(Volné místo po elektronu je zaplněné jiným elektronem, který sem přeskočil z jiné slupky atomového obalu. Při tomto
přeskoku se vyzáří energie ve formě charakteristického záření. Místo charakteristického záření může dojít k
alternativnímu jevu - energie se předá některému elektronu na vyšší slupce, který se pak uvolní a vyzáří jako tzv.
Augerův elektron.)

Foton interaguje s elektronem na slupkách K, L a M. Tedy s elektrony, které leží blízko jádru atomu. Nejčastěji
probíhá na slupce K (80% pravděpodobnost).

Fofoefekt je pravděpodobnější v materiálech s vyšším protonovým číslem absorpčního materiálu (kost, kontrastní
látky).

Podle představ klasické fyziky by elektronům měla být předána kinetická energie dopadajícího elektromagnetického
vlnění. Energie elektromagnetických vln souvisí s intenzitou záření, tzn. energie vyzařovaných elektronů by měla
záviset na intenzitě dopadajícího záření. Experimenty však ukázaly, že kinetická energie vyzařovaných elektronů
je závislá na frekvenci a nikoliv na intenzitě dopadajícího záření.

Pro každý kov existuje určitá mezní frekvence f0 taková, že elektrony se uvolňují pouze při frekvenci f0 a frekvencích
vyšších. Na frekvenci použitého elektromagnetického záření závisí také energie emitovaných elektronů. Pokud je
frekvence f dopadajícího záření vyšší než mezní frekvence f0, mají fotoelektrony energii v rozmezí od nuly do určité
maximální hodnoty Emax.

Závislost pozorovaného jevu na frekvenci záření nebylo možné vysvětlit klasicky.

Druhy fotoefektu

Podle způsobu vzniku elektronů vlivem dopadajícího elektromagnetického záření můžeme rozlišit:

1. vnější fotoelektrický jev − jev probíhá na povrchu látky, elektrony se uvolňují do okolí
2. vnitřní fotoelektrický jev − uvolněné elektrony zůstávají v ní jako vodivostní elektrony (např. polovodiče, v
nichž jsou tímto způsobem uvolňovány elektrony zejména z přechodu PN)

Inverzní fotoelektrický jev

Inverzní (obrácený) fotoelektrický jev je jev, kdy na látku dopadají elektrony, které způsobují vyzařování fotonů.

Vysvětlení jevu

Závislost kinetické energie elektronu


na frekvenci dopadajícího světla

V roce 1905 Albert Einstein vyšel z Planckovy kvantové hypotézy a z představy, že elektromagnetická vlna o
frekvenci f a vlnové délce λ se chová jako soubor částic (světelných kvant), z nichž každá má svou energii a
hybnost. Tyto částice mají zvláštní vlastnosti, především se stále pohybují rychlostí světla a nelze je žádným způsobem
zastavit, zpomalit ani urychlit. Podle teorie relativity musí mít nulovou klidovou hmotnost. Tyto částice byly v roce
1926 nazvány fotony. Velikost kvanta energie závisí na frekvenci (vlnové délce) elektromagnetického záření, přičemž
platí:

Světlo při dopadu na povrch látky předává energii povrchovým elektronům zkoumané látky. K uvolnění elektronu z
vazby v atomu je potřeba tzv. ionizační energie. Tato nutná energie k uvolnění elektronu může vzniknout, jestliže je
vlnová délka světla dostatečně malá. V tom případě může frekvence a energie dosáhnout dostatečně vysoké
hodnoty. Předáním takové energie elektronům je možné překonat tzv. fotoelektrickou bariéru k uskutečnění
výstupní práce. Minimální frekvence, při níž dopadající fotony předávají elektronům výstupní energii se označuje
jako prahová frekvence. Jestliže je energie předaná elektronu větší než energie potřebná k jeho uvolnění, pak
fotoelektronu zůstane část energie jako kinetická energie.

Rovnice fotoelektrického jevu: (hf je energie dopadajícího fotonu,


hf0 je výstupní práce − minimální energie potřebná k uvolnění elektronu, Emax je maximální možná energie
uvolněného elektronu) Z této rovnice vyplývá, že energie uvolněného elektronu závisí pouze na frekvenci
dopadajícího záření, a nikoliv na intenzitě tohoto záření.

Využití
Fotoelektrický jev hraje významnou úlohu na poli biofyziky. Příkladem je uplatnění těchto jevů při radiačních
vyšetřeních pacienta. Rentgenové snímky vznikají na principu obráceného fotoelektrického jevu, kdy se povrch
ostřeluje elektrony a uvolňují se paprsky X. Různé tkáně mají jinou absorbci, proto můžeme na snímcích rozeznat
struktury. Elektron zcela pohltí foton a Rtg foton zaniká. Absorbce fotoelektrického jevu je na rozdíl od Comptonova
rozptylu, který probíhá také, žádoucí. Při Comptonově jevu zůstávají volné elektrony a foton nezaniká, dochází tedy ke
srážkám těles a mění se jejich směr a vlnová délka.

Comptonův rozptyl

Comptonův rozptyl popisuje srážku fotonu s např. elektronem za následné změny vlnové délky vzniklého fotonu.

Comptonův rozptyl

Zjednodušené schéma Comptonova


jevu
Historie

V roce 1905 zavedl Albert Einstein myšlenku korpuskulárně vlnového charakteru částic pro vysvětlení
fotoelektrického jevu. Vzhledem k tomu, že podle ní bylo možné foton pokládat zároveň za vlnění i částici, mělo by
docházet mezi ním a například elektronem k interakcím, které by svým charakterem odpovídaly pružným srážkám, při
niž dochází v rámci izolované soustavy k zachování celkové hybnosti a energie.

Schéma Comptonova experimentu

Avšak dle představ klasické fyziky by po srážce fotonu s elektronem měl být elektron rozkmitán frekvencí
dopadajícího fotonu a následně vyslat fotony opět se stejnou frekvencí.

Roku 1922 se rozhodl tuto teorii prověřit Arthur Holly Compton. Vytvořil experiment s rozptylem rentgenového záření
na volných elektronech. Bylo třeba využít dopadu záření na materiály s velmi slabě vázanými elektrony. Rentgenové
záření (λ = 0,07 nm ) dopadalo na uhlíkový terčík. Compton byl schopen zachytit zdvojené spektrální čáry: jedna
odpovídala původní vlnové délce (rozptyl na pevně vázaných elektronech), druhá měla vlnovou délku vyšší (rozptyl na
volných elektronech). Byla tak experimentálně potvrzena správnost Einsteinovy teorie a Compton roku 1927 získal
Nobelovu cenu za fyziku.

Comptonův posun

Existence druhé vlnové délky byla vyjádřena rovnicí pro Comptonův posun:

λ... vlnová délka fotonu před srážkou

λ´ … vlnová délka fotonu po srážce

φ … úhel rozptylu

h/m0c... Comptonova vlnová délka (pro elektron = 2,4262 · 10-12 m)

Dodatky k teorii

Teoreticky ke Comptonovu jevu dochází při každé srážce fotonu s elektronem, je-li však hmotnost fotonu velmi malá v
porovnání s hmotností elektronu, je tento posun minimální. Vzhledem k tomu lze Comptonův jev pozorovat pouze za
použití záření s vysokou hmotností fotonů, např. záření rentgenové nebo gama.

Demonstrace Comptonova jevu při


použití gama záření

Sekundární foton se vychyluje v intervalu 0–180° a na odchylce je závislá jeho energie. Pokud dochází ke zpětnému
rozptylu (tj. 180° úhel), má foton nejmenší energii. Sekundární foton může být schopen znovu opakovat jev, pokud má
dostatečnou energii, nebo zaniká fotoelektrickým jevem.

Využití

Comptonova jevu se využívá v mnoha vědních oborech. Jako příklad můžeme uvést zejména radioterapii (cílené
poškozování DNA např. rakovinných buněk), spektroskopii (detekce ionizujícího záření) a astronomii (Comptonova
gama observatoř).

Elektron-pozitronové páry
Ke tvorbě elektron-pozitronových párů dochází při interakci vysokoenergetického γ záření s elektronovým obalem
atomu. Je to energeticky nejvyšší možnost ze tří interakcí γ záření s obalem.
Tvorba elektron-pozitronového páru

Při energiích fotonů teoreticky nad 1,02 MeV, prakticky však mnohem vyšších, dochází k přeměně fotonu blízko
atomového jádra na pozitron a elektron. Přitom je nutné, aby se tak stalo v blízkosti atomového jádra nebo jiné
částice, která může převzít část hybnosti fotonu (jelikož hybnost pozitronu a elektronu je nižší). Samovolná přeměna
fotonu na elektron a pozitron není možná při jeho pohybu ve vakuu z důvodu zákona zachování hybnosti (součet
hybností vzniklého elektronu a pozitronu je menší než hybnost dodaná fotonem). Samotná proměna probíhá v
důsledku elektrického pole atomového jádra (čím větší náboj jádro má, tím je větší pravděpodobnost proměny).
Kinetická energie vytvořeného elektron-pozitronového páru je rozdělena mezi obě částice náhodně.

Pomocí následující rovnice lze vyjádřit energetickou bilanci daného děje:

Z uvedeného vztahu vyplývá, že energie fotonu musí být větší než energie, která představuje součet dvou klidových
hmotností elektronu (součet klidové energie elektronu a pozitronu jsou stále stejné).

Vzniklé částice ztrácejí svou energii při interakcích s okolním prostředím, tj. ionizací nebo excitací. Pozitron se však
většinou spojuje s elektronem za procesu anihilace a vyzáří tak dvě kvanta elektromagnetického záření o energii
511 keV. Tato kvanta se pohybují opačným směrem.

Interakce nabitých částic


Těžší částice, nesoucí náboj, interagují s hmotou nepružnými nárazy. Tím předávají okolí svou kinetickou energii.
Tento děj nazýváme srážkové ztráty energie. Náboj se nemění.

Interakce může proběhnout také formou tzv. radiační ztráty, kdy spolu interagují pouze elektromagnetická pole
částic. K tomu dochází často u lehkých částic, elektronů.

Částice záření nemusí předat celou svou energii najednou. Energie se v cílové struktuře projeví jako excitace buď
jádra nebo elektronů v obalu. Vždy dochází ke ztrátám energie v podobě tepla. Pokud je předaná energie dostatečně
velká, může dojít k odtržení elektronu, který se pak chová jako β- částice, jeho kinetická energie je rovna energii
předané nárazem. Toto takzvané sekundární elektronové záření je někdy označováno jako záření δ.

Těžší částice nesoucí větší náboj interagují častěji, svou energii předají okolí na krátké vzdálenosti a pak zanikají.

Podrobnější informace naleznete na stránce LET.

Interakce nenabitých částic


Neutrony, jako nejvýznamnější zástupci skupiny nenabitých částic, interagují s okolní hmotou jen na základě silných
a slabých jaderných sil.

Interakce může probíhat formou pružného a nepružného rozptylu, emisí nabité částice, radiačního
(neutronového) záchytu, nebo dojde k rozštěpení jádra.

Pružný rozptyl

Nejpravděpodobnějším typem interakce je pružný rozptyl. Dochází k němu na velmi malých jádrech, která se svou
velikostí blíží neutronu, jako například vodík. Energie, předaná neutronem, se celá přemění na kinetickou energii
zasažené částice. Atom se neexcituje. Odražený neutron pokračuje dále se zbytkem energie. Tomuto ději se říká
moderace neutronové rychlosti. Děj pokračuje dokud se neutron nezpomalí natolik, že může být absorbován
jádrem. Moderace se využívá v 235uranových jaderných reaktorech, kdy atomy vodíku v molekule vody zpomalují
rychlé neutrony, vzniklé štěpením.

Podrobnější informace naleznete na stránce Jaderný reaktor.

Nepružný rozptyl

K nepružnému rozptylu dochází na jádrech těžkých prvků. Neutron, obdobně jako při pružném rozptylu, předá část
své kinetické energie a jako zpomalený pokračuje dál. Zasažené jádro se ale excituje, část předané energie je
vyzářena v podobě γ fotonu, zbytek se změní v kinetickou energii jádra.

Emise nabité částice


Neutron má tolik energie, že při zásahu jádra vyrazí jeden nebo i několik jaderných elementů. Kinetická energie
neutronu je tedy spotřebována na vyražení protonu, α částice nebo deuteronu (jádro deuteria, jeden proton a jeden
neutron), zbytek předané energie se změní v kinetickou energii vyražené částice. Tím může dojít ke vzniku
nestabilního nuklidu a jeho dalšímu rozpadu.

Radiační záchyt

Neutron je zachycen jádrem, jeho kinetická energie je vyzářena v podobě γ fotonu.


Jaderné štěpení

Při vhodné rychlosti neutronu, v poměru k cílovému atomovému jádru, může dojít k rozštěpení jádra za vzniku
štěpných produktů, kterými jsou většinou radioaktivní izotopy. Při štěpení se z jádra uvolní tolik energie, že vzniklé
neutrony mají i vyšší energii, než ten, který způsobil štěpení. Obvykle je emitováni foton γ záření. Pokud se uvolní víc
než jeden neutron schopný štěpení, dochází k tzv. lavinovému efektu s exponenciálním nárůstem interakcí. Této
řetězové štěpné reakce se využívá u jaderných zbraní. V moderované podobě (= ne všechny vzniklé neutrony štěpí
další jádra) je základem jaderného reaktoru.

Odkazy
Související články
Ionizující záření

Použitá literatura

PODZIMEK, František. Biofyzika ionizujícího záření. 1. vydání. Hradec Králové : Vojenská lékařská akademie


Jana Evangelisty Purkyně, 1990. 119 s. ISBN 80-85109-24-7.

BENEŠ, Jiří, Pravoslav STRÁNSKÝ a František VÍTEK. Základy lékařské biofyziky. 2. vydání. Praha : 


Karolinum, 2007. 201 s. ISBN 978-80-246-1386-4.

NAVRÁTIL, Leoš a Jozef ROSINA, et al. Medicínská biofyzika. 1. vydání. Praha : Grada, 2005. 524 s. ISBN 80-
247-1152-4.
Detekce ionizujícího záření
Detekce ionizujícího záření funguje na principu interakce záření s látkou čidla. Čidlo neboli detektor přemění
zářivou energii na jinou formu energie, která je dále snadno registrovatelná běžnými čidly. Použití různých typů
detektorů uvažujeme podle druhu interakce , např. ionizační, scintilační.

Po detekci na vstupu měřící soustavy a absorpci fotonu nebo částice vznikne na výstupní části detektoru elektrický
impuls. Dále jsou elektrické impulsy registrovány a počítány čítačem impulsů (v případě měření aktivity), nebo je
měřena střední četnost elektrických impulsů integrátorem (v případě měření střední intenzity záření). Tento průběh
můžeme pozorovat u detektorů v impulzním zapojení.

V případě použití více detektorů rozlišujeme dva druhy zapojení:

koincidenční zapojení registruje pouze impulzy, které vznikly souběžně ve dvou a více detektorech;
antikoincidenční zapojení nedovoluje registraci impulzů, které vznikly současně, a naopak přijímá impulzy,
které nevznikly současně ve dvou a více detektorech.

Ionizační komory
Ionizační komora je elektrodový systém sloužící k detekci ionizujícího záření, který může být izolovaný od prostředí,
nebo volně umístěný ve vzduchu.

Podrobnější informace naleznete na stránce Ionizační komora.

Geiger-Müllerovy počítače
Geiger-Müllerův počítač (GM počítač) je detektor ionizujícího záření (obvykle β a γ, ale při vhodném uspořádání i
částice α). Zařízení se skládá z trubice naplněné inertním plynem (Geigerovy trubice).

Podrobnější informace naleznete na stránce Geiger-Müllerův počítač.

Scintilační detektory
Scintilační detektor se skládá z luminiscenčního scintilačního krystalu (většinou thalliem aktivovaný jodid sodný
NaI(Tl)), schopného zachytit ionizující záření ve formě γ nebo rentgenového paprsku.

Podrobnější informace naleznete na stránce Scintigrafie.

Odkazy
Související články

Ionizující záření

Použitá literatura

BENEŠ, Jiří, Pravoslav STRÁNSKÝ a František VÍTEK. Základy lékařské biofyziky. - vydání. Karolinum, 2005. 
196 s. ISBN 9788024610092.
Ionizační komora

Schéma ionizační komory

Ionizační komora je elektrodový systém sloužící k detekci ionizujícího záření, který může být izolovaný od prostředí,
nebo volně umístěný ve vzduchu.

1. statické ionizační komory


2. impulzní komůrky – počítají průlety jednotlivých častic – průlet částice komorou totiž způsobuje proudové
impulzy
3. náprstkové komůrky

Tvoří ji dvě elektrody (anoda a katoda), na kterých se měří napětí voltmetrem. Při průniku ionizačního záření do
komory se z atomů nevodivého plynu vyráží elektrony, čímž vznikají kladné ionty. Kladně nabité ionty směřují ke
katodě a záporně nabité ionty k anodě, dochází k rekombinaci.

Vlastnosti
Ionizační komora má složitou voltampérovou charakteristiku, jejíž průběh dělíme na 3 části:

1. Ohmův zákon – proud je přímo úměrný napětí;


2. nasycený proud – přestává platit Ohmův zákon, zvyšování napětí nevede k zvyšování proudu, v této části pracují
ionizační komory;
3. nárazová ionizace – ionty ionizují nárazem, se zvyšujícím se napětím prudce vzrůstá napětí, mohou ionizovat i
elektricky neutrální molekuly.

Proporcionalita – koncentrace primárních a sekundárních iontů je v rovnováze, sekundární ionty přispívají k


vodivosti, v této části pracují proporcionální počítače.
Nepřímá proporcionalita – není užíván k měřením.
Lavinové množení – ionizace i neutrálních molekul, v této části pracují Geiger-Müllerovy počítače.

Odkazy
Související články
Geiger-Müllerův počítač
Geigerův-Müllerův počítač

Geiger-Müllerův počítač

Geiger-Müllerův počítač (GM počítač) je detektor ionizujícího záření (obvykle β a γ, ale při vhodném uspořádání i
částice α). Zařízení se skládá z trubice naplněné inertním plynem (Geigerovy trubice). Na jednom konci je v ose
trubice izolovaně upevněna drátová elektroda spojená se zdrojem vysokého napětí (500 V). Na druhém konci trubice
je vstupní okénko. Trubice je obvykle vyplněna héliem, neonem nebo argonem s příměsí například par metylalkoholu
či bromu, sloužících jako tzv. zhášedlo. Zhášedlo ukončuje vzniklý výboj a zabraňuje tak vzniku stálého proudu, který
by znemožnil další měření a mohl by poškodit elektrody nebo plynovou náplň.

Vznik impulzu

lavinový efekt
Loading

Zvuk Geiger-Müllerova počítače při


nízké úrovni záření
Loading

Zvuk Geiger-Müllerova počítače při


vysoké úrovni záření

Ionizující záření proniká okénkem do trubice. Při srážce s atomy plynu dochází k ionizaci plynu. Uvolněné elektrony
jsou urychlovány k anodě, kladné ionty ke katodě. Po nárazu těchto primárních urychlených elektronů do dalších
atomů se vyrážejí z dosud neionizovaných atomů sekundární elektrony, které mohou po urychlení vyrážet další
elektrony. Tento proces se nazývá lavinový efekt. Zároveň se vznikem volných nosičů náboje obou znamének dochází i
k jejich zániku vzájemnou rekombinací elektronů a kationtů. Pro vznik výboje je pak podstatné to, že vznik volných
nosičů náboje ionizací převáží jejich zánik rekombinací.

Zhášedlo slouží k tomu, aby se omezilo trvání výboje na několik mikrosekund.

Počítač nemůže detekovat dvě částice přicházející bezprostředně po sobě. Po ionizaci jednou částicí je detektor
krátkou dobu necitlivý – tato doba se nazývá mrtvá doba. Délka mrtvé doby patří mezi důležité charakteristiky GM
trubic. V průběhu mrtvé doby dochází k průletu částic, tím vznikají chyby v měření. Chyby se odstraňují korekčním
výpočtem.

Použití
Jako měřiče kontaminace, hlásiče radiace, monitorovací systémy. Nepoužívají se k přesnějším měřením, protože
dochází k systematickým chybám. Výhodou je schopnost měření v rozsahu několika řádů.

Synonymní názvy
Geigerův-Müllerův počítač
Geiger-Müllerův čítač
Geigerův-Müllerův čítač
Geigerova-Müllerova trubice
Geiger-Müllerova trubice
GM trubice
Scintigrafie
Jedná se o diagnostickou metodu k detekci záření γ a rentgenového záření. Využívá se soustavy, která se skládá ze
scintilačního detektoru a převodně-zesilovací soustavy.

Stavba a princip scintilačního detektoru


Scintilační detektor se skládá z luminiscenčního scintilačního krystalu (většinou thalliem aktivovaný jodid sodný
NaI(Tl)), schopného zachytit ionizující záření ve formě γ nebo rentgenového paprsku. Pohlcením záření se excitují
elektrony krystalu a při jejich následné deexcitaci emitují fotony viditelného světla.[1]Tyto velice slabé záblesky
světla jsou vhodnými světlovodiči převedeny do fotonásobiče.

Úlohou fotonásobiče je znásobit a transformovat paprsky viditelného světla na elektrický impuls, který se sestává z
velkého množství elektronů. Děje se tak při dopadu světelných záblesků z krystalu na fotokatodu. Z fotokatody se tak
uvolní velmi malé množství elektronů, které interagují s dynodami (elektrodami), jejichž povrchová úprava umožňuje
násobení impulsu. Uvolňuje se tak stále více elektronů (ke konci řádově 106–107), které jako salva dopadají na anodu
fotonásobiče. Vytvoří tak měřitelný elektrický impuls, který se zpracovává v zesilovací soustavě. [1]

Mezi fotokatodu a anodu je přivedeno vysoké napětí o velikosti asi 1000 V. Prostředí fotonásobiče je udržováno ve
vakuu.

Fotonásobič

Zesilovací soustava se skládá z předzesilovače. V předzesilovači se amplituda elektrických impulzů upravuje přímo
úměrně ve vztahu k počtu světelných fotonů dopadajících na fotokatodu. Zároveň je i počet světelných fotonů z
krystalu úměrný energii fotonů na krystal dopadajících.
Princip scintilačního detektoru

V zesilovači se signál impulsu z předzesilovače zvyšuje a propouští se do analyzátoru impulsů. Analyzátor impulsy
třídí v závislosti na amplitudě.

Rozlišuje se analyzátor:[1]

Jednokanálový amplitudový – užívá horního a dolního diskriminátoru (tvoří hranici), amplitudy leží mezi těmito
hranicemi. Velikost těchto dvou hranic se nazývá kanálem a je dána v eV. Zaznamenává se počet impulsů v
jednom kanálu, poté se jeho hranice posouvají a vzniká tak postupně amplitudové spektrum.
Vícekanálový amplitudový – mnoho jednotlivých analyzátorů zapojených paralelně, umožňuje tak mnohem
rychleji získat amplitudové spektrum.

Impulsy jsou propouštěny do koncové jednotky, kterou může být čítač, integrátor nebo paměťová jednotka.

Užití scintilačního detektoru


Scintilační detektor se využívá v mnoha diagnostických oborech, zejména v nukleární medicíně. Lze zde využít jako
měřič radioaktivity látek, zejména radiofarmak nebo aktivity biologických materiálů (např. v těle pacienta).
Stanovení aktivity se používá jako běžný postup před dalším zpracováním radiofarmak (tj. ředěním, aplikací), a je
proto zásadní v oboru nukleární medicíny. Používají se:[2]

automatické měřiče aktivity – založené na principu ionizační komůrky nebo scintilátoru, aktivita se měří
nejčastěji v roztoku,
scintilační studnové detektory – odstíněné měřiče aktivity malých objemů radiofarmak založené na
scintilačním detektoru.

V moderní době se používají celotělové detekční systémy, které měří aktivitu látek v těle pacienta bez ohledu na
distribuci v organismu. Lze tak velmi výhodně měřit například kontaminace osob, sledovat vyšetřovaný orgán
označený radioaktivní látkou nebo provádět různé metabolické studie. K těmto měřením se výhradně užívá γ záření.

Scintigrafické zobrazovací systémy

Dělíme je na:[3]

Planární zobrazovací systémy

Založeny na detekci záření a jeho převedení do dvojrozměrných obrazů. Těmto detektorům se také říká
gamakamery. Jedná se o systém složitějších zobrazovacích zařízení. S úspěchem se gamakamery používají i pro
detekci rychlých dynamických dějů radiofarmak, bolusových technik nebo celotělových scintigramů.

Tomografické zobrazovací systémy

Umožňují sledovat i třetí rozměr obrazu na tomografických řezech. Jedná se o emisní počítačové tomografy (ECT),
u kterých je záření emitováno z pacienta. Podle použitého radiofarmaka se užívá SPECT (zde běžně používané 99mTc)
a PET (β+ zářiče).

Odkazy
Související články

SPECT
PET
Gamakamera
Kolimátor

Externí odkazy
Scintigrafie (česká wikipedie)
Scintigraphy (anglická wikipedie)

Zdroj
1. NAVRÁTIL, Leoš a Jozef ROSINA, et al. Medicínská biofyzika. 1. vydání. Praha : Grada, 2005. 524 s. s. 422-
424. ISBN 80-247-1152-4.
2. NAVRÁTIL, Leoš a Jozef ROSINA, et al. Medicínská biofyzika. 1. vydání. Praha : Grada, 2005. 524 s. s.
425. ISBN 80-247-1152-4.
3. NAVRÁTIL, Leoš a Jozef ROSINA, et al. Medicínská biofyzika. 1. vydání. Praha : Grada, 2005. 524 s. s. 430-
431. ISBN 80-247-1152-4.

Použitá literatura

NAVRÁTIL, Leoš a Jozef ROSINA, et al. Medicínská biofyzika. 1. vydání. Praha : Grada, 2005. 524 s. s. 422-
435. ISBN 80-247-1152-4.

KUPKA, Karel, Jozef KUBINYI a Martin ŠÁMAL, et al. Nukleární medicína. 1. vydání. vydavatel, 2007. 185 s. s.
36-37. ISBN 978-80-903584-9-2.
Integrální a selektivní detekce záření gama
Amplitudový analyzátor
Amplitudový analyzátor je v současnosti důležitým krokem ke zpracování elektrických impulsů, které vznikají ve
scintilačním detektoru registrováním paprsků γ. Hodnota elektrických impulsů závisí na elektrickém napětí
impulsu, který je snímán anodou fotonásobiče. Tato hodnota je přímo úměrná intenzitě luminiscence v krystalu
(většinou NaI/Ta) a energii záření γ, která je v scintilačním detektoru absorbována. Detektor zachycuje nejen
paprsky vysílané radionuklidem, který se hromadí v pozorovaném orgánu, ale také zachycuje i paprsky jiných energií.
Paprsky jiných energií (kosmické záření, přirozená radioaktivita prostředí) by mohli zkreslovat výsledky měření a
právě proto se používá amplitudový analyzátor. Pomocí amplitudového analyzátoru můžeme nastavit libovolnou
hodnotu impulzů (napětí), které bude detekováno a ostatní s hodnotou vyšší nebo nižší se zadrží. Úlohou
amplitudového analyzátoru je tedy propustit z celého spektra záření γ jen paprsky zvolené energie. Na základě
tohoto postupu je pak možné vyhodnocovat impulsy dvojím způsobem.

Integrální detekce

Při integrální detekci se pomocí čítače impulsů nebo integrátoru registrují všechny impulsy odpovídající vyšším
amplitudám než je zvolená diskriminační hodnota. Na základě tzv. diskriminační hladiny tato metoda odpovídá
integrální amplitudové diskriminaci. Plocha omezená osou energie a křivkou vyjadřující počet impulsů je úměrná
celkovému počtu detekovaných impulsů.

Amplitudové spektrum impulzů -


měření ve fotopíku

Selektivní detekce

Selektivní detekce se užívá na registraci těch impulsů, které odpovídají hodnotám v rozmezí zvolené dolní a horní
diskriminační hodnoty. Detekují se tedy jenom hodnoty impulsů v daném energetickém rozsahu. Použití
selektivní detekce (detekce ve fotopíku) odpovídá metodě diferenciální amplitudové diskriminaci. Celkový počet
zaregistrovaných impulsů je vymezen fotopíkem.

Energetická rozlišovací schopnost R vyjadřuje vlastnosti scintilačního detektoru. Je dána vztahem:

přičemž ΔE je šířka fotopíku měřená v polovině jeho výšky a Es je střední energie kanálu. Energetická rozlišovací
schopnost se udává v procentech.

Pokud máme radionuklid složený z více izotopů, můžeme detekcí ve fotopíku (při vhodné volbě energie a šířky
fotopíku) měřit záření pouze jednoho radioizotopu. To platí v případě, že dané izotopy emitují záření γ o dostatečně
rozdílné energii. Pak nedochází k překrývání jejich fotopíků.

Odkazy
Související články

Scintigrafie
Detekce ionizujícího záření
Záření gama

Použitá literatura
BENEŠ, Jiří. Základy lékařské biofyziky. - vydání. Karolinum, 2007. 201 s. ISBN 9788024613864.

NAVRÁTIL, Leoš. Medicínska biofyzika. - vydání. Grada, 2005. 524 s. ISBN 9788024711522.


Jednotky radiační dozimetrie
Pro úplnou charakterizaci biologického účinku ionizačního záření je nutno znát veličiny definující míru účinku.

Jednotky radiační dozimetrie

Emise

Mezi základní veličiny, které charakterizují zdroj záření, patří emise. Hodnota emise udává počet částic záření
emitovaných zdrojem za jednotku času. Rozměr emise je s-1.

Aktivita

Pokud je zdroj záření radioaktivní, emise se obsahově pojí s veličinou aktivita A zdroje. Aktivita zdroje je definovaná
jako počet radioaktivních přeměn v daném množství radionuklidu za jednotku času. Jednotkou aktivity je 1 Bq
(becquerel). 1 Bq představuje jednu radioaktivní přeměnu za 1 s. Rozměr aktivity je s-1.

Podrobnější informace naleznete na stránce Aktivita radionuklidu.

Objemová aktivita

Objemová aktivita av vyjadřuje poměr mezi aktivitou zdroje A a objemem V určité látky. Jednotkou je Bq.m3. Rozměr
objemové aktivity je s-1.m-3.

Expozice

Expozice X (ozáření) je charakteristikou pole rentgenového a γ záření. Tato veličina se definuje pomocí ionizačního
účinku záření v daném prostředí (standardně vzduch). Expozice je daná poměrem elektrického náboje ΔQ iontů,
které vznikly zabrzděním elektronů nebo pozitronů a určité hmotnosti Δm objemového elementu vzduchu.

X=ΔQ/Δm

Hlavní jednotkou je 1 C.kg-1. Rozměr expozice je A.s.kg-1.

Grafické znázornění vztahů mezi


jednotkami radiační dozimetrie

Expoziční rychlost

Expoziční rychlost (expoziční příkon) dX/dt je charakterizován jako nárůst expozice v daném časovém intervalu.

dX/dt = ΔX/Δt

Jednotkou je A.kg-1. Rozměr expoziční rychlosti je A.m2.s-2.

Absorbovaná dávka

Absorbovaná dávka D je definovaná jako poměr střední energie ΔE ionizujícího záření absorbované objemovým
elementem vzduchu o určité hmotnosti Δm.

D=ΔE/Δm

Jednotkou je 1 Gy (grey) = 1J.kg-1. Rozměr absorbované dávky D je m2.s-2.

Dávková rychlost

Dávková rychlost (dávkový příkon) dD/dt je vyjádřená poměrem přírůstku dávky ΔD v časovém intervalu Δt.
dD/dt = ΔD/Δt

Jednotkou je Gy.s-1.

Dávkový ekvivalent

Dávkový ekvivalent H koreluje s velikostí biologických účinků různých druhů ionizujícího záření. Používá se pro
účely radiační hygieny a má význam modifikované dávky H = D.Q.N, kde Q je jakostní faktor (tzv. faktor kvality),
který vyjadřuje kvalitu záření z hlediska biologických účinků. N představuje součin dalších faktorů, které popisují
podmínky ozáření. Jednotkou dávkového ekvivalentu je 1 Sv (sievert). Rozměr veličiny je m2s-2.

Odkazy
Související články

Ionizující záření
Deterministické účinky ionizujícího záření
Dozimetrie

Použitá literatura

JIŘÍ, Beneš, Kymplová JAROSLAVA a Vítek FRANTIŠEK. Základy fyziky pro lékařské a zdravotnické obory : pro
studium i praxi. - vydání. Grada Publishing, a.s., 2015. 236 s. ISBN 9788024747125.

BENEŠ, Jiří. Základy lékařské biofyziky. - vydání. Karolinum, 2007. 201 s. ISBN 9788024613864.


Měření aktivity in vitro a in vivo

Článek byl označen za rozpracovaný,


od jeho poslední editace však již uplynulo více než 30 dní
Chcete-li jej upravit, pokuste se nejprve vyhledat autora v historii a kontaktovat jej.
Podívejte se také do .
Pokud vše nasvědčuje tomu, že původní autor nebude v editacích v nejbližší době
pokračovat, odstraňte šablonu {{Pracuje se}} a stránku .
Stránka byla naposledy aktualizována ve čtvrtek 7. února 2019 v 13:14.

Měření aktivity in vitro


Měření aktivity in vitro je měření radioaktivity vzorků. Při výběru vhodné metody měření záleží na různých
faktorech - na energii a druhu záření, velikosti vzorku, jeho aktivitě. Dále záleží na tom, zda chceme provádět
měření absolutní nebo relativní, přičemž na lékařských pracovištích se v naprosté většině případů jedná o měření
relativní aktivity, která je určitou částí absolutní aktivity.

Faktory určující podmínky měření jsou geometrie měření, mrtvá doba detektoru a samoabsorbce záření ve
vzorku. Je nezbytné zajistit, aby při měření relativní aktivity byly tyto podmínky stejné pro vzorek i standard.

Měření vzorku i standardu je prováděno v olověném či v ocelovém krytu, v jehož horní části se nachází detektor -
buď se jedná o scintilační detektor, nebo o Geiger-Müllerův počítač. Pod detektor se v určité vzdálenosti vkládá
vzorek, nebo standard. Vzdálenost volíme v závislosti na tom, aby vzorek poskytoval vhodnou četnost impulzů.

Měření aktivity in vivo


Měření aktivity in vivo je měření radioaktivních látek uložených uvnitř organismu na základě zevní detekce
vycházejícího záření gama. Radioaktivní látky se mohou do organismu dostat buď cílenou aplikací při diagnostickém
či terapeutickém využití ionizujícího záření, nebo při kontaminaci radioaktivní látkou - například při nedodržení
bezpečnosti práce s radionuklidy, při radiační nehodě atd.

Při měření aktivity in vivo se využívají kromě scintilační sondy také speciální stínící kryty. Ty dohromady tvoří tzv.
směrový detektor, jehož citlivost je prostorově závislá. Díky tomu dochází k selektivní detekci záření - záření je
měřeno jenom z určitého místa (směru) v organismu. Aby k tomuto došlo, je třeba odstínit záření, které přichází z
ostatních směrů. K tomu se používá kolimátor, stínící kryt, který má ve směru detekce válcový otvor.

Kolimátor tvoří nejjednodušší směrovou scintilační sondu a jejím postupným přikládáním k různým místům těla
můžeme zjistit distribuci radioindikátoru v jednotlivých orgánech uvnitř organismu po jeho aplikaci, nebo v případě
nasměrování na jednotlivý orgán můžeme sledovat časový průběh distribuce radioindikátoru.

V případě některých vyšetření, například při vyšetření funkce štítné žlázy (stanovení akumulace 131J - jaké množství
radiojodu štítná žláza vychytala), postačuje využití jedné kolimované scintilační sondy. U složitějších vyšetření se
využívá scintilační nebo rotační gamakamera nebo pozitronová emisní tomografie (PET).

Odkazy
Použitá literatura

BENEŠ, Jiří, Pravoslav STRÁNSKÝ a František VÍTEK. Základy lékařské biofyziky. 2. vydání. Praha:
Nakladatelství Karolinum, 2007. ISBN 978-80-246-1386-4

Externí literatura

Detekce a spektrometrie ionizujícího záření (na serveru AstroNuklFyzika.cz


Osobní dozimetrie
Dozimetrie je souborem metod používaných k měření absorbované dávky záření a využívaných zejména v lékařství či
jaderné energetice.

Smyslové ústrojí člověka není schopno vnímat účinky ionizujícího záření. Jeho nadměrné působení zvyšuje riziko
ohrožení zdraví jedince. Osobní dozimetrie slouží k měření absorbované dávky záření. Informace o jednotlivých
typech ionizujícího záření získáváme díky interakci látky s vhodnou absorbující látkou – detektorem.

Druhy osobních dozimetrů


Dozimetry měří dávku záření, které dopadá na lékaře či pracovníka v jaderné elektrárně.

Filmové dozimetry

Filmové dozimetry jsou nejstarším používaným typem detektoru. Jedná se o obdobu rentgenového filmu. Pracovník
nosí zařízení na přední straně pláště okénkem dopředu a film je po měsíci odeslán k analýze. Vyrábějí se v rozměru
3x4 cm. Dozimetr je umístěn ve světlotěsném papírovém obalu, pokryt obalem z umělé hmoty a má okénko na přední
části. Film je velice citlivý na působení záření. Hustota zčernání filmu je úměrná míře expozice. Vyhodnocení probíhá
na základě míry zčernání. Výhodou filmových dozimetrů je nízká finanční náročnost, lehká manipulace, uspokojivá
přesnost. Nevýhodou je omezená životnost a zvýšená náchylnost k poškození vlhkostí a vyššími teplotami.

Osobní dozimetr

Termoluminiscenční dozimetry

Termoluminiscenční dozimetry jsou založeny na citlivosti některých látek na ionizující záření. Příkladem je využití
krystalické formy lithia (LiF). Po vystavení účinkům záření a následném zahřátí na vysoké teploty dochází k deexcitaci
elektronů v krystalech, kterou doprovází emise viditelného světla. Fotonásobič měří intenzitu emitovaného světla,
která je úměrná dávce záření absorbovaného krystalem. Je používán ve formě prstového dozimetru, kdy měří dávku
dopadající na ruce exponované osoby. Výhodou je jejich jednoduchost, citlivost a zároveň odolnost vůči vlhkosti.
Nevýhodou je vyšší cena a citlivost na viditelné a ultrafialové světlo. Nošeny jsou po dobu tří měsíců.

Tužkové dozimetry

Tužkové dozimetry jsou ionizační komory ve formě tužky.

Slepé dozimetry
Slepé dozimetry jsou malé komůrky ovoidního či zaobleného tvaru. Dozimetry je možné rozšroubovat a nabít na
zvolené napětí. Po určité době působení záření elektrometr změří úbytek napětí i elektrostatického náboje, který je
úměrný ozáření komůrky.

Odkazy
Použitá literatura

BENEŠ, Jiří, et al. Základy lékařské biofyziky. 3. vydání. Praha : Karolinum, 2011. 200 s. ISBN 978-80-246-


2034-3.

ROSINA, Jozef a Hana KOLÁŘOVÁ. Biofyzika pro studenty zdravotnických oborů. 1. vydání. Praha : Grada, 
2006. 232 s. ISBN 80-247-1383-7.

NAVRÁTIL, Leoš a Jozef ROSINA, et al. Medicínská biofyzika. 1. vydání. Praha : Grada, 2005. 524 s. ISBN 80-
247-1152-4.

You might also like